Vous êtes sur la page 1sur 527

SMLE Q BANK Group

Up to 16th of December

7th UPDATE



( SMLE Q Bank group ) Leaders
(
)

" "
( + )
( )
:
Smle2015.2016@gmail.com

2016 2015

New to the 7th edition:


" Questions Up to 16th of December "

Corrected some questions


Added Un-classified Questions at the end.
Added questions in Internal Medicine section
Added questions in Surgery section
Added questions in Basic Sciences
Added questions in Orthopedic
Added questions in OBS/GYN
Added questions in Pediatric

Corrections:

Medicine: old Qs: Q1,Q51,Q53,Q172.


th
added Qs(6 update) section: Q14, Q35, Q60, Q61, Q64.

Ophtha

old: Q15, Q33


added Qs: Q2

ENT

old: Q4
added Qs: Q9

Anesthesia: old: Q11


Added: Q2

Surgery old: Q111

OB/GYNE old: Q49,Q106

Pedia

added: Q24

We need you to send us all the questions that you think are reapeated or
wrong with the answer suggested and the reference:
smle2015.2016@gmail.com

For Updated Versions:


http://cutt.us/nOtw9

For the New questions :


http://cutt.us/JrP7b
2

Introduction
This is a collaboration between medical interns. This collaboration helps in gathering SMLE
Questions as much as possible from multiple sources including Saudi Medical Student
website.
These questions have already been reviewed and answered. However, there is a chance of
mistakes in the answers chosen especially that some of the questions we receive have
missing information that might change the answer.
Some Questions are updated and added frequently, considering previously answered questions
as well.
*Highlights = Some of doubtful answers.

Table of Contents
Medicine ......................................................................................................................................
Surgery .......................................................................................................................................
Pediatrics ...................................................................................................................................
Obstetrics and Gynecology ........................................................................................................
Emergency .................................................................................................................................
Anesthesia .................................................................................................................................
Family Medicine and Statistics .................................................................................................
Ethics .........................................................................................................................................
Psychiatry .................................................................................................................................
Orthopedics ..............................................................................................................................
Dermatology .............................................................................................................................
Ophthalmology .........................................................................................................................
ENT ..........................................................................................................................................
Basic Science ............................................................................................................................
Embryology .......................................................................................................................
Histology ...........................................................................................................................
Anatomy ............................................................................................................................
Biochemistry ......................................................................................................................
Immunology ......................................................................................................................
Pathology ..........................................................................................................................
Pharmacology ....................................................................................................................
Genetics ...............................................................................................................
Un-classifided Questions .....................................................................................................

Internal Medicine

1- Diabetic patient developed fever, productive cough and SOB. Labs show high WBC. CXR (picture was given which
showed lower lobe infiltrates + air-fluid level).The drug that will be given to the patient acts on which of the
following?
a.
DNA gyrase
b.
30 S ribosome
c.
50 S ribosome
d.
Transpeptidase
Answer: C
http://emedicine.medscape.com/article/428135-overview#showall
http://www.uptodate.com/contents/lung-abscess?source=outline_link&view=

Reference: Toronto Notes


2- Patient developed nausea and vomiting then developed cranial nerve palsies then bilateral symmetrical
progressive LL paralysis. What is the most likely diagnosis?
a.
Tetanus
b.
Botulism
c.
Lead poisoning
Answer: B
Botulism is an acute neurologic disorder that causes potentially life-threatening neuroparalysis due to a neurotoxin
produced by Clostridium botulinum. The 3 main clinical presentations: Infant botulism, Foodborne botulism and Wound
botulism.
Signs and Symptoms:
occur 6-48 h after ingestion
difficulty with convergence, ptosis, paralysis of extraocular muscles
dilated, poorly reactive pupils
other autonomic dysfunction: jaw weakness, dysarthria, dysphagia.
spreads to trunk and limbs
abdominal cramps with nausea and vomiting
symmetric weakness with paralysis and absent/decreased deep tendon reflexes
anticholinergic symptoms: dry mouth, constipation, urinary retention
rarely respiratory distress, potentially advancing to respiratory failure
Reference: Toronto Notes and Medscape.

3- Patient with ventilator associated pneumonia. Culture showed lactose non-fermenting, gram negative motile
bacilli producing greenish colony + Oxidase positive. What is the organism?
a.
Haemophilus influenzae
b.
Streptococcus pneumoniae
c.
Klebsiella or other gram negative bacteria
d.
Pseudomonas aeruginosa
Answer: D

Reference: http://www.columbia.edu/itc/hs/medical/pathophys/id/2008/utiGNR.pdf
4- HIV patient, presented with SOB and productive cough. Lung biopsy showed soap bubble like intra-alveolar
lesions with exudates, small cyst, stained by silver stain.
a.
Pneumocystis jiroveci
b.
Aspergillus fumigatus
c.
Cryptococcus neoformans
Answer: A
Pneumocystis jiroveci (previously named P. carinii), is the most common opportunistic infection in patients with AIDS,
and it is an increasing cause of disease in other immunosuppressed persons. It cannot be cultured and most clinical
laboratories rely on microscopic examination of stained material from the respiratory tract. The most widely used stain
techniques are those that stain the cyst wall, such as Gomori methenamine silver (GMS), toluidine blue, and certain
fluorescent brighteners (FB). Reference: American Journal of Clinical Pathology (AJCP).
One of the special stains is silver stain (fungi. Pneumocystis carinii) for HIV/ immunocompromised patients. Reference:
Step Up to Medicine.
5- Breast cancer patient who receives many medications (cyclophosphamide, fluorouracil, ..etc). In order to avoid the
adverse effect of having hemorrhagic cystitis which of the following will be given to this patient?
a.
Aldesleukin
b.
Mesna
Answer: B
Mesna: Protects the bladder from damage that may be caused by some cancer medicines.
6- Long scenario about a patient who presented with gout. Inhibition of which enzyme will treat this disease:
a.
PRPP synthase
b.
Adenosine deaminase
c.
Xanthine oxidase
d.
Orotate phosphoribosyltransferase
Answer: C

The enzyme xanthine oxidase catalyses the oxidation of hypoxanthine to xanthine and then to uric acid, which plays a
crucial role in gout. Reference: Pubmed. Thats why we give Xanthine Oxidase inhibitors such as Allopurinol.
7- Patient with shoulder pain and pleurisy. Which part of the pleura causes radiation of the pain to shoulder:
a.
Visceral
b.
Mediastinal
c.
Costal
d.
Anterior
Answer: B
Visceral pleura: insensitive to pain due to autonomic innervation.
Parietal pleura:
Costal and peripheral parts of diaphragmatic pleura are referred along intercostal nerves to thoracic and
abdominal wall.
Mediastinal and central diaphragmatic pleural pain referred to root of neck and over shoulder (Dermatomes C3C5). Reference: Lippincott's Concise Illustrated Anatomy: Thorax, Abdomen & Pelvis.
8- Which one of these can cause LBBB:
a.
Aortic stenosis
b.
PE
c.
Cardiomyopathy
Answer: A (Most common of these choices)
Arteriosclerotic coronary artery disease is the most common adult cardiovascular disease. As a result, it is the most
common cause of left bundle branch block. Hypertension, aortic valve disease and cardiomyopathies continue to be
important but less common etiologic disorders. Reference: Journal of the American College of Cardiology.
9- Patient is hypertensive. In addition to antihypertensive medications the patient was given phytosterol. What is the
mode of action of phytosterol?
a.
Decrease plasma triglycerides
b.
Decrease plasma cholesterol
c.
Inhibit fatty acid synthesis
d.
Decrease de novo synthesis of cholesterol
Answer: B
Phytosterols are plant sterols structurally similar to cholesterol that act in the intestine to lower cholesterol absorption.
Reference: Pubmed.
10- A patient with tuberculosis on medication for 3 months. He developed pins and needles sensation of his lower
limbs. Deficiency of which of the following caused his symptoms?
a.
Niacin
b.
Folic acid
c.
Iron
d.
Pyridoxine (B6)
Answer: D
Vitamin B6 (pyridoxine) supplementation during isoniazid (INH) therapy is necessary in some patients to prevent the
development of peripheral neuropathy. Reference: Pubmed.
11- right coronary artery dominance" is explained as right coronary artery giving branch to:
a.
Circumflex
b.
Anterior Descending
c.
Posterior Descending
d.
Marginal
Answer: C

12- How to monitor the response to iron treatment?


a.
Ferritin
b.
Hct
c.
RBC
d.
Reticulocyte count
Answer: D
Monitoring response:
reticulocyte count will begin to increase after one wk
Hb normalizes by 10 g/L per wk (if no blood loss)
Iron supplementation required for 4-6 mo to replenish stores.
Reference: Toronto notes
13-What gene is related to Celiac Disease?
Answer: ?
HLA-DQ2 (chromosome 6) found in 80-90% of patients compared with 20% in general population; also associated with
HLA-DQ8. Reference: Toronto Notes
14- According to the new classification of lung cancer, which of the following is considered carcinoma in-situ?
a.
Adenocarcinoma less than 2 cm.
b.
Atypical hyperplasia
Answer: A
Adenocarcinoma in situ (AIS) with no invasive features is a localized, small (3 cm) adenocarcinoma with growth
restricted to a noninvasive lepidic pattern and an absence of papillary or micropapillary patterns or intraalveolar tumor
cells. Reference: UpToDate.
15- What is the optimal duration of antibiotic treatment in strep throat?
a.
3 days
b.
5 days
c.
7 days
d.
10 days
Answer: D
Penicillin is the treatment of choice for strep throat. It is usually given in pill or liquid form, two to four times per day for
10 days. Reference: UpToDate.
16- Patient presented with non traumatic acute urinary retention. How will you treat him/her?
a.
Foley catheter
Answer: A
Reference: UpToDate
17- Diabetic patient who is allergic to sulfa drugs, on metformin but its not controlled. What will you add to control
his diabetes?
a.
chlorpropamide
b.
glyburide
c.
rosiglitazone
Answer: C or Gliptin?
Gliptin (DPP-4 inhibitor) is a better choice (Answered by an endocrine consultant).
Chlorpropamide (sulfonylurea)
Glyburide (sulfonylurea)
Rosiglitazone (thiazolidinedione)

18- What is the anti influenza medication that is given intranasally?


Answer: ?
Live Attenuated Influenza Vaccine (Nasal spray flu vaccine) provide protection against four flu viruses: an influenza A
(H1N1) virus, an influenza A (H3N2) virus and two influenza B viruses. Reference: Centers for Disease Control and
Prevention.
19- What is the mechanism of action of metformin on the cellular level?
a.
decrease Muscle uptake
b.
increase Muscle gluconeogenesis
c.
Enhance muscle use of fatty acid
Answer: C
Reference: Pubmed
20- Patient with arthritis and rash on the face. ANA is positive. What should you do next?
a.
Anti DNA
Answer: A
21- What is the most important risk factor for stroke?
a.
Hypertension
b.
DM
c.
Smoking
Answer: A
Hypertension emerges as the single most powerful and reversible risk factor in stroke and for survival after stroke.
Impaired cardiac function is the second most important precursor of stroke. Reference: Pubmed.
22- Young patient has hypertension, high Na and low K. What is the treatment?
a.
Spironolactone
Answer: A
This patient has hyperaldosteronism.
Spironolactone is a potassium sparing diuretic.
23- Chemotherapy can case which type of anemia?
a.
aplastic anemia
Answer: A
It will cause bone marrow suppression.
24- Old lady living alone for 5 years. She has memory problem and looks pale.
a.
B12 deficiency
Answer: A
This patient is pale, this is a hint for anemia, vit-B12 deficiency can cause memory problems plus anemia.
Thats why if you have old patient with memory problems, before you say he has dementia, you should exclude:
Hypothyroidism and vit-B12 deficiency.
Reference: Step-up to Medicine.
25- Cardiac patient on Aspirin, no new complaints. He is having low platelet (less than 10) for the last 6 months. What
is your management?
Answer: ? the diagnosis is rare and called aspirin induced thrombocytopenia
Stop aspirin and find the cause (Answered by a senior cardiologist consultant)

26- Patient with Crohn's disease. Most relevant and associated with CD?
a.
Positive Family History
b.
Smoking
c.
Alcohol
Answer: A
Positive family history is the largest independent risk factor for CD and UC (Kumar)

27- Which medication will delay the surgery for chronic aortic regurgitation?
a.
Digoxin
b.
Verapamil
c.
Nifedipine
d.
Enalapril
Answer: C
Nifedipine is the best evidence-based treatment in this indication.
ACE inhibitors are particularly useful for hypertensive patients with AR.
Beta blockers may be indicated to slow the rate of aortic dilatation and delay the need for surgery in patients with AR
associated with aortic root disease. Furthermore, they may improve cardiac performance by reducing cardiac volume
and LV mass in patients with impaired LV function after AVR for AR. Reference: 3rd Edition UQU>Cardiology> Q3
In severe aortic valve insufficiency, the excess in afterload increases burden on the left side of the heart. Theoretically,
any medication that can reduce afterload could be expected to improve left ventricular function and decrease
regurgitant backflow from the aorta. This would provide a temporizing measure by which surgical intervention can be
postponed. One study showed that the use of nifedipine in asymptomatic patients with severe aortic regurgitation who
had normal LV function could delay the need for surgery by 2-3 years. This result may also be expected with the use of
similar vasodilating agents. Reference: Medscape.
28- Diabetic patient has history of weakness and dizziness. what anti-diabetes can cause it?
a.
Sulfonylurea (glipizide)
b.
Metformin
c.
Thiazolidinediones (rosiglitazone)
d.
Insulin
Answer: A? (Symptoms are consistent with hypoglycemia)
Sulfonylurea side effects: Hypoglycemia Weight gain.
Metformin can cause unusual tiredness or weakness. Glucagon-like peptide (GLP)-1 analogue: Exenatide or
Liraglutide causes muscle weakness.
Pioglitazone, rosiglitazone and troglitazone can cause myalgia and muscle weakness.
References: Toronto Notes + Mayoclinic + Pubmed
29- Which of these anti HTN medications decrease afterload and preload?
a.
ACEI
Answer: A
30- Most common cause of DKA in adult:
a.
Missing insulin
b.
Dietary
c.
Increase physical activity
Answer: ?
The most common precipitating factor is infection, followed by noncompliance with insulin therapy.
Reference: http://www.aafp.org/afp/2005/0501/p1705.html

10

31- Why are we concerned about anti hypertensive agents in elderly patients?
a.
Hypotension
b.
Hypokalemia
c.
CNS side effect
Answer: A
Reference: http://www.ncbi.nlm.nih.gov/pubmed/11574742
32- Patient with chest pain and ST changes, you will find elevation in:
a.
ALT
b.
AST
c.
Troponin
Answer: C
33- Prophylactic antiarrhythmic therapy:
a.
Procainamide
b.
Lidocaine
c.
Amiodarone
Answer: ?
Dont give the patient any anti-arrhythmic medication unless he has an arrhythmia. Reference: Step up to Medicine.
C is the best answer compared to A and B (if not post MI)
The question might be similar to this (Post MI):

In this case the answer will be Metoprolol, which is a beta blocker (Prophylaxis for arrhythmias after MI: Beta blocker).
Reference: Lippincott Illustrated Reviews Pharmacology.
34- Asymptomatic Patient. Chest X ray shows a unilateral calcified nodule on the upper zone of his lung?
a.
Adenoma
b.
Granuloma
c.
Hamartoma
d.
SCC
Answer: B
The most common cause of nodule calcification is granuloma formation, usually in the response to healed infection.
Reference: http://radiopaedia.org/articles/calcified-pulmonary-nodules

11

35- Elderly patient presented with typical symptoms of multiple myeloma. X ray showed lytic lesions, positive M
protein and hypercellular bone marrow. What other findings can be found in the blood?
a.
Increase peripheral blood B cell
b.
Rouleaux formation
Answer: B
Reference: Pubmed
36- Elderly female complaining of depression, bilateral shoulder and hip pain. Normal blood workup.
a.
polymyalgia rheumatica
b.
fibromyalgia
Answer: A

37- Which of the following is the the most specific for Rheumatoid arthritis?
a. HLA-DR4.
b. Rheumatoid factor.
c. CRP.
d. Anti cyclic citrullinated peptide. (Anti-CCP)
Answer: D
Anti CCP is the most specific for RA

12

38- Patient on diuretics developed palpitation. Due to the disturbance of which of these electrolytes lead to his
presentation?
a.
Na
b.
K
c.
Cl
d.
Ca
Answer: B
Hyperkalemia: usually asymptomatic but may develop nausea, palpitations, muscle weakness, muscle stiffness,
paresthesias, areflexia, ascending paralysis, and hypoventilation. Can be caused by K+-sparing diuretics such as
Spironolactone, Amiloride and Triamterene. Reference: Toronto Notes.
39- Treatment for trigeminal neuralgia:
a.
Carbamazepine
Answer: A
1st line: carbamazepine or oxcarbazepine. Reference: Toronto notes
40- Schistosoma Antibiotics:
a.
Praziquantel
Answer: A
Praziquantel.
Add glucocorticoid if acute schistosomiasis or neurologic complications develop. Reference: Toronto notes
41- Patient on central cath developed infection. Blood gram stain shows budding yeast.
a.
Fluconazole
Answer: A
Reference: Toronto notes
42- Most specific test for TB:
a.
PCR
b.
Chest X ray
c.
Sputum culture
d.
PPD
Answer: C
Reference: http://www.cdc.gov/tb/publications/factsheets/testing/diagnosis.htm
43- Medication for resistant hiccups:
Answer:?
Chlorpromazine is the most thoroughly studied and appears to be the drug of choice. Reference: Medscape
44- 19 years old male cant bring the spoon infront of himself to eat after a bike accident. Where is the site of his
lesion?
a.
Parietal lobe
b.
Temporal lobe
c.
Occipital lobe
d.
Cerebellum
Answer D
It is a problem of coordination.
References:
http://www.healthline.com/health/movement-uncoordinated#Overview1
Al-Qassim MCQs > Q 342

13

45-According to the modified criteria of rheumatic fever which is considered as a minor criteria ?
a.
Carditis
b.
Fever
c.
Arthritis
Answer: B

46- Coarctation of aorta, what can cause:


a.
coronary artery disease
b.
aortic dissection
Answer: ?
Complications of untreated patient include: HTN, stroke, aortic aneurysm, aortic dissection, premature coronary artery
disease, HF, brain aneurysm or hemorrhage. The most common complication in adult is HTN and CAD.
47- Asymptomatic patient, known case of chronic gastritis, has positive occult blood stool and his Hb=9. You will
manage him by:
a.
IM iron
b.
Oral iron
c.
Erythropoietin
d.
Blood transfusion
Answer: A (IV iron would be more appropriate answer)
Similar question in another exam with different choices:
Elderly man on NSAIDs developed dyspepsia. Endoscopy showed gastritis. Labs showed iron deficiency anemia with
Hb= 9. What is the treatment?
a.
IV iron
b.
IM iron
c.
Erythropoietin
d.
Oral Iron
Answer: A
Oral ferrous sulfate associated with a significantly higher risk of GI side effects than IV iron.
2- Acquired malabsorption for iron with autoimmune atrophic gastritis or Helicobacter pylori infection
Reference: http://www.uptodate.com/contents/treatment-of-the-adult-with-iron-deficiency-anemia
48- A patient presented with frothy red sputum, flushed cheeks, etc (long scenario). What is the diagnosis?
a.
mitral stenosis
Answer: A
Reference: Master the boards + Medical diagnosis and Management by Mohammad Inam Danish

14

49- A male patient presented with symptoms. Labs showed 80% blasts with 20% aurer rodes. What is the diagnosis?
a.
CML
b.
AML
c.
CLL
d.
ALL
Answer: B
Auer rods are a hallmark of acute myeloid leukemia.
50- A patient with SLE with rash on her cheeks, etc. What will you advise her?
a.
avoid sun exposure as much as she can
Answer: A
Reference: Medical diagnosis and Management by Mohammad Inam Danish
51- A farmer presented with 2 weeks history of fever, headache and one more symptom. What is the most likely
diagnosis?
a.
Brucellosis
b.
Meningitis
Answer: A, The trick in the duration, bacterial meningitis usually present over several hours ( Cryptococcal meningitis
may be present for several weeks - but it's not one of the options). In the other hand, brucellosis may show up anytime
from a few days to a few months after the person get infected.
Brucellosis is an infectious disease. People can get the disease when they are in contact with infected animals or
animal products contaminated with the bacteria. Animals that are most commonly infected include sheep, cattle,
goats, pigs, and dogs, among others. Signs and symptoms include fever, headache, sweats, malaise, anorexia pain in
muscles, joint, and/or back, fatigue, Reference: Centers for Disease Control and Prevention.
52- Dental caries caused by which organism?
a.
candida
b.
HSV
c.
Streptococcus mutans
Answer: C
The group of "mutans streptococci" was described as the most important bacteria related to the formation of dental
caries. Reference: Pubmed.
53- An asthmatic who needs daily short acting beta 2 inhalers, oral steroids and daily spirometry monitoring of PFTs.
What is his asthma stage?
a.
Mild intermittent
b.
Mild persistent
c.
Moderate
d.
Severe
Answer: D
Once the patient on oral steroid he/she classified as a severe asthma.
Reference: http://www.med.umich.edu/1info/FHP/practiceguides/asthma/EPR-3_pocket_guide.pdf
54- A patient with signs and symptoms of renal and respiratory involvement. What is the diagnosis?
a.
Glomerulonephritis
b.
Wegener's granulomatosis
Answer: B? Incomplete question but you should to keep one thing in your mind if they mentioned there is a hx. of
sinusitis the diagnosis with be 100% Wegners
Reffrence: First Aid step 1
Reference: Master the boards

15

55- A patient with dyspnea, pallor, edema of legs and itching. What is the diagnosis?
a.
scabies
b.
renal failure
Answer: B
56- A patient with gout. What drug should be avoided?
a.
thiazide
b.
furosemide
c.
ACEI
Answer: A
Hyperuricemia is a relatively common finding in patients treated with a loop or thiazide diuretic and may, over a period
of time, lead to gouty arthritis. Furosemide can elevate uric acid levels as well. Reference: UpToDate.
57- You read a PPD result after 48 hours. It showed 10 mm induration. What does that indicate?
a.
negative
b.
weakly positive
c.
strongly positive
Answer: ?
The person's medical risk factors determine the size of induration the result is positive (5 mm, 10 mm, or 15 mm).
Five mm or more is positive in:
HIV-positive person
Recent contacts of active tuberculosis cases
Persons with nodular or fibrotic changes on Chest X-ray consistent with old healed TB
Organ transplant recipients and other immunosuppressed patients who are on cytotoxic immune-suppressive
agents such as cyclophosphamide or methotrexate.
Patients on long term systemic corticosteroid therapy (> than six weeks) and those on a dose of prednisone 15
mg/day or equivalent.
End stage renal disease
Ten mm or more is positive in:
Recent arrivals (less than five years) from high-prevalence countries
Injectable drug users
Residents and employees of high-risk congregate settings (e.g., prisons, nursing homes, hospitals, etc.)
Mycobacteriology lab personnel
Persons with clinical conditions that place them at high risk (e.g., diabetes, prolonged corticosteroid therapy,
leukemia, end-stage renal disease, chronic malabsorption syndromes, low body weight, etc.)
Children less than four years of age, or children and adolescents exposed to adults in high-risk categories
Infants, children, and adolescents exposed to adults in high-risk categories
Fifteen mm or more is positive in:
Persons with no known risk factors for TB. (Reactions larger than 15 mm are unlikely to be due to previous BCG
vaccination or exposure to environmental mycobacteria). Reference: Pubmed
58- A patient with renal function test abnormalities. Tests show beads on string appearance. What is the diagnosis?
a.
renal artery disease
b.
fibromuscular dysplasia
Answer: B
The string-of-beads appearance is considered pathognomonic for medial fibroplasia on diagnostic angiography.
Reference: http://emedicine.medscape.com/article/417771-overview#a
59- Diabetic patient on metformin and another drug but still blood sugar not controlled. He is allergic to sulfa,
what to add?
Answer: ?

16

Metformin should be used as initial therapy for type 2 diabetes unless contraindications exist. Second-line agents
include sulfonylureas, DPP-4 inhibitors, GLP-1 receptor agonists, meglitinides, and -glucosidase inhibitors. TZDs are
no longer recommended because of potential increases in cardiovascular risk and fracture risk.
The incretin agents are a reasonable second choice if the main problem is PPG elevation and if hypoglycemia and
weight gain are concerns. If FPG and postprandial glucose are elevated and if cost is a concern, sulfonylureas are a
reasonable second-line agent.
If a patient has symptomatic hyperglycemia, poor control despite two to three oral agents, or an A1C > 8.5%,
insulin should be considered.
Reference: http://m.clinical.diabetesjournals.org/content/30/2/72.full
60- A patient with myasthenia gravis presents with myasthenic crisis. What is the cause?
a.
anticholinesterases
b.
pyridostigmine
Answer: ?
Numerous medications may exacerbate MG, including quinidine, procainamide, -adrenergic antagonists, calcium
channel antagonists (verapamil, nifedipine, felodipine), magnesium, antibiotics (ampicillin, gentamicin, streptomycin,
polymyxin, ciprofloxacin, erythromycin), phenytoin, gabapentin, methimazole, -interferon and contrast media.
Reference: Pubmed
Myasthenia gravis (MG) is a disorder of muscular weakness from the
production of antibodies against acetylcholine receptors at the
neuromuscular junction.
Best initial treatment of myasthenia gravis: Neostigmine or
pyridostigmine (anticholinesterase agents / acetylcholinesterase
inhibitors). These are longer acting versions of edrophonium.
Acute myasthenic crisis presents with severe, overwhelming disease
with profound weakness or respiratory involvement. It is treated with
IVIG or plasmapheresis. Reference: Master the boards
61-

what is diagnosis?
a.
SVT
b.
AF
c.
VT
d.
VF
Answer: A
same ECG that shows SVT in UQU cardiology section << couldnt find it, so I uploaded another ECG strip.
Read about other arrhythmias.
62- The most specific/ best diagnostic test for syphilis?
a.
Fluorescent treponemal antibody absorption (FTA-ABS)
Answer: A
Nontreponemal testing is commonly used for serologic screening for syphilis, which include VDRL, RPR, and TRUST.
(These tests must be confirmed with specific treponemal testing since false positive nontreponemal tests can occur.
Nontreponemal testing is performed initially due to its low cost.)
Specific treponemal tests include FTA-ABS, MHA-TP, TP-PA, and TP-EIA. Reference: UpToDate

17

63- Signs and symptoms of 1st syphilis?


a.
Painless genital ulcer
Answer: A
Reference: http://www.cdc.gov/std/syphilis/stdfact-syphilis-detailed.htm
64- 37 years old presented with back pain. On examination there was tenderness when palpating paraspinal muscles,
neurovascular exam was normal. What is the treatment?
a.
Physiotherapy
Answer: A
65- Patient on warfarin 7 mg presented with melena. INR was very high. What will you do?
Answer:?
The treatment of excessive anticoagulation is based on the level of the INR, the presence or absence of bleeding, and
clinical circumstances. Reversal of Warfarin sodium anticoagulation may be obtained by discontinuing Warfarin sodium
therapy and, if necessary, by administration of oral or parenteral vitamin K1.
Reference: http://www.drugs.com/pro/warfarin.html

Reference: Toronto Notes.


66- A girl with migraine. She doesnt want to take prophylactic medicine.
a.
biofeedback
b.
sumatriptan
c.
ergotamine
d.
propranolol
Answer: A
67- Patient developed dysphagia. On examination there was deviation of the uvula to the left side. Which nerve is
affected?
a.
Lt. Vagus
b.
Rt. Vagus
c.
Hypoglossal
d.
Glossopharyngeal
Answer: B
In glossopharyngeal nerve (sensory) involvement, there will be no response when touching the affected side. With
vagal nerve damage, the soft palate will elevate and pull toward the intact side regardless of the side of the pharynx
that is touched. If both CN IX and X are damaged on one side (not uncommon), stimulation of the normal side elicits
only a unilateral response, with deviation of the soft palate to that side; no consensual response is seen. Touching the
damaged side produces no response at all. Reference: Disorders of the Nervous System-Dartmouth.
68- Loss of sensation over the maxilla and mandible:
a.
trigeminal
Answer: A

18

69- Obese patient recently diagnosed to have DM II. He is following a diabetic diet regimen and he exercises
regularly. When he came to you in the next visit... His blood sugar was high and he gained 5 kgs... He was also
complaining of thirst and hunger, what would you give him:?
a.
Metformin
Answer: A
70- Most common cause of excessive sleepiness in the daytime is?
a.
Narcolepsy
b.
Obstructive Sleep Apnea (OSA)
Answer: B
The most common causes of excessive daytime sleepiness are sleep deprivation, obstructive sleep apnea, and sedating
medications. Reference: American Family Physician Journals.
71- A patient with facial nerve involvement. Presents with loss of taste sensation of the anterior 2/3 of tongue. There
is loss of function of stapedius as well. At what point is the injury?
*options includes different points of facial nerve course
Answer: ?
Facial canal between geniculate ganglion and nerve to stapedius muscle.
Reference: http://www.ncbi.nlm.nih.gov/books/NBK385/

72- Patient with needle sensation after TB drug.


a.
Isoniazid
Answer: A
73- What causes polyuria in DM?
a.
Increase glucose in urine
b.
Increase glucose in serum
c.
Increase ketones in serum
Answer: A

19

74- 60 years old man with a fractured thoracic vertebrae. T Score = -2.6. What is the diagnosis?
a.
Osteoporosis
b.
Established osteoporosis
c.
Osteopenia
Answer: B
Dexa scan interpretation:

T-score
>1.0
<1.0, >2.5
<2.5
<2.5 plus fragility fractures

Diagnosis
Normal
Osteopenia
Osteoporosis
Severe/ Established
osteoporosis

75- Why inulin is used in estimation of GFR?


Answer: ?
The perfect filtration marker: is not protein bound, is freely filtered by the glomerulus, is without any tubular secretion,
is not metabolised by the kidneys, and is physiologically inert. Very few substances fulfil these criteria: the gold
standard has been a plant polysaccharide called inulin. Reference:
http://www.ncbi.nlm.nih.gov/pmc/articles/PMC3100284/
76- Best treatment for allergic rhinitis.
Answer: ?
The management of allergic rhinitis consists of 3 major categories of treatment, (1) environmental control measures
and allergen avoidance, (2) pharmacological management, and (3) immunotherapy.
Glucocorticoid nasal sprays are presently the most effective single-agent maintenance therapy for allergic rhinitis
Reference: http://www.uptodate.com/contents/pharmacotherapy-of-allergic-rhinitis#H25107621
77- A patient with vit B 12 deficiency. what is the type of gastric cell that will be affected?
a.
Chief cell
b.
Parietal cell
Answer: B
Patient with pernicious anemia is usually directed against the intrinsic factor or parietal cell themselves.
78- Which one of the following can cause liver cirrhosis?
Answer:?
Fatty liver (alcohol, metabolic syndrome)
Chronic viral hepatitis (B, B+D, C; not A or E)
Autoimmune hepatitis
Hemochromatosis
1-antitrypsin deficiency
Primary biliary cirrhosis
Chronic hepatic congestion
a. cardiac cirrhosis (chronic right heart failure, constrictive pericarditis)
b. hepatic vein thrombosis (Budd-Chiari),
Idiopathic
Rare: Wilsons disease, Gauchers disease . Reference: Toronto Notes.
79- A patient known case of anemia on medications. Later he came complaining of dark stools. What is the
medication?
a.
Ferrous sulphate
b.
Folic acid
c.
Iron dextran?
Answer: A

20

80- A Patient had an MI and was treated for it, after that he developed chest pain that worsen with movement and
taking deep breath. On examination there was distant heart sounds and pericardial rub. What is the most ECG
changes associated with this condition?
a.
ST changes
b.
PR prolongation
Answer: A
A case of pericarditis.
ECG: initially diffuse elevated ST segments depressed PR segment, the elevation in the ST segment is concave
upwards >> 2-5 d later ST isoelectric with T wave flattening and inversion. Reference: Toronto Notes.
81- bacterial meningitis in LP
a.
decrease glucose and increase protein
Answer: A
Glucose level: Low (< 40% of serum glucose) and Protein level: Elevated (>50 mg/dL)
Reference: http://emedicine.medscape.com/article/2172226-overview
82- Mediterranean blood disease
a.
Thalassemia
Answer: A
Reference: Toronto Notes
83- A patient known case of aortic stenosis had syncope. What is the cause?
a.
systemic hypotension
Answer: A
Syncope from aortic stenosis often occurs upon exertion when systemic vasodilatation in the presence of a fixed
forward stroke volume causes the arterial systolic blood pressure to decline. It also may be caused by atrial or
ventricular tachyarrhythmias.
Syncope at rest may be due to transient ventricular tachycardia, atrial fibrillation, or (if calcification of the valve extends
into the conduction system) atrioventricular block. Another cause of syncope is abnormal vasodepressor reflexes due to
increased LV intracavitary pressure (vasodepressor syncope).
Reference: http://emedicine.medscape.com/article/150638-clinical
84- What is the most common symptom of factor 8 deficiency?
a.
hemarthrosis
Answer: A
In severe hemophilia A, spontaneous joint or deep-muscle bleeding is the most frequent symptom.
Reference: http://www.ncbi.nlm.nih.gov/books/NBK1404/
85- Dog bite infection caused by?
a.
viral
Answer: A
Viral (rabies)
86- Osmotic fragility test.
a.
Hereditary spherocytosis
Answer: A

21

87- What type of anemia is associated with rheumatoid arthritis?


a.
normocytic normochromic
Answer: A
88- A male patient who is a smoker, developed symptoms. Ca: High. CXR showed solitary nodule. What is the most
likely diagnosis?
a.
Squamous Cell Carcinoma SCC
b.
Adenocarcinoma
Answer: A
High Ca > Paraneoplastic of SCC.
89- How is giardia diagnosed?
a.
3 different stool sample
Answer: A
Because Giardia cysts can be excreted intermittently, multiple stool collections (i.e., three stool specimens collected on
separate days) increase test sensitivity. Reference: Centers for Disease Control and Prevention
90- 70 years old man complaining of back pain. Calcium: High, IgA,IgM and IgG: Low (i.e. three types of
immunoglobulins are low). What is the diagnosis? (They didnt give any other details)
Answer: ?
Multiple myeloma
Reference: http://www.cancernetwork.com/hematologic-malignancies/non-secretory-myeloma-clinician%E2%80%99sguide#sthash.nmvYh621.dpuf
Monoclonal Gammopathy of Unknown Significance (MGUS) is the least aggressive subclass of plasma cell dyscrasia.
Patients have only a small abnormal protein spike (<3.0 gm/dl and <2.0 gm/dl for IgG and IgA and IgM, respectively),
minimal or no bone marrow involvement (<10% plasma cells), no bony involvement, normal blood counts and usually
normal levels of unaffected antibodies. The urine is usually free of monoclonal protein, however, it is not unusual to
find small amounts of monoclonal light chains in the urine of MGUS patients. Patients with MGUS have a 20-25%
chance of developing multiple myeloma or a related lymphoproliferative disorder. The remainder lead a normal life.
Reference: https://www.clevelandclinic.org/myeloma/mm-pt.htm
91- A patient working in a new place up the hill. Recently, he had raised hemoglobin. What is the reason?
a.
Due to increased production of erythropoietin.
Answer: A
92- Hb F is present, Hb A2 is present also, what is the diagnosis?
a.
Thalassemia minor.
b.
Thalassemia major.
Answer: B (if Hb A was absent)

Beta-thalassemia trait: Hb A2 (Hb A2) values of 3.5% to 9% and Hb F is usually between 5% to 15%
Beta-thalassemia major: Hb F may be 30% to 90% or even more of the total hemoglobin
Reference: http://www.mayomedicallaboatories.com/test-catalog/Clinical+and+Interpretive/83341

22

93- A woman with jaundice and high liver enzymes. Her husband has +ve Hep B surface antigen but she does not
have +ve markers for Hep A, B, C. What will you do now?
a.
Check for anti Hep B core antibody (IgM)
Answer: A
Because it will become +ve early.
94- 52 years old woman recently diagnosed to have DM came with high ketones and hyperglycemia. She was treated
for it but she developed DKA again. The doctor is confused whether she have type 1 or 2 DM. What test should be
ordered?
a.
Insulin
b.
HbA1c
c.
C-peptide
Answer: C
C-peptide is commonly used in preference to insulin measurement when assessing -cell function in clinical practice. In
patients on insulin, C-peptide measurement must be used as exogenous insulin will be detected by insulin assays.
Reference: http://www.ncbi.nlm.nih.gov/pmc/articles/PMC3748788/
95- A patient with HTN (3rd reading). Labs: K=3 Na=147 (normal up to 146). What is your diagnosis?
a.
Primary Hyperaldosteronism.
b.
Essential
Answer: A
96- A patient presented with hip and shoulder pain. ESR: high (Polymyalgia Rheumatica case). In addition to these
symptoms, what else can be there in the history?
a.
Proximal muscle weakness.
b.
Proximal muscle Tenderness.
Answer: B
Muscle weakness is not a feature of PMR.
Reference: http://emedicine.medscape.com/article/330815-clinical
97- A patient known case of HTN controlled with CCBs. Developed BPH. How will you treat?
a.
Prazosin
Answer: A
98- Patient with oral ulcers. Culture showed herpesvirus.
a.
HSV2
b.
VSV
c.
HSV1
Answer: C
99- A patient with glomerulonephritis developed hemoptysis. What is the most likely diagnosis?
a.
Goodpasture syndrome.
Answer: A
Frank hemoptysis suggests Goodpasture syndrome (glomerulonephritis and pulmonary hemorrhage associated with
anti GBM antibody) but this also can be a prominent feature of systemic vasculitis. Reference: Diseases of the Kidney
and Urinary Tract textbook edited by Robert W. Schrier.
NB. Wegners was not included in the choices.

23

100- A patient presented with hemoptysis and signs of nephropathy. Biopsy of the lung showed presence of antiGBM antibodies. What is the most likely diagnosis?
a.
Goodpasture syndrome.
Answer: A
Reference: http://emedicine.medscape.com/article/981258-overview
101- A patient with symmetrical joint involvement, complaining of morning stiffness that is relieved with movement.
MCP and PIP are involved as well. What is the diagnosis?
a.
RA
Answer: A
102- Young adult came complaining of painless penile ulcer, what is the appropriate investigation to do?
a.
Blood culture.
b.
Excisional biopsy.
c.
Swab culture and urinalysis.
d.
Darkfield microscope.
e.
CBC & ESR.
Answer: D
Primary syphilis usually begins with a single, painless, well-demarcated ulcer (chancre) with a clean base and indurated
border. Treponema pallidum is identified on darkfield microscopy or direct fluorescent antibody testing of a chancre or
lymph node aspirate. Reference: http://www.aafp.org/afp/2012/0201/p254.html
103- What hormone increases body cells sensitivity and response to insulin?
a.
Leptin
b.
Lipase
Answer: A

Reference: http://www.ncbi.nlm.nih.gov/pmc/articles/PMC3602983/
104- 50 years old woman with no issues except for HgA1c 7.3, LDL and triglyceride are high. What is the next test you
want to order?
a.
TST
b.
LFT
Answer: ?
In diabetic patients with clinical CVD or over age 40 years, statin therapy should be added to lifestyle intervention
regardless of baseline lipid levels. Reference: UpToDate
All patients started on statins should have their AST and ALT tested as a matter of routine monitoring, even if no
symptoms are present. Reference: Master The Board USMLE Step 2 CK

24

105- Old guy with lymph node enlargement and B-symptoms. What is the treatment?
a.
CHOP-R
b.
ABVD
c.
Rituximab
d.
CHOP
Answer: ? (It is not clear from the question whether it is HL or NHL)
B symptoms: fever, night sweats, weight loss
NHL:
Local disease (stage Ia and Ila): local radiation and small dose/course of chemotherapy
Advanced disease (stage III and IV, any "B" symptoms): combination chemotherapy with CHOP and rituximab, an
antibody against CD20 (CHOP-R)
HL:
Stage Ia and Ila: local radiation with a small course of chemotherapy
Stage III and IV or anyone with "B" symptoms: ABVD
*A = adriamycin (doxorubicin)B = bleomycin V = vinblastine D = dacarbazine
Reference: Master The Board USMLE Step 2 CK
106- Readstring cells
a.
Hodgkin lymphoma
Answer: A
The histological hallmark of HL is the presence of ReedSternberg cells, large malignant lymphoid cells of B cell origin.
Reference: Davidson's Principles and Practice of Medicine
107- College student have meningitis. What to do as a prophylaxis next?
a.
Isolate all contacts for 4 weeks
b.
Immunize all contacts
c.
Give antibiotic
Answer: C
Close contacts of patient with meningococcal meningitis should receive rifampin. Second-line prophylaxis regimens
include ciprofloxacin, ceftriaxone or azithromycin. Reference: First Aid USMLE step 2 CK P189
108- Postherpetic neuralgia treatment
a.
Antiviral
Answer: TCA (Amitriptyline or Nortiptyline), Antiviral aims to shorten the clinical course, prevent complications, prevent the
development of latency and/or subsequent recurrences, decrease transmission, and eliminate established latency.
(Medscape)

Reference: Master the board

25

109- A patient was diagnosed with enteric fever. What is the presentation that he will have?
a.
Confusion (or other CNS problems)
b.
Maculopapular rash
c.
Nausea, vomiting and loose stools
d.
Fever with
e.
Abdominal pain ( newly added )
Answer: ?
Signs and symptoms of Salmonella typhi (aka Enteric Fever, Typhoid) include "Rose spot" rash (on anterior thorax,
upper abdomen), sustained fever 39 to 40 C (103 to 104 F), abdominal pain precedes diarrhea/ constipation,
headache, loss of appetite and cough. Toronto Notes. If not treated the patient may become delirious. Reference:
Mayoclinic
110- 40 years old man presents with persistent lesion on the forearm that started 3 months ago. He came from a
deserted area that has sandfly infestation. Giemsa stain showed Donovan bodies inside and outside monocytes.
What will you use to treat him?
a.
Miltefosine
Answer: A?
This patient has leishmaniasis.
Uncomplicated cutaneous leishmaniasis: Topical application of
paromomycin or Intralesional antimony (Sodium stibogluconate,
Meglumine antimoniate).
Complicated cutaneous leishmaniasis: Oral systemic therapy; include
azoles and miltefosine
Agents with activity against visceral leishmaniasis (VL) include
amphotericin B, pentavalent antimonial drugs, Paromomycin (a
parenteral aminoglycoside), Miltefosine (the first oral drug for treatment of VL)
Reference: http://www.uptodate.com/contents/treatment-of-cutaneous-leishmaniasis
111- Which antilipid medications decrease LDL and Triglycerides, and increase HDL?
a.
Statin
b.
Fibrate
Answer: A
Statins block the production of cholesterol in the liver itself. They lower LDL and triglycerides, and have a mild effect in
raising HDL.

26

112- A man came from Africa with some symptoms. Vital signs were provided. What is the diagnosis?
a.
Yellow fever
b.
Ebola
c.
Lassa fever
Answer: ?

Reference:
http://lectures.shanyar.com/3rd_Stage/Medicine/Dr._Muhammad_Shaikhani/5._Viral_Hemorrhagic_Fevers.pdf
113- What is the first structure you will hit after lumbar puncture?
a.
Interspinous
b.
Ligamintum flavum
c.
Anterior spinal ligament
d.
Posterior spinal ligament
Answer: A
1- Skin
2- Fascia and SC fat
3- Supraspinous ligament
4- Interspinous ligament
5- Ligamentum flavum
6- Epidural space and fat (epidural anesthesia needle stops here)
7- Dura
Reference: USMLE step 1 + Wikipedia
114- What is the best anti-diabetic regime that resembles the normal physiology?
a.
Lispro & glargine
b.
NPH & glargine
c.
Lispro & NPH
Answer: A or C?
NPH and regular insulin
NPH with aspart or lispro
Glargine with aspart or lispro
Reference: https://books.google.com.sa/books?id=8CbYBAAAQBAJ&pg=PA88&dq=#v=onepage&q&f=false
115- A patient with signs of TB. What vaccination you would give to his family?
a.
MMR
b.
DTaP
c.
BCG
d.
Polio
Answer: C

27

116- Reference: kumar and clarks A patient with signs and symptoms of Cushing syndrome. What is the best next
investigation?
a.
Brain MRI
b.
Adrenal scan
c.
Adrenal MRI
Answer: A
In adults, 80% of CS is due to ACTH-dependent causes and 20% due to adrenal causes. Since the majority of patients
with ACTH-secreting tumors have a pituitary lesion (often very small), a MRI of the pituitary gland with gadolinium
enhancement is always the initial approach.
References:
www.ncbi.nlm.nih.gov/pubmed/18209870
https://csrf.net/understanding-cushings/diagnostic-testing/
117- A patient with lung cancer. Lab results: low PTH and High Calcium. What is the reason?
a.
PTH related peptide for lung ca
Answer: A
Reference: kumar and clarks p:556
118- A patient with signs and symptoms of Atopy. Which cell produce these mediators?
a.
Mast cells
b.
Nk cells
c.
Macrophage
d.
B cells
Answer: A
119- What is the oncogene for pancreatic ca?
a.
Ras
b.
Myc
Answer: A
There are abnormalities in the structure and/or function of several oncogenes and growth factors in human pancreatic
cancer, notably the EGF receptor and its ligand TGF alpha, c-erb B-2 proto-oncogene, Ki-ras oncogene and the tumour
suppressor gene p53.
Reference: http://www.ncbi.nlm.nih.gov/pubmed/1964102
120- What is the treatment for Chlamydia?
a.
Azithromycin
Answer: A
Reference: Toronto Notes
121- Patient with bronchiectasis. What else beside medical treatment can benefit this patient?
a.
Chest physiotherapy
Answer: A
Reference: Toronto Notes
122- Patient known case of Diabetes Type 2 suffer from recurrent hypoglycemia. Which drug responsible?
a.
Sulphonylureas
Answer: A
Reference: Davidson's.

28

123- What is the difference between type I&II DM?


a.
Endogenous insulin secretion
b.
Weight
Answer: A

124- A patient on Digoxin and Lisinopril or CCB??. Most probable electrolyte imbalance that will affect his ECG?
a.
K
b.
Na
c.
Mg
Answer: A
Digoxin side effect.
125- Celiac disease affects which of the following locations?
a.
Distal SB
Answer: ?
Affect the upper small bowel (Duodenum and proximal jejunum) more than lower small bowel.
Reference: WebMed website and digestive disease center

29

126- Characteristic finding in Behet disease?


Answer: ?
Leukocytoclastic vasculitis, multi-system disorder presenting with ocular involvement (uveitis), recurrent oral and
genital ulceration, venous thrombosis, skin and joint involvement, more common in Mediterranean and Asia, average
age 30s, M>F Reference: Toronto Notes
127- Raynaud phenomena. Which antibody will be positive?
a.
SCL-70
Answer: ?
If its scleroderma case the answer will be A
ANA is ordered for Raynaud phenomenon, although it's not specific. The anti-centromere antibodies are ordered for
CREST syndrome. (R for Raynaud)
128- Case of polyuria, polydipsia and weight loss. Na: . What is the diagnosis?
a.
DI
Answer: A (Depends on the case)
Diabetes Insipidus (DI) clinical features: Passage of large volumes of dilute urine, polydipsia, dehydration;
hypernatremia can develop with lack of access to water or impaired thirst mechanism. Reference: Toronto Notes.
129- Best drug to decrease bronchial secretion in COPD?
a.
Ipratropium
Answer: A
Reference: Master the board.
130- Female patient complaining of constipation, weight gain and fatigue. What is the most likely diagnosis?
a.
Hypothyroidism
Answer: A
131- A patient presented with STEMI. What is the first priority to do?
a.
ECG
Answer: A
132- A patient with chronic retrosternal pain, cough and metallic taste in mouth. What is the most likely diagnosis?
a.
GERD
Answer: A
heartburn (pyrosis) and acid regurgitation (together are 80% sensitive and specific for reflux) sour regurgitation,
water brash, sensation of a lump in the throat (bolus sensation) and frequent belching. Usually a clinical diagnosis
based on symptom history and relief following a trial of pharmacotherapy (proton pump inhibitor (PPI): symptom relief
80% sensitive for reflux) . NB. 24-h pH monitoring is the most accurate test, but rarely required.
PPIs are the most effective therapy. Reference: Toronto Notes.
133- Which of the following is a primary cause of osteoporosis?
a.
Age
Answer: A
Primary type 1: most common in post-menopausal women, due to decline in estrogen, worsens with age
Primary type 2: occurs after age 75, seen in females and males at 2:1 ratio, possibly due to zinc deficiency
Reference: Toronto Notes.

30

134- 70 years old smoker woman with low vit-D and osteoporosis. Which of the following has the highest risk for
osteoporosis?
a.
Smoking
b.
Age
c.
Vit D
Answer: B?
135- A case of low platelets, low RBCs and low WBCs. What is the diagnosis?
a.
Iron def anemia
b.
Aplastic anemia
Answer: B
Aplastic anemia (AA) is characterized by diminished or absent hematopoietic precursors in the bone
marrow, most often due to injury to the pluripotent stem cell (Decrease RBCs, WBCs, Platelets).
Reference: Uptodate.
136- A patient with decrease in factor Va. The etiology is due to:
a.
Inherited
b.
Immune
c.
Infection
Answer: A
137- Henoch schonlein purpura
a.
IgA Vasculitis
Answer: A
138- Which of the following is considered to be beneficial in DM and hypertensive patients?
a.
ACEI
Answer: A
Hypertensive patients with DM and tight BP control at <150/85 mmHg by use of ACEI or B-blocker reduced risk of
diabetic complications and death related to DM and reduced risk of end-organ damage. Reference: Toronto Notes.
139- SCA patient presents with bloody urine. What is the cause?
a.
Recurrent UTI
Answer: ?
Typically, the hematuria is mild and self-limited in SCD. As a rule, the hematuria originates from the left kidney; this
has been attributed to the greater length of the left renal vein and compression of the left renal vein between the
aorta and superior mesenteric artery (ie, the nutcracker phenomenon).
Hematuria can also be secondary to papillary necrosis:
The renal medulla contains the vasa rectae, that is, the renal tubules and blood vessels located therein. In SCD,
increased blood viscosity contributes to ischemia and eventual infarction that involves the renal microcirculation.
Medullary ischemia and infarction cause papillary necrosis.
Renal medullary carcinoma is an uncommon cause of gross hematuria.
It can result from other problems such as urinary infections, renal stones or glomerulonephritis
References:
http://emedicine.medscape.com/article/1957952-overview#a7
https://scinfo.org/problem-oriented-clinical-guidelines/specific-problems-hematuria-and-nephropathy

31

140- Which bleeding disorder presents with prolonged BT & aPTT?


Answer: ?
vWD

141- A patient presented with macular papular rash and fever. (case of rubella)
Answer: ?
Clinical Features: Rash (pink, maculopapular rash 1-5 d after start of symptoms. The rash starts on face and spreads to
neck and trunk), Prodrome of low grade fever and occipital/retroauricular nodes.
STAR complex (sore throat, arthritis, rash) and Positive serology for rubella IgM.
Managament:
For infected patients: Symptomatic Rx.
For Prevention: MMR vaccine.
For rubella-exposed pregnant women: Serologic testing,
Reference: Toronto Notes
142- 40 years old man, Hb: low, MCV: high, No megaloblast. What is the most likely diagnosis?
a.
Alcoholic anemia
Answer: A
Non-megaloblastic DDx: Liver disease, Alcoholism, Reticulocytosis, Hypothyroidism and Myelodysplasia. Reference:
Toronto Notes.
143- What is the treatment of choice for Traveler's diarrhea?
a.
Ciprofloxacin.
b.
Amoxicillin.
c.
Metronidazole.
Answer: A
The most common organism is enterotoxigenic E. coli and it is treated with fluoroquinolones (e.g. Ciprofloxacin).
Reference: Uptodate.
Initial treatment: rehydration, the Abx only will decrease the duration of symptoms;
1st line: antibiotics include fluoroquinolones, such as ciprofloxacin or levofloxacin or ofloxacin
2nd line: azithromycin. 3rd line: Rifaximin.
144- A patient with leukemia, his peripheral blood smear showed blasts more than 80 %, and Auer Rod
bodies, what is the type of leukemia?
a.
AML
b.
ALL
c.
Hairy Cell leukemia
d.
CML
Answer: A
Reference: FA for the USMLE step 2.
145- Boutonniere deformity:
a.
Flexion of PIP joint & hyperextension of DIP
Answer: A

32

146- What the investigation should be done before lumbar puncture?


a.
platelets
Answer: A
We recommend NOT performing an LP in patients with coagulation defects who are actively bleeding, have severe
thrombocytopenia (eg, platelet counts <50,000 to 80,000/L), or an INR >1.4, without correcting the underlying
abnormalities. Reference: UpToDate
147- What is the drug of choice for absence seizure?
a.
Phenytoin
b.
Carbamazepine
c.
Ethosuximide
Answer: C
Reference: Master the Boards.
148- Which of the following can be found on smear in sickle cell disease?
a.
Bite cells
b.
Howell-Jolly bodies
c.
Acanthocyte
d.
Spherocyte
Answer: B
Reference: Master the Boards.
149- A patient with an infection that is resistant to Beta-lactam antibiotics, what antibiotic should be given?
a.
Azithromycin
b.
Vancomycin
c.
Gentamicin
Answer: B
Reference: Master the boards.
150- Pyoderma gangrenosum is associated with which one of these diseases?
a.
Ulcerative colitis.
Answer: A
PG is more common in UC (512%) than CD (12%).
Reference: http://www.ncbi.nlm.nih.gov/pmc/articles/PMC3273725/table/T1/
151- Old patient presented with agitation, urinary incontinence, confusion and impaired short term memory. Long
term memory is intact. CT shows temporal and hippocampal atrophy. Which of the following genes might be
affected?
a.
13
b.
15
c.
18
d.
X
Answer: 19?
Alzheimers Disease:
Early-onset, (age 30 to 60): 3 major genes for autosomal dominant AD have been identified: amyloid precursor
protein (chromosome 21), presenilin 1 (chromosome 14) and presenilin 2 (chromosome 1).
Late-onset (mid-60s and later): The E4 polymorphism of apolipoprotein E is a susceptibility genotype (E2 is
protective). The APOE gene is located on the long (q) arm of chromosome 19. Reference: Toronto Notes

33

152- Treatment of HCV?


Answer: ?
Interferone + ribavirin (Kumar)
Patients with acute hepatitis C virus (HCV) infection appear to have an excellent chance of responding to 6 months of
standard therapy with interferon (IFN). Reference: http://emedicine.medscape.com/article/177792-treatment
153- A patient with right knee osteoarthritis, presented with swelling of the right knee. On examination the right
knee is swelled with no change in temperature. What will you do?
a.
Aspiration of the knee fluid
b.
Bilateral Knee X-Ray and Ibuprofen.
Answer: B
Because there is no change in temperature.
Its normal for osteophyte to cause swelling of the knee in OA
154-A patient who was treated from TB, came to you complaining of eye pain. What is the cause?
a.
Isoniazid
b.
Rifampicin
c.
Ethambutol
d.
Pyrazinamide
Answer: C
Reference: http://www.drugs.com/sfx/ethambutol-side-effects.html
155- 40 years old patient complaining of weight loss, nausea, lethargy and jaundice. When he was asked about
similar attack in the past, he mentioned 4 episodes during the past two years. What is the most likely diagnosis?
a.
Acute pancreatitis.
b.
Cancer head of pancreas.
c.
Peptic ulcer disease.
d.
Chronic pancreatitis
Answer: B
The most characteristic sign of pancreatic carcinoma of the head of the pancreas is painless obstructive jaundice.
Reference: http://emedicine.medscape.com/article/280605-clinical
156- 27 years old smoker who was studying in a foreign country for two years and lived in a student housing. He
returned home two months ago. The patient complains of 4 day mid epigastric pain, what is the most likely
diagnosis ?
a.
Viral Hepatitis.
b.
H. pylori infection.
c.
Acute Pancreatitis.
d.
Myocardial infarction.
Answer: B
Epigastric pain is the most common symptom of both gastric and duodenal ulcers. It is characterized by a gnawing or
burning sensation and occurs after mealsclassically, shortly after meals with gastric ulcer and 2-3 hours afterward
with duodenal ulcer. Reference: http://emedicine.medscape.com/article/181753-overview

34

157- How you can confirm Brain stem death?


a.
Absence of doll eye
b.
Active coughing & gag reflex
c.
One inactive pupil
Answer: A
The process for brain death certification includes 3- Performance of a complete neurological examination.
Components of a complete neurological examination are: iii. Absent corneal, oculocephalic (doll's eye ), cough and gag
reflexes. The corneal reflex may be altered as a result of facial weakness.
Reference: http://www.ncbi.nlm.nih.gov/pmc/articles/PMC2772257/
158- Man came to the hospital after he got bitten by a wild cat. What is the most likely organism ?
a.
Pasteurella caballi.
b.
Pasteurella Multocida.
c.
Pasteurella canis.
d.
Pasteurella avium.
Answer: B
Reference:
http://www.ncbi.nlm.nih.gov/pmc/articles/PMC3122494/
http://emedicine.medscape.com/article/224920-treatment
159- (long scenario) adult with right toe tenderness for 1 week and he have ear pain also (investigation included).
What is the best treatment for him ?
a.
Cortisone.
b.
NSAIDs.
c.
Allopurinol.
Answer: B (For acute attack. Allopurinol can be given after attack resolution)
Treatment of acute gout attacks does not differ substantially in patients with or without clinically apparent tophi,
although the presence of tophi is an indication for the initiation of long-term urate-lowering therapy after attack
resolution to prevent or reverse chronic gouty arthropathy.
Reference:
http://emedicine.medscape.com/article/329958-treatment#showall
http://www.uptodate.com/contents/treatment-of-acute-gout
160- Case of DKA with metabolic acidosis. What is the early mechanism to restore blood pH?
a.
Excretion of CO2 through lungs.
b.
Excretion of lactic acid through kidneys.
Answer: A
Reference: https://en.wikipedia.org/wiki/Acid%E2%80%93base_homeostasis
161- (long scenario) case of endocarditis with negative bacterial culture, there is diastolic murmur radiate to the
left axilla. What is the most likely diagnosis?
a.
SLE
b.
Rheumatoid arthritis.
c.
Acute myocarditis.
Answer: A
Libman-Sacks endocarditis (otherwise known as verrucous, marantic, or nonbacterial thrombotic endocarditis) is the
most characteristic cardiac manifestation of the autoimmune disease systemic lupus erythematosus.
One cohort study reported that pure mitral regurgitation was the most common valvular abnormality, followed by
aortic regurgitation, combined mitral stenosis and regurgitation, and combined aortic stenosis and regurgitation.
Reference: http://emedicine.medscape.com/article/155230-overview#showall
Blood culture-negative infective endocarditis (IE):
http://www.uptodate.com/contents/epidemiology-microbiology-and-diagnosis-of-culture-negative-endocarditis

35

162- (long scenario) man with chest pain and abnormal EKG. Which one of the following will be elevated?
a.
ESR.
b.
M2 Protein.
c.
CRP.
d.
Creatinine.
Answer: C?
References:
http://circ.ahajournals.org/content/113/6/e72.full
http://circ.ahajournals.org/content/123/10/1092.full
http://circ.ahajournals.org/content/107/3/499.figures-only
163- (long scenario) 55 year old known diabetic patient came for checkup. What is the earliest effect of Diabetes
Mellitus on the kidney ?
a.
Hydronephrosis with protein excretion.
b.
Hydronephrosis with protein excretion.
c.
Sclerosis with protein excretion.
d.
Sclerosis with protein excretion.
Answer: ? Hyperfiltration with normoalbuminuria or protein excretion.
(I think they meant by hydronephrosis = hyperfiltration).

References:
Toronto Notes + http://www.pathophys.org/ckd/ + http://emedicine.medscape.com/article/238946-overview#showall

36

164- A patient came to you complaining of morning stiffness of the PIP and DIP that decreases and goes away with
activity. On x-ray you observed a bone growth. What is the name of that growth ?
a.
Heberden node
Answer: ? (depends on the X-ray given)
It is a case of arthritis.

Osteoarthritis Hand

Cartilage loss with narrowing of interphalangeal joints


B: Bouchard nodes (osteophytes proximal interphalangeal joints)
H: Heberden nodes (osteophytes distal interphalangeal joints)

165- Peptic ulcer patient with Anemia, what you will do regarding his anemia?
a.
Oral iron supplement.
b.
I.M iron.
c.
Blood Transfusion
Answer: B
Reference : uptodate
166- Man eating rice only, he has gingival and tongue lesions. Which of the following deficiency you will find ?
a.
Vitamin A.
b.
Vitamin C.
c.
Thiamin(B1).
d.
Niacin (B3).
Answer: C
In Asia when white rice became more popular than brown rice, people started to develop a nerve damage disease
called beriberi. It can cause hypersensitivity of oral mucosa, burning sensation of the tongue and loss of taste.
References: http://www.dietobio.com/vegetarisme/en/vit_b1.html +
https://books.google.com.sa/books?id=EBqYie7BGsYC&pg=PA12&lpg=PA12&dq=#v=onepage&q&f=false
167- What is the preferable imaging to diagnose pituitary microadenoma?
a. Contrast with enhanced brain CT
b. Brain CT
c. Contrast with enhanced brain MRI
d. Unenhanced brain MRI
Answer: C
Reference: UpToDate , Usmle 2 first aid

37

168- In hemolytic anemia which enzyme will be noticed?


a.
erythropoietin
b.
bilirubin
Answer: B
169- What is the treatment of choice for Kawasaki?
Answer: ?
Aspirin and IVIg
Reference: CanadaQbank
170- A patient presented with high parathyroid hormone and high calcium what is the diagnosis ?
a.
primary parathyroid
b.
secondary parathyroid
Answer: A
High ca + high PTH = primary hyperparathyroid
Reference: medlineplus medical encyclopedia
171- A male patient known case of HTN on ACEi but with poor control, which drug you'll add ?
a.
b.
c.

Thiazide
Furosemide
Vasodilator

Answer: A

172- young female came to your office complaining of swelling in front of her parotid gland. No tenderness, no
secretion coming out, What is the most likely diagnosis ?
a.
Parotid CA
b.
Sialadenitis
c.
Mumps
d.
L.N enlargement
Answer: A
the most common presentation of parotid ca. is a painless, asymptomatic mass; >80% of patients present because of a
mass in the posterior cheek region. Some people infected with the mumps virus have either no signs or symptoms or
very mild ones. When signs and symptoms do develop, they usually appear about two to three weeks after exposure to
the virus and may include: Swollen, painful salivary glands on one or both sides of your face (parotitis).
Refference: Medscape and myoclinic

38

173- A male patient came to the ER complaining of palpitations, tachycardia ... ECG shows deep S wave in lead (?)
and tall R wave in lead (?) , Dx ?
a.
LBBB
b.
RBBB
Answer: ?
LBBB:
QRS duration >120 msec
V1 and V2: W pattern and wide deep slurred S wave
V5 and V6: wide QRS complex with M pattern or rabbit ear pattern
RBBB: the opposite of LBBB
QRS duration >120 msec
V1 and V2: wide QRS ( more than 3 small squares) with ear rabbit pattern or M shape
pattern
V5 and V6: wide and deep/slurred S
LVH:
S inV1 + R in V5 or V6 > 35mm above age 40, (>40 mm for age 31-40, > 45 mm for age 21R in aVL >11mm
R in I + S in III >25mm
RVH:
Right axis deviation
R/S ratio > 1or qR in lead V 1
RV strain pattern: ST segment depression and T wave
Reference: Toronto Notes

30)

inversion in leads V1-2

174- Women coming with elevated indirect bilirubin:


a.
Rotor syndrome
b.
Crigler Najjar
c.
Dubin Johnson
Answer: B
Differential diagnosis:
- Unconjugated (indirect) hyperbilirubinemia: Hemolytic, physiologic (newborns), Crigler-Najjar, Gilbert syndrome
- Conjugated (direct) hyperbilirubinemia:
1- Biliary tract obstruction: gallstones, cholangiocarcinoma, pancreatic or liver cancer, liver fluke.
2- Biliary tract disease: 1 sclerosing cholangitis and 1 biliary cirrhosis
3- Excretion defect: Dubin-Johnson syndrome, Rotor syndrome.
- Mixed (direct and indirect) hyperbilirubinemia: Hepatitis, cirrhosis.
175- Which disease involves antibiotic in treatment regimen?
a.
Crohn
b.
Ulcerative colitis
c.
Celiac
d.
Whipple
Answer: D
Reference: http://emedicine.medscape.com/article/183350-treatment

39

176- How to differentiate a large ovarian cyst from ascites?


a.
Dull anteriorly resonant laterally
b.
Resonant anteriorly and dull laterally
Answer:A (midabdominal dullness and lateral tympany)

Reference: https://books.google.com.sa/books?id=qW4IKSfM8x8C&pg=PA472&lpg=PA472&dq#v=onepage&q&f=false
177- A patient is coughing bloody frothy sputum. He has pulmonary edema, + hepatojugular reflux and lower limb
edema. Capillary pressure is 3 times more than oncotic pressure. What is the type of edema ?
a.
Venous
b.
Arterial
c.
Interstitial
d.
Capillary
Answer: ?
Pulmonary edema in heart failure patient caused by increase of pulmonary venous pressure lead to pulmonary venous
distention and transudation of fluid. Also, lead to pulmonary capillaries rupture.
Extra information:

Reference: Rapid Review Pathology, 4th edition


178- In Guillain-Barr syndrome, which cell will be affected?
Answer: ?
Schwann cells
Guillain-Barr syndrome; Rapidly progressive limb weakness that ascends following GI/ upper respiratory infection.

40

179- (Long scenario) Patient with fever, lymphadenopathy and hepatosplenomegaly :


Answer: ? (Case of Mononucleosis)
See: http://emedicine.medscape.com/article/222040-overview
180- BCG vaccine is contraindicated in which of the following deficiencies?
a.
IL
b.
TNF
c.
INF
Answer: C
Reference:https://books.google.com.sa/books?id=MRTa5qwTc7AC&pg=PA459&lpg=PA459&dq#v=onepage&q=BCG%2
0contraindicated&f=false P: 459
181- Urine sample from a woman and there was positive epithelial cells. What is the source of these cells?
a.
cervical erosion
b.
renal colic
c.
trachomatis
d.
Urethral contamination
Answer: D
182- A patient with high ICP (I think it was due to a brain tumor), which of the following nerves is most likely to be
affected?
a.
trochlear
b.
6 th cranial
c.
optic
d.
facial

Answer: B
increase ICP leads to CN III and VI deficient. Reference: First Aid USMLE 2.
183 - case about LP what the investigation should be don before it ______
A: platelets
Answer:A
Source:
http://www.hopkinsmedicine.org/neurology_neurosurgery/centers_clinics/headache/procedures/lumbar_pu
ncture.html
184 -

Common cause of bronchiolitis?

a. :Para influenza virus


b. RSV
Answer: B
Bronchiolitis is an acute inflammatory injury of the bronchioles that is usually caused by a viral infection (most
commonly respiratory syncytial virus and human metapneumovirus) Medscape
185 - Pt with type 1 DM , what's the diagnostic antibody
A: Glutamic Acid Decarboxylase
B: others antibodies not related to DM1
Answer: A
Reference: Islet-cell (IA2), anti-GAD65, and anti-insulin autoantibodies can be present in early type 1 but not type 2 DM.
Measurements of IA2 autoantibodies within 6 months of diagnosis can help differentiate between type 1 and type 2
DM. These titers decrease after 6 months. Anti-GAD65 antibodies can be present at diagnosis of type 1 DM and are
persistently positive over time
http://emedicine.medscape.com/article/117739-workup#c8

41

186 - "symmetric joint pain and swelling worse at morning. Dx? 2 times
Answer: RA
Signs and symptoms of rheumatoid arthritis may include the following:
Persistent symmetric polyarthritis (synovitis) of hands and feet (hallmark feature)
http://www.mayoclinic.org/diseases-/rheumatoid-arthritis/basics/symptoms/con-20014868
5187 - Case of Absence seizure What is best treatment for him: " 4 times
A- Phonation
B- Carbazapin
C- Ethosuximide
Answer: C
Ref: Only 2 first-line AEDs have approval from the US Food and Drug Administration (FDA) to be indicated for absence
seizures: ethosuximide (Zarontin) and valproic acid (Depakene, Depacon). Ethosuximide has efficacy for absence
seizures only and valproic acid has efficacy for absence, generalized tonic-clonic, and myoclonic seizures.
Ethosuximide (Zarontin) is effective only against absence seizures.
http://reference.medscape.com/article/1183858-overview#a9

188 - "Most common cause of pneumonia? 2 times


Answer: Streptococcus pneumonia
Streptococcus pneumonia is by far the most common cause of typical bacterial pneumonia.
http://emedicine.medscape.com/article/300157-overview#a4
189 -

"Sickle cell disease and joint pain? 2 times


a.

Due to avascular necrosis

Answer:
not clear but Pain can be mild or severe and usually develops gradually. Pain associated with avascular necrosis
of the hip may be focused in the groin, thigh or buttock. In addition to the hip, the areas likely to be affected
are the shoulder, knee, hand and foot. Some people develop avascular necrosis bilaterally for example, in
both hips or in both knees. http://www.mayoclinic.org/diseases-conditions/avascularnecrosis/basics/symptoms/con-20025517
190 -

Gout metabolism of
A)purine

Answer: Overproduction accounts for only a minority of patients presenting with hyperuricemia. The causes
for hyperuricemia in overproducers may be either exogenous (diet rich in purines) or endogenous (increased
purine nucleotide breakdown). A small percentage of overproducers have enzymatic defects that account for their
hyperuricemia. These include a complete deficiency of hypoxanthine guanine phosphoribosyltransferase (HGPRT)
as in Lesch-Nyhan syndrome, partial deficiency of HGPRT (Kelley-Seegmiller syndrome), and increased production
of 5-phospho-alpha-d-ribosyl pyrophosphate (PRPP) activity. Accelerated purine degradation can result from rapid
cell proliferation and turnover (blast crisis of leukemias) or from cell death (rhabdomyolysis, cytotoxic therapy).
Glycogenoses types III, IV, and VII can result in hyperuricemia from excessive degradation of skeletal muscle ATP.
http://emedicine.medscape.com/article/241767-overview#a5

42

191 -

Swimming in river pool and got infection

a. Streptococcus
b. H influenza
c. N gonorhea
d. Something i have never heard of
Answer: Most common organisms acquired from swimming pools Bacteria: Shigella and E.coli / Protozoa :
Cryptosporidium and Giardia./ Viruses : Adenovirus , Echovirus , Norwalk virus, and Hepatitis A.
Reference: WHO and CDC
http://www.who.int/water_sanitation_health/bathing/recreaII-ch3.pdf
http://www.cdc.gov/healthywater/swimming/rwi/
192 - Leukemia pt with 80 blast and Auer bodies in peripheral smear what type of leukemia
Answer: A. Acute Myeloid Leukemia (AML).
Peripheral blood smear shows : circulating blasts with Auer rods (azurophilic granules) are pathognomonic for AML.
Bone marrow aspirate show : blast count >20%.
Reference: Toronto Notes Hematology (H38)

193 - 45 years old with proximal muscle weakness,tongue fasciculation,with history of recurrent aspiration
pneumonia. what is the diagnosis
a.mononeuropathy
b.myasthenia gravis
c.mysthenic syndrome
Answer: B. Myasthenia Gravis
Reference: http://m.jnnp.bmj.com/content/73/6/766.full
194- What of the following present in in peripheral blood smear of Sickle cell disease patient?
a.Howell jolly bodies
b.spherocytes and.
c.acanthocyte
Answer: A. Howell-Jolly Bodies (hyposplenism : in Sickle Cell disease)
Reference: Tornoto Notes Hematology (H4)
195 - a known case of sickle cell disease presented with unilateral lower limb pain since (short period:acute) . Vital
signs: Tachycardia . Fever 38.3 . the range of movement is intact with no signs of inflammation over the limb. what is
the diagnosis:
a:Vaso-occlusive Crisis
b_Osteomylitis
Answer: A
Findings secondary to vaso-occlusion.
a. Painful crises involving bonebone infarction causes severe pain. This is the
most common clinical manifestation.
Bone pain usually involves multiple sites (e.g., tibia, humerus, femur). It may
or may not be bilateral.
The pain is self-limiting and usually lasts 2 to 7 days.
b. Handfoot syndrome (dactylitis).
Painful swelling of dorsa of hands and feet seen in infancy and early childhood
(usually 4 to 6 months).
Often the first manifestation of sickle cell disease.
Caused by avascular necrosis of the metacarpal and metatarsal bones.
precipitated by infection , fever , dehydration , pregnancy and alcohol
Reference : step up to medicine

43

196 - Sickle cell disease patient with multiple gall bladder stones. what is the Best thing to do:
A:hydroxyurea
B:Cholysystectomy
Answer: B
If the patient does not have symptoms, no treatment is usually necessary. If there is recurrent or severe pain from
gallstones, the gallbladder may need to be removed. Minimally invasive procedures (using laparoscopy) reduce possible
complications
Reference: https://umm.edu/health/medical/reports/articles/sickle-cell-disease
197 - Cause of 2ry HTN
Answer:

Reference: American Academy of Family Physicians


198 - Chlamydia ttt
Answer: Azithromycin (oral one dose ) or doxycycline (oral for 7 days ) + treat all sexuall parners
Reference : step up to medicine
199 - Drug to delay surgery: nefidipine
200- Scenario about diabetic what is the Tx: metformin
Answer: Metformin
201 - Scenario pt. came with melena and bleeding take NSAID for 3 weeks
what is the cause ?
Answer: NSAID use is associated with an increased risk of gastric or duodenal ulcer by inhibition of prostaglandins that
leads to symptomatic chronic ulceration ,also inhibit platelet function, and their use has been associated with GI
bleeding from throughout the GI tract.
http://www.uwgi.org/guidelines/ch_07/ch07txt.htm

44

202- What is the best ttt for traveler diarrhea ? 3 times


ciprofloxacin
cipro
amoxycillin
metronodazole
??
Answer: fluoroquinolones, such as ciprofloxacin or levofloxacin
http://www.uptodate.com/contents/travelers-diarrhea-clinical-manifestations-diagnosis-and-treatment
203 - bronchial cancer mets to sypathatic plexus what the sign ?
ptosis
dilated pupil
Answer: the answer is horners syndrome ptosis ,anhydrosis,miosis
Toronto Notes Neoplasm (R28)
http://www.uptodate.com/contents/superior-pulmonary-sulcus-pancoast-tumors
204 - :20 yrs old girl her parent have dyslipedimia and she denies if have dyslipidemia lpid profile showing high
triglyc,cholestrol,high LDL ,low HDL what is the best to check next ?
GH
TSH
FSH
ACTH
Answer: TSH to rule out hypothyroidism
http://www.uptodate.com/contents/lipid-abnormalities-in-thyroid-disease
205 - ebola very clear. Answer: http://www.who.int/mediacentre/factsheets/fs103/en/
206 - spoon shaped nail ?
Answer: iron deficiency
207- patient for surgery known case of DM2 on glimepiride you will shift patient to which drug during and after
surgery?
A-insulin
B-metformin
Answer: On the morning of surgery hold oral hypoglycemic agent because Sulfonylureas will increase the risk for
hypoglycemia,and metformin will increase the risk for lactic acidosis.
If the patient develop hyperglycemia give SC insulin, and correction insulin is given until the patient is eating and either
can resume oral agent.
Reference : uptodate "perioperative management of blood glucose in adult with DM"
208 - patient with murmure in midsternal not radiatin high pitched first sound with decrascindo:crascindo ?
ejection diastol
ejection systolic
physiologic
innocent
Answer: Systolic ejection ( not sure 100%) maybe some points missed in the questions because answer
overlapping. Systolic ejection murmurs may be functional or organic.The most common functional systolic ejection
murmur in adults is probably a variant of Still's murmur, the so-called innocent murmur of childhood. It is a short,
buzzing, pure, medium-pitched, nonradiating, midsystolic murmur heard best along the upper left sternal border.

209- "symmetric joint pain and swelling worse at morning. Dx?


Answer:
rheumatoid artheritis
http://www.webmd.com/rheumatoid-arthritis/guide/diagnosing-ra

45

210 - Best Type of carbohydrat for Diabetic pt & " Which type of Carbohydrates is better for diabetic patient":
Polysaccharides
Answer: Polysaccharides, a type of carbohydrate that includes starch and cellulose, may benefit people with
diabetes because they help retard absorption of glucose
http://www.sciencedaily.com/releases/2009/07/090728172604.htm
211 - parasite infection in under cooked food
Answer: Trichinellosis/Trichinosis
Infection is initiated by ingestion of viable larvae in raw or undercooked meat. Digestive action liberates the
larvae. http://emedicine.medscape.com/article/787591-overview#a5
212 - Pt have lesion in right upper lung look like calcium how to treated ?
Answer:

The first step in the evaluation of a pulmonary nodule is to look for a prior x-ray. Finding the same pulmonary
nodule on an x-ray done years ago may save you from doing any further workup. If no prior x-ray is available,
then consider whether this patient is high or low risk for lung cancer.

In low-risk patients, <35 years of age and nonsmokers with calcified nodules, you may follow
the patient with chest x-rays or chest CT every 3 months for 2 years. Stop the follow-up if after
2 years there is no growth.

High-risk patients >50 years of age with a smoking history and a nodule are likely to have bronchogenic cancer. The best diagnostic procedure is open-lung biopsy and removal of the nodule
at the same time.
Reference : Kaplan Internal Medicine 2013

213 - In hemolytic anemia which enzyme will be noticed:


a -erythropoietin .....
b-bilirubin
Answer: B
Standard blood studies for the workup of suspected hemolytic anemia include the following:
Complete blood cell count
Peripheral blood smear
Serum lactate dehydrogenase (LDH) study
Serum haptoglobin
Indirect bilirubin
Reference : http://emedicine.medscape.com/article/201066-workup
214- risk factor for stroke (most important)
Hypertension, is the single most important treatable risk factor for stroke
Reference : uptodate.com
215 - Clear case of absence seizure, what happens if we give fentanyl
Answer: fentanyl-induced epileptiform activity on the electrocorticogram
Reference :http://www.epilepsy.com/information/professionals/diagnosis-treatment/procedures-epilepsypatients/general-anesthetics-4

46

216- Asthma on monteleukast and BRONchodialtor has dry couph every day came to ICU, what to give for long term?
Answer: high-dose inhaled corticosteroid plus a leukotriene receptor antagonist plus an oral corticosteroid. Consider
omalizumab for patients who have allergies. Medscape
217 - Head trauma with low urine osmolality?
Answer: Central DI can be an acute or chronic complication of head injury or subarachnoid hemorrhage.
Medscape
218 - LOW BACK PAIN IN THE MORNING THAT RESOLVES IN 30 mins TWO CASES
what to do? Pt?
Answer : Physiotherapy to strengthen muscles
219 - Triple therapy for gastric ulcer in 10 yo boy? Ppi + metro + ?
Answer: treatment of PU is the same for adults and children
PPI-based triple therapy regimens for H pylori consist of a PPI, amoxicillin, and clarithromycin for 7-14 days
Medscape
220 - Gradual Hearing LOSS in a 17 yo male with normal tympanic membrne.
Answer if unilateral: acoustic neuroma, If with noise exposure and bilateral: noise induced
Ref: http://www.aafp.org/afp/2003/0915/p1125.html
221- Elderly heavy smoker with pain on walking and o/e loss of hair and pallor of feet whats dx?
Answer: PVD
Ref: http://www.healthline.com/health/peripheral-vascular-disease#Overview1
222 - T1DM whats the antigen he has? DR 4, DR 7?
Answer: HLA-DR 3 & 4
Ref: 1ST AID USMLE STEP 2
223 - Elderly came with leukocytosis... What supports the diagnosis of CML
Answer: The diagnosis of CML is suspected based on the results of a simple blood test The test may show an
abnormally high white blood cell count in blood samples examined under a microscope, less mature white
blood cells, normally found only in bone marrow, are seen
Ref: Medical council of canada exam
224- patient has splenomegaly and teardrop RBC on blood film:
ITP
Myelofibrosis
Answer: Myelofibrosis
Ref: http://www.pathologystudent.com/?p=607
225 - patient with honeymoon cystis . Which of following could be the organism?
Staph.saprophticous
E,coli
Answer: Honeymoon cystitis is a bladder infection that results from sexual activity. It occurs when the bacteria travels
in the upward direction from the urethra to the bladder. The bacteria mainly responsible for such infections are
Escheria Coli.
http://www.m.webmd.com/a-to-z-guides/tc/understanding-bladder-infections-basic-information?page=2
http://www.urineinfection.net/honeymoon-cystitis-symptoms-and-relief/

47

226 - old patient with poor control of DM1, compline of SOB and hemoptysis . x:ray show lung consolidation .
culture show non septa fungal hypha . what is the diagnosis?
Aspiragillus
Candida
Zymgomycetes
Answer:Zymgomycetes
Opportunistic infection especially associated with diabetes; other predisposing factors are neutropenia, corticosteroid
therapy, iron overload and mucocutaneous trauma.
Large, non-septa hyphae with 90 degree angle branching and non-parallel walls, angioinvasive causing tissue necrosis
and hemorrhage
http://www.pathologyoutlines.com/topic/lungnontumormucor.html
227 - cholera vibro antibiotic
Doxycycline
Answer: Doxycycline inhibits protein synthesis and, thus, bacterial growth by binding to 30S and possibly 50S ribosomal
subunits of susceptible bacteria.
In areas of known tetracycline resistance, therapeutic options include ciprofloxacin and erythromycin. Strains resistant
to ciprofloxacin have been reported from Calcutta, India.
Pharmacotherapy plays a secondary role in the management of cholera; fluid replacement is primary.
http://emedicine.medscape.com/article/962643-medication#2
228 - pt with dyslipedemia ,,which hormone will be screen :
TSH (correct answer
testosterone
Answer: Patient with hypercholestrolemia should be screened for hypothyroidism before being given any specific lipidlowering drug therapy.
http://www.uptodate.com/contents/lipid-abnormalities-in-thyroiddisease?source=outline_link&view=text&anchor=H6582291#H6582291
229 - histopathology of minimal change nephrotic syndrome
The glomeruli appear normal on light microscopy in patients with and there are no complement or
immunoglobulin deposits on immunofluorescence microscopy. Glomerular size is usually normal by
standard methods of light microscopy, although enlarged glomeruli may be observed
The characteristic histologic lesion in MCD is diffuse effacement (also called "fusion") of the epithelial foot
processes on electron microscopy. More specifically, there is retraction, widening, and shortening of the
foot processes. The spaces between flattened podocyte foot processes are reduced in number and support
the burden of plasma filtration, which may play a role in the excess albumin load into the urinary space. The
degree of effacement does not correlate with the degree of proteinuria. Foot processes regain a normal
appearance with remission of proteinuria
http://www.uptodate.com/contents/etiology-clinical-features-and-diagnosis-of-minimal-change-disease-inadults?source=machineLearning&search=minimal+change+nephrotic+syndrome&selectedTitle=2~104&secti
onRank=5&anchor=H2#H58052618
230 - organ responsible for multiorgan failure:
heart
lung
kidney
liver
Answer: Unclear question

48

231 - Pt with vit b 12 deficiency what gastric cell type will be affected
Cheif cell
Parietal cell
Answer: Parietal cell deficiency as its responsible for intrinsic factor synthesis which is required for vit. B12 to be
absorbed in terminal ileum
Autoimmune metaplastic atrophic gastritis A major component of PA (pernicious anemia) is chronic atrophic
gastritis, which is associated with autoantibodies directed against gastric parietal cells in approximately 90 percent of
patients with PA.
http://www.uptodate.com/contents/etiology-and-clinical-manifestations-of-vitamin-b12-and-folatedeficiency?source=machineLearning&search=megaloblastic+anemia&selectedTitle=3~91&sectionRank=1&anchor=H21
25777#H2125777
232 -Acute management of gout
Answer: INITIAL TREATMENT CHOICES The choice of medications depends upon the comorbidities that are present,
the effectiveness of past treatments, patient preferences for use, and the experience of the clinician with joint
injection. Despite wide use in the treatment of acute gout attacks, the various antiinflammatory agents have only
infrequently been compared with placebo or with each other in randomized trials. We take the following approach to
the initial treatment of acute attacks of gout (algorithm 1):
We treat most patients able to take an oral medication with a nonsteroidal antiinflammatory drug (NSAID). (See
'NSAID therapy' below.)
An oral low-dose colchicine regimen may be used in patients who are able to take an oral medication but who
have contraindications to NSAIDs (eg, moderate or more severe chronic kidney disease [CKD], active peptic ulcer
disease, or a history of NSAID-intolerance). (See 'Colchicine therapy' below.)
In patients with contraindications to the use of both NSAIDs and colchicine, we prefer intraarticular, oral, or
parenteral glucocorticoids, depending upon the number of involved joints, the experience of the clinician with
joint injection techniques, and the need, if present, for parenteral rather than orally administered therapy. (See
'Glucocorticoids' below.)
http://www.uptodate.com/contents/treatment-of-acute-gout

233- Chronic Gout >allopuriol


Answer: Allopurinol inhibition of uric acid production is in large part due to inhibition of xanthine oxidase (xanthine
dehydrogenase) by both the native drug and the active metabolite oxypurinol. Allopurinol and oxypurinol are pyrazolopyrimidine analogs of the purine bases hypoxanthine and xanthine, respectively
Ref :uptodate
234- Pyseodogout >pyrophysphate crystel
Answer: PSEUDOGOUT DIAGNOSIS A healthcare provider can confirm or rule out a diagnosis of pseudogout by
performing an examination and tests. In many patients, a sample of joint fluid is obtained in order to determine
whether calcium pyrophosphate dihydrate (CPP) crystals are present and to exclude arthritis due to other causes, such
as gout or joint infection.
Ref: uptodate
235- Female pt diagnosed with IBD on ceftriaxon with no benefit wt is the organism
herpes
can't recall others
Answer: Pseudomonas aeruginosa
ceftriaxone does not have useful activity against Pseudomonas aeruginosa.It is generally not active against
Enterobacter species, and its use should be avoided in the treatment of Enterobacter infections even if the isolate
appears susceptible because of the emergence of resistance.Some organisms, such as Citrobacter, Providencia, and
Serratia, have the ability to become resistant through the development of cephalosporinases (these enzymes hydrolyze
cephalosporins and render them inactive)

49

236 - pt k/c of DM with uncontrolled blood sugar With figure shows high at the 6am wt to do?
long acting night insulin **
short acting night insulin
long acting morning in
short acting morning in
Answer:A
The Dawn Effect: In the dawn hours, many people experience a release of counterregulatory hormones. These
hormones, such as human growth hormone (HGH), help with the growth, maintenance, and repair of our bodies while
we sleep. They also make muscle tissue and the liver more resistant to the effects of insulin, meaning that more insulin
is needed to maintain normal blood glucose levels. If there isnt enough insulin to counteract the resistance, the result
is morning hyperglycemia.
Ref:
http://www.todaysdietitian.com/newarchives/111412p18.shtml
237
K/c of DM pt on Glipizide want to go for elective surgery you want to control his blood sugar during the su
rgery what to add ?
Insulin **
Metformin
Sulfonylurea
..
Answer: A
Patients with type 2 diabetes who take oral hypoglycemic drugs or noninsulin injectables are advised to hold their
oral hypoglycemic and noninsulin injectable drugs on the morning of surgery. For patients who develop
hyperglycemia, supplemental short or rapid-acting insulin may be administered subcutaneously (typically every six
hours), based on frequently (every one to two hours) measured glucose levels which are often obtained on
capillary "fingerstick" samples .
Reference: http://www.uptodate.com/contents/perioperative-management-of-blood-glucose-in-adults-withdiabetes-mellitus?source=preview&search=%2Fcontents%2Fsearch&anchor=H20#H20
238- Q about young boy present with Abd pain and vomiting
DKA **
Answer: very short question cant decide but the answer depends on the clinical scenario, Common causes of
abdominal pain include constipation, gastrointestinal (GI) infections, infections outside of the GI tract, and colic.
Less common GI conditions (ie, inflammatory bowel disease, pancreatitis, cholecystitis, intraabdominal abscess,
dietary milk protein allergy, malabsorption, and Meckel's diverticulum)
and conditions outside of the GI tract (ie, diabetic ketoacidosis, painful crisis with sickle syndromes, Henoch Schnlein
purpura (IgA vasculitis), tumors, urolithiasis, ovarian torsion, testicular torsion, and some toxic ingestions) can present
with abdominal pain.
Reference: http://www.uptodate.com/contents/causes-of-acute-abdominal-pain-in-children-andadolescents?source=preview&search=%2Fcontents%2Fsearch&anchor=H30#H30
please read here for more specific classification according to age:
http://www.uptodate.com/contents/image?imageKey=EM/65488&topicKey=EM%2F6454&source=outline_link&search
=%2Fcontents%2Fsearch&utdPopup=true
239 Patient with watery diarrhea I think
If asking about the most common cause: so choose the ( rotavirus)
Reference : http://www.medicinenet.com/rotavirus/article.htm

240 -about which marker indicate chronic hepatitis B?


A.IgM
B.hep s ab
Answer:
The diagnosis of chronic HBV infection is based on persistence of HBsAg for more than six months; IgG anti-HBc is
positive, while IgM anti-HBc is negative.
Reference: medical diagnosis and management book ( moh. Danish)
http://www.uptodate.com/contents/overview-of-hepatitis-b-virus-infection-in-children-andadolescents?source=preview&language=en-US&anchor=H6&selectedTitle=1~150#H6

50

241 Patient has HTN and BPH. .WHICH DRUG CAN USE.?
A.PRAZUSIN
B...
C...
D....
Answer: A
Reference: www.drugs.com
242 - .Patient with chest pain .. Pic of ECG ..NO ST ELEVATION LAP RESULT, HIGH
TROPONIN, HIGH LDH, HIGH ASPARTAT
Answer : Increase LDH, AST ,Troponin in skeletal muscle injury and MI (NSTEMI)
So answer according the pic.
Reference: http://www.ayubmed.edu.pk/JAMC/PAST/14-4/Rehan%
243 - A.HIGH RISK UNSTBLE angina
B.low risk =========$=====
C.NSTEMI
D.STEMI
Other
Answer: C
NSTEMI indicate myocardial necrosis marked by elevation in troponin I and CK-MP without ST-segment elevation
on the ECG
Ref: First Aid USMLE STEP 2 CK

244 - Sputum culture for TB


Answer: Culture of sputum is the gold standard but can take weeks to obtain , acid fast stain can yield rapid
preliminary results but lacks sensitivity
Ref: First Aid USMLE STEP 2 CK

245 - Rheumatic fever acute management


Answer: 1- Oral penicillin 2- Amoxicillin 3- Intramuscular penicillin, single dose 4- Cephalexin 5- Aspirin
*http://www.uptodate.com/contents/treatment-and-prevention-of-acute-rheumaticfever?source=search_result&search=rheumatic+fever&selectedTitle=2~111#H2
246 - Infective endocarditis (migratory arthritis)
Answer: Duke criteria for diagnosis of infective endocarditis
Major: at least 2 separate + blood culture for a typical organism , Evidence of endocardial ( via echo or new
murmur )
Minor ( fever >38.5 , vascular and immunolical phenomena as septic emboli , janeway lesion , osler nodes ,
roth spot )
The presence of 2 maajor or 1 major + 3 minor or 5 minor , diagnosis endocardits
Ref: First Aid USMLE STEP 2 CK

51

247 - Case of rheumatoid arthritis


Answer: Rheumatoid arthritis is a chronic inflammatory condition. its symptoms develop gradually and may include
joint pain, stiffness, and swelling. The condition can affect many tissues throughout the body, but the joints are usually
most severely affected. The cause of rheumatoid arthritis is unknown.
A person with well-established rheumatoid arthritis typically has or has had at least several of the following:
Morning stiffness that lasts at least one hour and that has been present for at least six weeks
Swelling of three or more joints for at least six weeks
Swelling of the wrist, hand, or finger joints for at least six weeks
Swelling of the same joints on both sides of the body
Changes in hand x-rays that are characteristic of rheumatoid arthritis
Rheumatoid nodules of the skin
Blood test positive for rheumatoid factor and/or anti-citrullinated peptide/protein antibodies (ACPA)
Not all of these features are present in people with early RA, and these problems may be present in some people with
other rheumatic conditions.
In some cases, it may be necessary to monitor the condition over time before a diagnosis of rheumatoid arthritis can be
made with certainty.
Laboratory tests Laboratory tests help to confirm the presence of rheumatoid arthritis, to differentiate it from other
conditions, and to predict the likely course of the condition and its response to treatment.
Rheumatoid factor An antibody called rheumatoid factor is present in the blood of 70 to 80 percent of people with
rheumatoid arthritis. However, rheumatoid factor is also found in people with other types of rheumatic disease and in a
small number of healthy individuals.
Anti-citrullinated peptide/protein antibody test Blood tests for ACPA are more specific than rheumatoid factor for
diagnosing rheumatoid arthritis. Anti-ACPA antibody tests may be positive very early in the course of disease. The test is
positive in most patients with rheumatoid arthritis.
Ref: uptodate
248 - Case of juvenile arthritis & ttt of JRA
Answer: Oligoarticular juvenile idiopathic arthritis (JIA) is usually responsive to intra-articular glucocorticoids.
Methotrexate and other immunosuppressive drugs are recommended for children with disease that extends to involve
five or more joints or require repeat injections. Biologic agents are typically reserved for patients with uveitis and are
also used in some patients with extended oligoarticular JIA.
systemicJIA
Mild-to-moderate disease nonsteroidal anti-inflammatory drug (NSAID) monotherapy as the initial treatment in
children with possible sJIA who have mild-to-moderate, nondisabling symptoms on presentation and no evidence of
macrophage activation syndrome (MAS)
Moderate-to-severe disease For patients whose initial symptoms include high fevers, other systemic manifestations
including serositis and possible early MAS, and/or moderate-to-severe polyarthritis, we suggest adding one of the
biologic agents that inhibit interleukin (IL)-1 or IL-6, such as anakinra, canakinumab, or tocilizumab, rather than the
nonbiologic disease-modifying antirheumatic drug (DMARD), methotrexate. NSAIDs can be continued in conjunction
with other agents if needed for pain control.
Ref:uptodate
249 - B6 & B12 deficiency
B6 deficiency:
Overt deficiencies of vitamin B6 are probably rare. Marginal deficiencies may be more common, manifested as
nonspecific stomatitis, glossitis, cheilosis, irritability, confusion, and depression.
The classic clinical picture of cobalamin deficiency, mentally sluggish, shiny tongue (atrophic glossitis), and a shuffling
broad-based gait. hematologic changes (eg, macrocytic anemia with oval macrocytes and increased neutrophil
lobulation) and neurologic abnormalities classic picture of subacute combined degeneration of the dorsal (posterior)
and lateral spinal columns, neuropathy which is symmetrical and affects the legs more than the arms. It begins with
paresthesias and ataxia associated with loss of vibration and position sense, and can progress to severe weakness,
spasticity, clonus, paraplegia, and even fecal and urinary incontinence.
Ref: uptodate

52

250 - differentiate between DM 1 and 2


Answer: see the two pictures

251 - wide complex ECG


Answer: see the picture below

53

252 - hypersplenism and aplastic anemia how to differentiate between them


Answer: Aplastic Anemia is characterized by diminished or absent hematopoietic precursors in the bone marrow,
most often due to injury to the pluripotent stem cell. major causes :Idiopathic ,Cytotoxic drug & radiation ,Drug
reaction (Anticonvulsants: carbamazepine Antibiotics: sulfonamides, chloramphenicol, NSAIDs
phenylbutazone, indomethacin , anti thyroid methimazole, propylthiouracil ) Toxic chemical , viral infection
(EBV,HIV , other herpes) , Immune disorders.
Hypersplenism is a condition in which the spleen becomes increasingly active and then rapidly removes the blood cells.
It can result from any splenomegaly. It is most common with splenomegaly secondary to portal hypertension and
haematological disorders
Reference : http://www.researchgate.net/publication/237841754_Hypersplenism_Review_article
253 - meningitis finding in csf due to bacteria
Answer: Appearance: Clear, cloudy, or purulent
Opening pressure: Elevated (>25 cm H 2 O)
WBC count: >100 cells/L (>90% PMN)
Glucose level: Low (< 40% of serum glucose)
Protein level: Elevated (>50 mg/dL)
254 - ABOUT ANTI LIPID FROM COMMON

reference: First aid USMLE step 1

54

ADDED QUESTION: ( need to be more checked )


1. most common tumor intercranial in adult ?
A. Hemangiomblastoma
B. Epnduoma
C. Shwanoma
Answer: C
Gliomas are the most prevalent type of adult brain tumor, accounting for 78 percent of malignant brain tumors.
Meningiomas are the most common benign intracranial tumors
Pituitary adenomas are the most common intracranial tumors after gliomas, meningiomas and schwannomas.
So: the most common brain tumors are (in sequence)
1.
Glioma
2.

Meningioma

3.

Schwannoma

4.

Pituitary adenoma

Gliomas include: Astrocytomas, Ependymomas, Glioblastoma multiforme, Medulloblastomas, and


Oligodendrogliomas.
In children: Medulloblastoma
American Association of Neurological Surgeons
http://www.aans.org/patient%20information/conditions%20and%20treatments/brain%20tumors.aspx

2. characteristic of perforated duodenal ulcer?


A. Mid epigastric pain
B.Statorha
C.Melena
answer: A
Symptoms include sudden, severe abdominal pain, a rapid heartbeat, and a low body temperature. Pain
may radiate to one or both shoulders, and the abdomen may become rigid. The abdominal pain is usually
sudden, sometimes producing collapse or syncope. Localization is usually epigastric, but it quickly
becomes generalized
uptodate
3. Hodgkin lymphoma with no fibrosis and . Esenophial ,rs cell, histiocytes A persent which type?
A.Mixed
B.
D. Nodular sclerosis
answer: A
Mixed cellularity Mixed cellularity HL (MCHL) is a heterogeneous category of classic HL with a
diffuse or vaguely nodular growth pattern without band-forming sclerosis . Fine interstitial fibrosis may
be present, and classical diagnostic Reed Sternberg cells are easily identified. The background infiltrate is
variable, but typically consists of eosinophils, neutrophils, histiocytes, and plasma cells, as shown in the
left pane
uptodate
4. which type of anemia associated with hyposplenism?
A.Sickle cell anemia
B. Thalassemia
C. Spherocitosis
answer: A
Autosplenectomy, which refers to the physiological loss of spleen function (hyposplenism) - eg, associated with
sickle cell anaemia (chronic damage to the spleen results in atrophy)
(master the boards)

55

5. Lung disease causes clubbing


Answer : A. Bronchiectasis
It usually begins in the thumb and index fingers and is most often associated with pulmonary or cardiovascular
diseases, including lung cancer, interstitial pulmonary fibrosis, lung abscess, pulmonary tuberculosis, pulmonary
lymphoma, congestive heart failure, infective endocarditis, and cyanotic congenital heart disease . Less
frequently, digital clubbing may occur in patients with extrathoracic disease, including inflammatory bowel
disease, liver cirrhosis, and gastrointestinal neoplasms
Uptodate
6. woman with recarrant UTI , WHY ?
Because it cleans itself of the from anus to vulva ( my answer )
Answer:
Biologic or genetic factors Women with recurrent UTI have been shown to have an increased susceptibility to vaginal
colonization with uropathogens
Behavioral risk factors
Spermicide use during the past year
Having a new sex partner during the past year
Having a first UTI at or before 15 years of age
Having a mother with a history of UTIs
Pelvic anatomy Pelvic anatomy may predispose to recurrent UTI in some women,
Postmenopausal women
Urinary incontinence
Presence of a cystocele
Postvoiding residual urine

No associations were found between a history of recurrent UTI and pre- and postcoital voiding patterns,
frequency of urination, delayed voiding habits, wiping patterns, douching, use of hot tubs, frequent use of
pantyhose or tights, or body mass index.
http://www.uptodate.com/contents/recurrent-urinary-tract-infection-in-women#H5

7. Most common site of metastasis:


A .Bone
B .Kidney
C.Breast
Answer: A
The most common sites of cancer metastasis are, in alphabetical order, the bone, liver, and lung.
http://www.cancer.gov/about-cancer/what-is-cancer/metastatic-fact-sheet#q3
8.Most sign renal cancer
A. Cachexia
B Hematuria
C Abdomen mass
Answer: B
The classic triad of RCC (flank pain, hematuria, and a palpable abdominal renal mass) occurs in at most 9 percent of
patients; when present, it strongly suggests locally advanced disease..
Hematuria is observed only with tumor invasion of the collecting system. In an early series, hematuria was observed in
almost 40 percent of patients
http://www.uptodate.com/contents/clinical-manifestations-evaluation-and-staging-of-renal-cell-carcinoma
9. Hypothyrodism pt on dose of thyroxine 75 .. Missed the dose 2 days because he dose not have the drug , lap
result show high TsH and normal t4 .. What dose should be taken ?
A-25
B-50
C-75
D-100
Answer: C

56

10. Pt with past hx of hodgikon lymphoma .. But cured completely .. Presented with back pain ... Examination and
evaluation show paraspinus Edema and fluid collection -ve burecella titer and tuberclin test ,, what the cause ?
a- burecellosis
b- breast cancer
c- recurrent hodgikon lymphoma
Answer: b long term complication of hodgkin lymphoma is solid tumor (breast cancer metastasis to bone) due to
radiotherapy
11.Ttt of meningitis ?
A- ampecillin
B- doxycycline
Answer:
less than 1m amoxacillin + ceftriaxone or genta
1m- adulthood vancomycin + ceftriaxone or cefotaxime
more than 60Y ampicillin + vancomycin + cefotaxime or ceftriaxone
Ref: 1st aid USMLE step 2
12.Lap result show high aptt and bleeding time .. Factor deficiency?
A.8
B.7
C.9
D.10
Answer: A in vWF disease there is deficiency in vWF and factor 8 most common inherited bleeding disorder lab high
PTT and prolong bleeding time

13.Ischemic heart pt with lap result of high lipid .. What the next order ?
A. TFT
B. LFT
Answer: B
recommend treating all patients with CVD with high-dose statin therapy
In 2012, the US Food and Drug Administration revised its labeling information on statins to only recommend liver
function testing prior to initiation of statin therapy and to only repeat such testing for clinical indications
uptodate

14.Pt with 1st metatarsal joint pain , redness and erythema .. High temperatures? What is the cause ?
A- staph aruse
B- NA monourate crystal
C- Ca pyrophosphate crystal
Answer: B na monoureat crystals
source: Text book of disorders and injuries of ms system
15.charstic of perforated dudenal ulcer?
A - Mid epigastric pain
B - Statorha
C - Melena
Answer: A
uptodate
16.NF1 gene respons for ?
A - Nureofibromatosis type 1 (NF1)
Answer:
NF1 is due to mutations in the NF1 gene, located at chromosome 17
Ref: uptodate

57

17.what is genetic beta thalassmia?


A - Inseration
B - Mutation
C - Depleting
D - Fharm shaft
Answer: B
The deficiencies in hemoglobin biosynthesis that characterize the various forms of thalassemia arise from mutations in or
near the two globin gene clusters.
uptodate
18. atlatic come for check up all thing normal except Xanthelasma on achllis tendon and cholesterol?
A - Ldl resptor
B - Apo ll
C - Apo c
Answer: A
LDL receptor genetic defects
19.which type of anemia associated with hyposplenism?
A -Sickle cell anemia
B - Thalassmia
C - Spherocitosis
D - B12
Answer: A
20.Long scenario of patients labs show low hg low platelet with normal reticylcyte which Antibiotic cause this ?
a.Tetracycline
B.cloroampincoal
c.cefepime
Answer : B
21.Bacteria sexual like behavior ?
Conjunction
Answer: Bacterial conjugation.
22.Enzyme that get elevated in MI ?
Creatine phosphokinase
Note : no troponin in question
Answer: Creatine Kinase-MB, Serial measurements of CK-MB isoenzyme levels were previously the standard
criterion for the diagnosis of myocardial infarction. However, sensitivity and specificity are not as high as they are for
troponin levels. The trend has favored using troponins and they are now considered to be the criterion standard for
defining and diagnosing myocardial infarction, according to the American College of Cardiology.
Reference: http://emedicine.medscape.com/article/155919-workup#c9
23.Patient with 3 reading high bp all investigation normal except high Na What dx?
a-primary hyperaldostenism
B.essential hypertension
C.Secondary hypertension
Answer: Primary hyperaldosteronism. Mild serum hypernatremia in the 143-147 mEq/L range and mild
hypomagnesemia from renal magnesium wasting are other associated biochemical findings in established primary
aldosteronism
Reference: http://emedicine.medscape.com/article/127080-workup#c8
24.patient came from Sudan before two weeks and developed fever headache and vomiting what is the best
diagnostic test ?
A.-blood culture
B.Stools culture
C.Peripheral blood picture.
Answer C.
Malarial infection is suspected. Individuals are generally asymptomatic for 12 to 35 days but can commence symptoms
as early as 7 days (depending on parasite species) In most cases, the incubation period for P. falciparum infection is
about 12 to 14 days (range 7 to 30 days); most infections due to P. falciparum become clinically apparent within one
month after exposure. Detection of parasites on Giemsa-stained blood smears by light microscopy is the standard tool
for diagnosis of malaria and remains the most common onsite diagnostic method

58

References: http://www.uptodate.com/contents/clinical-manifestations-of-malaria
http://www.uptodate.com/contents/diagnosis-of-malaria
25.patient have cough and sob x-Ray show consolidation in right upper lob what ttt
Answer: Antituberculosis drugs. TB may be found in any part of the lung, but upper lobe involvement is most
common.
Reference: http://emedicine.medscape.com/article/230802-workup#c12
26.Young adult having episodic palpitation and fear and tightness . Btw the attack she feel fatige? What
investigation ?
A- urine catacholamins test
Answer
Pheochromocytoma is a nonmalignant lesion of the adrenal medulla autonomously overproducing catecholamines
despite a high blood pressure.
Symptoms : o Hypertension that is episodic in
nature
o Headache
o
Sweating
o Palpitations and tremor
Diagnostic Testing
Best initial tests:
- High plasma and urinary catecholamine
- Plasma-free metanephrine and VMA
Most accurate test:
- CT or MRI of the adrenal glands
- Metastatic disease is detected with an MIBG scan
Treatment
1. Phenoxybenzamine (alpha blockade) first to control blood pressure.
Without alpha blockade, patients blood pressure can significantly rise intraoperatively.
2. Propranolol is used after an alpha blocker like phenoxybenzamine.
3. Surgical or laparoscopic resection. Metastatic disease cannot be treated with surgery.
4. Metastatic disease is treated medically.
Reference : Master The Board
27.Patient came with severe pain involving the forehead to nose ( trigeminal distribution) what is the diagnosis ?
A- trigeminal neuralgia
Answer
Trigeminal neuralgia is an idiopathic disorder of the fifth cranial nerve resulting in severe, overwhelming pain in the
face. Attacks of pain can be precipitated by chewing, touching the face, or pronouncing certain words in which the
tongue strikes the back of the front teeth. Patients describe the pain as feeling as if a knife is being stuck into the face.
There is no specific diagnostictest. Treat with oxcarbazepine or carbamazepine. Baclofen and lamotrigine have also been
effective. If medications do not control the pain, gamma knife surgery or surgical decompression can be curative.
Reference : Master The Board
28. patient with asthma exacerbation ,Which drug will decrease the mucous secretion more the bronchodilation :
A-oral steroids
B- ipratropium
C- leukotriene
Answer A
Corticosteroids reduce the mucus secretion by inhibiting the release of secretagogue from macrophages.
Reference: http://www.ncbi.nlm.nih.gov/pubmed/3026210
29.patient with pheocromocytoma And high catecholamine in urine Initial medical management
A- ACEI
B-aldosteron blocker
C- Phenoxybenzamine
Answer C
Phenoxybenzamine (alpha blockade) first to control blood pressure. Without alpha blockade, patients blood pressure
can significantly rise intraoperatively.
Reference : Master The Board

59

30.smokers obese patient can't exercise with family history of MI came with vague chest pain . But ECG is normal
Next step ?
A- 24 hours ECG
B- exercise with ECG
C- perfusion cardiac scan
Answer : C
When this is the answer:
Patients who cannot exercise to a target heart rate of > 85% of maximum:
COPD
Amputation
Deconditioning
Weakness/previous stroke
Lower-extremity ulcer
Dementia
Obesity
Reference : Master The Board
31.With thyrotoxicosis, what is the most common arrhythmia:
A. SVT
B. VT
C. VFib
D. WPW
Answer: SVT ( toronto note mentions AF as most common)

60

32.An ECG with ST elevation in V1-V5. :


A. Extensive anterior MI
B. High lateral MI
Answer: Anterior or anteroseptal MI
Extensive MI there has to be ST elevation of 6 or more leads,

(torono notes C8)


33.A patient with sudden chest pain, diaphoresis, best test to order
A. Echo
B. ECG
C. Cardiac enzymes
Answer: ECG ( first investigation in chest pain is ECG, Master the board)
34.case of poor hygiene in and hallucinating, what medication to give ?
-lithium
Answer: ?
antipsychotic medications used to treat hallucinations include haloperidol, olanzapine and risperidone.
http://www.news-medical.net/health/Hallucination-Treatments.aspx
35.HIV presents commonly with? ( repeated )
A-opportunistic infection
B -chronic diarrhea
C -generalized lymphadenopathy
answer: C
The patient may present with signs and symptoms of any of the stages of HIV infection. Acute seroconversion manifests
as a flulike illness, consisting of fever, malaise, and a generalized rash. The asymptomatic phase is generally benign.
Generalized lymphadenopathy is common and may be a presenting symptom.
http://emedicine.medscape.com/article/211316-clinical
36.increase of which of the following prevalence cause reactivation of TB in
developed countries? ( repeated )
a-DM
b-HIV
Answer: B
(WHO & UpToDate: According to data from the United States National TB Surveillance System for 1993 to 2005,
7.7 percent of TB patients were HIV infected; greater than 80 percent of TB patients in the United States received
HIV testing [43]. HIV infection rates in TB patients were highest among injection drug users, homeless persons,
correctional facility inmates, and alcoholics (35, 22, 16, and 15 percent, respectively) [43].)
37.positive culture of budding yeast in urine what is the management ? ( repeted )
A-Flucanazole
B-caspofungin
Answer: A (UpToDate & Medscape)

61

38.X-ray of pericardial effusion ( repeated )


Answer: Plain film: chest radiograph
a very small pericardial effusion can be occult on plain film
there can be globular enlargement of the cardiac shadow giving a water bottle configuration
lateral CXR may show a vertical opaque line (pericardial fluid) separating a vertical lucent line directly
behind sternum (epicardial fat) anteriorly from a similar lucent vertical lucent line (pericardial fat)
posteriorly; this is known as the Oreo cookie sign 5
widening of the subcarinal angle without other evidence of left atrial enlargement may be an indirect clue
Ref: http://radiopaedia.org/articles/pericardial-effusion

39.what most commonly cause itching ?


Answer:?
40-most common cause of cough-in-adults-beyond-adult? ( repeated )
A-GERD
B-postnasal drip
C-asthma
Answer: B
http://www.uptodate.com/contents/chronic-cough-in-adults-beyond-thebasics?source=outline_link&view=text&anchor=H2#H2
41.hypertensive patient on ACEI but not controlled BP, what to add? ( repeated )
A-furosemide
B-thiazide
C-beta blocker
Answer: C http://www.ncbi.nlm.nih.gov/m/pubmed/11515986/
42.enteric fever best diagnosed in the first week of presentation by ?
A-blood culture
B-stool culture
C-multiple something?
D-bone marrow>>not sure
Answer:D (bone marrow culture)
43.rheumatoid arthritis with the loss of bones in joint? What is the cause ?
A-substance released by synovial cell
B-synovial fluid pressure
C-something prostaglandin?
Answer: because use corticosteroids in treatment
44. A 72 y male disoriented and hallucinating and disorganized thinking had aorto popletial graft and symptom
fluctuates in the 2 days what the cause ?
A.multi infraction demntia
B.mania
C.demensia
D.delirium
Answer: D (post operative delirium)
45.A case of a patient diagnosied to have cutenous lachmenia or bghlabar/ bhagdad type which type of lachmenia
?
A.Kalazar
B.donavan
and 2 more type of lachmenia
Answer:A
46.best initial screening test for pt suspected with coarctation of aorta
A. echo/doppler
B. ct cardiac
C. mri cardiac

62

D. cardiac angio
Answer:A
http://www.uptodate.com/contents/clinical-manifestations-and-diagnosis-of-coarctation-of-the-aorta
47.female non smoker with nodule by ct found calcium and fat
a. hamartona
B. mystheoma
C. no adeno
Answer: A
http://emedicine.medscape.com/article/356271-overview
48.smoker with hilar mass what suspect
A. lymph node
B. squamous cell ca
C. adenocarcinoma
Answer: B
http://emedicine.medscape.com/article/279960-overview
49.pt in ICU on ventilator develop yeast infection what is rx
a. fluconazole
B. itraconazole
Answer:A
http://emedicine.medscape.com/article/213853-treatment
50. which common feature of IBS?
A. diarrhea
B. constipation
C. vomiting
Answer: A or Abdominal Pain relived by defecation
51. which cause insomnia irritability and restless?
A. TCA
B. tetracyclin antidepressant?
C. SSRI
Answer: C
52.DM type 1 which of these confirm dx
a. acetone
Answer: A
53.patient is sezuir sence 35 mint , he take diazepam i.v but nor effective , what will you do ? Frn
Answer:
Phenyton
54.acetone + high glucose
A. dka
B. hyperosmolar
Answer: A
55.organism undercooked beef ?
A.Entamoeba B.Tinea
Answer: B (Teina saginara - beef Tenia solium- pork)

63

64

56. monospot test +ve what is the diagnosis


a. infectous mononeclosis
Answer :A
https://www.nlm.nih.gov/medlineplus/ency/article/003454.htm
57.70 years old female patient with osteprosis what is the treatment ?
A. estrogen
B. biophosphate
Answer: B
1st Aid p:320
58. patient irriagate , and weakness in lower and upper extermites , what is the cause ?
A. deficincy in vit d
B. deficincy in vit a
C. deficincy in b1
D. deficincy in b3
Answer: C
59.which of this drug cause sezuir ?
A. isoniazide
B. ethambutaol
C. ripaficin
D. pyrazinamide
Answer: A
http://reference.medscape.com/drug/isoniazid-342564#4
60. man got a bee sting then his wife trying look for the epinephrine what it
gonna inhibit?
A-luektrine release from macrophages
B-cross reactivity with the cardiac..
C-inhibit immunocomplex formation
Answer: here in the link belw the same Q but the is extra choise and it is the Right Ans D. Widespread
histamine release.

https://quizlet.com/8362963/immuno-block-4-practice-exam-flash-cards/

61.Niphritic syndrome?
A.HTN
B. Hypobilirubinemia
C. Edema
Answer: A
HTN is common in nephritic while edema is common in nephrotic
Reference: FA. step 1
62.First sign of portal HTN
A- spleenomegaly
B- hepatomegaly
C- ascites
Answer : A (Davidsons Principles and Practice of Medicine: Says splenomegaly is the most cardinal sign of portal
hypertension) no clear mentioning of any of the above options as being the first sign anywhere else.
63.what is the commonest cause of HTN in adolcent :
A.idipatic
B.renal
Answer : A
(american family physician/university of rochestor: essential hypertension is more common in adolescents)

65

64.MI patient within 6 hours what is the most expected complication :


A. ichose ?
B.PE .
C. Arrhythmia
Answer : C Arrhythmia is the most expected complication of MI within 6 hours
Reference: FA step 1
65.in factor V11 defiency what elevated :
A.pt
B.ptt......
Answer :
A.elevated PT
emedicine.medscape.com/article/209585-workup
66.patient has history of mi suddenly he become breahlesness and harshsystolic murmur heard what t
he cause :
A.PE
B.rupture aortic cusp
C.tricu regurge
D. rupture .......
Answer :-Rupture of papillary muscle ~>MR (harshsystlic )
https://books.google.com.sa/books?id=ANbZecMvdyIC&pg=PA107&lpg=PA107&dq=myocardial+infarction+harsh+sy
stolic&source=bl&ots=Zc4qWUx2t6&sig=6Vaup9OPqC3FhVYwI7ElgI7b2bA&hl=en&sa=X&redir_esc=y#v=onepage
&q&f=true
67.Pt with retrosternal chest pain for 3days increase in the last 24 hrs relieved by sublingual nitro Dx?
A.Unstable angina
B. Pericardidits
C. MI
Answer The duration of chest pain here is more than 3 days so unlikely to be a cardiac cause.
I suggest Non Cardiac cause eg. esophageal spasm that will relieved by nitro .
68.Celiac pt what is safe for him?
A.Rice
Answer : A. rice is safe for celiac patients
http://wheat.pw.usda.gov/ggpages/topics/Celiac.vs.grains.html
69.Pt c/o of loin pain & hematuria and inherited autosomal dominant what the diagnosis ?
kidney

A.polycystic

Answer A.polycystic kidney disease.Uptodate


http://www.uptodate.com/contents/polycystic-kidney-disease-beyond-thebasics?source=outline_link&view=text&anchor=H1#H1
70.sickle cell anemia false positive test because
A-high protein level
B-protein c
C-protein d
Answer : A
A positive test is consistent with sickle cell trait (hemoglobin A/S), sickle cell anemia (hemoglobin S/S), hemoglobin
S in combination with another hemoglobin variant, or hemoglobin C Harlem. Positive results also occur in
polycythemia, multiple myeloma, or cryoglobulinemia.
http://education.questdiagnostics.com/faq/FAQ99
71.sickle cell anemia came with hepato splenomegaly And low platelets- HGB -WBC
A- splenectomy
B-blood transfusion
Answer : B
-Hepatic or splenic sequestration BT is used acutely for the treatment of severe anemia that cannot be adequately
compensated by increased red cell production.
-splenectomy is often removed after a person has survived such a crisis to try and prevent another one.
-http://www.ncbi.nlm.nih.gov/pubmedhealth/PMH0012048/

66

72. Spoon shaped nail?


A-Iron def anemia
Answer A
73.what is HELLP syndrome
answer :
HELLP is an acronym that refers to a syndrome characterized by Hemolysis with a microangiopathic blood
smear, Elevated Liver enzymes, and a Low Platelet count [1]. It probably represents a severe form of
preeclampsia.
ref: uptodate
74.best diagnosis tool for thalacemia is
Answer : electrophorisis
75.Definitive diagnosis of TB.
Answer : culture is done on all initial samples to confirm the diagnosis. (However, a positive culture is not always
necessary to begin or continue treatment for TB.) A positive culture for M. tuberculosis confirms the diagnosis of
TB disease.
http://www.cdc.gov/tb/publications/factsheets/testing/diagnosis.htm
76. bilateral pneumonia treatment
Answer: ??
77.what factor cause thrombosis ?
Answer:
Factor V Leiden mutation or activated protein C resistance *
Prothrombin 20210 mutation
Hyperhomocysteinemia
FVIII, FIX, FXI, FVII, VWF
78. what medication dissolve thrombos ?
Answer:
STREPTOKINASE , ALTEPLASE , RETEPLASE , TENECTEPLASE , LANOTOPLASE
Ref: uptodate
79- Smoking increases risk of:
A- BLADDER CANCER
Answer: A
Smoking is the most important risk factor for bladder cancer. Smokers are at least 3 times as likely to get bladder cancer
as nonsmokers. Smoking causes about half of the bladder cancers in both men and women.
Reference: http://www.cancer.org/cancer/bladdercancer/detailedguide/bladder-cancer-risk-factors
80-What is the treatment of temporal arteritis?
A- ORAL STEROIDS
B- TOPICAL STEROIDS
Answer: A
The universally accepted treatment of giant cell arteritis (GCA) is high-dose corticosteroid therapy. The major
justification for the use of corticosteroids is the impending danger of blindness in untreated patients.
Few studies exist regarding dosing protocols for corticosteroids in GCA. It is generally agreed that most patients with
suspected GCA should be started on oral prednisone 40-60 mg/day, with a temporal artery biopsy performed within 1
week.
Reference: http://emedicine.medscape.com/article/332483-treatment

67

81- CLUSTER HEADACHE TREATED BY:


A- Oxygen
Answer: A
Pharmacologic management of cluster headache (CH) may be classified into 2 general approaches as follows:
Abortive/symptomatic (eg, oxygen, triptans, ergot alkaloids, and anesthetics)
Preventive/prophylactic (eg, calcium channel blockers, mood stabilizers, and anticonvulsants)
Reference: http://emedicine.medscape.com/article/1142459-treatment
82. Calculate anion gap
Na: 135 Cl: 100 HCO3: 12
A.
23
B.
10
C.
6
Answer: A
AG = Na - (Cl + HCO3)
Range of normal AG: 10 2
Reference: http://emedicine.medscape.com/article/2087291-overview
83. A patient present with fatigue, palpitation, SOB and pallor, Hgb 9. Shown is the peripheral film. What is the
type of anemia?

A.
B.
C.
D.

Megaloblastic anemia
Hypochromic microcytic
Sickle cell
G6PD

Answer: A
Note that RBCs are as large as the neutrophil and lymphocyte.
84: Most common cause of acute bronchiolitis:
A- RSV
B- Adeno
C- Parainfluenza
Answer: A
Reference: http://emedicine.medscape.com/article/961963-overview
85- Rhinorrhea, cough and conjunctivitis etiology?
A- Rhinovirus
B- Adenovirus
Answer: B
The most common cause of rhinorrhea and sinusitis is Rhinovirus. But since there is also conjunctivitis, then
Adenovirus is more appropriate.
Reference: http://emedicine.medscape.com/article/302460-clinical

68

86- old patient agitated, urine incontinence, confused, impaired short memory, intact long memory, CT temporal
and hippocampal atrophy, which gene is affected
A-13
B- 21
C-18
D-X
Answer: B
Early-onset Alzheimer's disease occurs in people age 30 to 60 and represents less than 5 percent of all people with
Alzheimer's. Implicated chromosomes: 21, 14, and 1.
Most people with Alzheimer's have the late-onset form of the disease, in which symptoms become apparent in the mid60s and later. The causes of late-onset Alzheimer's are not yet completely understood, but they likely include a
combination of genetic, environmental, and lifestyle factors that affect a person's risk for developing the disease.
Reference: https://www.nia.nih.gov/alzheimers/publication/alzheimers-disease-genetics-fact-sheet
87- long standing trip, swelled LL, no pain, high D-Dimer management?
A- aspirin
B- LMWH
C- warfarin
D- unfractioned heparin with warfarin
Answer:D
Admitted patients may be treated with a LMWH, fondaparinux, or unfractionated heparin (UFH). Warfarin 5 mg PO
daily is initiated and overlapped for about 5 days until the international normalized ratio (INR) is therapeutic >2 for at
least 24 hours.
For admitted patients treated with UFH, the activated partial thromboplastin time (aPTT) or heparin activity level must
be monitored every 6 hours while the patient is taking intravenous (IV) heparin until the dose is stabilized in the
therapeutic range. Patients treated with LMWH or fondaparinux do not require monitoring of the aPTT.
Platelets should be monitored. Heparin or LMWH should be discontinued if the platelet count falls below 75,000.
Fondaparinux is not associated with hepatin-induced thrombocytopenia (HIT).
Reference: http://emedicine.medscape.com/article/1911303-treatment
88- HIV presents commonly with?
A-opportunistic infection
B-chronic diarrhea
C-generalized lymphadenopathy
Answer: A
In general, people with CD4 counts greater than 500 cells/mm3 are not at risk for opportunistic infections. For
people with CD4 counts around 500, however, the daily fluctuations in CD4 cell levels can leave them vulnerable
to minor infections, such as candidal vaginitis or yeast infections.
Reference: https://www.aids.gov/hiv-aids-basics/staying-healthy-with-hiv-aids/potential-related-healthproblems/opportunistic-infections/
89- Man got a bee sting then his wife trying look for the epinephrine what it gonna inhibit?
A-leukotriene release from macrophages
B-cross reactivity with the cardiac..
C-inhibit immunocomplex formation
Answer: A
Epinephrine maintains blood pressure, antagonizes the effects of the released mediators, and inhibits further release of
mediators.
Reference: http://emedicine.medscape.com/article/135065-treatment#d9

69

90- Increase of which of the following prevalence cause reactivation of TB in developed countries?
A-DM
B-HIV
Answer: B
Reference: http://www.medscape.com/viewarticle/443137_2
91- post streptococcal infection generalized petechia and plt =15 Management?
A-splenectomy
B-cyclo
C-VIII
D-IVIG
Answer: ?

Newly added QS 7th update:


1 - You give pt ACI for treating hypertension , what you'll add :
Answer: we recommend therapy with a long-acting ACE inhibitor or ARB in concert with a longacting dihydropyridine calcium channel blocker. Combination of an ACE inhibitor or ARB with a
thiazide diuretic can also be used but may be less beneficial. ACE inhibitors and ARBs should not be
used together.
Ref: uptodate
http://www.pharmacology2000.com/Cardio/antihyper/antihype.htm

70

2- obese male pt presented to family physician complaining of chest pain for 2 days ..ECG normal
..what to do next ?
A-Treadmill
b- Coronary angio
c- 24 hrs monitoring
Answer
3- Pt has a family hx of hemochromatosis presented with abdominal pain ,fatigue ..what to check ?
A-Ferritin .
B transferrin
Answer :
Transferrin saturation Transferrin is a protein that binds iron and transports it between the
tissues. Transferrin saturation is calculated from iron levels in the blood. The transferrin saturation
increases as the body's iron stores increase. This test is one of the most sensitive tests for
detecting early hemochromatosis. A transferrin saturation greater than 45 percent should be
investigated further.
Ref : http://www.uptodate.com/contents/hemochromatosis-hereditary-iron-overload-beyond-thebasics#H10
4- What is the best to dx coarctation of aorta ?
A-Cardiac MRI
B- echo
Answer:
Postnatal diagnosis The clinical diagnosis of coarctation of the aorta is based upon the
characteristic findings of systolic hypertension in the upper extremities, diminished or delayed
femoral pulses (brachial-femoral delay), and low or unobtainable arterial blood pressure in the
lower extremities. The diagnosis is confirmed by noninvasive imaging methods, particularly
echocardiography.
Ref: http://www.uptodate.com/contents/clinical-manifestations-and-diagnosis-of-coarctation-ofthe-aorta#H17
5-What valve lesion you'll find in acute infective endocarditis ?
A- Mitral stenosis
B- mitral regurgue
C- aorta stenosis
D- aorta regurge .
Answer: MR
Mitral regurg. ( short textbook of medical diagnosis and management, INAM DANISH, page 101)

71

7- Pt had URTI ,2 weeks later developed orthopnea , severe pulmonary edema


-What is the dx .
Infective endocarditis
acute epica rditis
acute myocarditis
acute bronchitis
Answer ??
8- Pt healthy pt with no symptoms x ray is normal ,has negative hx of tuberculin test now
has positive test ..?
- Reassure
- give rifampicin and izo
-give izo for 6 months
Answer :
Isoniazid One of the most commonly used treatments for LTBI is isoniazid (INH). Isoniazid is a pill
that is taken once per day for nine months. It is important to take the medicine every day and to
finish the entire course of treatment since missing days or discontinuing the medicine early may not
prevent active TB. This medication interacts with seizure medications. Antacids may decrease the
absorption of this medication.
Rifampin Another treatment option is a medication called rifampin. Rifampin is taken as two
capsules every day for four months. Rifampin interacts with many other medications, such as
seizure medications, hormonal birth control methods (skin patch, pills, vaginal ring), blood thinners,
certain medications for high blood pressure, and many others. Therefore, it is important to discuss
possible drug interactions with a healthcare provider. This medication also alters the color of urine,
sweat, and tears, giving them an "orange" tinge.
Rifapentine A rifampin-like medication, rifapentine, may be prescribed along with INH in some
situations. It usually is taken once a week with the INH under direct observation by a trained
healthcare worker. Its toxicities and drug interaction properties are similar to those of rifampin.
Ref : http://www.uptodate.com/contents/tuberculosis-beyond-the-basics#H15
9- How to dx hepatitis B ?
Answer :
HBsAg, hepatitis B core antibody (anti-HBc), and hepatitis B surface antibody (anti-HBs)
Ref : http://www.uptodate.com/contents/diagnosis-of-hepatitis-b-virusinfection?source=see_link#H9
10- Pt with heart valve lesion developed endocarditis ..whats the organisms ?
Answer: streptococcus verdins????

72

11- Pt witb absence seizure the dr wants to start him on sodium valproate ..what test should
be done before starting the tx ?
A) Liver function ,
B) creatinine ,
C) urea ..
Answer: A
Side effects of VPA include nausea, vomiting, hair loss, easy bruising, and tremor VPA is associated
with weight gain, obesity, insulin resistance, and the metabolic syndrome . VPA can also cause
thrombocytopenia and other coagulation disturbances and has also been associated with
subclinical hypothyroidism with mild to moderate elevations in thyrotropin (TSH) levels . VPA has
also been linked to the polycystic ovarian syndrome . A number of case reports have linked VPA to
Fanconi syndrome in children with severe disabilitie.
VPA-exposure in utero is associated with major malformations and other adverse effects, including
neurodevelopmental abnormalities. VPA should be avoided in pregnancy when possible
Approximately 5 to 10 percent of patients develop ALT elevations during long term VPA therapy;
most of the time these abnormalities are asymptomatic and can even resolve with continuation of
the drug. In addition, there are more serious forms of hepatotoxicity that can occur with VPA:
_VPA-related hyperammonemic encephalopathy (VHE
_Acute hepatocellular injury with jaundice can occur
****Although routine monitoring of hepatic function has not been shown to permit early
identification of serious toxicity or improve outcome, many physicians choose to obtain liver
function tests (LFTs) once or twice a year in patients who are clinically asymptomatic. The FDA
recommends checking LFTs prior to initiating treatment and at frequent intervals thereafter,
especially during the first six months
REf :UPTODATE
12- Infection in venous lines ? With needle insertion . (Something like that .)
Answer: pseudomonas according to UAQ questions if staph not present in choices
Gram-positive organisms currently account for ~ 5060% of nosocomial bacteremic events.
Staphylococcus epidermidis is the most common gram-positive organism isolated from blood (~
30% of isolates) and accounts for the majority of infections that are associated with an intravascular
catheter. Staphylococcus aureus also causes a significant number of bloodstream and intravascular
catheter infections and the incidence of clostridial and streptococcal infections has increased .
Enterococci have become a significant cause of bacteremia in surgical patients and have been
isolated increasingly from patients with burns or multiple injuries
Gram-negative bacteria account for ~ 30% of all episodes of bacteremia at most institutions. The
mortality associated with gram-negative bacteremia in normal individuals is ~ 10% and may exceed
50% in immunocompromised patients. The most common causative microbes include Escherichia
coli, Klebsiella pneumoniae, Pseudomonas aeruginosa, Enterobacter aerogenes and cloacae,
although a vast array of organisms can be responsible. Infections caused by Pseudomonas and
Klebsiella species occur more commonly in patients with serious underlying diseases (e.g., extensive
trauma, burns, or malignancy
73

A.

13- Pheochromocytoma ..what is the initial tx ?


Alpha antagonist .
Answer: A
alpha adrenrgic blocker to control hypertension follow by beta blocker to control tachycardia
never give BB first ( unoppsed action lead to refractory hypertension.

usmle step 2 first aid

14- Gastoentritis ddx


Answer:
DIFFERENTIAL DIAGNOSIS The differential diagnosis of acute viral gastroenteritis includes
other causes (infectious and non-infectious) of acute diarrhea [12,14]. Diarrhea that lasts over
a week in an individual with a history of travel, hiking, or oral-anal sexual activity should
prompt evaluation for protozoa such as Giardia and cryptosporidium [9]. Recent antibiotic use
or hospitalization should prompt consideration of Clostridium difficile infection. Common
foodborne illnesses (eg, Staphylococcus aureus) need to be considered, particularly when the
incubation period is shorter than is typical for viral illness (ie, within 8 to 16 hours). The
presence of alarm symptoms or signs should prompt further investigation for an alternate
diagnosis.
Causes of chronic diarrhea that may less commonly masquerade as acute viral gastroenteritis
include: colorectal cancer, irritable bowel syndrome, inflammatory bowel disease,
microscopic colitis, malabsorption syndromes, post-cholecystectomy related diarrhea,
medication-induced diarrhea, laxative abuse, and chronic infections. Patients with acute viral
gastroenteritis may also present with isolated vomiting without prominent diarrhea. Clinicians
should consider adverse effects of medications and acute vestibular disorders in the
differential diagnosis of these patients.
uptodate
15- Anomaly between aortic arch and pulmonary trunk ?
A.

Patent ductus arteriosus .


Answer: A

74

16- Q about pathophysiology of DM 1 I Don't remember the options


Answer:

75

17- long Scenario about old male bedridden on foly's catheter he develop Gram -ve bacteria
what is the organism
A.
B.
C.

E.coli
pseudomonas aergonsa
strep. Puomonia

Answer : A
E. coli present in 27 percent of cases
Enterococcus spp 15 percent
Candida spp 13 percent
Pseudomonas aeruginosa 11 percent
Klebsiella spp 11 percent
18- long scenario about pt coming from Africa 3 wks ago with fever no other +ve points What is
the Dx?
1. Ebola
2. yellow fever
3.
answer:
yellow fever incubation period 3-6 days and in this scenario 3 weeks so we exclude it
ebola from 2 to 21 days so it could be
if malaria present most likely 630 days (98% onset within 3 months of travel)

76

DISEASE

USUAL INCUBATION PERIOD


(RANGE)

DISTRIBUTION

Incubation <14 days


Chikungunya

24 days (114 days)

Tropics, subtropics

Dengue

48 days (314 days)

Topics, subtropics

Encephalitis, arboviral (Japanese


encephalitis, tickborne encephalitis,
West Nile virus, other)

314 days (120 days)

Specific agents vary by


region

Enteric fever

718 days (360 days)

Especially in Indian
subcontinent

Acute HIV

1028 days (10 days to 6 weeks)

Worldwide

Influenza

13 days

Worldwide, can also be


acquired while traveling

Legionellosis

56 days (210 days)

Widespread

Leptospirosis

712 days (226 days)

Widespread, most common


in tropical areas

Malaria, Plasmodium falciparum

630 days (98% onset within 3


months of travel)

Tropics, subtropics

Malaria, P. vivax

8 days to 12 months (almost half


have onset >30 days after
completion of travel)

Widespread in tropics and


subtropics

Spotted-fever rickettsiae

Few days to 23 weeks

Causative species vary by


region

Incubation 14 Days to 6 Weeks


Encephalitis, arboviral; enteric fever;
acute HIV; leptospirosis; malaria

See above incubation periods for See above distribution for


relevant diseases
relevant diseases

Amebic liver abscess

Weeks to months

Most common in
developing countries

Hepatitis A

2830 days (1550 days)

Most common in
developing countries

Hepatitis E

2642 days (29 weeks)

Widespread

Acute schistosomiasis (Katayama


syndrome)

48 weeks

Most common in subSaharan Africa

Table 5-03. Common infections, by incubation period


77

19- long scenario I don't remember exactly but there is upper limb hypertension and low or absent
lower limb pulse What is Dx?
A.

Coarctation of aorta

answer : A

20- Lap result show high aptt and bleeding time .. Factor deficiency?
A.
B.
C.
D.

8
7
9
10

Answer : A
vWD associated with factor 8 deficiency
21-Patients with epilepsy, which of the following receptors most likely is stimulated?
A.Protein G
B.glutamate
C.serotonin
D.kinase
Answer: B, some antiepileptic drugs are glutamate receptor antagonists works specifically
on glutamate receptors, some other AEDs (such as topiramate) work on glutamate
receptors and other targets.
Ref:http://www.epilepsysociety.org.uk/how-anti-epileptic-drugs-work#.VmyX0K2e1D8
22-Patient 3 weeks after URTI develop rash, knee pain, and hematuria. What' the Dx?
A- Henoch-Schnlein purpura ( specific type of hypersensitivity vasculitis)
Answer: A
Henoch-Schnlein purpura IgA vasculitis characterized by a tetrad of clinical
manifestations:
Palpable purpura in patients with neither thrombocytopenia nor coagulopathy 95-100%
Arthritis/arthralgia especially involving the knees and ankles 60-84%
Abdominal pain usually diffuse, with acute-onset 35-85%
Renal disease proteinuria, hematuria
In one half to two thirds of children, an upper respiratory tract infection (URTI) precedes the
clinical onset of HSP by 1-3 weeks.
http://www.uptodate.com/contents/henoch-schonlein-purpura-immunoglobulin-a-vasculitisclinical-manifestations-and-diagnosis
23- 3-4 cases about IDA
Answer: iron deficiency anemia
ttt:
1-Oral iron therapy: initial therapy, gastrointestinal side effects are extremely common and
may result in poor adherence.
2-Parenteral iron: for those unresponsive to or intolerant of oral iron, for pt with IBD, gastric
bypass surgery.
3-BT: for pt who are hemodynamically unstable because of active bleeding.
http://www.uptodate.com/contents/treatment-of-the-adult-with-iron-deficiency-anemia
78

24-Dx of RA :
Answer: Anti-CCP Anti-citrullinated peptide/protein antibody
- more specific than rheumatoid factor, positive very early in the course of disease.
http://www.uptodate.com/contents/rheumatoid-arthritis-symptoms-and-diagnosis-beyondthe-basics#H19
25-case of fanconi syndrome
Answer:
a type of RTA a disease of the proximal renal tubules in which glucose, AA, uric acid, P and
bicarbonate are passed into the urine, instead of being reabsorbed. It may be inherited, or
caused by drugs or heavy metals.
clinical features of proximal renal tubular acidosis are:
Polyuria, polydipsia and dehydration
Hypophosphatemic rickets (in children) and osteomalacia (in adults)
Growth failure
Acidosis
Hypokalemia
Hyperchloremia
Other features of the generalized proximal tubular dysfunction of the Fanconi syndrome are:
Hypophosphatemia/phosphaturia
Glycosuria
Proteinuria/aminoaciduria
Hyperuricosuria
- treatment :
mainly consists of the replacement of substances lost in the urine. mainly fluids and
electrolytes.
Dehydration due to polyuria must be prevented by allowing free access to water;
treat dehydration with either oral or parenteral solutions.
Metabolic acidosis due to the loss of bicarbonate is corrected by 3-10 mg/kg/d of
sodium bicarbonate in divided doses.
Diuretic, as 1-3 mg/kg/d of hydrochlorothiazide, may be necessary to avoid volume
expansion, need to augment potassium supplementation in the form of potassium
bicarbonate, citrate, or acetate.
Phosphate and vitamin D supplementation are necessary, 1-3 g/d of supplemental
phosphate
https://en.wikipedia.org/wiki/Fanconi_syndrome
http://emedicine.medscape.com/article/981774-treatment#d6
26-case of pneumonia, what ur finding on the auscultation?
A.dispred crackles
B.bronchial breath sound
C.absence of vesicular breath sound
Answer: B
(short textbook of medical diagnosis and management, INAM DANISH, page 344)

79

27-case of multiple myeloma


Answer:
-cancer of plasma cells in the bone marrow.
-Criteria for diagnosis:
- A bone marrow aspirate or biopsy showing that at least 10% of the cells are plasma cells
or the presence of a plasma cell tumor (called a plasmacytoma), plus at least one of the
following two features:
- Evidence of damage to the body as a result of the plasma cell growth, as severe bone
damage, kidney failure, anemia, or high calcium in the blood, and/or
- Detection of one of the following findings: 60% plasma cells in the bone marrow; free
light chain ratio of 100 or more (provided involved FLC level is at least 100 mg/L); or MRI
showing more than one lesion (involving bone or bone marrow).
variety of symptoms:
-bone pain in the back or chest, or less commonly, the arms and legs. The pain is usually
triggered by movement and is absent at night, except when changing positions, The bone
loss and erosions can lead to osteoporosis and fractures pathological fracture
-High blood calcium levels : hyperCa symptoms loss of appetite, nausea, vomiting, frequent
urination, increased thirst, constipation, weakness, confusion, stupor, or coma.
-Anemia: paleness, weakness, and fatigue.
-Impaired kidney function: Occasionally, kidney failure is the first sign of MM.
-hyperviscosity syndrome The symptoms may include bleeding from the nose and mouth,
blurred vision, neurologic symptoms, and heart failure.
-Neurologic symptoms radiculopathy causing numbness or tingling, pain, or muscle
weakness.
- Generalized symptoms The generalized symptoms of MM include an increased
susceptibility to infections (especially during chemotherapy) and weight loss.
Dx: Blood and urine tests for monoclonal protein , Bone marrow examination (aspiration
and biopsy)
http://www.uptodate.com/contents/multiple-myeloma-symptoms-diagnosis-and-stagingbeyond-the-basics#H3498669556
28-What the scientific term of chewing ice?
A.Pagophagia
Answer: A, is a form of the disorder pica involving the compulsive consumption of ice or
iced drinks.
29- HIV patient test to confirm :
A.PCR
B.western blot
C.elisa
answer: B western blot is confirmatory.
, ELisa is for screening.. If s negative, other tests are not usually needed.
30- test for Sickle cell anemia :
A- Hb electrophoresis
B- bone marrow aspiration
answer: A

80

31- patient with lymphadenopathy , splenomegaly and fever:


A- infectious mononucleosis
answer:A
infectious mononucleosis by EBV , early sign is a fever, lymphadenopathy. Later finding
include hepatomegaly and splenomegaly. Reference: medscape
32- woman with recurrent UTI , WHY ?
A- Because it cleans itself of the from anus to vulva
answer:
Strongest risk for recurrent UTIs in young women is frequency of sexual intercourse.
Other risk include age at first UTI more than 15 years , maternal history of UTI.
there is no proven association between recurrent UTIs and wiping patterns , douching ,
frequency of urination
Reference ;
http://www.aafp.org/afp/2010/0915/p638.html
33-mid diastolic murmur in left sternum ?
A- Mitral stenosis
B- mitral reguarge
C- aortic stenosis
D- aortic regurage
answer A
MS: mid diastolic to late diastolic , rumbling
MR: Holosyatolic pansystolic , blowing radiate to left axilla
AS: Midsystic , harsh radiated to carotid artery
AR: early diastolic
34- 70 years old female patient with osteoporosis what is the treatment ?
A- ESTROGEN
B- BIOPHOSPHATE
answer is B
- Bisphosphonates (e.g. alendronate) is first line
-HRT: second-line treatment (unless for vasomotor instability as well)
Toronto notes 2015 p 516

81

35-patient is seizure sence 35 mint , he take diazepam I.V but not effective , what will you do ?
Phenytoin is not in the choices
answer:
Seizures occurring continuously for at least 30 minutes, or two or more seizures occurring
without full recovery of consciousness between attacks.
First-line therapy = benzodizepinesdiazepam, lorazepam, or midazolam.
- Lorazepam has a relatively longer duration of seizure suppression.
Second-line therapy = phenytoin and/or phenobarbital
-Phenytoin: Rapid administration may cause hypotension and cardiac dysrhythmias this
may be avoided with fosphenytoin .
- Phenobarbital: Anticipate sedation, respiratory depression and hypotension.
Drug-induced coma (pentobarbital, midazolam, propofol) or general anesthesia, if
resistant to above
Reference : first aid of emergency
36- What factor cause thrombosis ?
answer :
Risk Factors for VTE
Stasis
Age > 40
Immobility
CHF
Stroke
Paralysis
Spinal Cord injury
Hyperviscosity
Polycythemia
Severe COPD
Anesthesia
Obesity
Varicose Veins
Hypercoagulability
Cancer
High estrogen states
Inflammatory Bowel
Nephrotic Syndrome
Sepsis
Smoking
Pregnancy
Thrombophilia
Endothelial Damage
Surgery
Prior VTE
Central lines
Trauma
Anderson FA Jr. & Wheeler HB. Clin Chest Med 1995;16:235.
82

37- Bilateral pneumonia treatment


answer :( not clear question )
38- A case of a patient diagnosed to have cutaneous leishmania or baghdad boil which type of
leishmania ?
A- Kalazar /
B- Donavan
Answers is

Cutaneous leishmaniasis (also known as oriental sore, baghdad boil)i s the most common form
of leishmaniasis affecting humans caused by (e.g., L. major and L.tropica)
Visceral leishmaniasis (VL), also known as kala-azar caused by (e.g., L. infantum and L.
donovani)

39-type of lachmenia
answer : see above

83

40- which common feature of IBS:


A-diarrhea
B-constipation
C-vomiting
answer : A

http://www.ncbi.nlm.nih.gov/m/pubmed/21929652/
41- pt in icu on ventilator develop yeast infection what is Rx -fluconazole -itraconazole -...
answer : Fluconazole
(UpToDate: The most common antifungal agents used currently for the treatment of
candidemia are fluconazole and the echinocandins (caspofungin, micafungin,
anidulafungin). Formulations of amphotericin B are given less often due to the risk of
toxicity. Both the echinocandins and the azoles are better tolerated than amphotericin B
formulations.
& Oxford Journals: Overall, fluconazole-susceptible Candida albicans remains the
most common species causing candidaemia in ICU patients.)
42- calculate anion gap with corrected NA 138
answer :
43-best initial screening test for pt suspected with coarctation of aorta
-echo/Doppler
-ct cardiac
-MRI cardiac
-cardiac angio
answer : Chest XRay (best initial test according to Master the Boards, First Aid, Kaplan)
UpToDate: In children with coarctation, echocardiography often provides adequate anatomic
and hemodynamic information for the surgeon or interventional cardiologist without the
need for a further imaging study. However, MRI or CT is generally used as a
complementary diagnostic tool in adolescent and adult patients, and provides important
anatomic data prior to intervention.
In adults, cranial MR angiography (or CT angiography) is also appropriate to search for
intracranial aneurysms.
44- Young pt with HTN discrepancy , what is the tx ?
a- Trans-Ortic stenting
b- Thiazides
answer :b

84

45- Examination and evaluation show paraspinus Edema and fluid collection -ve brucella
titer and tuberculin test ,, what the cause ?
1- brucellosis
2- breast cancer
3- recurrent hodgkin lymphoma
answer: Recurrent Hodgkins Lymphoma ?
(I tried searching for this question, but couldnt find anything proper to answer it.)
46- Pt with past hx of hodgkin lymphoma .. But cured completely .. Presented with back
pain
Answer:
47-patient can't take BCG vaccine Because he deficiency in
A- IL
B- TNF gama
C IFN
Answer:
UpToDate: Studies among infants demonstrate BCG-associated induction of CD4+ and
CD8+ T cells, interferon (IFN)-gamma+, interleukin (IL)-2+, tumor necrosis factor (TNF)alpha+, and polyfunctional CD4+ T cells
Studies in adults indicate that BCG induces CD4+, IFN-gamma responses, and IFNgamma and TNF-alpha secreting CD8+ cells with cytotoxic activity; data on polyfunctional T
cells has been conflicting
In infants & adults, BCG works on: CD4 & CD8 T cells, IFN-gamma, IL 2, TNF-alpha.
48- diabetic patient what type of carbohydrates is recommended
Answer:
49- most common risk factor for stroke
A-HTN
B-atrial fibrillation
C- LDl
Answer: A
50- HTN patient develops gout Which drug side effect
Answer: thiazide
51- patient with pheocromocytoma And high catecholamine in urine Initial medical
management
A- ACEI
B-aldosteron blocker
Answer: Phenoxybenzamine
(alpha blocker - best initial therapy - Master the Boards & UpToDate)
52- Pt HTN came with uric acid 200 you prescribe antihypertensive drug for him after 1
weak uric acid 400 what is the drug ?
Answer: Thiazide diuretics
53- ITP case management >> prednisolone
85

54- Pt with high aptt what mechanism of action of that drug?


" unfractionated heparin" Antithrombin
Answer:
Medscape: Unfractionated Heparin = Its anticoagulation effect is mediated by the activation
of antithrombin III, which then inactivates with relatively equal potency the coagulation
enzymes thrombin (factor IIa) and factor Xa. Other antithrombotic effects include inhibition
of platelet aggregation and additional antithrombin III-independent mechanisms. The partial
thromboplastin time may be elevated because of inactivation of thrombin.
55- Aspirin inhibit which product formation Thromboaxan A2
answer:
56- what is the commonest cause of HTN in adolescent :
A-idiopathic
B -renal-.....
Answer: idiopathic ???
57- best prophylactic against traveller's diarrhea :
A_fresh fruit and vegetable daily antibiotic
B-peeled fruit
? drinks with rice ....
Answer: B
UpToDate: Basic advice for travelers to moderate or high-risk regions for travelers'
diarrhea includes eating only food that has been thoroughly cooked and served hot, fruits
that the traveler peels just prior to eating, and pasteurized dairy products. Beverages
should be bottled or disinfected. Bottled drinks should be requested without ice and should
be drunk from the bottle with a straw rather than from a glass. Hot tea and coffee are
usually safe alternatives to boiled water.
Although antibiotics and other agents (namely bismuth salicylate) are effective in reducing
the rate of travelers' diarrhea for individuals traveling from resource-rich to resource-poor
areas, we do not routinely recommend chemoprophylaxis. Use of daily antibiotics is
expensive, has potential side effects, can wipe out normal gastrointestinal flora that may be
beneficial, and can promote bacterial resistance.
However, chemoprophylaxis may be a reasonable approach in the setting of an underlying
medical condition that would increase the risk of complications from diarrhea or would be
severely exacerbated by dehydration from diarrhea such that the benefits of using antibiotic
prophylaxis outweigh its risks. Such situations include known severe inflammatory bowel
disease that could be exacerbated by an episode of infectious diarrhea; severe vascular,
cardiac, or renal disease that would be seriously compromised by dehydration; or a severe
immunocompromised state, such as advanced HIV disease or after a complicated organ
transplant.
If prophylaxis is administered, the options include:
Ciprofloxacin 500 mg once daily
Norfloxacin (not available in the US) 400 mg once daily
Rifaximin 200 mg once or twice daily
Bismuth subsalicylate two tablets chewed four times daily

86

58- parasite in soil contamination :


A-tenia saginatm
B -ascaris-bancrofti .....
Answer:
59- female patient with high prolactin What to exclude
1- thyroid disease 2- pituitary tumors
Answer: 2
60- Lipid profile of a patient shows high level and patient is on simvastatin What to add?
Answer:
UpToDate: Thus, in patients who do not achieve a particular LDL-C goal on statin therapy
alone, we suggest not adding a nonstatin lipid-lowering medication for primary prevention. That
is, the patient should be maintained on statin therapy as his/her only lipid-lowering medication.
61- Pt had head trauma , he has nausea, vomiting and decreased level of consciousness ,
ICP was suspected and CT was arranged , what cranial nerve examination can confirm the Dx
?
A.
Optic
B.
Oculomotor
C.
Trochlear
Answer: CN 5 (Trigeminal) & CN 6 (Abducent)
UpToDate: Global symptoms of elevated ICP include headache, which is probably mediated
via the pain fibers of cranial nerve (CN) V in the dura and blood vessels, depressed global
consciousness due to either the local effect of mass lesions or pressure on the midbrain
reticular formation, and vomiting.
Signs include CN VI palsies, papilledema secondary to impaired axonal transport and
congestion (picture 1), spontaneous periorbital bruising [16] and a triad of bradycardia,
respiratory depression, and hypertension (Cushing's triad, sometimes called Cushing's reflex or
Cushing's response)
Medscape: Occasionally, patients with diplopia present with oculomotor or trochlear nerve
palsy.
answer : A .. by checking the papilledema
62- Pt with barking cough and 38 temb which of the following symptoms is associated with
this disease ? Cyanosis
Answer:
Croup usually begins with nonspecific respiratory symptoms (ie, rhinorrhea, sore
throat, cough).
the characteristic signs of hoarseness, barking cough, and inspiratory stridor along
with a variable degree of respiratory distress. worsening at night,
http://emedicine.medscape.com/article/962972-clinical
63- KSA have implemented strong regulations regarding worker health cares , which of the following
diseases if the worker had , he can't work ? HeB HeC HIV
Answer:

87

64- pt with long hx of uncontrolled HTN , he presented to you with headache and 160/90 BP ,
what you will see in his kidneys :
A) Decrease sclerosis
B) Increase hyalination of arterioles
Answer: B
intimal thickening and luminal narrowing of the large and small renal arteries and
the glomerular arterioles, AND
Glomerulosclerosis: both focal global (involving the entire glomerulus) and focal
segmental sclerosis http://www.uptodate.com/contents/clinical-features-diagnosis-andtreatment-of-hypertensivenephrosclerosis?source=preview&search=%2Fcontents%2Fsearch&anchor=H1126882
4#H2
65- what is the most specific investigation for TB ?
Answer: sputum Culture for AFB is the most specific test for TB and allows direct
identification and determinatio
n of susceptibility of the causative organism.
http://emedicine.medscape.com/article/230802-workup#c10
66- MI patient within 6 hours what is the most expected complication? :
Answer: Arrhythmia ?? no reference
67- patient has history of MI suddenly he became breathless and harsh systolic murmur heard
what the cause :
a- PE
b - rupture aortic cusp
c -tricu regurge
d - rupture
Answer : interventricular septum rupture?? due to harsh systolic murmer
68- best diagnosis tool for thalassemia is :
Answer : Hemoglobin electrophoresis
69- in factor V11 defiency what elevated :
a-pt
b-ptt
Answer: A PT
A normal aPTT and a prolonged PT in a patient with a lifelong history of a tendency
for mild or severe bleeding is consistent with the diagnosis of factor VII deficiency or
the presence of an inhibitor to factor VII.
Bleeding time is usually within the reference range.
With no significant clinical bleeding but a prolonged PT and a normal aPTT, the
patient has either mild factor VII deficiency or is taking oral anticoagulants.
http://emedicine.medscape.com/article/209585-workup

88

70- splenctomy what vaccin he should took ?


Answer:
71- What is the most accurate test for carpal tunnel syndrome:
A. Tinel
B. Compression test
C. Durkan's carpal test
D. Phallens test
Answer: B ?? due to higher specificity than phalen and tinel tests in uptodate
The manual carpal compression test is performed by applying pressure over the transverse
carpal ligament, and it is deemed positive if paresthesia occur within 30 seconds of
applying pressure. The average sensitivity and specificity of the manual carpal compression
test is 64 and 83 percent.
http://www.uptodate.com/contents/carpal-tunnel-syndrome-clinical-manifestations-anddiagnosis?source=outline_link&view=text&anchor=H2#H2
72- Female pt diagnosed with IBD on ceftriaxon with no benefit wt is the organism
- herpes
Answer:
!!
73- Pt came with cough and she take anti cholesterol medication I cant remember the name
, she started it since 3 weeks , the Dr should worry about what ? " not sure maybe it's 2
different Q "
Answer: statin = LFT
74- One Small thyroid nodule , we do investigation and its Increase in iodine uptake , what
is the best treatment ?
a.
Conservative
b.
Antithyroid drug
c.
Iodine radiotherapy
Answer: B
http://emedicine.medscape.com/article/924550-overview#a1
75- Q pt k/c of DM with uncontrolled blood sugar With figure shows high at the 6am wt to
do?
a- long acting night insulin
b- short acting night insulin
c- long acting morning in...
d- short acting morning in
Answer: A by senior medical residant
76- Mycobacterium tuberculosis , that is the best culture media ?
a.
Blood agar ?
Answer: lwenstein Jensen agar
89

77What is the best ttt for traveler diarrhea ?


- ciprofloxacin
Answer: ciprofloxacin
78- Cancer of lung with high keratin ?
answer:
Non small cell cancer
http://www.ncbi.nlm.nih.gov/pubmed/12367790
79-Calcified lesion in the upper lung?
A. Broncopscoe bionsy
B. Percutanus baiopsy
C. Thoracotmy
D. Follow up with serial x-ray
Answer:
A !?
80- Renal or small cell cancer stage III with bone pain what is the immediate action?
A. MRI only
B. Radiothyrpy
C. Iv steriod and MRI
D. No imediate action
Answer: !!! A?
81- Pt with 1st metatarsal joint pain , redness and erythema .. High temperatures? What is
the cause ?
A.
Staph aruse
B.
NA monourate crystal
C.
Ca pyrophosphate crystal
Answer:!!
B came with gout or A becaue of high temp
http://worldwidescience.org/topicpages/a/acute+gouty+arthritis.html
82- Pt does not complain of anything, has sudden knee swelling? What is the best thing to
do?
Answer:!!!
83- Fever and cough then facial nerve then loss of reflexes?
A.Tetanus
B. Botulinsim
Answer: B
84- Long ceniro, bloody diarrhea and rbc in urine after 7 days hx of food posing, rx?
A. STERIOD
B. ANTIBIOTIC
answer: conservative
85- Brucellosis prevention?
a.
Pastrization
answer:
90

86- Low ph, bicarb, co2?


A. Compensated metabolic acidosis
answer:
87- Tb case, what is the next appropriate step to get a definitive dx?
A.
Speutm smear under microscope
B.
Sputum culture
answer:b
88- Asyptomaric pt. With Positive HBV antigen?
A. Acute hepatitis
B. Chronic hepatitis
C. Active carrier
D. Non active carrier
answer: INACTIVE carrier asymptomatic , carry HBV antigen more than 6m and HBVeAB
89- Pt. With HBA after 3 week we take bipsy, what is show?
A. Normal architecture
B. Fibrosis something
C. Another somethig
Answer:A
90- What is the common dz to make the pt. Retire in KSA?
A. HBV
B. HBC
C. HIV
D. hep a
Answer:C
91- Patient with acute rheumatic fever show acute cardiac symptoms, what is the treatment?
A.
IV penecillin
B.
IM steriod
Answer: A??
Treatment of ARF consists of anti-inflammatory therapy, antibiotic therapy, and heart
failure management.
Patients with severe carditis (significant cardiomegaly, congestive heart failure, and/or thirddegree heart block) should be treated with conventional therapy for heart failure.
Corticosteroids should be reserved for the treatment of severe carditis.
92- Cushing case which skin manifestation is associated with
it Vitiligo Telenagectasia Acropathy Something Derma
Answer:
??

91

93- 50 years old patient with third reading of persistent hypertension wasn't started on
medication yet, lab shows high Na of 147 and low k of 3 other parameters were
normal What's most likely the diagnosis?
a.
Essential hypertension
b.
Hyperaldorensism
Answer: B
94- 3rd Cranial palsy ?
Answer: Out and down
95-pneumonia , the organism ? repeated?
Answer: Pseudomonas
96-Addison disease:
Answer:
97- Primary hyperaldostironism.
Answer:
98- In addition to anti- HTN , what to advice ?
Answer: Restrict Na to 9mg ,Walking ( such meters)
99- Most common cause of obstructive sleep apnea ?
Answer:
Most patients have OSA because of a small upper airway. As the bones of the face and
skull develop, some people develop a small lower face, a small mouth, and a tongue that
seems too large for the mouth. These features are genetically determined, which explains
why OSA tends to cluster in families. Obesity is another major factor. Tonsil enlargement
can be an important cause, especially in children.
Reference: Uptodate
100- 3 Questions with scenarios with ABG to interpret
Answer:
?

101- Patient with chest pain and ST changes, you will find elevation in:
Answer: C
A.
ALT
B.
AST
C.
Troponin
c
102- 2 questions has the same idea with the same options about polymyalgia rheumatica
(they mention that ESR was high)
Answer:

92

103- Most common cause of itching:


Answer:
a.
eczema
b.
b- bile salt can't recall the rest
a
104- Patient on warfarin 7 mg presented with melena. INR was very high. What will you do?
Answer:
a.
give vit. K
b.
b- lower the dose of warfarin
answer : stop warfarin + Vit K
105- similar scenario 37 years old presented with back pain. On examination there was
tenderness when palpating paraspinal muscles, neurovascular exam was normal. What is
the treatment?
Answer:A
a. Physiotherapy
106- Best investigation to measure GFR.
Answer:A OR Inuline
A- 24 urine creatinine cllection other options were irrelevant

107- Primary biliary cirrhosis options were about pathophysiology.


Answer:
intrahepatic - T lymphocyte mediated attack on small intralobular duct
108- Bacterial meningitis in LP
Answer:A
a. decrease glucose and increase protein
b. increase glucose and decrease protein
109- Patient with osteoarthritis, you found that he developed anemia which type it would be:
Answer:A
a- normochromic normocytic
b- microcytic hypochromic ( if with NSAiD)
c- macrocytic hyperchromic
110- 70 years old smoker woman with low vit-D and osteoporosis. Which of the following
has the highest risk for osteoporosis?
Answer: B
a. Smoking
b. Age
c. Vit D

93

111- Patient is concerned about osteoporosis as her mother had it, what you will do:
Answer:A
a- give vit D, calcium
b- give estrogen postmenopausal

94

113- Patient with prolonged PT and PTT 3


Answer:

http://www.practical-haemostasis.com/Screening%20Tests/aptt.html
114- Patient with back pain that improves with walking, you find that it is muscle strain how
will you treat him:
Answer: A
a- physiotherapy
b- surgery
115- question about osteoporosis
answer:

95

116- X-ray of pericardial effusion

answer
117-most common cause of chronic cough in adult?
a GERD
b -postnasal drip
c asthma
answer:B
http://www.uptodate.com/contents/chronic-cough-in-adults-beyond-thebasics?source=outline_link&view=text&anchor=H3#H3

96

118- hypertensive patient on ACEI but not controlled BP, what to add?
A-furosemide
B -thiazide
C-beta blocker
answer: B
119-adolescent male with swelled parotid and salivary gland with dry eye and dry mouth, labs
HLA, ANA and RF are positive which of the following is appropriate treatment?
-a-Physostegmine
b -Artificial eye and saliva drops Answer: B

97

120- - rheumatoid arthritis with the loss of bones in joint? What is the cause ?
a -substance released by synovial cell
b -synovial fluid pressure
Answer: A

http://www.hopkinsarthritis.org/arthritis-info/rheumatoid-arthritis/ra-pathophysiology-2/
121-something prostaglandin?
( q not clear)
122-which part of bone is firstly affected in hematogenous osteomyelitis .
a epiphysis
b-metaphysis
c-diaphysis
Answer:
In children: metaphysis
http://www.uptodate.com/contents/hematogenous-osteomyelitis-in-children-epidemiologypathogenesis-andmicrobiology?source=search_result&search=hematogenous+osteomyelitis&selectedTitle=1%7E3
0
In adults: diaphysis
98

Source: http://www.uptodate.com/contents/hematogenous-osteomyelitis-inadults?source=search_result&search=hematogenous+osteomyelitis&selectedTitle=3%7E30
123- what is the commonest cause of HTN in adolescent:
a idiopathic
b -renal-.....
Answer: a
http://www.stanfordchildrens.org/en/topic/default?id=high-blood-pressure-in-children-andadolescents-90-P01609
124-Patients with epilepsy, which of the following receptors most likely is stimulated?
A.Protein G
B.glutamate
C.serotonin
D.kinase
Answer: B
http://www.uptodate.com/contents/pathophysiology-of-seizures-andepilepsy?source=search_result&search=seizure&selectedTitle=11%7E150
125- Patient is brought by his parent because of loss of hair. On examination: he had localized patch
of hair loss at temporal area, the end of hair looked broken and tapered. What is the diagnosis?
Alopecia Areata.
Trichotillomania.
Answer: Trichotillomania
http://www.uptodate.com/contents/telogeneffluvium?source=machineLearning&search=tapered+hair&selectedTitle=1%7E150&sectio
nRank=3&anchor=H1164291246#H1164291246
126-Patient 3 weeks after URTI develop rash, knee pain, and hematuria. What' the Dx?
A.hypersensitivity vasculitis
Answer: mostly henoch-shonlein purpura (HSP) also known as IgA vasculitis
http://www.uptodate.com/contents/henoch-schonlein-purpura-immunoglobulin-a-vasculitisclinical-manifestations-anddiagnosis?source=machineLearning&search=hsp&selectedTitle=1%7E124&sectionRank=3
&anchor=H26#H3
127- 3-4 cases about IDA
( q not clear)

99

128 -Dx of Rheumatoid arthritis


Answer: Anti-CCP
129-case of fanconi syndrome
( q not clear)
130 -case of pneumonia, what ur finding on the auscultation?
A.dispred crackles
B.bronchial breath sound
C.absence of vesicular breath sound
Answer: B
Source: http://emedicine.medscape.com/article/234240-overview#a8
131 - case of multiple myeloma
Answer:
Myeloma is an abnormal proliferation of plasma cells. These plasma cells are
unregulated in their production of useless immunoglobulin that is usually IgG or IgA.
IgM is a separate disease called Waldenstrom macroglobulinemia. These
immunoglobulins do not fight infection but clog up the kidney. The most frequent
presentation of MM is with bone pain caused by a fracture occurring under normal
use.. Also may present with Hyperuricemia ,anemia and renal failure .
Diagnostic Testing :
Skeletal survey to detect punched out osteolytic lesions. (Osteoblastic lesions
suggest metastatic prostate cancer.)
Serum protein electrophoresis (SPEP):You are looking for elevated levels of
monoclonal antibody (usually IgG). 20% are IgA.
Urine protein electrophoresis (UPEP):Detects Bence-Jones protein.
Peripheral smear:Shows rouleaux formation of blood cells.
-The single most specific test is the bone marrow biopsy,which detects high
numbers of plasma cells (10 percent).
Treatment :
Treat with melphalan and steroids. Thalidomide, lenalidomide, or bortezomibmay be
added The most effective therapy is an autologous stem cell bone marrow
transplantation.
Reference : master the boards usmle .
132 - What is the scientific term of chewing ice?
A.Pagophagia
Answer: A
Its might be a sign of medical condition like iron deficiency anemia.
Reference : mayoclinic
133 - HIV patient test to confirm :
A.PCR
B.western blot
C.elisa
answer: B
The best initial test for HIV is the ELISA test. This is confirmed with Western blot
testing. Infected infants are diagnosed with PCR or viral culture.
Reference : master the boards usmle step 2
100

134- test for Sickle cell anemia :


1- Hb electrophoresis
2- bone marrow aspiration
answer: A
The best initial test is a peripheral smear. Sickle cell trait (AS disease) does not give
sickled cells. The most accurate test is the hemoglobin electrophoresis.
Reference : master the boards usmle step 2
135- patient with lymphadenopathy , splenomegaly and fever :
1- infectious mononucleosis
answer:
Epstein-Barr virus causes infectious mononucleosis. Incubation period is 12 months. patients
have pharyngitis (which is commonly exudative) or tonsillitis, fever, lymphaden
opathy, and abnormal liver function. Splenomegaly may be seen.
Lymphocytosis is commonly found in acute EBV infection
usually with > 10% "atypical lymphocytes." Atypical lymphs are enlarged with
abundant cytoplasm, vacuoles, and indentations of the cell membrane.
Diagnosis of infectious mononucleosis may be made
clinically and confirmed by serology. Heterophile
antibody titers (monospot) are nonspecific antibodies that crossreact with RBCs of other mammals.
The diagnosis can be made by testing for IgM capsid antibody in
these patients. Treatment of acute EBV infection remains supportive because of
its excellent prognosis and unavailability of any antivirals active against EBV .It
causes oral hairy leukoplakia which seen as early manifestation of HIV disease.
Reference: MedStudy 16th Edition Internal Medicine Review Core Curriculum
136- woman with recurrent UTI , WHY ?
1 - Because it cleans itself of the from anus to vulva
Answer:
Recurrent urinary tract infection (UTI) refers to 2 infections in six months or 3
infections in one year.
Risk Factors for Recurrent UTI :
The frequency of sexual intercourse ( strong RF )
Spermicide use ( strong RF )
Women with recurrent UTI have been shown to have an increased
susceptibility to vaginal colonization with uropathogens compared with
women without a history of recurrences.
Pelvic anatomy may predispose to recurrent UTI in some women, with a
shorter distance from the urethra to the anus being associated with
increased risk.
Among postmenopausal women, mechanical, and/or physiological factors
that affect bladder emptying are associated with recurrent UTI.
Reference: UPTODATE

101

137 -mid diastolic murmur in left sternum ?


A- Mitral stenosis
B- mitral reguarge
C- aortic stenosis
D- aortic regurage
answer: A
causes of mid diastolic murmur :
Mitral stenosis is best heard at apex.
Tricuspid stenosis is best heard along the left sternal border.
Atrial myxoma.
Reference : UPTODATE
138 - 70 years old female patient with osteoporosis what is the treatment ?
1- ESTROGEN
2- BIOPHOSPHATE
answer: B
For the treatment of osteoporosis in postmenopausal women, we suggest oral
bisphosphonates as first-line therapy. We prefer oral bisphosphonates as initial
therapy because of their efficacy
Reference : UPTODATE
139-patient is having seizure since 35 minutes , he had diazepam I.V. but no benefit , what will
you do?
answer:

Reference : UPTODATE
140 - What factor cause thrombosis ?
answer :
Risk factors (causes) for the development of venous thrombosis :
Inherited : Factor V Leiden mutation
Prothrombin gene mutation
Protein S & C deficiency
Antithrombin (AT) deficiency
Acquired : Malignancy
- Surgery
- Trauma
Pregnancy
OCP
- Hormone replacement therapy
- Polycythemia vera IBD
-Antiphospholipid antibody syndrome
-Nephrotic syndrome
Reference : UPTODATE

102

141- Bilateral pneumonia treatment


answer : virus pneumonia or aspiration pneumonia
viral pneumonia : antiviral
aspiration pneumonia : clindamycin
142- A case of a patient diagnosed to have cutenous lachmenia or bghlabar/ bhagdad type
which type of lachmenia ?
A- Kalazar /
B- Donavan /
C-and 2 more type of lachmenia
answer : tropica
143- which common feature of ibs?
A- diarrhea
B- constipation
C- vomiting
answer : A
MYO CLINIC
144- pt in icu on ventilator develop yeast infection what is Rx
-fluconazole
-itraconazole
answer :
Amphotericin B is initial therapy
aspergillosis > voriconazole
medscape
145- calculate anion gap with corrected NA 138
answer : anion gap ;Na - ( Cl + HCO3 )=
anion gap should not be corrected to calculate anion gap it will lead to false elevated
calculation
146- best initial screening test for pt suspected with coarctation of aorta
A -echo/Doppler
B-ct cardiac
C-MRI cardiac
D-cardiac angio
answer : A dopplarechocardiograph establish the diagnosis
uptodate
147-Young pt with HTN discrepancy , what is the tx ?
A- Trans-Ortic stenting
B- Thiazides
answer : ?
common cause B
of HTN at adolescent is essential .
103

148- Pt with past hx of hodgkin lymphoma .. But cured completely .. Presented with back
pain Examination and evaluation show paraspinus Edema and fluid collection -ve
brucella titer and tuberculin test ,, what the cause ?
1- brucellosis
2- breast cancer
3- recurrent hodgkin lymphoma
answer: B
149- patient can't take BCG vaccine Because he deficiency in
A- IL
B- TNF gama
C
Answer:
150-diabetic patient what type of carbohydrates is recommended
Answer:

151- most common risk factor for stroke


A-HTN
B-atrial fibrillation
C- LDl
Answer:

152- HTN patient develops gout Which drug side effect


Answer: thiazide
Hyperuricemia is common finding in patient treated with thiazide and may lead to gouty
arthritis
Reference: uptodate

Q- patient with pheochromocytoma And high catecholamine in urine Initial


medical management
A- ACEI
B-aldosterone blocker
Answer: alpha blocker : phenoxybenzamine
All patients with pheochromocytoma need to undergo preoperative alpha-adrenergic
blockade; we suggest phenoxybenzamine as the first line drug
other alpha blocker medication : ( prazosin , terazosin , doxazosin )
after alpha blocker is given we give beta blocker
but never start beta blocker before the alpha blocker
surgical resection with early ligation of venous drainage
is the treatment of choice in pheochromocytoma
Reference: uptodate
104

153- Pt HTP came with uric acid 200 you prescribe antihypertensive drug for him
after 1 weak uric acid 400 what is the drug ?
Answer: Thiazide diuretics
the answer explained in Q152
154- ITP case management
Answer : Glucocorticoids ( Prednisone)
Reference: uptodate

155- Pt with high aptt what mechanism of action of that drug?


" unfractionated heparin" Antithrombin
Answer:
low dose heparin : inactivates factor Xa and inhibits conversion of prothrombin to thrombin
high dose heparin : inactivates factors IX , X , XI , XII and thrombin and inhibit conversion
of fibrinogen to fibrin .
156- Aspirin inhibit which product formation Thromboaxan A2
answer:
157- what is the commonest cause of HTN in adolescent :
A-idiopathic
B -renal-.....
Answer: idiopathic
158- best prophylactic against traveller's diarrhea :
A_fresh fruit and vegetables
B-peeled fruit
c- daily antibiotic
d- drinks with rice
Answer: B
Reference: uptodate
159- - parasite in soil contamination :
A-tenia saginatm
B -ascaris-bancrofti .....
Answer:

105

160- female patient with high prolactin What to exclude


1- thyroid disease
2- pituitary tumors
Answer: Pituitary tumors
A. Causes of hyperprolactinemia
1. Prolactinoma
a. Most common cause of hyperprolactinemia
b. Most common type of pituitary adenoma (up to 40%)
2. Medications (e.g., psychiatric medications, H2 blockers, metoclopramide, verapamil,
estrogen)
3. Pregnancy
4. Renal failure
5. Hypothyroidism
Reference : step up to Medicine
162-Lipid profile of a patient shows high level and patient is on simvastatin What to
add?
Answer: If the patient has high triglyceride level we can add a fibiric acid derivative but
with caution increased risk of myositis
if the patient has persistent high LDL we can add ezetimibe although it has no
mortality benefit but it is an LDL lowering agent
MTB

163- Pt had head trauma , he has nausea, vomiting and decreased level of consciousness ,
ICP was suspected and CT was arranged , what cranial nerve examination can confirm the
Dx ?
A.Optic
B.Oculomotor
C.Trochlear
Answer: A?
Looking for Papilloedema which is swelling of the optic disc
164-Pt with barking cough and 38 temp which of the following symptoms is associated with
this disease ?
A.Cyanosis
B.Wheezing
Answer: A
explanation : scenario of barking cough + fever suggestive of croup which is
associated with stridor and possibly cyanosis
165- KSA have implemented strong regulations regarding worker health cares , which of the
following diseases if the worker had , he can't work ? HeB HeC HIV
Answer: All of them are correct ?

106

166- pt with long hx of uncontrolled HTN , he presented to you with headache and 160/90
BP , what you will see in his kidneys :
A,Decrease sclerosis
B.Increase hyalinization of arterioles
Answer: B
hyaline arteriosclerosis is associated with aging , long standing HTN and DM
167- what is the most specific investigation for TB ?
A. Sputum culture
B. PPD
Answer: A
MTB
168- MI patient within 6 hours what is the most expected complication? :
Answer: Arrhythmias mostly VF
MTB
169- patient has history of MI suddenly he became breathlessness and harsh systolic
murmur heard what the cause :
a- PE
b - rupture aortic cusp
c - tricu regurge
d - rupture
Answer : Acute SOB with High pitched systolic murmur post MI is usually due to
acute Mitral Regurgitation with or without injury to the papillary muscle chordae
tendineae
170-best diagnosis tool for thalassemia is :
Answer : Haemoglobin electrophoresis is the gold standard
Source : uptodate
171- in factor VII deficiency what is the Lab abnormality we will detect?
a- increased pt
b- increased ptt
Answer: A
Explaination: prothrombin time (PT) to measure the functioning of factors I, II, V, VII, and X
partial prothrombin time (PTT) to measure the functioning of factors VIII, IX, XI, XII, and von
Willebrand factors
172- splenectomy case, what vaccines should be given afterwards or prior to spleen removal?
Answer: Spleen acts as a Macrophage to Encapsulated organisms, of those organisms the ones we
should vaccinate with are as follows:
1. pnuemococcal vaccine
2. meningococcal vaccine
3. Hemophilus influenza vaccine

107

173-What is the most accurate test for carpal tunnel syndrome:


A. Tinel
B. Compression test
C. Durkan's carpal test
D. Phallens test
Answer: c
There isnt a single best test for diagnosing Carpal tunnel syndrome, everywhere on
the internet the best method to diagnose has been mentioned to be a proper History
from the patient, we can also diagnose with these methods (electrodiagnostic tests)
which includes both Tinel and Phallen tests, so keep that in mind.
174- Female pt diagnosed with IBD on ceftriaxone with no benefit wt is the organism
- herpes
Answer: ??
Antimicrobial Spectrum:
Staphylococcus aureus (methicillin susceptible), Coagulase negative Staphylococci,
Streptococcus pneumoniae (penicillin susceptible), Streptococcus spp., Haemophilus influenzae,
Moraxella catarrhalis, Neisseria meningitides, Neisseria gonorrhoeae, Enterobacteriaceae, E.
coli
*************** so the answer will be by exclusion of the previous organisms
****************
175-Pt came with cough and she takes an anti-cholesterol medication (statins), she started
it 3 weeks ago, What should the doctor monitor?
A.
Liver function test
Answer: a
explanation: LFT should be carried out before and within 4-6 weeks of starting statin
therapy (1). Thereafter at intervals of 6 months to 1 year - earlier if clinical features of
hepatotoxicity; also at the first review at 4-6 weeks - enquire about adverse effects
such as itching, rash, myalgia, arthralgia, insomnia (1)
if satisfactory lipid control and no evidence of adverse effects then review again
at 4-6 months, then 6-12 monthly
if unsatisfactory lipid control then measurements should be repeated 6 weeks
after dosage adjustments are made until the desired lipid concentrations are
achieved (2)
however NICE state that LFTs only need to be measured on three occasions:
o baseline liver enzymes should be measured before starting a statin.
Liver function (transaminases) should be measured within 3 months of
starting treatment and at 12 months, but not again unless clinically
indicated
o people who have liver enzymes (transaminases) that are raised but are
less than 3 times the upper limit of normal should not be routinely
excluded from statin therapy
treatment should be discontinued if serum transaminase concentrations rise to, and
persist at, 3x normal range

108

176- Single Small thyroid nodule, investigations revealed an Increase in iodine uptake, what
is the best treatment ?
a.
b.
c.

Conservative
Antithyroid drug
Iodine radiotherapy

Answer: in the case scenario if:


If the patient is well and asymptomatic choose A
If the patient is symptomatic mild to moderate choose B
If the patient is severely symptomatic consider C or Surgical removal
Explanation:
Radioactive Iodine uptake test:
Abnormality: The test shows either more or less uptake of tracer than normal in the
thyroid gland. If hyperthyroidism is present, abnormal test results may
mean certain conditions are present.
A low uptake of tracer by the thyroid gland may mean that
hyperthyroidism is caused by inflammation of the thyroid gland
(thyroiditis), taking too much thyroid medicine, or another rare
condition.
A high uptake of tracer spread evenly in the thyroid gland may
mean that hyperthyroidism is caused by conditions such as
Graves' disease.
Treatment of Graves disease that is best will depend on many factors. Antithyroid
drugs and RAI or a mix of both often are preferred..
177- pt k/c of DM with uncontrolled blood sugar.. With figure shows high at the 6am wt to
do?
a- long acting night insulin
b- short acting night insulin
c- long acting morning insulin
d- short acting morning insulin
Answer: A
Explanation:
-We should increase the long acting night insulin as the patient is sleeping for hours and it
wouldnt be ideal to wake him/her up daily in the middle of the night to take medications.
- Cortisol among other counter-regulatory hormones such as epiniphrine, glucagon, or
growth hormone are anti-insulin in the sense that they elevate Blood glucose, Cortisol will
work mainly during the early hours of the morning peaking espacaily around 6 AM.
- Adjusting the long acting insulin night dose should be able to adjust this physiologic
hyperglycemia

109

178- Mycobacterium tuberculosis , that is the best culture media ?


a.
Blood agar
b.
Lwenstein-Jensen medium
Answer: B
Explanation:
Mycobacterium tuberculosis on Lwenstein-Jensen medium after 6 weeks of cultivation,
37C. Typical nonpigmented, rough, dry colonies on Lwenstein-Jensen medium. The
green color of the medium is due to the presence of malachite green which is one of the
selective agents to prevent growth of most other contaminants. Unlike many other solid
cultivation media used in clinical microbiology Lwenstein-Jensen medium (or e.g., Ogawa
medium) doesn't contain any agar (solid consistence is attained by heat coagulation of the
egg albumin).
-While the LJ medium is the most popular means of culturing Mycobacteria, as
recommended by the International Union against Tuberculosis (IUAT), several alternative
media have been investigated.
Solid media
Egg-based Petragnini medium and Dorset medium
Middlebrook 7H10 Agar
Middlebrook 7H11 Agar
Blood based Tarshis medium
Serum based Loeffler medium
Potato based Pawlowsky medium
Liquid media
Dubos' medium
Middlebrook 7H9 Broth
Proskauer and Beck's medium
Sula's medium
Sauton's medium

110

179- What is the best treatment for travellers diarrhea ?


A.
ciprofloxacin
Answer: A
Explanation: Because traveler's diarrhea tends to resolve itself, you may get better without
any intervention. It's important to try to stay hydrated with safe liquids, such as bottled water
or canned juice. If you don't seem to be improving quickly, you can turn to several
medications to help relieve symptoms such as Anti-motility agents, however you can use
Abx to decrease the duration and severity of the disease.

Antibiotics Used for the Treatment of Travelers Diarrhea:

Antibiotic

Dosage

Comments

Ciprofloxacin
(Cipro)

500 mg twice daily for


one to three days

Other quinolones (e.g., ofloxacin [Floxin],


norfloxacin [Noroxin], and levofloxacin
[Levaquin]) are presumed to be effective as
well.

Rifaximin
(Xifaxan)

200 mg three times daily


for three days

Not effective in persons with dysentery

Azithromycin
(Zithromax)

In adults: 500 mg daily


Antibiotic of choice in children and pregnant
for one to three days or
women, and for quinolone-resistant
1,000 mg in a single dose Campylobacter
In children: 10 mg per kg
daily for three days

Source: http://www.aafp.org/afp/2005/0601/p2095.html
180- Cancer of lung with high keratin ?
A.
lung adenocarcinoma
source: http://www.ncbi.nlm.nih.gov/pmc/articles/PMC3155291/
answer: NSLC

111

181- Calcified lesion in the upper lung?


A. Bronchoscope biopsy
B. Percutaneous biopsy
C. Thoracotomy
D. Follow up with serial x-ray
Answer: A
if there is HRCT it is the right answer
http://www.clevelandclinicmeded.com/medicalpubs/diseasemanagement/hematologyoncology/pulmonary-nodules/
182- Renal or small cell cancer stage III with bone pain what is the immediate action?
A. MRI only
B. Radiotherapy
C. Iv steroid and MRI
D. No immediate action
Answer:B
Radiation therapy can also be used to help reduce pain or other symptoms of
metastases in some other places, such as the bones.
http://www.cancer.org/cancer/kidneycancer/detailedguide/kidney-cancer-adult-treating-bystage
183- Pt with 1st metatarsal joint pain , redness and erythema .. High temperatures? What is
the cause ?
A.
Staph aruse
B.
NA monourate crystal
C.
Ca pyrophosphate crystal
Answer: the q is not complete if the diagnosis go with osteomyelitis the answer will
be A
anyway in the link there is Dx of metatarsal joint pain
http://www.mayoclinic.org/diseases-conditions/metatarsalgia/basics/causes/con20022369
184- Pt does not complain of anything, has sudden knee swelling? What is the best thing to
do?
Answer:
Treatment will depend on the cause of the swollen knee, but the most
common ways to reduce the swelling are:
1) Ice: can be used to slow down the blood flow and therefore reduce swelling
and pain. It is important to use it properly otherwise it can make things worse
see the Ice Therapy section for more details and the ice wraps section for the
best ways to apply ice
2)Compression: Tubigrip and knee braces can be used to provide compression
to the knee which helps reduce swelling. Click the links to find out more
3)Medication: Non-steroidal anti-inflammatories e.g. ibuprofen may be
prescribed to reduce the knee swelling
4)Aspiration: Water on the knee can be drained by your doctor with a needle,
but it does sometimes come back
112

5)Cortisone Injections: Cortisone is a steroid hormone that suppresses the


immune system, reducing inflammation and pain
6) Elevation: Keeping the leg elevated, ideally with the knee higher than the
level of the heart can help treat a swollen knee as gravity draws the fluid down
away from the knee
7) Rest: Reducing your activity levels helps to take pressure of the knee which
can reduce swelling otherwise the knee keeps getting irritated
185- Fever and cough then facial nerve then loss of reflexes?
A.Tetanus
B. Botulism
Answer:B
but i think it is most going with Guillain-Barr syndrome ?!
http://www.cdc.gov/nczved/divisions/dfbmd/diseases/botulism/
186- Long ceniro, bloody diarrhea and rbc in urine after 7 days hx of food poisoning, rx?
A. STEROID
B. ANTIBIOTIC
answer: B
As the infection progresses, diarrhea becomes watery and then may become
grossly bloody; that is, bloody to the naked eye. E. coli symptoms also may
include vomiting and fever, although fever is an uncommon symptom.
On rare occasions, E. coli infection can cause bowel necrosis (tissue death) and
perforation without progressing to hemolytic uremic syndrome (HUS)a
complication of E. coli infection that is now recognized as the most common
cause of acute kidney failure in infants and young children.

187- Brucellosis prevention?


a.
Pastrization
answer: A
The best way to prevent brucellosis infection is to be sure you do not consume:
undercooked meat
unpasteurized dairy

products

http://www.cdc.gov/brucellosis/prevention/index.html

113

188- Low ph, bicarb, co2?


A. Compensated metabolic acidosis
answer: NON COM

189- Tb case, what is the next appropriate step to get a definitive dx?
A.
Speutm smear under microscope
B.
Sputum culture
answer:B
The presence of acid-fast-bacilli (AFB) on a sputum smear or other specimen
often indicates TB disease. Acid-fast microscopy is easy and quick, but it does
not confirm a diagnosis of TB because some acid-fast-bacilli are not M.
tuberculosis. Therefore, a culture is done on all initial samples to confirm
the diagnosis. (However, a positive culture is not always necessary to begin or
continue treatment for TB.) A positive culture for M. tuberculosis confirms the
diagnosis of TB disease.
http://www.cdc.gov/tb/publications/factsheets/testing/diagnosis.htm

114

190- Asymptomatic pt. With Positive HBV antigen?


A. Acute hepatitis
B. Chronic hepatitis
C. Active carrier
D. Non active carrier
answer: A/C ??
Transient HBsAg positivity (lasting <18 days) might be detected in
some patients during vaccination.
http://www.cdc.gov/vaccines/pubs/surv-manual/chpt04-hepb.html
hepatitis B surface antigen (HBsAg)
This tests for the presence of virus. A "positive" or "reactive" HBsAg test result
means that the person is infected with the hepatitis B virus, which can be an
"acute" or a "chronic" infection.
hepatitis B surface antibody (HBsAb
A "positive" or "reactive" HBsAb (or anti-HBs) test result indicates that a person
has successfully responded to the hepatitis B vaccine or has recovered from an
acute hepatitis B infection.
hepatitis B core antibody (HBcAb)
A "positive" or "reactive" HBcAb (or anti-HBc) test result indicates a past or
present infection, but it could also be a false positive.
191-Pt. With HBA after 3 week we take biopsy, what is show?
A. Normal architecture
B. Fibrosis something
C. Another something
Answer:

115

192-What is the common disease to make the patient retire in KSA?


A. HBV
B. HBC
C. HIV
D. hep a
Answer: C?
193-Pt with acute rheumatic fever show acute cardiac symptoms, rx?
A.Iv penicillin
B. Im steroid
answer: A?
Depends on the severity of carditis.
Treatment of rheumatic fever : bed rest is essential especially in cardiac involvement.
-Eradication of streptococcal pharyngitis, using penicillin for 10 days OR single dose of IM
benzathine benzylpencillin
Carditis management :
= mild : salicylates for 4-6 weeks then taper the dose and monitor the degree of carditis with
ECHO.
= severe : corticosteroids for 2-4 weeks , if Heart failure develops consider diuretics and digoxin if
necessary.
http://www.medscape.com/viewarticle/406404_5
194-Cushing case which skin manifestation is associated with it
a.
Vitiligo
b.
Telangiectasia
c.
Acropathy
d.
Something Derma
Answer: B
Skin changes In Cushing's syndrome, the skin tends to become thin, fragile, and more
susceptible to bruises and infections. Wounds heal poorly. Wide, reddish-purple streaks,
called striae (stretch marks), can develop in areas of weight gain.
http://www.uptodate.com/contents/cushings-syndrome-beyond-the-basics#H7
195-50 years old patient with third reading of persistent hypertension wasn't started on
medication yet, lab shows high Na of 147 and low k of 3 other parameters were
normal What's most likely the diagnosis?
A.
Essential hypertension
B.
Hyperaldosteronism
Answer: B
http://emedicine.medscape.com/article/127080-overview#a4

116

196-Lab result show high APTT and BT (bleeding time) .. Factor deficiency?
A.
8
B.
7
C.
9
D.
10
Answer: 8
In hemophilia a (F8 deficiency) * Prolonged aPTT *Normal PT and BT. But in Von
Willebrand disease both aPTT and BT are prolonged * Normal Pt
197-A married woman was diagnosed with UTI. Urine culture revealed Staphylococcus
Saprophyticus. What you should ask this patient about?
A.
Use of condom and spermicides.
B.
Alcohol consumption.
C.
Fecal incontinence.
Answer: a
Honeymoon cystitis
Sexual activity increases the risk of S. saprophyticus UTI because bacteria are
displaced from the normal flora of the vagina and perineum into the urethra. Most
cases occur within 24 hours of sex, earning this infection the nickname "honeymoon
cystitis".
S. saprophyticus has the capacity to selectively adhere to human urothelium.
(wikipedia)
Sexual intercourse contributes to increased risk, as does use of a diaphragm and/or
spermicide (medscape)
198- A patient developed rhinorrhea and itching immediately after moving into new
apartment. There was Molds spores in the apartment.
Answer: allergic Rhinitis
199- Which of the following toxins has many uses?
A.
Botulinum
B.
Tetanus
Answer: a
http://www.ncbi.nlm.nih.gov/pmc/articles/PMC1128745/
200- Patient has been diagnosed with Lymphoma in the past and has received full course
of chemotherapy. Now complaining of painless facial swelling, cough and flushing, what is
the diagnosis?
A.
Superior vena cava obstruction
B.
Inferior vena cava obstruction
C.
Some type of facial tumor
Answer: A. SVC obstruction http://radiopaedia.org/articles/superior-vena-cavaobstruction
Superior vena cava (SVC) obstruction can occur from extrinsic compression,
intrinsic stenosis or thrombosis. Malignancies are the main cause and is considered
an oncologic emergency.
Clinical features of acute superior vena cava obstruction include: facial and neck
swelling, facial flushing, bilateral upper extremity swelling, neurological signs,
dyspnoea, headache and cough.

117

201- Which of the following is most likely to be the presentation of a patient with early
STEMI?
A.
Troponin of 0.12 with T inversion in V1-V4
B.
Pathological Q wave with subsided chest pain
C.
Presence of chest pain with 0.3 mg elevation in ST segment in leads 2,3, AVf
D.
ST depression in 2,3, AVf
Answer: C?
1. Normal ECG prior to MI
2. Hyperacute T wave changes - increased T wave amplitude and width; may also
see ST elevation
3. Marked ST elevation with hyperacute T wave changes (transmural injury)
4. Pathologic Q waves, less ST elevation, terminal T wave inversion (necrosis)
o (Pathologic Q waves are usually defined as duration 0.04 s or 25% of
R-wave amplitude)
5. Pathologic Q waves, T wave inversion (necrosis and fibrosis)
6. Pathologic Q waves, upright T waves (fibrosis)
202- Patient with fever, pre-auricular swelling, (description of the swelling was provided), what is
the diagnosis?
A.
Mumps
B.
Pre-auricular lymphadenopathy
Answer: B. Preauricular lymphadenopathy.
203- Patient is allergic to sulfa drugs and penicillin and shellfish. She has UTI what antibiotic you will
give?
A.
Nitrofurantoin
B.
Trimethoprim Sulfamethoxazole
C.
Amoxicillin
Answer: A. Nitrofurantoin
204- pt with bilateral flank pain for 6 months and there is gene 16 mutation , what is the
disease ?
A. Adult polycystic kidney disease
B.
C.
Answer : A
205-man wants to increase awareness about stroke prevention , what will he do ?
A. HTN campaign in mole
B.
C.
Answer:
Repeated, HTN is the most important factor for stroke prevention.
206- pt had MI , he was given sublingual nitrate and analgesia , the MOA of the
analgesic that was given ?
A.
B.
C.
Answer: morphin bind to Mu receptor
side effect of sublingual .....
118

207- which of the following antibiotics is DNA gyrase and it works on what organism ?
A.
B.
C.
Answer: fluroquinolon = pseudomonus

A.
B.
C.

208- enterococcus faecalis antibiotic and the pt is allergic to ampicillin , what to give ?
Vancomycin

Answer: A
209- old pt with recent memory loss and poor self care and social withdrawal , what to
give him ?
A.
Neostigmen
B.
Rivastigmen
C.
Answer: b
210- 29 yrs old female , came for her annual check up , her father was dx with
dislypidemia one year ago and she is anxious about she will have the same thing , he
lab were all normal except for high triglycerides , what will you give her ?
A.
B.
C.
Answer: 6 week trial of diet and exercise if very high not respond start fibrate
211- clear scenario about hyperthyroidism
Q about cardiac markers

Q about thalassemia major ( blood electrophoresis was provided )


119

212- to follow respond to iron therapy , you will order which of the following ?
Reticlocyte
Ferritin

A.
B.
C.

Answer: A
213- pt with low hemoglobin and low MCV , which of the following will confirm the dx ?
Iron level and TIBC

A.
B.
C.

Answer: ferrtin
214- Patient had 1.5 cm calcified lesion in the routine chest x ray . He's symptomless . Next
action will be.
Answer:
a - observation
b- percutaneous biopsy
c- transbronchial biopsy
answer: A
215- Generalized cervical lymphadenopathy + mild tenderness + low grade fever. What's the
most likely diagnosis?
a- small lymphocytic lymphoma ( presented by generlaized lymphoadenopathy)
b- hodgkin's lymphoma
answer: B or EBV
217-

Which of the following is side effect of atropine :

a- vasoconstriction
b- decrease IOP
c- decrease urine output
d- dry of mouth ( my answer )
answer : C and D
218- Which of the following is side effect of morphine:
a- dry cough
b- nausea , vomiting
c- tachypnea
d- anxiety
answer: B
219- medical student diagnosed as meningitis , what you do for him ?
answer: a
a- start antibiotics
b- isolate him for 4 weeks
c- give him influenza vaccine for his colleagues
220old lady with osteoarthritis and risk for osteoporosis , what you will given :
answer: B
a- calcium , TSH , dihydroxy vit d
b- bisphosphonate , vit d , calcium
120

221- which of the following is major criteria of rheumatic fever : #Repeated


a- fever
b- subcutaneous nodule
answer:b

222- bilateral shoulder and hip stiffness and pain what is Dx :


a- polymyalgia rheumatica
b- OA
Answer: A
Polymyalgia rheumatica (PMR) is characterized by aching and morning stiffness in the
shoulders, hip girdle, neck, and torso in patients over the age of 50
Reference: Uptodate http://goo.gl/ZZPjlI
223loss of adduction of fingers caused by injury to:
a- ulnar
b- median
answer: A
Ulnar nerve supplies all the intrinsic muscles of the hand except the first two lumbricals and
muscles of thenar eminence
224patient with numbness of index finger when he scissor .... What is Dx:
a- OA
b- ducyptus
answer: triger finger . tenosynovitis

225patient HTN and hyperlipidemia with chest pain and when he is coming to hospital
take drug relieve his symptoms which drug :
a- nitric oxide
b- digoxin
answer: Nitrates.
121

226patient with chest pain for 6 hours what you will take :
a- tpa
b- aspirin
answer:b. Aspirin improves survival & lowers mortality in patients with acute coronary
syndrome and should be administered immediately upon arrival to the hospital.
Reference: Medescape
227female came from 18 hours flight and she feel leg pain what you will take :
a- warfarin
b- LMWH
c- Unfractionated heparin and warfarin
answer: Unfractioned heparin and warfarin.
Reference: uptodate.
228which heart disease is common in down syndrome :
a- VSD
b- atrioventricular septal defect
c- coartication of aorta
answer:b ASD. Reference: First Aid.
229- patient with murmur in left sternal border change with stand or sitting what is Dx:
a- pulmonary stenosis
b- aortic stenosis
c- cardiomyopathy
answer:incent murmur
Cardiomyopathy. Reference: http://emedicine.medscape.com/article/152913-clinical#b3
230- treatment of cluster headache :
a- 100% oxygen
answer: A 100% oxygen by mask is the abortive thearpy for cluster headache.
Reference: Master the board

231- antidote of opiate :


a- naloxone
answer: A
232which of the following drugs causes hair growth :
a- phenytoin
b- phenobarbital
c- valproic acid
d- carbamazepine
answer: a
234- best exercise to patient with HTN :
a- weight loss
b- aerobic excercise
answer: b
anearobic increase blood pressure
122

235academic professor complaining of headache relieved by 2 tablets of panadol which


type of HTN:
a- malignant
b- essential
answer:b
236patient with OA which type of exercise is the best :
a- high repitition and .........
b- low repition and .......
answer: a?
237- Hypothyroidism pt on dose of thyroxine 75 .. Missed the dose 2 days because he dose not have
the drug , lab result show high TsH and normal t4 .. What dose should be taken ?
a-25
b-50
c-75
d-100
Answer: 75
238- patient has history of mi suddenly he become breahlesness and harsh systolic murmur heard
what the cause :
A) PE
B) rupture aortic cusp
C)tricu regurge
D) rupture .......
answe is: rapture of pappilary muscle
239-MI patient within 6 hours what is the most expected complication : ()
A) ichose PE .
B)
C)
D)
Answer is: arrhthmia
240--renal colic( was easy)
A)
B)
answer is: not complete Q

123

241-- in factor V11 defiency what elevated :


A) pt
B)ptt......
C)
D)
answer is: A
The prothrombin time (PT) is prolonged in factor VII (FVII) deficiency and the international
normalized ratio (INR) is elevated. The activated partial thromboplastin time (aPTT) is within
the reference range in isolated factor VII deficiency.
( http://emedicine.medscape.com/article/960592-workup )
242Pt with past hx of hodgikon lymphoma .. But cured completely .. Presented with
back pain ... Examination and evaluation show paraspinus Edema and fluid collection -ve
burecella titer and tuberclin test ,, what the cause ?
A)burecellosis
B) breast cancer
C) recurrent hodgikon lymphoma
answer: B
243- Ischemic heart pt with lap result of high lipid .. What the next order ?
TFT
LFT
answer: B

244- Pt dose not complain of anything ,, has sudden knee swelling ? What is the best
thing to do ?
answer: xray
245scenario about tb and the pt take 4 medication for Tb he develope numbness in
hand and feet
A.
rifmpacin
B.
INH ( the answer )
C) ethambutol
answer: b
246Pt with MI and take morphine (not sure about route of administration ) What is the
side effect of morphine
dry cough
tachypnea
anixity& .....
answer: nausia and vomiting
( side effects of morphin: http://reference.medscape.com/drug/ms-continastramorph-morphine-343319#4 )

124

247-pt with hodgen lymphoma with aur reeds no fibrosis There is eosinophils bsao .....
What is the of hodegen lymphoma?
The answer is Mixed type
answer: mixed
248-diagram about ( window period) Ask about +ve antigen in this period?
answer:HBcAb and HBeAb
249effect of metformin on DM ?
a.
Decrease hepatic gluconeogenesis
answer:A
Action of Metformin :
1.Decrease insulin resistance.
2.Decrease appetite.
3.Increase anerobic glycolysis
At molecular level:
1- increase intracellular adenosine monophosphate (AMP)
2- Reduce adenosine triphosphate (ATP)
3- activate the intracellular energy sensor AMP-activated protein kinase (AMPK)
(Davidsons principles & Practice of Medicine 22nd edtion)
250-

prophylactic of rheumatic disease Normal pt ?

answer: pencillin

251- long Scenario about old male bedridden on foly's catheter he develop Gram ve bacteria what is the organism
A.
E.coli
B.
pseudomonas aergonsa
C.
strep. Puomonia
answer: a
Enteric pathogens (eg, Escherichiacoli) are most commonly responsible
( http://emedicine.medscape.com/article/2040035-overview )

125

252- long scenario about pt coming from Africa 3 weeks ago with fever no other +ve points
What is the Dx?
Ebola
yellow fever
answer: 18
253-mode of inheritance for Wilson's disease
The answer :- autosomal recessive

254-best screening method for prostate cancer?


a) digital rectal examination
b) cytology for prostate cancer
c) specific prostate antigen
answer: Digital rectal examination
First Aid
255Ttt of traveler's diarrhea ?
answer: ciprofloxacin
256Drug uesd in break down thrombus ?
answer:
257Lap result show high aptt and bleeding time .. Factor deficiency?
8
7
9
10
answer: 8
258Q about pathophysiology of DM 1
I Don't remember the options
answer: 16

A.
B.
C.

259-the main tx for Kawasaki :


IVIG
Aspirin
Answer: A
Cost-benefit analysis reveals that IVIG treatment of KD is one of the most costeffective medical therapies available, leading to tremendous short- and long-term
savings
UPTODATE
260- what medication that cause hypertensive crisis with Tyramine
A. MAOI
B. TCA
Answer: A
126

261- dog bit , what is the most common organism ?


A. Rabies
B.
Yersinia pestis ( rodent - plaque )
C.
Toxoplasma gondii ( cat )
Answer: A
Pasteurella ( 50% )
strept 46% , staph 46%
neiserria 32%
corynebasterium 12%
Bite wound infections are usually polymicrobial, with a mix of animal oral flora, recipient skin flora
and environmental organisms. The most common pathogens in dog bites arePasteurella spp. (both
Pasteurella multocida and Pasteurella canis), Staphylococcus and Streptococcusspp., and the
fastidious Gram-negative rod Capnocytophaga canimorsus. The dog is the most common transmitter
of rabies to humans worldwide, with greater than 95% of reported cases being due to these
animals.Reference:
http://www.medscape.com/viewarticle/739023_4
262- -Post MI, patient develops Atrial Fibrillation. Which medication is required to prevent stroke?
A) Warfarin
B) Heparin
C) Aspirin
D) Dabigatran
Answer: A
Taking an anticoagulant (blood thinner) can reduce the risk of having a stroke by approximately 50 to 70
percent.
Reference:
http://www.uptodate.com/contents/atrial-fibrillation-beyond-the-basics#H17
263- Asthma medication question. Patient on SABA PRN, symptoms not controlled. What to add?:
A) Inhaled Corticosteroid
B) Long acting beta agonist
C) Iatropium Bromide
D) Steorid Oral
Answer: A
Reference:
http://www.nhlbi.nih.gov/health-pro/guidelines/current/asthma-guidelines/quick-reference-html

127

264- Which of the following renal stones is associated with infection and alkaline urine?
A- Calcium oxalate.
B- Uric acid.
C- Cystine.
D- Struvite.
Answer: D
magnesium ammonium phosphate (MAP)
alkaline urinary pH due to infection with urea-splitting organisms precipitates MAP
Proteus, Pseudomonas, Providencia, Klebsiella, Mycoplasma, Serratia, S. aureus, NOT E. coli
perpetuate UTI because stone itself harbours organism, therefore must remove stone to cure
infection stone and all foreign bodies must be cleared to avoid recurrence associated with staghorn
calculi.
Reference: Toronto Notes
265- Patient with recurrent UTI. Bilateral hydronephrosis. Renal biopsy showed multiple cysts with some
description, what is the diagnosis?
A-Renal cell carcinoma.
B-Polycystic kidney disease.
C-Renal dysplasia.
Answer: B
Approximately 30 to 50 percent of patients with autosomal dominant polycystic kidney disease (ADPKD)
will have a urinary tract infection (UTI) during their lifetime.
Reference: http://www.uptodate.com/contents/urinary-tract-infection-in-autosomal-dominantpolycystic-kidney-disease
266-Patient is referred to Orthopedics because of abnormally high bone density scan. She has no history
of fractures. She takes analgesics and is waiting for hip replacement surgery. Her dietary calcium and
serum biochemistry are normal. What is the the cause of her high density result? (no numbers were
provided).
A- Osteoarthritis.
B- Pagets disease.
C- Osteoporosis.
D- Osteopenia.
Answer: B
Paget disease is a localized disorder of bone remodeling that typically begins with excessive bone
resorption followed by an increase in bone formation. This osteoclastic overactivity followed by
compensatory osteoblastic activity leads to a structurally disorganized mosaic of bone (woven bone),
which is mechanically weaker, larger, less compact, more vascular, and more susceptible to fracture
than normal adult lamellar bone.
Reference: http://emedicine.medscape.com/article/334607-overview
128

267- Patient has high Alkaline Phosphatase, in order to confirm the hepatic origin of this elevation the
doctor ordered one more test. What is it?
A- Lactate dyhydrogenase.
B- Creatine kinase
C- Gamma-glutamyl transpeptidase
D-Lipase
Answer: C
The gamma-glutamyl transferase (GGT) test may be used to determine the cause of elevated alkaline
phosphatase (ALP).
Both ALP and GGT are elevated in disease of the bile ducts and in some liver diseases, but only ALP
will be elevated in bone disease. Therefore, if the GGT level is normal in a person with a high ALP, the
cause of the elevated ALP is most likely bone disease.
Reference: https://labtestsonline.org/understanding/analytes/ggt/tab/test/
--268- clear scenario about RT
--269--exercise in pt e CAD Iso
A.
Anaerobic
B.
Iso
C)
D)
Answer: Isotonic exercise and aerobic

Additional Questions 2 ( 7th update)


1)Which of the following transmitted through Un cooked meat ?
a. Entameba Coli
b. Entamebea.h
c. Teania
(Important foodborne parasites: Trichnella (undercooked pork), Tinea saginata
(undercooked beef), Tinea solium (undercooked pork), Toxoplasma gondii (undercooked
meat).)
Answer: C. Refernce: CDC
2) Pt post hystrectomy and oophorectomy to be started on HRT how you gonna give ?
Cyclic estrogen & progestrone
Continuous estrogen & progesterone
Estrogen alone
Answer: C, Post-Hysterectomy pts have no uteri so no point in giving PROGESTERON to
prevent endometrial hyperplasia. UpToDate.
129

3) Pt with pic of osteoarthritis on examination there is nodule in the tips of the finger the
name of the finding?
- heberden
boutchared -?? -??
Answer: A
Note:
OA PIP nodules- Boutchard (The P/B sound )
OA DIP nodules: Heberden.
Reference: UpToDate
4) pt with testicular mass after exesion you took a sample and see it under microscope it
showed (I can't recall) what is the type of malignancy....? - seminoma - yolk sac - ???
Inadequate information.
Info:
Histology
o seminoma
large cells in lobules with watery cytoplasm
"fried egg"
analogous to dysgerminoma of the ovary
o embryonal carcinoma
glandular/papillary
o yolk sac (endodermal sinus) tumor
yellow, mucinous
Schiller-Duval bodies resemble primitive glomeruli
o choriocarcinoma
disordered syncytiotrophoblastic and cytotrophoblastic elements
o teratoma
contain three tissue types
o leydig cell
contains Reinke crystals
Reference: UsmleBullets.com
5)ovarian follicular cell origenate from ?? No answers for this one.
OVARIAN Follicular cells (granulosa cells) develop from the secondary SEX CORD
(which develops in gonadal ridge).
Reference: Embryology websites, Basic embryology
6) Lung biopsy showed (something)what is the diagnosis ?? Something!
Not enough information.
7) if a breast lump felt by self exam for how many years it's there ?? - 1 -2 -4
4 years.
Source: No clear source. References generally indicate that it is between 2 and 5 years.
Mammograms can detect breast CA 2 years before being felt as a lump.
8) Pt with HbA1c within pre diabetic range .when to repeat the test ? - 3months -6 months 1 year - ??
1 Year.
Reference: American diabetes association guidelines: At least annual monitoring for the
development of diabetes in those with prediabetes is suggested. E
130

9)most common cause of epistaxis in children ?


- self-induced trauma
- polyps -??
Answer: A.
Reference: Medscape
10) most common cause of central line infection
- during the insertion
- migration of bacteria from other site
- while giving the medication through the line
Answer: A.
Reference: http://www.uptodate.com/contents/epidemiology-pathogenesis-andmicrobiology-of-intravascular-catheter-infections
1-which is Hinge Joint? shoulder elbow ankle knee
Elbow. An another hinge joint example is the interphalangeal joints. Snell Anatomy
Ankle joint formed of ?
The tibia and fibula of the leg, and the talus. Snell Anatomy
3-IHD or (DM +HTN) with BPH ? medication? THE CHOISES WERE : 2 types of ACE 1
BETA BLOKCKER 1 CHOICE OF I DO NOT KNOW THE OPTIMAL RX IS ALPHA
BLOCKER (Prazocin Etc).
Answer: Alpha blockers are preferred for such a scenario (HTN & BPH)(UpToDate,
Medscaperetc). In practice, a prostate selective alpha blocker like Tamsulsin is used for
BPH along with 1st line Antihypertensive drugs for HTN (ACE/ARB, CCB, Thiazide.) If no
Alpha blockers in the answers, then avoid BB because they mask the symptoms of DM.
ACEI would be the first choice especially that they are first line in the new HTN guidelines
(JNC8) plus the fact that they are useful for diabetic nephropathy, and they are not diuretics
(BPH).
4- MOST COMMON CAUSE OF HTN IN SAUDI ARABIA.
Essential HTN.
5- MOST COMMON INFECTIOUS DISEASE AMONG MEDICAL STAFF IN SAUDI
ARABIA
HBV
6- LONG SCENARIO OF ORTHOPAEDIC SURGERY HAD DONE TO PT THEN AFTER
HE DEVOLVED AN INFECTION, WHAT COMMON CAUSE?
CONTAMINATED HANDS OF SURGEONS DURING EXAMINATION
CONTAMINATED TOOLS
Answer. ?
In the majority of surgical site infection cases, the pathogen source is the native flora of the
patients skin, mucous membranes, or hollow
viscera. http://www.ncbi.nlm.nih.gov/pmc/articles/PMC2812878/

131

7- LONG SCENARIO OF PREGNANT IN 1ST TRIMESTER WHILR CHICK UP SHE HAD


HIGH BP IN NEXT VISIT HIGH BP BUT LOWER THAN THE 1ST VIZIT, DX ?
ESSINTIAL HTN
GASTENTIONAL HTN
CHRONIC HTN
PREECALMPCIA
Answer: C. Chronic HTN
Source: Medscape http://emedicine.medscape.com/article/261435-overview#a7
8-SENARIO OF ATYPICAL PNEUMONIA, CAUSATIVE ORGANISM ?
MYCOPLASMA PNEUMONEA
Source: Medscape
9-GLUTEAL MUSCLE INSUFECINCY ? INFER& SUPERIOR GLTEAL ARTERY >>
INTERNAL & EXTERNAL ILIAC ARTERY .
Answer: ? Deficient information.
10-INGUNAL LEMPHEDENOPATHY in the medial part of the transverse group ? ANAL
AREA below pectinate line.
11-SCENARIO OF LEUKAEMIA WITH AUER RODS ? AML
12-DM WITH UTI, CLTURE SHOWN, ANTISEPTIC ? PSEDOUMONAS.
True. Pseudomonas is known to persist in hospital solutions and antiseptics.
13-SUBMENTAL PAIN, TENDER, PALPABLE---- ? SUBMENTAL Calculi (Sialolithiasis)
14-Scenario OF PULSELESS, COLD LESION IN THE FOOT ? Arterial INSUFENCI.
Medscape PAD presentation.

1- fracture at wrist involved both ulnar and radius bone with 1cm injured skin : A- closed
reduction and apply cast below elbow joint. B- closed reduction and apply cast above elbow
joint. C- open fixation .... D- open fixation ..... ( forget )
2- patient has fracture shoulder with winged scapula what is the nerve affected ? A- anterior
B- posterior C- roots.
(Anterior Rami of 5,6,7)
3- side effect of anticholinergic. Hot as a hare: increased body temperature
Blind as a bat: mydriasis (dilated pupils)
Dry as a bone: dry mouth, dry eyes, decreased sweat
Red as a beet: flushed face
Mad as a hatter: delirium

5- constant defect in Von wel brand : A- pt B- Ptt C prolonged bleeding D- factor VIII.
Answer: Factor VIII activity
Refernce: Cecil Medicine Table in Hemorrhagic Dz.
132

6- child with jaundice and kieser ring in iris.


Answer: Wilson Disease.
7- allergy to peanut butter what is the mechanism ?
(Not written from the original writer, but it is type I IgE mediated hypersensitivity, initial
sensitization with the Ag passing through the GI tract)
8- indirect hernia , location of the sac to the cord ?? Anteriomedial.
9- pt with dysphagia decrease MANOMETRY in pharynx and upper esophagus what is the
diagnosis?
No options for this one.
Possible causes: Mysthenia graves, stoke, or Dermatomyositis, Parkinsonetc
10- synthesized inside the nucleus : A- DNA B- m RNA C- t RNA Other types of RNA.
mRNA. DNA is not in the nucleus in the metaphase.
11- Fracture of head and neck of fibula which artery is affected? - deep something superficial something
- Superior segment of Circumflex fibular artery which comes from posterior tibial A.
- Also anterior tibial A passes medial to fibula neck.
* If the question is asking about the nerve -_-, the nerve is common peroneal nerve.
12- DM chohort study during 3 years of 10.000 DM pts , number increase about 2000 pt at
the beginning of the study and 1000 at the end , how much percent increase ?? A- 10.2 B12.2 C- 20.2 D-22.2
Unclear scenario
13- post coital bleeding + vaginal discharge ( no special color or odor was mentioned ) Atractio .... B- vaginosis C- candida
Answer: A (Chlamida Tachomatis) UpToDate: Postcoital bleeding, The most common
etiology is chlamydial infection(cervicitis), 2nd most common cause is Bacterial vaginosis.
14- gluteus muscle supplied by : A- internal iliac B- external iliac C- femoral
15- frontal bone mass which lymph node going to be enlarged ? A- mastoid B- sub mental
C- sub mandible D- parotid
Answer: Submandibular.
Source: Google pictures!
16- which drug will increase cholecystitis pain: A- acetaminophen B- morphine. Theoretical
increase in biliary colic by causing sphincter of Odi spasm using morphine? Not found to be
significant in studies, old studies prefer pethidine (meperidine) because it has an atropine
like effect on sphincter of odi.
17- child or neonate diagnosed with UTI which next step before treatment ? A- urethral
culture B- cystoscopy C- catheter drainage D- Renal US
Answer: D
Refernce http://www.uptodate.com/contents/urinary-tract-infections-in-neonates#H14
133

18- which type of thyroid cancer will responds to radioactive therapy ?


19- pt with epistaxis ( they describe the site and asked about the artery affected ) Vague
description.
Kiesselbach's plexus, in Little's area, is a region in the anteroinferior part of the nasal
septum where four arteries anastomose to form a vascular plexus of that name. The
arteries are
Anterior ethmoidal artery (from the ophthalmic artery)
Sphenopalatine artery (terminal branch of the maxillary artery)
Greater palatine artery (from the maxillary artery)
Septal branch of the superior labial artery (from the facial artery)
Posterior ethmoidal artery

20- pt has renal stenosis next step of investigation after angiography?


Angioplasty with stenting?
Refernce: Medscape
When renal function is normal or nearly normal, specialists recommend revascularization
for prevention of renal insufficiency if the patient meets the following criteria:
The degree of stenosis is more than 80-85%
The degree of stenosis is 50-80%, and captopril-enhanced scintigraphy demonstrates
an activation of intrarenal renal artery stenosis

38 years old female had amenorrhea for two months after 1 year of irregularity of the
menstrual cycle.she is a mother of 3 children, he has one history of dilatation and curettage
after Cesarean section. she have thinning in the vaginal secretion labs were done and
shows normal finding except high FSH & LH low estrogen radiology revealed normal uterus
and cervix what is the diagnosis A)premature failure of the ovary B)asherman's syndrome
c)androgenic cause
Answer: A
Refernce: http://www.ncbi.nlm.nih.gov/pmc/articles/PMC1502130/

134

There were also missing questions about the following:


Corona virus
http://www.uptodate.com/contents/middle-east-respiratory-syndrome-coronavirus
Ebola virus
http://www.uptodate.com/contents/clinical-manifestations-and-diagnosis-of-ebola-virus-disease
Alport disease
hereditary nephritis (Alports disease): X-linked nephritis often associated with sensorineural hearing loss;
proteinuria <2 g/d
DIC
Addison disease.
Cardiomyopathy
Iron deficiency with koilonychia.
Types of bacteria and antibiotics
SIADH
Osteoarthritis
Guillain barre syndrome
Liver cirrhosis
HBV
Pneumothorax

135

Surgery

136

1- Patient had injury to the ulnar nerve?


a.
Complete claw hand
b.
Partial claw
Answer: B
Effects of the ulnar nerve injury: ulnar claw hand (Partial claw hand) and hollowing of skin in the first web space on
dorsal aspect of hand.

Complete claw hand results from combined lesions of the median and ulnar

Reference: Textbook of Anatomy - Upper Limb and Thorax


2- Pronator teres syndrome, which nerve is entrapped:
a.
Ulnar
b.
Radial
c.
Median
Answer: C
Reference: https://www.youtube.com/watch?v=ZqhO1dzqTtY
Pronator teres syndrome is a compression neuropathy of the median nerve at the elbow. It is rare compared to
compression at the wrist (carpal tunnel syndrome) Reference: Wikipedia.

137

3- Basal skull fracture with loss of sensation under the eye. Which of the following nerves is affected?
a.
frontal
b.
trochlear
c.
infraorbital
d.
supraorbital
Answer: C

4- 16 years old female complaining of abdominal pain for 2 days. It started in the periumbilical area then it radiated
to the right lower quadrant. It is associated with anorexia. On examination, right lower mass was felt. What is the
management?
a.
surgery immediately
b.
conservative
*Antibiotics werent mentioned in the choices.
Answer: ?
Appendectomy remains the only curative treatment of appendicitis, but management of patients with an appendiceal
mass can usually be divided into the following 3 treatment categories:
Patients with a phlegmon or a small abscess: After intravenous (IV) antibiotic therapy, an interval appendectomy
can be performed 4-6 weeks later.
Patients with a larger well-defined abscess: After percutaneous drainage with IV antibiotics is performed, the
patient can be discharged with the catheter in place. Interval appendectomy can be performed after the fistula is
closed.
Patients with a multicompartmental abscess: These patients require early surgical drainage.
Reference: Medscape.
5- The most common cause of breast bloody discharge:
a.
ductal papilloma
Answer: A
6- A mass at the middle of the Esophagus:
a.
Adenocarcinoma
b.
Squamous
Answer: B
We have two types of Esophageal CA:
Adenocarcinoma usually at the GE junction, due to chronic reflux that cause dysplasia.
Squamous cell carcinoma in most of the esophagus
th
Reference: Surgical Recall, 6 edition

138

7- Which of these supplements protect against colorectal cancer?


a.
folic acid
b.
vit E
Answer: A
Reference:http://www.hopkinscoloncancercenter.org/CMS/CMS_Page.aspx?CurrentUDV=59&CMS_Page_ID=1293D6
14-71B1-4A5A-8CFD-7BF8760295FA
8- Median nerve injury leads to:
a.
Complete claw hand
b.
Ape hand
c.
Partial claw
d.
Ulnar Claw
Answer: B
Reference: Textbook of Anatomy - Upper Limb and Thorax
9- Cancer of the lower lip. What are the lymph nodes that you will examine first?
a.
submental, submandibular.
b.
Submental, Buccal.
c.
Buccal, parapharyngeal.
Answer: A
The lymphatic drainage of both the upper and lower lips is primarily to the submandibular group of lymph nodes. To a
lesser extent, drainage may go to submental intraparotid, or internal jugular lymph nodes.
Reference: http://www.ncbi.nlm.nih.gov/pubmed/8460042
10- A patient with achalasia. Repeated attempt of balloon expansion relapses again. What is the management?
a.
stent insertion
b.
myotomy
c.
nasogastric tube
Answer: B
11- Patient with chronic osteomyelitis of her leg and there is sinus draining. She has cancer of skin near the sinus
which has has eosinophilic inclusions. What is the type of cancer?
a.
Squamous cell carcinoma.
b.
Basal cell carcinoma.
Answer: A
De novo squamous cell carcinoma emerge in the setting of long-standing ulcers, burn scars, or osteomyelitis.
Reference: Journal of Skin Cancer.
12- A patient was not able to move her arm above shoulder after breast CA surgery that involved axillary
dissection. What is the nerve injured?
a.
long thoracic nerve
b.
rotator cuff
c.
supraspinatus
d.
brachial plexopathy
Answer: A
We have four nerves must the surgeon be aware of during an axillary dissection:
1. Long thoracic nerve
2. Thoracodorsal nerve
3. Medial pectoral nerve
4. Lateral pectoral nerve
th
Reference: Surgical Recall, 6 edition

139

13- Patient underwent orthopedic surgery. 2 days after the surgery he started to complain of swelling, erythema and
tenderness at the site of surgery. Which organism can cause the wound infection?
a.
Clostridium perfringens
b.
Other choices included different subtypes of clostridium
Answer: A
C. perfringens is the only Clostridium species can cause wound infection (Gas gangrene; A medical emergency)
14- A 32 years-old alcoholic male patient is brought to the emergency department with the history of vomiting large
amount of bright red blood. Physical examination revealed splenomegaly and ascites. Which of the following is the
most likely source of bleeding?
a.
Duodenal ulcer.
b.
Proton pump.
c.
Esophageal varices.
d.
Gastric cancer.
Answer: C
15- A patient with indirect inguinal hernia. What artery runs medial to it?
a.
inferior epigastric artery
Answer: A
16- What is the commonest nerve injured after thyroidectomy
a.
recurrent laryngeal nerve
Answer: A
17- Patient came with abdominal swelling and weak abdominal wall. Whats the diagnosis?
a.
direct inguinal hernia
Answer: A
18- Winging scapula:
a.
long thoracic nerve injury
Answer: A
19- DVT in pregnant. What will you do?
a.
Compression ultrasound
Answer: A
20- Obturator nerve injury. Which muscle will get full paralysis?
a.
Adductor Magnus
b.
Adductor longus
Answer: B (Adductor longus)
NB. Adductor magnus has nerve supply from 2 nerves.

140

21- 23 years old woman with cyclic bilateral nodularity in her breast since 6 months. On examination there is tender
3 cm mobile subareolar mass on her right breast. What will you do next?
a.
FNA with cytology
b.
Mammogram
c.
Biopsy
d.
Follow up for next cycle
e.
Observation
Answer: ?
Ultrasound is the initial test to determine if its a simple benign cyst or complex that are suspicious of breast carcinoma.
If it is suspicious it should be examined surgically or with an US guided biopsy. Confirmation can be made by FNA but
bloody fluid may be an indication for a biopsy.

Reference: https://books.google.com.sa/books?id=rkT3HWNSBP4C&pg=PT1027&dq=#v=onepage&q&f=false
22- Long thoracic nerve damage with winging of scapula, this nerve arises from which part of the brachial plexus?
a.
Upper trunk.
b.
Posterior cord.
c.
Medial cord.
d.
Root
Answer: D
Root: C5, C6, C7
23- Pectoralis major muscle was removed during mastectomy. Which action will be lost?
a.
Abduction
b.
Adduction
c.
Extension
d.
Internal rotation or flexion
Answer: B
adduction and medial rotation

141

24- A patient presented complaining of dark stool and vomiting blood. He is stable now. What is your investigation?
a.
Upper GI endoscope.
Answer: A
Mostly from upper GIT bleeding
25- In which position will you apply a splint in patients with carpal tunnel syndrome.
a.
Dorsiflexion.
b.
Plantarflexion
c.
Extension
d.
Abduction
Answer: A
The ideal position of the wrist is close to neutral with 2 + 9 of dorsiflexion and 2 + 6 ulnar deviation to prevent
prolonged flexion or extension that prevents any raise in the pressure.
Reference: Occupational Therapy for Physical Dysfunction.
26- Which one of these layers is involved in chagas and hirschsprung disease?
a.
muscularis externa
b.
mucosa
c.
submucosa
Answer: A
Hirschsprung disease (congenital megacolon) and Chagas disease have different etiologies, but both inhibit intestinal
motility by affecting the myenteric (Auerbach's) plexus located between the layers of the muscularis externa.
Reference: Berkeley CourseHero.
27- Child was crying since 4 hours ago. He has fever and vomiting. On examination there was a mass on the RUQ.
What is your diagnosis?
a.
Appendicitis
b.
Gastroenteritis
c.
Pancreatitis
d.
Intussusception
Answer: D
28- Treatment of hirschsprung disease
a.
Surgical repair
Answer: A
29- 40 years old woman with no pain but you noticed jaundice. She has high direct bilirubin and high ALT what is
your diagnosis?
a.
Gilberts disease
b.
Biliary stone
Answer: ?
- Gilberts disease classified as a prehepatic (associated with high indirect bilirubin)
- Biliary stone (It can be the answer to this Q, exclude other DDx first): asymptomatic (80%), Biliary colic (10 - 25%, painfull).
- Other DDx: pancreatic head CA, Cholangiocarcinoma and primary biliary cirrhosis

142

30- You did rectal examination to someone and found there is a mass in front of anterior rectal wall which was
diagnosed as adenoma. What is the location of this neoplasm?
a.
Anterior
b.
Posterior
c.
Lateral
d.
Median
Answer: B?
The prostate is divided into lobes:
The anterior lobe is the portion of the gland that lies in front of the urethra. It contains no
glandular tissue but is made up completely of fibromuscular tissue.
The median or middle lobe is situated between the two ejaculatory ducts and the urethra.
The lateral lobes make up the main mass of the prostate. They are divided into a right and
left lobe and are separated by the prostatic urethra.
The posterior lobe is the medial part of the lateral lobes and can be palpated through
the rectum during digital rectal exam (DRE).
Reference: http://www.cdc.gov/cancer/npcr/pdf/abstracting/prostate.pdf

31- A patient with diaphragmatic hernia and you want to cut the phrenic nerve. Where can you do that?
a.
Anterior to scalenus anterior
b.
Anterior to scalenus medius
c.
posterior to scalenus anterior
d.
posterior to scalenus medius
Answer: A

143

32- 24 years old girl came to you with a painless mobile breast mas that does not change with menstrual cycle. It
started increasing since 3 months. Now its 3 cm in size. What is the most likely cause?
a.
Fibroadenoma
b.
Fat tissue
c.
Cyst
Answer: A
Fibrocystic: changes with menstrual cycles (hormones), lumps in both breasts
Fibroadenomas. These are the most common benign tumors. They are solid, round, rubbery lumps that move
freely. Theyre usually painless.
Simple cysts: Simple cysts are fluid-filled sacs that usually happen in both breasts. There can be one or many. They
can vary in size. Tenderness and size often change with menstrual cycle.
Intraductal papillomas: These are small, wart-like growths in the lining of the mammary duct near the nipple. They
usually affect women who are 45 to 50. They can cause bleeding from the nipple.
33- They want to stop screening for breast cancer for women under 47 to decrease unnecessary anxiety to the
public. You didnt agree because there is a gene that cause cancer in young women. What is the gene?
a.
APC
b.
BRCA2
Answer: B
BRCA1 or BRCA2 mutation.
34- What will tell you tell a young lady in regard to breast ca?
a.
Self exam is obsolete now
b.
Self exam as well as mammography are important
c.
Only mammography
d.
CT scan
Answer: B (Were not sure about the guidelines they are following)
35- A patient with thenar atrophy but with no numbness. What is the nerve affected?
a.
Axillary
b.
Ulnar
c.
Median (Anterior interosseous nerve)
d.
Radial
Answer: C
Thenar atrophy is an eroding of muscle tissue, which can impair control over the thumb and leave the hand disfigured.
The problem can be a complication of several different conditions and disorders, including carpal tunnel syndrome,
acromegaly, or direct trauma to the wrist or thumb.
36- 12 years old male with undescended testes. The surgeon excised it. What is the most likely histology?
a.
Malignant transformation
b.
Normal histology
Answer: ?
6 % of testicular tumors develop in patients with a history of cryptorchidism (undescended testes).
th
Reference: Surgical Recall, 6 edition
37- 45 years old female presented with neck swelling and anxiety. On examination the swelling is moving with
swallowing and lateral to midline. What is the most likely diagnosis?
a.
Thyroglossal cyst
b.
Branchial cyst
c.
Thyroid nodule
Answer: C

144

38- Which of the following suggest ductal papilloma on breast examination?


a.
Blood stain on nipple
b.
Hemorrhagic discharge
c.
Serous discharge
d.
Pus from nipple
Answer: A
39- A patient came to your clinic with inflammation, redness and swelling around the nail. You gave 1 week course of
augmentin but of no use. What should be done? (Picture was also attached)
a.
give augmentin for one more week
b.
incision and drainage
c.
warm soaks
d.
splint
Answer: B
40- What is the best study to determine cystic breast lesion?
Answer: ?
US is the best to evaluate cystic lesions
41- You need to put cannula in the great saphenous vein. Where can you find its branch?
a.
in front of medial malleolus
b.
in front of lateral malleolus
c.
behind medial malleolus
d.
behind lateral malleolus
Answer: A
42- A patient presented with hard mass on the outer upper area of the breast. which lymph node you have to
examine?
a.
posterior axillary
b.
anterior axillary
c.
lateral axillary
d.
medial axillary
Answer: B
Anterior (pectoral) group: from the lateral quadrants of the breast.
43- What is the most common site of Meckels diverticulum?
a.
lower ileum
Answer: A
Most (75%) Meckel diverticula are found within 100 cm (two feet) of the ileocaecal valve
44- What is the most common site of breast mass?
a.
superior and lateral
Answer: A
Approximately one half of breast cancers develop in the upper outer quadrants.
th
Reference: Surgical Recall, 6 edition

145

45- Case of burn (Know how to calculate the BSA and how to determine the degree of burn).

Reference: Toronto Notes


46- Patient with adrenal mass on examination. What is the next step before you do FNA?
a.
renin/aldosterone ratio
b.
dexamethasone stress test
c.
free metanephrines
Answer: C
Pheochromocytoma should always be suspected to avoid the risk of lethal hypertensive crises, especially during biopsy
or surgery. Plasma-free metanephrines provide the best test for excluding or confirming pheochromocytoma.
Reference: http://www.medscape.com/viewarticle/442383_3

47- A patient presented with lymph node enlargement on the horizontal line of the inguinal ligament. What is the
structure that you must examine?
a.
anal canal
b.
gluteal folds
c.
anterior medial leg
d.
anterior medial thigh
Answer: A
Superficial inguinal lymph nodes: Anal canal (below pectinate line), skin below umbilicus (except popliteal territory),
scrotum. Reference: FA USMLE step1

48- Patient complaining of pain in the 2nd digit. Which tendon is affected?
Answer: ?

146

49- Patient with abdominal pain and fever. Lab showed high amylase. What is the diagnosis?
a.
Acute pancreatitis
Answer: ?
50- Patient had Upper GI bleeding. What is the diagnosis?
a.
Zollinger ellison syndrome
Answer: ?
Read about Upper GI bleeding.
51- A patient presented with RLQ pain, guarding, tenderness and positive obturator sign. What is the most likely
diagnosis?
a.
Appendicitis.
Answer: A
52- Cancer associated with Smoking?
a.
Bladder
b.
Colorectal
Answer: A
53- A patient presented with splenic injury after abdominal trauma. The surgeon decided to embolize the splenic
artery. Which of the following will be compromised?
a.
Stomach fundus
b.
Splenic flexure
Answer: A
Reference: https://en.wikipedia.org/wiki/Short_gastric_arteries
54- A patient with a stabbed wound to the Gluteus. Examination: The patient tilt to the unaffected side while
walking. Which nerve is affected?
a.
Femoral N
b.
Obturator N
c.
Superior Gluteal N
d.
Inferior Gluteal N
e.
Peroneal N
Answer: C
Superior gluteal nerve is a nerve that originates in the pelvis and supplies the gluteus medius, the gluteus minimus
(abductor muscles), and the tensor fasciae latae muscles. (Trendelenburg gait)

147

55- 2 years old with crampy abdominal pain, bleeding per rectum. What is your next step?
a.
barium enema
Answer: A (Its a case of intussusception, there were more details in the history)
56- 40 years old male patient work on the computer a lot. After that get tingles around lateral aspect of the hand.
What is the most accurate test?
a.
Tinels sign
b.
Phalen Test
c.
Durkan's carpal test
d.
Compression
Answer: C
Durkan: sensitivity: 89% and specificity: 96%. It is considered by some to be a better provocative test than tinel and
phalen tests.
Reference:https://books.google.com.sa/books?id=D4mQCgAAQBAJ&pg=PA110&dq=tinel+phalen+compression&hl=en
&sa=X&redir_esc=y#v=onepage&q&f=false
56- A patient had appendectomy. The artery that supplies the appendix is a branch from which of the following?
a.
Superior epigastric Artery
b.
Inferior epigastric Artery
c.
Superior Mesenteric Artery
d.
Inferior Mesenteric Artery
Answer: C
The appendicular artery (appendiceal artery) is a terminal branch of the ileocolic artery which is a branch of Superior
mesenteric artery. Reference: Wikipedia.

58- A patient had prostate CA and it was removed. Now he presented with metastasis. What is the most probable
location?
a.
Scalp
b.
Brain
c.
Bone
d.
Lung
Answer: C
Two triangle Rule that love bone. Thyroid, Breast. Kidneys and Prostate
59- A patient received clindamycin before surgery. 3rd day post-op patient developed watery diarrhea. What test will
you order?
a.
Stool ova and parasite
b.
EIA for C.difficile
c.
Viral
Answer: B

148

60- Patient came with acute urinary retention. Urologist did catheterization for the patient. He had BPH. Surgery was
done and they injured the prostatic plexus. Whats the complication?
a.
Stress incontinence
b.
Urinary incontinence
c.
Erectile Dysfunction
Answer: C
Erectile Dysfunction then Retrograde ejaculation. (Answered by a urologist).
61- Patient with chronic osteomyelitis of her leg and there is sinus draining. She has cancer of skin near the sinus
which has has eosinophilic inclusions. What is the type of cancer?
a.
Squamous cell carcinoma.
b.
Basal cell carcinoma.
Answer: A
De novo squamous cell carcinoma emerge in the setting of long-standing ulcers, burn scars, or osteomyelitis.
Reference: Journal of Skin Cancer.
61- Pelvic fracture with blood coming out from urethral meatus. What will be your initial step?
Answer: ?
Rectal and pelvic examinations are of utmost importance during the initial evaluation to rule out the presence of an
open fracture. Blood in the vaginal vault or in the rectum should raise the level of suspicion for an open injury. Palpable
bony spicules within the rectum or vagina may be present indicating an open injury. A high-riding prostate may also be
detected on rectal examination, indicating the presence of a periurethral or periprosthetic hematoma occurring
secondary to genitourinary injury >> Pelvic stabilization for patients with pelvic fractures who are hemodynamically
unstable. Reference: http://www.sportssurgerynewyork.com/articles/diagnosis-management-pelvic-fractures.pdf
In any male patient with suggestive symptoms or signs urethral injury, the diagnosis is confirmed by retrograde
urethrography. This procedure should always precede catheterization. Urethral catheterization in a male with an
undetected significant urethral injury may potentiate urethral disruption (eg, convert a partial disruption to a complete
disruption). Female patients require prompt cystoscopy.
63- Cancer of the lower lip. What are the lymph nodes that you will examine first?
a.
submental, submandibular.
b.
Submental, Buccal.
c.
Buccal, parapharyngeal.
Answer: A
The lymphatic drainage of both the upper and lower lips is primarily to the submandibular group of lymph nodes. To a
lesser extent, drainage may go to submental intraparotid, or internal jugular lymph nodes.
Reference: http://www.ncbi.nlm.nih.gov/pubmed/8460042
64- loss of sensation in medial part of thigh nerve affected.
Answer: ?
Obturator nerve: Medial thigh
Anterior cutaneous branches of the femoral nerve: Anteromedial thigh

149

65- urge and stress incontinence treatment.

Reference: Toronto Notes.


66- Patient underwent orthopedic surgery. 2 days after the surgery he started to complain of swelling, erythema and
tenderness at the site of surgery. Which organism can cause the wound infection?
a.
Clostridium perfringens
b.
Other choices included different subtypes of clostridium
Answer: A
C. perfringens is the only Clostridium species can cause wound infection (Gas gangrene; A medical emergency)
67- Elderly patient is having bilateral Hydronephrosis:
a.
BPH
Answer: A
Reference: PubMed
68- Old patient with COPD, DM and now hes diagnosed to have BPH, what medication will you use?
a.
B-Blocker
b.
Terazosin
Answer: B
Terazosin is an alpha blocker
69- A man presented with pain and mass in the flank. What is the diagnosis?
Answer: ?
DD of flank pain and mass:
Renal abscesses: patient have pain because inflammation and edema produce stretch of renal capsule.
Sometimes, a flank mass may be palpable.
Polycystic kidney disease.
Renal vein thrombosis.
Renal cell carcinoma (pain, mass and hematuria)
Reference: Medscape + RightDiagnosis
70. (long scenario) lactating women 10 days after delivery complaining of fever and rigors. On examination: tender
left breast and nodules in upper outer area (investigations result included). What is the most likely diagnosis?
a.
Postpartum sepsis.
b.
Breast abscess.
c.
Inflammatory breast cancer.
Answer: B
Postpartum fever is defined as a temperature greater than 38.0C on any 2 of the first 10 days following delivery
exclusive of the first 24 hours. (which is not clearly met here).
Reference: http://emedicine.medscape.com/article/796892-overview#showall

150

71. Patient have blunt in right 4 intercostal space. Which of the following is most likely affect affected?
a.
Upper lobe of lung
b.
Lower lobe of lung
c.
Horizontal
Answer: C
The horizontal fissure arise from the right oblique fissure and follow the fourth intercostal space from the sternum until
it meets the oblique fissure as it crosses right 5th rib. Reference: http://radiopaedia.org/articles/horizontal-fissure
72- (long scenario) man with solid thyroid nodule, what is the most appropriate thing to do?
a.
Incisional biopsy.
b.
Excisional biopsy.
c.
Fine needle aspiration.
Answer: C
Fine-needle aspiration biopsy is used for definitive diagnosis. FNAB has attracted much attention in the adult
population in the evaluation of thyroid nodules. Reference: http://emedicine.medscape.com/article/924550workup#c5
73- Patient have appendectomy what is the most common infection will be come after operation?
a.
B. fragilis
b.
Staph aureus
c.
Shigella
d.
Pseudomonas
Answer: A
Enterococcus faecalis, Escherichia coli and B. fragilis are common pathogens in Surgical Site Infections after clean
contaminated surgery.
Reference:
https://books.google.com.sa/books?id=n9Y58-950vYC&pg=PA543&lpg=PA543&dq#v=onepage&q&f=false
http://www.cdc.gov/hicpac/SSI/table7-8-9-10-SSI.html
74- What is the common presentation of perforated duodenal ulcer?
Answer: ?
The most characteristic symptom is the suddenness of the onset of epigastric pain. The pain rapidly becomes
generalized although occasionally it moves to the right lower quadrant.
Reference: http://www.ncbi.nlm.nih.gov/books/NBK6926/
75- In appendectomy. Which artery you have to pay attention to during the procedure?
Answer: ?
Gonadal vessels
Iliac artery/vein
One must be cognizant of anatomical structures in the right lower quadrant, in order to avoid injuries during an
appendectomy. These include the right ureter, gonadal vessels, iliac artery/vein,psoas muscle, ileum and cecum.
Reference: http://www.eaes-eur.org/getmedia/7994a09c-344d-4c5e-81d54b9a75b8304b/Grantcharov_Appendectomy.pdf

76- During a cholecystectomy, there was an injury to the cystic artery. The surgeon applied pressure on the free
margin of the lesser omentum. What is the name of the artery compressed in this maneuver?
a.
The right gastric artery
b. Coeliac
c.
Hepatic
d. Splenic
e.
Gastro-epiploic/duodenal
Answer: C

151

Reference: Farquharson's Textbook of Operative General Surgery, 10th Edition and Wikipedia.
77- 17 year old complaining of abdominal pain that shifts to the right iliac fossa. what is the most likely diagnosis?
Answer: ?
Appendicitis
Features of the abdominal pain are as follows:
Typically begins as periumbilical or epigastric pain, then migrates to the RLQ
Patients usually lie down, flex their hips, and draw their knees up to reduce movements and to avoid worsening
their pain Reference: http://emedicine.medscape.com/article/773895-overview
78- A female patient who has high risk for breast cancer. Which gene is responsible for that?
Answer: ?
BRCA1
The family history characteristics that suggest increased risk of cancer are summarized as follows:
Two or more relatives with breast or ovarian cancer
Breast cancer occurring in an affected relative younger than 50 years
Relatives with both breast cancer and ovarian cancer
One or more relatives with two cancers (breast and ovarian cancer or 2 independent breast cancers)
Male relatives with breast cancer
BRCA1 and BRCA2 mutations
Ataxia telangiectasia heterozygotes (quadrupled risk)
Ashkenazi Jewish descent (doubled risk)
Reference: http://emedicine.medscape.com/article/1947145-overview#a6
79- 17 years old boy with acute appendicitis, appendectomy was done. If you take it under microscope, which of the
following will be found ?
a.
Neutrophils in muscularis propria.
b.
Mucus filled lumen.
c.
Neoplastic tumor at the tip.
Answer: A
Microscopy demonstrates neutrophil infiltrate of the mucosal and muscularis layers extending into the lumen.
Reference: http://emedicine.medscape.com/article/773895-workup#c20

152

80- 7 years old boy brought by his parents, he has nausea, severe vomiting for 20 minutes and now semi comatosed.
The parents mentioned that he has same episode two weeks ago for 5 minutes without deterioration in
consciousness. On examination there is right testicular mass that does not transilluminate with light. What is the
best action to do ?
a.
Radiology.
b.
ESR.
c.
Surgical exploration.
Answer: C
The most likely diagnosis is strangulated hernia
Once the diagnosis of a hernia is made, surgical repair (a herniotomy) will be performed.
Reference: http://www.rch.org.au/kidsinfo/fact_sheets/Inguinal_hernia/
81- Child was diagnosed to have meckel's diverticulae. What is the best diagnostic test?
Answer: ?
Meckels scan (technetium-99m pertechnetate scanning)
Meckel's scan has a sensitivity of 85 to 97 percent in pediatric patients
Reference: Uptodate
82- What is the cause of skin dimpling in breast cancer?
a.
Cooper ligament
b.
Lactiferous duct
Answer: A
83- A patient presented with urethral injury (already established) with blood coming out from the meatus, What will
you do for him?
a.
Insert suprapubic catheter.
Answer: ?

84- A patient presented with ischemia of the gluteus area. Whats the origin of gluteal artery?
a.
internal iliac artery
Answer: A
85- What area could be affected if there was occlusion of the internal iliac artery?
Answer: ?

153

86- Patient presents with blunt trauma that affect the inferior mesenteric artery, Which one of the following
branches NOT affected?
Answer: ?
Inferior mesenteric artery has 3 branches:
- Left colic artery
- Sigmoidal arteries
- Superior rectal artery
87- One cell type testicular mass, what is the diagnosis?
a.
Seminoma
Answer: ?

Most common urologic diagnosis in men <50 yr. Prevalence 2-12%

154

* KEEPS Klebsiella sp.E. coli (90%), other Gram-negatives EnterococciProteus mirabilis, PseudomonasS. saprophyticus

Reference: Toronto Notes

87- Prevent of hemorrhoid:


a. high fiber diets
Answer: a
The best way to prevent hemorrhoids is to keep your stools soft, so they pass easily. To prevent hemorrhoids and
reduce symptoms of hemorrhoids, follow these tips:
1.

Eat high-fiber foods. Eat more fruits, vegetables and whole grains. ...

2.

Drink plenty of fluids. ...

3.

Consider fiber supplements. ...

4.

Don't strain. ...

5.

Go as soon as you feel the urge. ...

6.

Exercise. ...

7.

Avoid long periods of sitting.


Ref : http://www.mayoclinic.org/diseases-conditions/hemorrhoids/basics/prevention/con-20029852
88 - Which vitamin can prevent or reduce risk of colorectal ca? 3 times
Folic acid
Vit.D
Vit.E
Answer: Vitamin D
Reference: uptodate
http://www.uptodate.com/contents/colorectal-cancer-epidemiology-risk-factors-and-protectivefactors?source=outline_link&view=text&anchor=H19#H19

155

89- which thyroid cancer have the best prognosis?


Answer: Papillary cancer
90 - Difficulty in initiation of urination which type of incontinence pt have
Urge incontinence
Overflow incontinence
Stress incontinence
Answer: non of the answers make sense due to the following
Stress Incontinence:
With this type, urine leaks due to weakened pelvic floor muscles and tissues. It can happen when pressure on your
bladder increases -- such as when you exercise, laugh, sneeze, or cough.
Urge Incontinence:
This is also called overactive bladder (OAB). With this type, you have an urgent need to go to the bathroom and may
not get there in time.
Overflow Incontinence:
If you can't empty your bladder, you may have overflow incontinence. This means you may dribble urine.
If there was no choice for non of the above the choose overflow incontinence .
91 mass in the neck in cervical area, the best initial test?
Answer: CT
Ref:
http://www.uptodate.com/contents/evaluation-of-a-neck-mass?source=outline_link&view=text&anchor=H13#H13
92- kidney tumor oncogene ?
ps53
cant remember
Answer:
-MET proto oncogene responsible for hereditary papillary renal cell cancer
Ref : pubmed
http://www.ncbi.nlm.nih.gov/pmc/articles/PMC2929006/
-Cont. alteration of p3 chromosome result in alteration of either tumor suppressor gene ( VHL , TSC ) or oncogene ( MET
)
Ref : medscape
http://emedicine.medscape.com/article/281340-overview#a2
93- The narrowest part of male urethra is?
A- membranous urethra.
B- penile urethra.
The answer is A
Answer : membranous urethra (1 cm long): passes through the urogenital diaphragm, surrounded by sphincter
urethrae

the shortest and narrowest portion.


Reference:
http://radiopaedia.org/articles/male-urethra
94- indirect hernia related to spermatic cord ?
anterio medial
posterio medial
something another something
Answer: Indirect inguinal hernia lies within the cord, Some websites mention that the hernia is located anterio-medial
to the cord.
95-30 years old female dancer with breast mass that disappeared by aspiration ?
Answer:Complete disappearance of the breast mass after aspiration of non bloody fluid , indicate simple cyst
References : pubmed & medscape
http://www.medscape.org/viewarticle/718139
http://www.ncbi.nlm.nih.gov/pmc/articles/PMC2855917/#!po=15.7407

156

96- gene mutation in breast cancer?


Answer: BRCA1 & BRCA 2
Ref: pubmed
97-30 years pt with cyclic breast pain and tenderness ..pain is relieved after the cycle..
By examination there is periaerular nodular mass 3cm ..what is invx?
A.FNA
B.mammogram followed by US ( newly added )
C.u/s
D.follow her to next period CT scan ( newly added )
Answer: U/S
The first line of investigation is imaging not cytology nor tissue biopsy, So that excludes FNA.
Because the pt is young (30 yr old) U/S is more sensitive. Because of the tissue density.
Ref: Baily & Love
98- Ulcerative colitis skin lesions :
Most frequent skin lesions associated with IBD inculde erythema nodosum and pyoderma gangrenosum
Source: uptodate
99- Which type of renal stones is associated with recurrent infections
Answer: struvite stones
Struvite stones account for 15% of renal calculi. They are associated with chronic urinary tract infection (UTI) with
gram-negative rods capable of splitting urea into ammonium, which combines with phosphate and magnesium
http://emedicine.medscape.com/article/437096-overview#a5
100- ttt of uncomplicated cystitis
Answer: The first-choice agents for treatment of uncomplicated acute cystitis in women include nitrofurantoin
monohydrate/macrocrystals, trimethoprim-sulfamethoxazole (TMP-SMX), or fosfomycin.
Ref: Medscape + uptodate
101- scrotal pain with dysuria ttt
Answer: It is mostly a case of epididymi-orchitis caused by STD ( gonorrhea or chlamydia ), both usually responsive to
doxycycline . pubmed
102 - sign of hyperthyroidism wth nodule in lateral to mid line
Answer: First rule any thyroid nodule not showing or changing hormone activity and signs it is most likely malignant
nodule
Hyperthyroidism signs and symptoms :
Common symptoms of thyrotoxicosis include the following:
* NervousnessAnxietyIncreased perspirationHeat intoleranceHyperactivityPalpitationsmedscape

103 - gastrectomy which one will be defecient


Lipase
Pepsin
Trypsin
Answer: pepsin
http://rfwdata.net/USMLEQBANK/block17questions.html

157

104-painful axillary lump with tender and erythematous with black head papule and large pores? repeated
A.Local antibiotic
B.Oral antibiotic
C.Surgery
Answer: C
It is most probably epidermoid cyst , if not infected it will resolve spontaneously but recurrence in high , if fluctuating I
and D is indicated .
Reference ; up to date
http://www.uptodate.com/contents/overview-of-benign-lesions-of-theskin?source=outline_link&view=text&anchor=H1101420445#H1101420445
105-old man complaining of inability to start urine. after voiding = rectal exam
revel distended full bladder ? repeated
A.Stress
B.overflow
C.reflex
Answer: B
overflow have full bladder , not A because there no past prostate surgery .
Reference: up to date
http://www.uptodate.com/contents/urinary-incontinence-inmen?source=outline_link&view=text&anchor=H9860233#H9860233
106-Post appendectomy .. penicillin resistance? What antibiotic use?
Answer: Broad-spectrum gram-negative and anaerobic coverage is indicated .Penicillin-allergic patients should avoid
beta-lactamase type antibiotics and cephalosporins. Carbapenems are a good option in these patients.
Referance: http://emedicine.medscape.com/article/773895-treatment#d10
nonPERFORATED
APPENDICITIS :
Postoperative management With both the open and laparoscopic approaches, most patients are discharged within 24
to 48 hours of surgery. Patients may be started on a clear liquid diet post-operatively and advanced to regular diet as
tolerated. Antibiotics are not required postoperatively in nonperforated appendicitis.
PERFORATED APPENDICITIS :
Postoperative management Postoperatively, these patients often have an ileus, and diet should only be advanced as
the clinical situation warrants. Patients may be discharged once they tolerate a regular diet, usually in five to seven days.
The duration of antibiotic therapy in such patients is discussed elsewhere. (See "Anaerobic bacterial infections", section
on 'Antibiotic treatment'.)
Referance: http://www.uptodate.com/contents/management-of-acute-appendicitis-in-adults
107-most common intracranial tumor in adult ?

A.Hemangiomblastoma
B.Ependymoma
C.Shwanoma
Answer: B
Glioblastoma multiforme (GBM) , also known as gliomas is the most common primary brain tumor in adults.
Ependymoma is a type of glioma.
http://braintumor.org/brain-tumor-information/understanding-brain-tumors/tumor-types/
108-during laparoscopic surgery of inguinal hernia you find artery superficial going upward ?
A.Inferior epigastric artery.
Answer: Inferior Epigastric vessels
Great care must be exercised as the dissection approaches the iliac vessels. In addition, obturator vessels
often cross the dissection planes and may need to be clipped and divided.
The inferior epigastric vessels are identified, and dissection lateral to the vessels leads to the space of Bogros, the cord
structures, and indirect hernias (see the video below). The proper plane of dissection is between the transversalis fascia
and the peritoneum. This is identified by retracting the inferior epigastric vessels upward against the rectus muscle. A
plane containing areolar tissue is identified, and this plane is dissected toward the pelvic sidewall
Referance : medscape
http://emedicine.medscape.com/article/1534321-overview

158

109-MVA diagnosed with fracture of base of skull and injury to nerve pass through jugular foramen
Answer:cranial nerve 9th , 10th , 11th
Vernet syndrome or jugular foramen syndrome is involvement of the IX, X, and XI cranial nerves with basal skull
fracture. Patients present with difficulty in phonation and aspiration and ipsilateral motor paralysis of the vocal cord, soft
palate (curtain sign), superior pharyngeal constrictor, sternocleidomastoid, and trapezius.
Referance : http://emedicine.medscape.com/article/248108-overview#showall

110- a patient has a trauma to his neck and he cant abduct his shoulder, what is the nerve that was injured?

A.Axillary
B.long thoracic
Answer: B ??
Abduction is carried out by the deltoid(Axillary) and the supraspinatus in the first 90 degrees. From 90-180 degrees it is
the trapezius and the serratus anterior(long thoracic).
Referance: https://en.wikipedia.org/wiki/Shoulder_joint
111-a patient weighs 70 kg had a circumferential 3rd degree burn in his trunck, how much lactate ringer you will
give him in the first 8 hours?
A.12.5 L

B.10.5
C.8.5
D.6.5
Answer: D
Resuscitation fluids - 3-4 mL Ringer lactate X weight (kg) X %TBSA burned (second-degree and third degree); half
administered over the first 8 hours (from time of injury), remaining half administered over the next 16 hours.
Referance: http://reference.medscape.com/article/934173-treatment
112-patient post lap choly develope sob cough respiratory distress What's dx

A.PE
B.Stroke
C.Pneumonia
Answer: D. Atelectasis.
Atelectasis is a common pulmonary complication in patients following thoracic and upper
abdominal procedures. General anesthesia and surgical manipulation lead to atelectasis by causing
diaphragmatic dysfunction and diminished surfactant activity.
PE is one of the correct options if the surgery is prolonged like in joint replacements (so unlikely
after lap chole) and usually present 5-7 days after surgery, unlike atelectasis which develop one day
after surgery.
Reference:
http://emedicine.medscape.com/article/296468-overview.
113-Pt with carpal tunnel what is the diagnostic test:
A.mri
B.ct
C.x-ray
D.nerve conduction
Answer: D
Electrophysiological tests (nerve conductive study). Electrical testing of median nerve function is
often done to help confirm the diagnosis.
Reference: http://orthoinfo.aaos.org/topic.cfm?topic=a00005
159

114-multiple myeloma in the spine


(Histopath report)
Answer:
Dx of multiple myeloma requires the following:

* bone marrow aspirate or biopsy showing that at least 10% of the cells are plasma cells or the
presence of plasma cell tumor ( plasmacyte ) plus at least one of the following
- evidence of the damage to the body by plasma cells ; bone damage , kidney failure , anemia or
high calcium .
- defection of one of the following ; > 60% plasma cells in bone marrow, free light chain ration of
100 or more, MRI showing involvement of more than one lesion in the bone or bone marrow
Reference: http://www.uptodate.com/contents/multiple-myeloma-symptoms-diagnosis-and-staging-beyond-thebasics?source=outline_link&view=text&anchor=H12#H12

115-honeycoomb cystitis which organism


Answer:
I think they mean Honeymoon cystitis*
Honeymoon cystitis (or "honeymoon disease") is cystitis caused by sexual activity.
The microbial spectrum of uncomplicated cystitis and pyelonephritis in women consists mainly of Escherichia coli (75 to
95 percent),
Reference: http://www.uptodate.com/contents/acute-uncomplicated-cystitis-and-pyelonephritis-inwomen?source=search_result&search=cystitis&selectedTitle=1~80
116-Rx uncomplicated 5 yrs cystitis

A. IM cefttiaxon
B. IV copra
C. Oral.
Answer: C
Children with cystitis usually do not require special medical care other than appropriate antibiotic therapy. A 4-day
course of an oral antibiotic agent is recommended for the treatment of cystitis. Antibiotic agents used include
Sulfamethoxazole and trimethoprim (SMZ-TMP), Amoxicillin and clavulanic acid, Cephalexin, Cefixime, Cefpodoxime,
Nitrofurantoin.
Reference: http://emedicine.medscape.com/article/969643-treatment#d12
117-Injury to jugular foramen which structure will not be affected all of option are muscle :
A.Strenochlinomastoid
B.Sphenioplatine and
C.2 more option
Answer:
sternocleidomastoid
is
least
muscle
affected
during
jagular
foramen
Jugular
foramen
muscle
sternocleidomastoid
sphenopalatine
Digastric
-If one of these choose SCM , if non of these choose it
Reference: by Neurosurgery consultant

injury
,
passing:
muscle

118-Injury to temporal area and superficial temporal artery bleed which layer of the skull where vessels are ?
A.
peri cranial
B.
epi cranial aponeurosis
Answer: B
Reference: http://emedicine.medscape.com/article/881374-overview#a9

160

119- surgeon take graft from rectus muscle which artery should be dissected

A.superior epigastric
B.inferior epigastric
C.superficial epigastric
Answer: B
Reference: http://www.ncbi.nlm.nih.gov/pmc/articles/PMC3365455/
120-what is the most common type of gallstone
A.cholesterol screening
B.bile pigmented
Answer: A
Reference: http://www.mayoclinic.org/diseases-conditions/gallstones/basics/causes/con-20020461
121-bladder ca surgical removed and the area of cancer now clear and no invasion of the muscle

A. follow up with cystoscopy and biopsy


B. intravesical chemo..
Answer: B intravesical chemotherapy
Reference : uptodate + toronto notes
http://www.uptodate.com/contents/overview-of-the-initial-approach-and-management-of-urothelial-bladdercancer?source=outline_link&view=text&anchor=H3#H3
122-lap-cholecystectomy injured cystic artery, surgeon put pressure with the free margin of lesser omentum,which
artery is compressed?

A.hepatic
B.coeliac
C.splenic
D.gastro-epiploic/duodenal ?
Answer: hepatic

123- What is the drug that can make a cholecystitis pain worse?
A.Acetaminophen
B.Morphine
C,Meperidine
Answer; B
Reference: http://www.meb.uni-bonn.de/dtc/primsurg/docbook/html/x3982.html

124-Tumor in rt testis, Hcg high, most likely?


A.Seminoma
B. Lydig cell
D.Teratoma
Answer: A
Reference: http://www.cancer.net/research-and-advocacy/asco-care-and-treatment-recommendations-patients/tumormarkers-testicular-cancer-and-extragonadal-germ-cell-tumors-teenage-boys-and-men
125-Ligament in inguinal canal?

A.Broad
B.Round
Answer: b

161

126-Ascites Bilateral ovarian mass, what is the most likely tumor?


Answer:

Reference: uptodate
127-Single thyroid nodule iodine sensitive. Best management?
A.Lobectomy
B.Radio iodine therapy
C.Anti thyroid drug
Answer:

Reference :american family physician

162

128-Perianal itching. Most likely?

A.Perianal abscess
B.Hemorrhoid
Answer:B Hemorrhoids, which cause painful swelling of blood vessels in the anal area, can cause
itching.
References:
http://www.emedicinehealth.com/script/main/mobileart-emh.asp?articlekey=58920&page=2
http://www.emedicinehealth.com/script/main/mobileart-emh.asp?articlekey=58920&page=2
129-Pt indwelling catheter having stone most common ?
A.Ca oxalate
B.Ca phosphate
C.Uric acid
Answer
Struvite stones account for 15% of renal calculi. They are associated with chronic urinary tract infection (UTI) with
gram-negative rods capable of splitting urea into ammonium, which combines with phosphate and magnesium
Reference: http://emedicine.medscape.com/article/437096-overview#a5
130-Pt co of infected wound ulcer ,intact pulse ,no improve by AB & pt known uncontrolled DM2 Amputation
A.AB
B.surgical debridement
Answer: B
A good clinical response for mild to moderate infections can be expected in 80%90% of appropriately treated patients
[10, 50] and, for deeper or more extensive infections, in 50%60% [64, 86]. When infection involves deep soft-tissue
structures or bone, more thorough debridement is usually needed. Bone resections or partial amputations are required in
about two-thirds of this patient group. Most of these amputations can be foot sparing, and long-term control of infection
is achieved in >80% of cases. Infection recurs in 20%30% of patients, many of whom have underlying osteomyelitis.
Factors that predict healing include the absence of exposed bone, a palpable popliteal pulse, toe pressure of >45 mm Hg
or an ankle pressure of >80 mm Hg, and a peripheral WBC count of <12,000/mm3 [19]. The presence of edema or
atherosclerotic cardiovascular disease increases the likelihood of amputation. Amputation may be more often required for
patients with combined soft-tissue and bone infection than for patients with either type of infection alone [86]. Patients
who have had one infection are at substantial risk of having another within a few years; thus, educating them about
prevention techniques and about prompt consultation when foot problems occur is critical.
Reference: http://cid.oxfordjournals.org/content/39/Supplement_2/S104.full
131-best imaging for cyst in the breast

A.US
B.mammogram
Answer: A
us is used to differentiate between solid and cystic lesion .
mammogram visualize large cyst but not small microcystic lesion .
Reference : up to date
http://www.uptodate.com/contents/breast-cysts-clinical-manifestations-diagnosis-andmanagement?source=outline_link&view=text&anchor=H91862703#H91862703

163

132-decrease colorectal cancer


A.vitamin
B.vitamin E
C.folic acid
Answer:
- folic acid decrease the risk of colorectal cancer .
- vit D act as inhibtor of colorectal cancer by influence the intiation and progrestion .
-also, "higher intake of vitamin B6 was associated with lower risk of colorectal cancer"
Reference: up to date
http://www.uptodate.com/contents/colorectal-cancer-epidemiology-risk-factors-and-protectivefactors?source=outline_link&view=text&anchor=H19#H19
Vit B6
Medscape
http://www.medscape.org/viewarticle/506337
UpToDate
http://www.uptodate.com/contents/colorectal-cancer-epidemiology-risk-factors-and-protectivefactors?source=outline_link&view=text&anchor=H20#H20
Um-alqura 3rd ed. :

133-female patient with high prolactin What to exclude

A.thyroid disease
B.pituitary tumors
Answer:B
most common cause of female hyperprolactinemia is pitutary tumor (microprolactinoma )
Reference : up to date
http://www.uptodate.com/contents/causes-ofhyperprolactinemia?source=outline_link&view=text&anchor=H1#H1
134-patient with BPH+HTN what TTT?
Answer: alpha blocker
Reference : up to date
http://www.uptodate.com/contents/image?imageKey=NEPH%2F63628&topicKey=NEPH%2F3869&source=see_link

164

135-case head trauma on parietal lobe subdural hematoma which artery is injured ?

A.superficial temporal .
B.mid cerebral
D.Rt.cerebral .......
Answer: b
The cortical branches of the MCA supply the lateral surface of the hemisphere, except for the medial part of the frontal
and the parietal lobe (anterior cerebral artery), and the inferior part of the temporal lobe (posterior cerebral artery)

http://www.radiologyassistant.nl/en/p484b8328cb6b2/brain-ischemia-vascular-territories.html
136-patient presented with appendicitis appendectomy done and send to pathology what you expect to see :
A.neutrophil in wall .
B.distended lumen with mucus ...........
C.renal colic was easy
D.splenctomy what vaccin he should took .
Answer: A
Neutrophils extend into and through the wall of the appendix in a case of acute appendicitis. Clinically, the patient often
presents with right lower quadrant abdominal pain. Rebound tenderness of the right lower quadrant is often noted on
physical examination, as well as positive obturator or psoas sign. An elevated WBC count is usually present.
http://library.med.utah.edu/WebPath/GIHTML/GI058.html

137-Trauma to shoulder PT cant raise hand the injury is in:

A.Posterior cord of biracial plexus


B.Medial cord
C.Lateral cord
Answer: A
The axillary nerve is a branch of the posterior cord of the brachial plexus
http://emedicine.medscape.com/article/1877731-overview#a2
138-gene mutation of suprarenal tumor?

The Li-Fraumeni syndrome is caused by inherited mutations that inactivate the p53 tumor
suppressor gene. This syndrome causes few cases of adrenal cancer in adults (1 of every 20), but is
often the cause of adrenal cancer in children. In fact, about 8 of every 10 cases of adrenal cancer in
children are caused by Li-Fraumeni syndrome. Many other adrenal cancers have also been found to
have abnormal p53 genes that were acquired after birth (not inherited).
http://www.cancer.org/cancer/adrenalcorticalcancer/detailedguide/adrenal-cortical-cancer-whatcauses
Mutations in the VHL gene cause von Hippel-Lindau syndrome.

http://ghr.nlm.nih.gov/condition/von-hippel-lindau-syndrome
many different syndromes
139-Patient with hematuria diagnosed bladder cancer did resection what is the next step ?
a-BCG intrbladder
b-Other choice
Answer: mitomycin-C
Within the first 24 hours, a single intravesical instillation of mitomycin-C (40 mg in 20 mL of saline) has been shown to
reduce the frequency of tumor recurrence.
Reference: http://emedicine.medscape.com/article/1951622-overview#a4
140- Patient diagnose with renal cell carcinoma which gene do u think have mutation ?
A-PKD
B-PKHD-1
C-VHL
Answer: C
Referance : http://emedicine.medscape.com/article/281340-overview#a3

165

141-Case Symptoms of cholecystitis what best initial modality ?


a-U/S
Answer: A
Sonography is the preferred initial imaging test for the diagnosis of acute cholecystitis, and scintigraphy is the preferred
alternative.
CT is a secondary imaging test that can identify extrabiliary disorders and complications of acute cholecystitis, such as
gangrene, gas formation, and perforation.
Reference:
http://emedicine.medscape.com/article/171886-workup#c7
142-Eldary Flank pain hematuria and mass what best imaging?
Answer: CT with contrast.
the pt. present with classical triad of renal tumor (hematuria, flank pain and flank mass)
References:
http://emedicine.medscape.com/article/281340-clinical
Contrast-enhanced CT scanning has become the imaging procedure of choice for diagnosis and staging of renal cell
cancer and has virtually replaced excretory urography and renal ultrasonography.
http://emedicine.medscape.com/article/281340-workup
143-mass in neck in cervicle the best intial test?
a-laryngioscope
b-ct
c-FNA
d-biopsy
Answer: c
Reference: https://www.med.unc.edu/surgery/education/files/articles/Neck%20Mass.pdf

166

144-pt noticed lump for three months the mass


freely mobile no discharge not related to menstrual?
a-cystic
b-fibroadenoma
c-ductal papilloma
d-something
Answer:b
Reference: http://www.mayoclinic.org/diseases-conditions/fibroadenoma/basics/definition/con-20032223

Added Qs 7th update


1-anterior abdominal stab wound omuntam come through the wound ?
A.Fast
B.Ct
C.Exploratory laparotomy
Answer: C
Exploratory laparotomy is the answer , conservative treatment is sometimes used in selective patients
Reference: http://www.ncbi.nlm.nih.gov/pmc/articles/PMC4379793/
2-old patient presented with congested neck veins bilateral upper limb swelling tumor in the right upper lung
which part it's compressing ?
A.superior mediastinal
B.anterior mediastinal
C.posterior mediastinal
Answer: A
This patient has superior vena cava (SVC) syndrome due to lung mass compression
The SVC is located in the anterior right superior mediastinum
Reference: http://www.ncbi.nlm.nih.gov/pmc/articles/PMC4093359/
3-best investigation of ovarian cancer:
a-U/s
b-CA125
c-?
d-?
Answer: Uptodate : -the only way to diagnose ovarian cancer is with exploratory operation.
-Intra operative biopsy is not usually done , biopsy may spread cancer cells.
-Ca125 is for monitoring ( pre and post operative ) - it is not diagnostic and not specific.
-Imaging like u/s is recommended initially , but can not be diagnostic .
References:
http://www.uptodate.com/contents/ovarian-cancer-diagnosis-and-staging-beyond-thebasics?source=outline_link&view=text&anchor=H4#H4
For screening :Check the NICE guide lines
http://www.nice.org.uk/guidance/cg122/resources/ovarian-cancer-recognition-and-initial-management-35109446543557
4-Patient history of trauma and right femur fracture with blood loss which is accepted change ?
a-Decrease
coronary blood flow
b-Decrease
venous capacity
Answer: The cardiovascular system initially responds to hypovolemic shock by increasing the heart rate, increasing
myocardial contractility, and constricting peripheral blood vessels. This response occurs secondary to an increased
release of norepinephrine and decreased baseline vagal tone (regulated by the baroreceptors in the carotid arch, aortic
arch, left atrium, and pulmonary vessels).

167

5- Patient did IVP shows filling defect then do us shows hypoecnecnty ?


a-Uric acid stone
b-Papillary
Answer: ??
6-Pt with high aptt what mechanism of action of that drug?
a-" unfractinated heparin"
b-Antithrombin
Answer: heparin act indirectly by binding to antithrombin rather than binding directly to coagulation factors.
Reference: http://www.uptodate.com/contents/therapeutic-use-of-unfractionated-heparin-and-low-molecular-weightheparin?source=outline_link&view=text&anchor=H3#H3
7-bronchial cancer mets to sypathatic plexus what the sign ?
a-ptosis
b-dilated pupil
Answer : the answer is horners syndrome (ptosis ,anhydrosis,miosis)
-it results from an interruption of the sympathetic nerve supply to the eye and is characterized by the classic triad of
miosis (ie, constricted pupil), partial ptosis, and loss of hemifacial sweating (ie, anhidrosis).
-Horner syndrome can be congenital, acquired, or purely hereditary (autosomal dominant). The interruption of the
sympathetic fibers may occur centrally (ie, between the hypothalamus and the fibers point of exit from the spinal cord
[C8 to T2]) or peripherally (ie, in cervical sympathetic chain, at the superior cervical ganglion, or along the carotid
artery).
Reference: http://emedicine.medscape.com/article/1220091-overview#a3
8-pt noticed lump for three mos the mass freely mobile no discharge not related to menstrual?
a-cystic
b-fibroadenoma
c-ductal papilloma
d-something
Answer:b
Reference: http://www.mayoclinic.org/diseases-conditions/fibroadenoma/basics/definition/con-20032223
9-read about lower limb dermatom20-knee dermatom?
a-L1-L2
b-L3-L4
c-L4-L5
d-L5-S1
Answer : b

168

10. 3 years old boy with UTI,, what youll do before starting treatment
-US
-culture urethra !!
-p.s. no assenting urethrogram within the answers
Answer : (the answer should be : send urine culture)
-Urine culture should be done beside urine analysis , but do not delay antibiotic.
- renal ultrasonography should be considered for any child with a first febrile UTI in whom good follow-up cannot be
ensured.
- blood cultures are not recommended .
Source :
http://emedicine.medscape.com/article/969643-workup
Toronto note :
U/s in recommended for all febrile "infants" with UTI
uptodate : US is done if :
child with recurrent UTI at any age
children younger than 2 yrs with a first febrile UTI
children who dont respond as expected to appropreate ttt .
children of any age with with a UTI who have family history of renal or urologic disease , poor growth or HTN.
11- Diabetic patient went for surgery; he was given insulin and dextrose. Then developed neurological symptoms
(Low Na). What is the mechanism?
-Water overload.
-SIADH.
Answer: incomplete Q the two pictures explain the possible answers

Reference: pics and toronto note

169

12-Most common neurosurgical tumor?


Astrocytoma.
Medullablastoma.
Answer:
Gliomas (principally astrocytoma) account for approximately 30 percent and meningioma 35 to 40 percent of
symptomatic primary brain tumors.

Reference : pic and uptodate


13- 60 y.o smoker with lung cancer. When the tumor compresses the sympathetic innervation, it will lead to ?
A. Anhydrosis
B. Ptosis
C. Hydrosis
D. Myadrasis
Answer : B
- Pancoast tumors leads to compression of sympathetic ganglion causing Horner's syndrome.
- Horner's syndrome presents with : miosis(constriction of the pupils), anhidrosis (lack of sweating), ptosis (drooping of
the eyelid) and enophthalmos (sunken eyeball).
14- 47 y/o female, her only child was born when she was 35 y, now she develop epithelial ovarian cancer, no
BARCA1 mutation, what will you tell her daughter about her risk to develop cancer?
a)
Family history has no risk
b)
OCP has protective effect
c)
Barrier contraceptive has protective effect
Answer: B
protective factors for ovarian, fallopian tube:
Oral contraceptives
Tubal ligation
Breastfeeding
Risk-reducing salpingo-oophorectomy
Reference : uptodate
15-55 y/o male present with erectile dysfunction, he mention that only worse with stress, medically free, not on any
medication, what is the best action?
a) Refer to endocrine
b) refer to urology
c) advice relaxation strategies
Answer :
In Om alqora : c
This article is discussing the management of erectile dysfunction in general:
http://www.journalofandrologicalsciences.eu/media/04_Gentile.pdf

170

16- Pheochromocytoma, associated with which neoplasm?


a)
Insulinoma
b)
Thyroid cancer
answer: B
Pheochromocytoma is associated with Medullary thyroid carcinoma , hyperparathyrodism .
Pheochromocytoma is a type of MEN IIA ( multiple endocrine neoplasia )
Reference: Step-up medicine P 185 .
17- man done vasectomy, change his mind want to reproduce again, they found antisperm antibodies what is the
cause ?
a.
antigen release or something like that?
b.
cross reactivity with viral infection
c.
inappropriate response of MHC II to antigen presenting cell
answer: b
18- Case Symptoms of cholecystitis what best intial modelity ?
A- U/S
answer: A
Ultrasound (US) is the preferred initial modality in the investigation of right upper quadrant pain.
19- 60 years old male presented with left flank pain, weight loss and hematurea, O/E palpable firm mass in the
lumbar area, BP 150/90
What is the best initial investigation?
A) ct scan
B ) mri
C) ultrasound
D) radionuclide scan
answer: C
by radiology Consultant
20- Chronic alcoholic wake up at 3 am with abdominal pain after a little while the pain generalized all over the
abdomen and is severely exacerbated with movement:
a. Peptic ulcer perforation
b.Choycystitis
c. DiverticulaTitis
d. Pancreatitis
answer : Pancreatitis
-Alcohol do NOT cause ulcers. They delay the healing of ulcers
-Alcohol is a major cause of acute and chronic pancreatitis
21- In thyroidectomy which nerve injury lead to horseness of voice?
A)Recurrent laryngeal
B)Right vagus
Answer :A
recurrent laryngeal nerve: Unilateral damage results in a hoarse voice. Bilateral damage presents as laryngeal obstruction
after surgery and can be a surgical emergency: an emergency tracheostomy may be needed.
22- Ureteral stone what is the rdiological study to detect it?
A)Abd.ultrasoubd
B)Ct with contrast
C)Ct without contrast
-There was No xray in options
answer: C
by radiology Consultant
23. most reliable screening for prostate cancer is:
- PR examination
answer : Prostate specific antigen (PSA)
DRE is less effective compared to PSA
Reference: http://www.uptodate.com/contents/prostate-cancer-screening-beyond-the-basics

171

24- patient lost sensation at the level of umbilicus after MVA . the lesion will be at the level of
- T4
- T10
- T11
-T 12
answer : T10

25- man came with loose stools , history of loose stools before ,, Its watery like with mucous not containing blood
( forget the other details ) what is you diagnosis:
a. IBS
b. chrons disease
c. ulcerative colitis
Answer : a
Because IBS ( irritable bowel syndrome ) characterized by change in stool frequency which we have in this Q ( loose
stool then watery like ) .
Ulcerative colitis (US) : bloody stool
Crohn's disease : diarrhea that may or may not be bloody .
So we need more details in this Q .
Reference: Usmle step1 P355-356 (2012)
26-pt came with history for alternative bowle habits some time loose and other day constipation for weeks your
diagnosis :
A/ IBS
B/ crohns disease
C/ ulcerative colitis
answer is: A
Reference:http://www.mayoclinic.org/diseases-conditions/irritable-bowel-syndrome/basics/tests-diagnosis/con-20024578
27- Patient came with history of upper abdominal pain the investigation show high amylase your diagnosis will be:
A/ acute pancreatitis.
B/ gastric ulcer.
C/ acute cholecystitis.
answer is: A
Reference: http://www.healthline.com/health/amylase-blood

172

28-) which of the following has strong association for colorectal cancer:
A/ familial polyposis
B/ ulcerative colitis
C/ chrons disease
answer is: A
Reference: http://ghr.nlm.nih.gov/condition/familial-adenomatous-polyposis
29- pt came with Lt flank pain radiated to groin no fever no change in urine color or amount no sign other than
tenderness this pain most likely:
a. uretric stone
b. renal colic
c. diverticulitis
answer ; b
The site of obstruction determines the location of pain. Upper ureteral or renal pelvic lesions lead to flank pain or
tenderness, whereas lower ureteral obstruction causes pain that may radiate to the ipsilateral testicle or labia.
Costovertebral angle tenderness on the affected side is common.
reference ; emedicine
30- patent came from RTA , in preparing of setting of intubation patient became desating more and ambobag not
maintaing saturation which should be done ?
A. more jaw thrust
B. more head tilt
C. precede immediate for intubation
answer ; if the patient's oxygenation cannot be maintained, immediate rescue by (usually) cricothyrotomy is necessary.
reference : uptodate
http://www.uptodate.com/contents/the-failed-airway-inadults?source=outline_link&view=text&anchor=H351136572#H351136572
31-When you are going to intubate unconscious male there were difficult ventilation with bag-mask what to do:
a. procced to intubation
b. apply cricoid pressure
c. head tilt
answer ; c
mask seal Mask seal requires reasonably normal anatomy, absence of facial hair, lack of interfering substances, such as
excessive vomitus or bleeding, and the ability to apply pressure to the face with the mask.
reference : uptodate
http://www.uptodate.com/contents/the-difficult-airway-in-adults?source=outline_link&view=text&anchor=H4#H4
32-best screening method for prostate cancer?
a.digital rectal examination
b.cytology for prostate
cancer
c.specific prostate antigen
answer:APSA is combined with digital exam in screening sometimes
PSA elevated in other prostate diseases other than cancer ( not specific )
http://www.uptodate.com/contents/prostate-cancer-screening-beyond-the-basics
Check Toronto note for screening recommendation
33-Pt with past hx of hodgikon lymphoma .. But cured completely .. Presented with back pain ... Examination
and evaluation show paraspinus Edema and fluid collection -ve burecella titer and tuberclin test ,, what the
cause ?
burecellosis
breast cancer
recurrent hodgikon lymphoma
answer: c
http://www.ncbi.nlm.nih.gov/pmc/articles/PMC3892516/

173

34- 18-year-old healthy male was playing baseball and suddenly he felt abdominal pain. On examination he
has para-umbilical mass. His vital signs are: BP 100/76, RR 30, HR 100, O2 sat is 95% with 100% oxygen
mask. What is your next step in the management?
Abdominal US.
CT scan.
Erect Chest x-ray.
Reassure and send home.
Answer :
C
Intestinal obstruction due to hernia
Erect chest xray looking for air fluid level
Source : Toronto note
35- Patient is having infertility came to infertility clinic. He is complaining of testicular pain. On examination the
testicle is swelled like a bag of worms. What is the diagnosis?
Varicocele.
Spermatocele.
Hydrocele.
Testicular cancer.
Answer: Varicocele
Palpating a varicocele can be likened to feeling a bag of worms
https://en.wikipedia.org/wiki/Varicocele
36- Patient is having unilateral testicular pain and fever. What is the diagnosis?
A-Epidedmo-orchitis.
B-Prostatitis.
C-Testicular torsion.
Answer: A Epidedmo-orchitis
Acute epididymo-orchitis is a clinical syndrome consisting of pain, swelling and inflammation of the epididymis,
with or without inflammation of the testes. The most common route of infection is local extension and is mainly
due to infections spreading from the urethra (sexually transmitted infections (STIs)) or from the bladder.[1]
Orchitis (infection limited to the testis) is much less common. Chronic epididymitis refers to epididymal pain
and inflammation (usually without scrotal swelling) that lasts for more than six months.
37- Patient after RTA had spleen injury, he underwent surgical exploration and splenectomy. Which of
the following should be given?
a. Some kind of antibiotic.
B. Meningiococcal vaccine.
Answer: Meningiococcal vaccine
Reference: uptodate
http://www.uptodate.com/contents/prevention-of-sepsis-in-the-asplenicpatient?source=outline_link&view=text&anchor=H16#H16
38-Rx of patient with lung ca stage IIIb came with sudden lower back pain?
a.MRI only
b.MRI with steroid
c.radiotherapy
Answer: MRI only
Reference: uptodate
http://www.uptodate.com/contents/overview-of-the-initial-evaluation-diagnosis-and-staging-of-patients-withsuspected-lung-cancer?source=outline_link&view=text&anchor=H57205858#H57205858

174

39-case of burn, how much fluids you will give in the 1st 8 hours?
Answer: of fluids
Parkland formula: 4 ml * kg * TBSA%
Reference: toronto notes

40-best screening method for prostate cancer?


A-digital rectal examination
B-cytology for prostate cancer
C-specific prostate antigen
answer: A

41-Theoretically which of the following cancer will prevented by vaccination?


A.ALL
B.CML
C.adult T cell leukemia
D.mycosis fungoides?
Answer: C
42-old man complaining of in ability to start urine after voiding = rectal exam revel distended full
bladder ?
A- Stress /
B- overflow /
C- reflex
answer: b

43- surgeon take graft from rectus muscle which artery should be dissected
A.
.superior epigastric
B.
inferior epigastric
C.
.superficial epigastric
answer : superior epigastric .. reviewed by my consultant
44-pt presented to you with Rt scrotal swelling , he said that it is increasing in size day after day , on
examination , it is not transilluminate , what will you do ?
Reassure him and get surgical opinion
Answer:
not translucent could be
Hemorrhage hydrocele further us or
Need to evacuate
Depends on other Q's
But most probable A
Since no trauma hydrocele
Source baily and love

175

45-Which of the following protect against some cancers?


Fibrate
Answer:
mcq is missing could be any vitamin or minerals
46-Pt complain of lower back pain after lifting heavy objects !
The vast majority of people with low back pain improve within 4 to 6 weeks without treatment or with simple
measures that can be performed at home.
Patients who are treated for acute back pain with bed rest have more pain and slower recovery than
ambulatory patients.
Therapy should focus on temporary symptomatic relief, to maximize patient comfort. We suggest a trial of
short-term treatment with either an NSAID or acetaminophen .
up to date
http://www.uptodate.com/contents/treatment-of-acute-low-backpain?source=outline_link&view=text&anchor=H25#H25
47-which indicate how long open wound ..in inflammatory process ?
* coverd by eshar
* covered by epithelium
* neovascularization
Answer: B
sorce : up to date
http://www.uptodate.com/contents/wound-healing-and-risk-factors-for-nonhealing?source=outline_link&view=text&anchor=H1070222553#H1070222553
48-RTA, splenic injury type 1, Rx?
* Laparotomy
* Diagnostic Laparoscopy with splenectomy
* Conservative
answer: C
Hemodynamically stable patients with low-grade (I to III) blunt or penetrating [32] splenic injuries without any
evidence for other intra-abdominal injuries, active contrast extravasation, or a blush on CT, may be initially
observed safely .
Hemodynamically unstable Based upon ATLS principles, the hemodynamically unstable trauma patient with
a positive FAST scan or DPA/DPL requires emergent abdominal exploration to determine the source of
intraperitoneal hemorrhage .
sorce : up to date
http://www.uptodate.com/contents/management-of-splenic-injury-in-the-adult-traumapatient?source=outline_link&view=text&anchor=H9#H9
49-Patient has deep ulcer with erythema on the sole of the foot, he is a known case of uncontrolled DM. what
is the treatment?
* Give oral antibiotics
* Admit the patient for debridement and deep culture
* Perform superficial culture
Answer: B
Adequate debridement, proper local wound care (debridement and dressings), redistribution of pressure on
the ulcer by mechanical off-loading, and control of infection and ischemia (when present) are important
components of treatment for all ulcers, regardless of stage and depth.
For most patients with diabetic foot ulcers, we suggest surgical (sharp) debridement rather than another
method .
up to date
http://www.uptodate.com/contents/management-of-diabetic-footulcers?source=outline_link&view=text&anchor=H28#H28

176

50-abdominal radiological investigation showed the inner and outer surface of intestine , what the pt has .
* Peroration
* Obstruction
B?
Proximal bowel dilation with distal bowel collapse Small bowel obstruction can be diagnosed if the more
proximal small bowel is dilated more than 2.5 cm (outer wall to outer wall) and the more distal small bowel is
not dilated .
up to date
http://www.uptodate.com/contents/epidemiology-clinical-features-and-diagnosis-of-mechanical-small-bowelobstruction-in-adults?source=outline_link&view=text&anchor=H2972181#H2972181
51- If there was inferior mesenteric artery thrombosis. Which artery will not be affected!
descending colon *IMA
sigmoid *IMA
splenic*IMA
cecum *SMA
answer:d cecum
http://radiopaedia.org/articles/inferior-mesenteric-artery

52- Bilateral breast cancer with :


paget disease *uni
lobular carcinoma * bilatral mucinus carcinoma *uni
ductal carcinoma*uni
answer: B .lobular ca

53- Case of RLQ pain and mass , what is Tx :


conservative .
surgery .
antibiotics .
answer: B . surgical drain then remove appendix
http://www.mayoclinic.org/diseases-conditions/appendicitis/basics/complications/con-20023582

177

54- which renal stone associated with infection and alkaline urine :
calcium oxalate
uric acid
struvite
cystine
answer: c .struvite
First aid

55- patient MVA and come with fracture of femur , tibia and fibula what is your action :
refer to orthopedic
answer: supportive then refer
http://www.ncbi.nlm.nih.gov/pmc/articles/PMC3524792/

56- patient did gastrectomy and need to take vit B12 for life ,, which cells are responsible :
goblet cells
chief cells
answer: parietal cells
https://en.m.wikipedia.org/wiki/Vitamin_B12

57-What is the best drug given to prevent postoperative thromboembolism?


LW heparine
Uf heparine
Warfarin
enoxaparin
Answer: d .enoxaparin answered by consultant
most to use now enoxaparin unless there is contraindication (Renal impairment results in decreased
elimination of enoxaparin and increased anticoagulant response or hx of heparin induced thrombocytopenia .
https://www.clevelandclinicmeded.com/medicalpubs/pharmacy/janfeb2003/enoxaparin.htm
34- done
35- done
36- done
37- done
58-Post appendectomy .. penicillin resistance? What antibiotic use?
Answer: Gentamicin

178

59- Long scenario about pt doing surgery, The key point in Q the pt 1st day develop fever + sob?
A.atelectasis
Answer: A

60-long scenario I don't remember exactly but there is upper limb hypertension and low or absent
lower limb pulse What is Dx?
A.Coarctation of aorta
Answer: A
Keys to diagnosis Coarctation of aorta include blood pressure (BP) discrepancies between the upper and
lower extremities and reduced or absent lower extremity pulses to palpation.
Reference: http://emedicine.medscape.com/article/895502-clinical#b4

61-Pt e CAD angio done for him Lift coronary 90% stenosis Right 40% What is the management :
A) Left coronary bypass
B) Bypass all vessel Stent
C)
D)
Answer: A
Reference: http://emedicine.medscape.com/article/164682-overview#a2
62- pt with melanoma above eyebrow which lymph node will examine ?
A) Cervical ..
B)
C)
D)
Answer: cervical
Reference: http://emedicine.medscape.com/article/849834-overview#a2
63-Generalized cervical lymphadenopathy + mild tenderness + low grade fever. What's the most likely
diagnosis:
A) lymphocytic lymphoma
B)hodgkin's lymphoma
Answer: difference between Hodgkin's and non-Hodgkin's lymphoma by examining the cancer cells under a
microscope.
Hodgkin's lymphoma associated with ReedSternberg cell.
References:
http://emedicine.medscape.com/article/201886-overview
http://emedicine.medscape.com/article/203399-overview

179

64-smoker with hilar mass what suspect


a.lymph node
b.squamous cell ca
c.adenocarcinoma
answer : squamous cell ca
It is impossible to defrentiate between them radiologicaly but squamous cell ca usually caused by heavy
smoking
Reference:http://radiopaedia.org/articles/squamous-cell-carcinoma-of-the-lung
65-what is the most common type of gall stone
a.Cholesterolbile pigmented
answer : cholesterol stone
Reference: http://www.mayoclinic.org/diseases-conditions/gallstones/basics/causes/con-20020461
66- fecal incontinence which level is affected
a.above c2
b.below c2
c.above t12
d.bolow t12
Answer: below t12
Pudendal nerve (s2-4) is responsible for inervation to pelvic floor muscles and external anal sphincter
Reference: http://www.nafc.org/spinal-cord/
67-Case Symptoms of cholecystitis what best intial modelity ?
Answer :U/S
Reference: Recall of surgery
68-best imaging for cyst in the breast
a. US
b. mammogram
Answer: A
-US is used to differentiate between solid and cystic lesion .
-mammogram visualize large cyst but not small microcystic lesion .
Reference : up to date
http://www.uptodate.com/contents/breast-cysts-clinical-manifestations-diagnosis-andmanagement?source=outline_link&view=text&anchor=H91862703#H91862703
69-Women with dysuria routine urine analysis show epithelial cell diagnosis :
a. Cerviacl erosion
b. Clymdia urethritis
c. Vaginal conta
d. renal Stone
Answer: C
Renal tubular cells are 1.5 to 3 times larger than white cells and are further distinguished by a round, large,
centrally-located nucleus.
Reference : up to date
http://www.uptodate.com/contents/urinalysis-in-the-diagnosis-of-kidneydisease?source=outline_link&view=text&anchor=H19#H19

180

70-testicular lymph node drainage ? ( I am not sure is it testicular or scrotum )


* .Left aortic lymph node
* .Superficial inguinal lymph node
* .Deep inguinal lymph node
Answer:B
Testis: to para-aortic lymph nodes
Lymphatic drainage of the testis follows the testicular vessels (in the spermatic cord) to the right and left
lumbar (caval/aortic) and preaortic lymph nodes at the second lumbar level.
scrotal: to superficial inguinal lymph nodes.
Reference: http://teachmeanatomy.info/pelvis/the-male-reproductive-system/testes-epididymis/
71-Which of the following protect against some cancers?
a. Fibrate
Reference : up to date
http://www.uptodate.com/contents/cancer-prevention?source=outline_link&view=text&anchor=H11#H11
72-which indcate how long open wound ..in inflammatory process ?
a.coverd by eshar
b.covered by epithilium
c.neovasculrization
Answer: B
Reference : up to date
http://www.uptodate.com/contents/wound-healing-and-risk-factors-for-nonhealing?source=outline_link&view=text&anchor=H1070222553#H1070222553
73-RTA, spleenic injury type 1, Rx?
a.Labratomy
b.Diagnostic Laproscpic with spleenectomy
c.Conservative
answer: C
Hemodynamically stable patients with low-grade (I to III) blunt or penetrating [32] splenic injuries without any
evidence for other intra-abdominal injuries, active contrast extravasation, or a blush on CT, may be initially
observed safely .
Hemodynamically unstable Based upon ATLS principles, the hemodynamically unstable trauma patient with
a positive FAST scan or DPA/DPL requires emergent abdominal exploration to determine the source of
intraperitoneal hemorrhage .
Reference : up to date
http://www.uptodate.com/contents/management-of-splenic-injury-in-the-adult-traumapatient?source=outline_link&view=text&anchor=H9#H9
74-Female with breast tender nodule 3*4 in the upper outer quadrant , what is the Next step?
a. FNA and cytology
b. Follow up
c. Reassurance
Answer: age !
For all women with a suspicious breast mass, a mammogram is the first diagnostic test performed. Frequently,
an ultrasound is also performed concurrently as a component of the evaluation. An MRI should be reserved
for diagnostic dilemmas. Breast masses in young women (under age 30 years) that are clinically consistent
with a benign lesion, such as a fibroadenoma, and in whom there is no family history of breast cancer, can be
first imaged by an ultrasound.
definitive diagnosis of a breast mass is made by a breast biopsy, which includes a fine needle aspiration, core
biopsy, or an open biopsy.
Reference : up to date

181

http://www.uptodate.com/contents/clinical-manifestations-and-diagnosis-of-a-palpable-breastmass?source=outline_link&view=text&anchor=H33#H33
75-Renal artery stenosis , Angie was done , what is next ?
A spiral (helical) computed tomography (CT) scan with intravenous contrast injection (called CT angiography,
or CTA) is a highly accurate noninvasive diagnostic test for imaging renovascular disease due to
atherosclerosis but less so for fibromuscular disease .
Reference : up to date
http://www.uptodate.com/contents/establishing-the-diagnosis-of-renovascularhypertension?source=outline_link&view=text&anchor=H760873#H16699522
76-Patient came for routine checkup CXR shows a unilateral calcified nodule on the upper zone of his
hung:
SCC
Hamartoma
Answer:hamartoma
Reference: http://emedicine.medscape.com/article/356271-overview
77-Something about post facial surgery, patient developed some neurological symptom, which branch
is effected options were branches of the main nerves?
Answer: Sensory or Motor?? Q is messing imp info
78-A patient presented with splenic injury after abdominal trauma. The surgeon decided to embolize
the splenic artery. Which of the following will be compromised?
Stomach fundus
Splenic flexure
Answer :stomach funds
Reference: http://radiopaedia.org/articles/splenic-artery

79-Patient underwent supradrelenectomy, you are worried not to injure which organ:
A-inferior vena cava
B-kidney
C-appendix
Answer: B
http://m.endocrinediseases.org/adrenal/surgery_complications.shtml

182

80-Old patient developed mass in the lung (CXR was attached)


:
SCC
adenocarcinoma
Answer: it depend on CXR and Hx
If it is arise in the peripheral, areas of the lungs. They also have a tendency to spread to the lymph nodes and
beyond. multiple sites in the lungs and spreads along the preexisting alveolar walls. It may also look like
pneumonia on a chest X-ray. and is very common in non-smoking women and in the Asian population.
adenocarcinoma
more common than adenocarcinomas arise in the central chest area in the bronchi. This type of lung cancer
most often stays within the lung, spreads to lymph nodes, and grows quite large, forming a cavity. SCC
http://www.webmd.com/lung-cancer/guide/lung-cancer-types
81- infant periumblical hernia , what you will do:
put plastic in mid abdomen
reassurance
do hernia repair before start school
answer: b?
Umbilical hernias: Most umbilical hernias do not require surgical repair until approximately age 5 years. For
that reason, managed by simple observation
http://emedicine.medscape.com/article/932680-treatment
82- 16 years old female . Fever and Chronic diarrhea for 10 months Post meal para umbilical
pain Sometimes blood mixed with stool
crohn
chronic pancreatitis
Answer: A
Crohn's disease Symptoms include watery diarrhea,abdominal pain,fever and weight loss.
http://www.webmd.com/ibd-crohns-disease/crohns-disease/crohns-disease

Read about:
Fertility Q
Case of cystitis.
Case of prostate cancer
Case about testicular torsion
case about renal colic
clear scenario about lateral epicondylitis
main difference between ( crohn's , ulcerative and IBS )
Dumping syndrome post surgery
Hernia (especially anatomical structures)
Carpal tunnel syndrome
Renal artery stenosis
Gallbladder stone
Prostatitis

183

Pediatrics

184

1- The most common parotid tumor in pediatrics?


Answer: ?
Pleomorphic adenoma then hemangioma.
Reference: http://www.ncbi.nlm.nih.gov/pubmed/10680869
*Mucoepidermoid if they asked about the most common malignant tumor.
2- Which of the following is the most common heart abnormality to get infective endocarditis?
a.
Tetralogy of fallot
Answer: A
In children, cyanotic heart disease is still the most common cause of endocarditis, and the risk does not diminish after
surgical repair as prostheses carry their own risk. Reference: http://www.ncbi.nlm.nih.gov/books/NBK2208/
Bacterial endocarditis can occur with many heart defects but is most common in aortic valve lesions, a patent ductus
arteriosus (unrepaired), tetralogy of Fallot, ventricular septal defects, coarctation of the aorta, and mitral valve prolapse
with mitral regurgitation. Reference: Bacterial Endocarditis by Ernest G. Brookfield, M.D.
3- Baby with greasy looking rash on face.
a.
Seborrheic dermatitis
Answer: A
4- Child presented with petechiae and his platelets is 15, otherwise healthy. What will you do for him?
a.
Splenectomy
b.
IVIG
Answer: ?
Observation vs. pharmacologic intervention highly debated; spontaneous recovery in >70% of cases within 3 months.
Treatment with IVIg or Prednisone if mucosal or internal bleeding, platelets <10, or at risk of significant bleeding.
Life-threatening bleed: additional platelet transfusion emergency splenectomy
Supportive: avoid contact sports and ASA/NSAIDs. Reference: Toronto Notes.
5- 4 weeks old boy with acute onset forceful non bilious vomiting after feeding. On abdominal examination: There is
olive mass at epigastric area. What is the 1st investigation should you do?
a.
PH monitoring
b.
Abdominal US
Answer: B (The diagnosis of choice is US and the most accurate test is an upper GI series)
Hypertrophic pyloric stenosis: When the vomiting persists, other clinical and biochemical findings may occur such as
dehydration, hypochloremic alkalosis and unconjugated jaundice. Late clinical manifestations include weight loss and
visible gastric peristaltic activity with a palpable pyloric olive. Prompt US diagnosis is important as these late findings
make the infants suboptimal candidates for surgery. Reference: Pubmed.
6- What is the most common site for mump?
a.
Parotid
Answer: A
Reference: http://reference.medscape.com/article/966678-clinical
7- Typical case of Poststreptococcal glomerulonephritis (PSGN). What will you do to confirm diagnosis?
Answer: ?
Occurs 1-3 wk following initial primary GAS infection of pharynx or skin.
Diagnosis is confirmed with elevated serum antibody titers against streptococcal antigens (ASOT, anti-DNAseB), low
serum complement (C3) Reference: Toronto Notes
If it Glomerular involvement occurred in less than a week after URTI its due to IgA Nephropathy

185

8- The most common chromosomal abnormality in a new infant:


a.
Down syndrome (Trisomy 21)
Answer: A
Most common abnormality of autosomal chromosomes. Reference: Toronto Notes.
9- Child came with wheezing and cough, diagnosed to have asthma and his dr. prescribed beclomethasone space
inhaler or nebulizer?? twice daily. you will be worried about:
a.
Growth retardation
b.
Extraocular problem
Answer: A
Reference: 3rd Edition UQU > Pediatrics > Q 58
NB. Corticosteroids inhalers can lead to oral thrush (yeast infection of the mouth).
10- Child presented with Asthma exacerbation. The patient did not respond to Beta agonist. What is your next step?
a.
Aminophylline
b.
Systemic steroid
Answer: B
Reference: First aid USMLE step 2CK
11- Cystic fibrosis
a.
chromosome 7 long q arm
b.
chromosome 7 short p arm
c.
chromosome 7 short q arm
d.
chromosome 7 long p arm
Answer: A
autosomal recessive, CFTR gene found on the long (q) arm of chromosome 7 resulting in a dysfunctional chloride
channel on the apical membrane of cells. Reference: Toronto Notes
12- Child was having rhinorrhea and then developed episodes of cough followed by vomiting. Which of these
vaccination may prevent him from having this disease?
a.
DTaP
Answer: A
A case of pertussis.
13- How to diagnose of pertussis?
a.
Nasopharyngeal swab
Answer: A
14- Child is complaining of severe throbbing unilateral headache, aggravated by light. What is the most likely
diagnosis:
a.
Migraine
b.
Cluster Headache
c.
Stress Headache
Answer: A
15- Tetralogy of fallot findings:
a.
Ventricular Septal Defect (VSD) + Overriding of the aorta + Pulmonary Stenosis + Right Ventricular
Hypertrophy (RVH)
Answer: A
Mnemonic: PROVe. PS, RVH, Overriding of the aorta, VSD.

186

16- Mother brought her 2 years old child to the ER with history of upper respiratory tract infection for the last 3 days
with mild respiratory distress. This evening the child started to have hard barking cough with respiratory distress. On
examination: RR 40/min, associated with nasal flaring, suprasternal & intercostal recessions. What is the most likely
diagnosis?
a.
Viral Pneumonia
b.
Bacterial Pneumonia
c.
Bronchiolitis
d.
Acute epiglottitis
e.
Laryngotracheobronchitis (croup)
Answer: E
Reference: 3rd Edition UQU > Pediatrics > Q 27
17- A case of TOF. How does it appear on X-Ray and echo?
Answer:?
ECG: RAD, RVH
CXR: boot shaped heart, decreased pulmonary vasculature, right aortic arch (in 20%)
Reference: Toronto Notes
18- Turner Features:
a.
Thick skin neck
Answer: A
Short stature, low set mildly deformed ear, triangular face, flat nasal bridge, epicanthal fold, web neck with or without
cystic hygroma, shield like chest with wide internipple space, puffiness of hands & feet, internal malformation mainly
coarctation of aorta in heart, horseshoe kidney. Reference: Nelson.
19- Child known case of DM 1, lost his consciousness at school. The last insulin injection is unknown.
a.
Take him to the hospital
b.
IV ranger lactate
c.
IM Glucagon
d.
Insulin
Answer: C
Because the patient could have hypoglycemic attack so give him shot of glucagon to increase his blood sugar.
Reference: http://www.healthofchildren.com/G-H/Hypoglycemia.html
20- A child with the history of repeated infections, failure to thrive and anemia. His older brother also has same
condition. what is the most likely diagnosis?
a.
Nutritional anemia
b.
Leukemia
c.
Lymphoma
d.
Haemoglobinopathy
Answer: ?
Repeated infection, Failure to thrive, Anemia and Family history of same condition make Hemoglobinopathy the best
answer. Reference : http://www.uptodate.com/contents/overview-of-the-clinical-manifestations-of-sickle-celldisease?source=machineLearning&search=haemoglobinopathy&selectedTitle=7%7E150&sectionRank=1&anchor=H13#
H19
21- A child was on clindamycin developed abdominal pain and watery diarrhea.
a.
Clostridium difficile
Answer: A
Pseudomembranous colitis

187

22- During delivery, when the doctor cut the umbilical cord, bleeding doesnt stop. Which of the following factors is
deficient in this case?
a.
Factor X
b.
Factor XI
c.
Factor XII
d.
Factor XIII
Answer: A?
Vit K deficiency: the synthesis of factors II (2) VII (7) IX (9) X (10), protein C, protein S.
Mnemonic: 1972 (in reverse order).
23- When does girls get puberty as compared to boys?
a.
1 to 2 yrs before
b.
2 to 3 yrs before
c.
Same time when boys do
d.
After boys
Answer: A
Average age for girls: 10 and a half years old.
Average age for boys: 11 and a half to 12 years old.
24- 6 year old child with fever, malaise, lymphadenopathy
hepatosplenomegaly. Shown is the CBC and bone marrow
aspiration slide. What is the most likely diagnosis ?

WBC

30,000 (4,500-11.000)

Hgb

Platelet

50,000 (150,000-400,000)

Peripheral Blast Cells

25%

a.
b.
c.
d.

and

Sickle sequestration.
Leukemia.
Malaria.
Leishmaniasis.

Answer: B
The diagnosis of acute lymphoblastic leukemia (ALL) is made when at least 30% lymphoblasts (French-American-British
[FAB] classification) or 20% lymphoblasts (World Health Organization [WHO] classification) are present in the bone
marrow and/or peripheral blood.
In addition, slides should be stained with myeloperoxidase (MPO) (or Sudan black) and terminal deoxynucleotidyl
transferase (TdT), unless another method is used, such as flow cytometry.
Bone marrow samples should also be sent for flow cytometry and cytogenetics. Approximately 15% of patients with ALL
have a t(9;22) translocation (ie, Philadelphia [Ph] chromosome).
Reference: http://emedicine.medscape.com/article/207631-workup#c11

25- Child came with Rt abdominal pain , jaundice, palpable tender liver, Dx ?
a.
HAV
Answer: A

188

26- An adolescent boy came to the clinic with unilateral gynecomastia. No other complaints. Everything was normal.
How will you manage?
a.
Reassure
b.
Give hormonal therapy
c.
Breast US
Answer: A
Gynecomastia (transient development of breast tissue) is a common self-limited condition seen in 50% of male during
puberty (but any discharge from nipple or fixed mass should be investigated). Reference: Toronto Notes.
27- Neonate born in home by a midwife presented with umbilical bleeding. What does he need?
a.
Vit K injection
Answer: A
28- Turner syndrome chromosomes?
a.
45X
Answer: A
29- Child with fever, malaise, LNs enlargement & mouth ulcers. What is the diagnosis?
a.
Herpes simplex virus infection type 1 (HSV 1)
Answer: A
Reference: Toronto Notes.
30- A boy with rickets (Picture was provided). What is the deficient vitamin?
a.
Vit D
Answer: A
31- Which of the following diseases has Mendelian mode of inheritance?
a.
Alpha thalassemia
Answer: A
Autosomal recessive
32- A child presented with fever and coryza, then watery diarrhea.
a.
Adenovirus
b.
Rotavirus
Answer: ?
Adenovirus: http://emedicine.medscape.com/article/211738-clinical
Rotavirus: http://emedicine.medscape.com/article/803885-clinical
33- Child presented with bronchiolitis. What is your management?
a.
Give Oxygen
Answer: A
34- 5 months old infant his parents were not able to bring him for his 4 months vaccination. What will you do?
a.
Arrange for 4 months vaccination
b.
Give missed vaccination and next appointment
c.
Give vaccine together during next appointment
Answer: ? Give missed vaccination (4 months) and arrange for the a next appointment (for the 6 months vaccination)

189

Children who have received some of their vaccines shots and then fallen behind schedule can catch up without having
to start over. Reference: http://www.medicinenet.com/childhood_vaccination_schedule/page3.htm
35- Child that throws a ball at you and draws a straight line and stacks few cubes on each other (they didnt
mention the number of cubes). What is the age?
a.
12 months
b.
14 months
c.
18 months
Answer: C
Check the table at the end of Pediatrics section.
NB. Drawing straight lines begins at the age of 2 years. Reference: illustrated textbook.
36- A child that can raise his head slightly when prone and smiles. He turns his head 180 degrees and has head lag
when you pull him to sit. How many old is he?
a.
4 weeks
b.
8 weeks
c.
12 weeks
d.
16 weeks
Answer: B
Check the table at the end of Pediatrics section
37- 14 years old boy comes to your clinic with swollen lips. He has similar episodes since 3 or 5 years. Deficiency of
which of the following caused his presentation?
a.
Factor D
b.
Anaphylatoxin inhibitor
c.
C1 esterase inhibitor
Answer: C
Hereditary angioedema is an autosomal dominant disease caused by low levels of the plasma proteins C1 inhibitor (C1INH). Reference: Medscape
38- Image of cells for a 2 years old with pancytopenia and something else. What is the most likely diagnosis?
a.
Malaria
b.
leishmaniasis
c.
Leukemia
Answer: ? Depends on the image.
Pancytopenia goes more with leishmaniasis
39- Mode of inheritance of neurofibromatosis.
Answer: ?
Autosomal dominant
40- 10 years old girl presented with fatigability, diarrhea and glossitis. What is the diagnosis?
a.
Vitamin B12 deficiency
Answer: A
Reference: Textbook of Oral Medicine
41- 6 years old boy presented with gingivitis, petechiae and rash. What is the diagnosis?
a.
Vitamin C deficiency
Answer: A

190

42- 15 years old has DM with dehydration?


a.
DKA
Answer: A

43- Infant suffer from groin rash that spare folds?


Answer: ?
Irritant diaper dermatitis: Shiny, red macules/patches, no flexural involvement. Reference: Toronto Notes.
44- A child with rheumatic heart disease allergic to Penicillin. What prophylaxis should be given before a procedure?
a.
IV amoxicillin
b.
IV vancomycin + IV gentamicin
c.
Oral vancomycin + gentamicin
d.
Oral amoxicillin
Answer: ? (Depends on the type of procedure and the ability to tolerate oral medications)
Most probable, the answer is B. Since Amoxicillin is type of penicillin and gentamicin generally not given PO.
Reference: http://www.cps.ca/documents/position/infective-endorcarditis-guidelines
Patients with rheumatic heart disease and valve damage require a single dose of antibiotics 1 hour before surgical and
dental procedures to help prevent bacterial endocarditis. Patients who had rheumatic fever without valve damage do
not need endocarditis prophylaxis. Do not use penicillin, ampicillin, or amoxicillin for endocarditis prophylaxis in
patients already receiving penicillin for secondary rheumatic fever prophylaxis (relative resistance of PO streptococci to
penicillin and aminopenicillins). Alternate drugs recommended by the American Heart Association for these patients
include PO clindamycin (20 mg/kg in children, 600 mg in adults) and PO azithromycin or clarithromycin (15 mg/kg in
children, 500 mg in adults). Reference: http://emedicine.medscape.com/article/891897-treatment
45- Whats the most common minimum side effect of DTP vaccine?
a.
low grade fever
b.
erythema over the injection site
Answer: ?
46- 7 years old boy developed Flu after receiving Flu vaccine. His father asked you about the reason. How will you
reply?
a.
Live attenuated vaccine has small risk of infection
Answer: A
47- Boy presented with unilateral nasal obstruction and foul smelling. What is the diagnosis?
a.
Foreign body
Answer: A
48- What is the inheritance mode of fanconi anaemia?
Answer: ?
Autosomal recessive.
49- Patient diagnosed with congenital adrenal hyperplasia. What is the next step?
Answer: ?
IV hydrocortisone

191

50- In Atrial septal defect (ASD) there will be:


a. fixed S2 split
Answer: A
Reference: Master the boards
51- Child with thumb sign on lateral Xray. What is the diagnosis?
a.
Epiglottitis
Answer: A
52- DTaP vaccine is against what?
a.
Whooping cough, tetanus, diphtheria
b.
Pertussis, tetanus, measles
c.
Rubella, tetanus diphtheria
d.
Whooping cough, rubella, diphtheria
Answer: A
53- A 2 years old child can run and play ball. Socially active but he refused to share his toys with other children. What
you will say to his parents?
a.
Delay motor
b.
Social impairment
c.
Well developed and normal response
Answer: C
54- Bilateral parotid swelling.
Answer: ?
http://www.ncbi.nlm.nih.gov/pmc/articles/PMC3528147/
local disease
mumps - more commonly children than
adults
parotitis
uveoparotid fever
sialectasis - especially if related to eating
Sjogren's syndrome
tumour infiltration

systemic disease
sarcoidosis
tuberculosis
alcoholism
myxoedema
cushing's disease
diabetes/insulin resistance - about 25% of patients with
overt or latent diabetes have bilateral asymptomatic
enlargement of the parotid glands
liver cirrhosis
gout
bulimia nervosa
HIV in children may cause bilateral parotid enlargement

55- Child presented with recurrent nasal congestion, rhinorrhea, sneezing, tearing eyes,..What is the diagnosis?
a.
allergic rhinitis
Answer: A
56- What is the causative organism of croup?
a.
Parainfluenza
Answer: A

192

57- Rheumatic fever prophylaxis?


a.
IM penicillin monthly
Answer: A
An injection of 0.6-1.2 million units of benzathine penicillin G intramuscularly every 4 weeks is the recommended
regimen for secondary prophylaxis for most US patients. Administer the same dosage every 3 weeks in areas where
rheumatic fever is endemic, in patients with residual carditis, and in high-risk patients.
Reference: http://emedicine.medscape.com/article/891897-treatment#showall
58- 11 years old patient with rheumatic fever and cardiac involvement. For how long he will require prophylaxis?
a.
5 years
b.
6 years
c.
10 years
d.
15 years
Answer: D
Rheumatic fever with carditis and clinically significant residual heart disease requires antibiotic treatment for
a minimum of 10 years after the latest episode; prophylaxis is required until the patient is aged at least 40-45 years
and is often continued for life.
Rheumatic fever with carditis and no residual heart disease aside from mild mitral regurgitation requires
antibiotic treatment for 10 years or until age 25 years (whichever is longer).
Rheumatic fever without carditis requires antibiotic treatment for 5 years or until the patient is aged 18-21
years (whichever is longer)
References: http://emedicine.medscape.com/article/236582-medication +
http://emedicine.medscape.com/article/891897-overview#showall
59- 10 years old child got strep throat infection followed by rheumatic fever. He was treated early without any
consequences. For how long he'll continue the rheumatic fever prophylaxis ?
a.
15 years
Answer: ?
8 - 11 years (requires antibiotic treatment for 5 years or until the patient is aged 18-21 years (whichever is longer))
Refer to Q58
60- Which of the following congenital heart disease is the least associated with infective endocarditis?
a.
ASD
b.
VSD
c.
PDA
d.
Pulmonary stenosis
Answer: A
Similar question: http://gradestack.com/Dr-Bhatia-Medical/Infective-endocarditis-is/0-3042-3177-15646-sf
Valvular aortic stenosis 13.3 percent
Coarctation of the aorta 3.5 percent
Primum atrial septal defect 2.8 percent
Ventricular septal defect (VSD) 2.7 percent
Tetralogy of Fallot (TOF) 1.7 percent
No child with secundum atrial septal defect, patent ductus arteriosus (PDA), or pulmonic stenosis had IE after
surgery.
61- 8 years old boy with petechiae all over his body. Lab results: low platelets and high creatinine level. what is
the diagnosis?
a.
ITP
b.
TTP
Answer: ?
HUS if no fever or altered mental status
HUS = Microangiopathic hemolytic anemia + thrombocytopenia + renal failure
TTP= HUS + fever and/ or altered mental status.

193

62- A boy came to your clinic with yellow discoloration of the eyes noticed 3 days back and hepatomegaly. His liver
enzymes are increased. What is the diagnosis?
a.
Hepatitis A
b.
Hepatitis B
c.
Hepatitis C
d.
Hepatitis D
Answer: A
63- What can increase fetal hemoglobin in sickle cell anemia (no hydroxyurea in the options)
a.
Folic acid
Answer: ?
5-azacytidine, butyric acid, erythropoietin and hydroxyurea

Reference: Advances in Pediatric


64- (long scenario) baby with cavernous hemangioma and have pleural effusion. ( he have other findings ). What you
will find in this baby?
a.
Pulmonary hemangioma.
Answer: ?
65- A child is always using abnormal sitting habits (W-Setting), what is the effect on the bones?
Answer: ?
Internal Femoral Torsion (Femoral Anteversion), it's the most common cause of intoeing > 2 years of age.
Management: Observation, takes 1-3 years to resolve. Surgery only if significant at > 10 years of age.
Reference: Pediatrics Kaplan for Step 2.
66- What is the Triple Antitoxoid?
a.
Tetanus, Diphtheria, Whooping cough
b.
Tetanus, Diphtheria, TB
c.
Diphtheria, Pertussis, Colorectal CA
d.
Diphtheria, Tetanus, Rabies.
Answer: A

194

67- Which of the following is most typically seen in 4 years old baby?
a.
Print name.
b.
Stand on one foot briefly.
c.
Copy triangle and square.
d.
Toilet trained.
Answer: D
Check the table at the end of Pediatrics section.
68- Child can set without support, cruises around furniture, uses chair to stand, say dada, crawl stairs. What is the
age of this child ?
a.
8 months.
b.
10 months.
c.
12 months.
d.
15 months.
Answer: B
Check the table at the end of Pediatrics section.
69- 2 weeks neonate passed unformed stool. What will you do?
a.
Prescribe formula milk.
b.
Give oral rehydration solution.
c.
Prescribe Lactose-free milk.
Answer: B?
70- (long scenario) child have 1mm defect in muscular atrial septum. What you will do?
a.
Surgical repair.
b.
Catheter repair.
c.
Reduce after load.
d.
Watchful waiting.
Answer: D
80-100% spontaneous closure rate if ASD diameter <8 mm. Reference: Toronto Notes.
71- A baby 8 month old breastfeed for 6 month normally. He devolved vomiting and jaundice after fruit juice.
What component in the juice the baby is allergic to?
a.
Glucose
b.
Fructose
c.
Sucrose
d.
galactose
e.
phenylalanine
Answer: B
Hereditary fructose intolerance due to deficiency of Fructose-1,6-biphosphate aldolase B in the Liver.
Reference: http://reference.medscape.com/article/944548-overview

195

72- What is the most common cause of acute bronchiolitis?


a.
Respiratory Syncytial virus (RSV)
b.
Adenovirus
c.
parainfluenza
d.
Mycoplasma pneumonia.
Answer: A
Reference: FA for the USMLE step 2 CK
73. To which part of body it can go ? (continuation of the previous question )
a. Spleen.
b. Bladder.
c.
kidney.
d. Liver.
Answer: D
RSV may be recovered from extrapulmonary tissues, such as liver, cerebrospinal fluid, or pericardial fluid
Reference: uptodate
74- What is the gene responsible for neurofibromatosis?
Answer: ?

The NF1 (peripheral neurofibromatosis or von Recklinghausen disease) gene is cytogenetically located
on the long (q) arm of chromosome 17, band q11.2 (17q11.2). The NF1 gene encodes for a cytoplasmic protein
called neurofibromin 1, which is a ras -GTPaseactivating protein that suppresses tumor growth, primarily by
inhibiting ras activity.

The gene for NF2 (bilateral acoustic neurofibromatosis or central neurofibromatosis) is cytogenetically
located on the long (q) arm of chromosome 22, band q12.2 (22q12.2). The NF2 gene codes for the protein
neurofibromin 2, also called merlin or schwannomin. Reference: Medscape.
75- A child came to you with Caf au lait spots in face and neck. Which of the following features can strengthen
your diagnosis?
a.
Port-wine stain.
b.
Axillary freckling.
Answer: B
Clinical diagnosis requires the presence of at least 2 of 7 criteria to confirm the presence of NF1. The 7 clinical criteria
used to diagnose NF1 are as follows, in the absence of alternative diagnoses:
Six or more caf-au-lait spots or hyperpigmented macules =5 mm in diameter in prepubertal children and 15
mm postpubertal
Axillary or inguinal freckles (>2 freckles)
Two or more typical neurofibromas or one plexiform neurofibroma
Optic nerve glioma
Two or more iris hamartomas (Lisch nodules), often identified only through slit-lamp examination by an
ophthalmologist
Sphenoid dysplasia or typical long-bone abnormalities such as pseudarthrosis
First-degree relative (eg, mother, father, sister, brother) with NF1
Reference: http://emedicine.medscape.com/article/1177266-overview
NF2: meningioma, schwannoma, glioma, neurofibroma, posterior subcapsular lenticular opacities accompanied by
external signs: hearing loss, ringing in the ears, and balance problems associated with vestibular nerve lesions, visual
deficits and cranial nerve palsies. Reference: Medscape.

196

76- Breastfeeding after delivery should start:


a.
Immediately
b.
6 hr
c.
8hr
d.
24 hr
Answer: A
Reference: https://www.betterhealth.vic.gov.au/health/healthyliving/breastfeeding-when-to-start
77- (long scenario) Child brought by his father in wheelchair complaining of knee swelling and history of falling on
his knee. What is the best investigation ?
a.
Joint Aspiration.
b.
X-ray.
Answer: B
A plain radiograph of the affected joint should be performed to rule out fractures, periostitis, avascular necrosis, bone
tumors, and bone dysplasias. Reference: Uptodate
78- A case of ambiguous genitalia. Which hormone you would like to check?
Answer: ?
17-hydroxyprogesterone (Hormone) should be measured promptly in all infants with nonpalpable gonads
presenting with genital ambiguity to exclude congenital adrenal hyperplasia (CAH) due to 21-hydroxylase deficiency.
This is the most common cause of genital ambiguity and can lead to life-threatening adrenal insufficiency within the
first weeks of life. Reference: Uptodate
79. 26 years old female G1P1 brought her two weeks old baby who cries a lot. He is on breastfeeding since birth,
he stop crying at night when she gave him formula milk. On examination the baby looks normal except for
increase gurgle sound. What is the most likely diagnosis?
a.
Paralytic ileus.
b.
Lactose malabsorption.
c.
Increase bowel gases.
d.
Breast milk jaundice.
Answer: B or C?

80- 5 years old girl with uncomplicated cystitis. what is the management?
a.
b.
c.
d.

Oral amoxicillin
IV cephalosporin
IM ceftriaxone
Sodium

Answer: A
81- Pediatric patient with classical symptoms of Diabetes + elevated blood glucose . What will you do next?
a.
Urine dipstick
b.
Genetic testing
c.
HbA1c
Answer: A
Urine dipstick to pick up DKA early and to prevent further complications.
Reference: http://guidelines.diabetes.ca/browse/Chapter34

197

82- What is the best initial investigation in a child presented with Croup ?
a.
X-Ray
b.
Pharyngeal Swab
Answer: A
Diagnosis of croup is clinical, based on the presence of a barking cough and stridor, and can be aided with a chest x-ray
showing a "steeple sign". Reference: Uptodate + Master the Boards.
83- A child presented with Croup. What to give in ER?
a.
epinephrine
Answer: A
Refer to the table at the end of Pediatrics section
84- Epiglottitis case.
a.
intubate
Answer: A
Refer to the table at the end of Pediatrics section
85- 7 years old girl was brought by her mother, she developed pubic hair and her height 70th percentile and weight
50th percentile. On examination there are no signs of puberty except pubic hair. Abdominal, chest, cardiac and renal
examination were normal. What is the most likely diagnosis?
a.
Congenital Adrenal hyperplasia
b.
45X (Turner syndrome).
c.
Premature Adrenarche.
d.
Normal puberty.
Answer: C
Premature adrenarche is when these changes begin early, before age 8 for girls and age 9 for boys.
Reference: http://www.medscape.com/viewarticle/759350_3
86- Malnourished baby with fair coiled hair and abdominal distension. What is the most likely diagnosis?
a.
Kwashiorkor
b.
Marasmus
Answer: A

87- Roll from prone to supine and vise versa + crab with only two fingers. What is the milestone.
Answer:?
4 - 6 months
Refer to the table at the end of Pediatrics section

198

88- Pediatric blunt trauma with duodenal coiled spring sign. what will you do?
Answer: ?
Duodenal hematoma:
In the pediatric age group, duodenal injury from a blunt abdominal trauma resulting in an intramural hematoma is rare.
This case illustrates the characteristic delayed presentation of a duodenal hematoma following a blunt injury from a
motor vehicle accident and the associated sequelae of delayed pancreatitis. Total parenteral nutrition and gastric
decompression provide an effective conservative treatment of the gastric outlet obstruction associated with this injury.
Surgical intervention is only reserved for those patients who continue to show the clinical and radiological signs of
complete high obstruction despite conservative management for three weeks.
For more details, See: http://www.ncbi.nlm.nih.gov/pmc/articles/PMC3088392/
89- Baby with tachypnea, cough, hemoptysis and bilateral lung infiltrates. what is the treatment ?
a.
Steroid
b.
Antibiotic
c.
Surgery
Answer:?
Reference: Hemoptysis in Children; See: http://medind.nic.in/ibv/t10/i3/ibvt10i3p245.pdf
90- Child with meningitis what to do immediately?
a.
Antibiotics
b.
Lumbar puncture
Answer: ?

Reference: Toronto notes.


91- Child sucking his thumb what to do?
Answer: ?
See: http://www.ncbi.nlm.nih.gov/pubmed/2676311
92- Which of the following is a cyanotic heart disease in children?
a.
ASD
b.
VSD
c.
Tetralogy of fallot
Answer: C
93- A child with developmental delay and constipation. Labs: low Na , low K and low ..?? what is the most likely
diagnosis ?
a.
CAH
b.
congenital chloride diarrhea
*other choices include syndromes
Answer: ?
Bartter syndrome? Reference: http://emedicine.medscape.com/article/238670-overview#showall

199

94- A child with normochromic normocytic anemia + splenomegaly. Blood smear was attached showing clear
spherocytosis. which of the following will be abnormal?
a.
plt
b.
retic
c.
wbc
d.
mcv
Answer: B
95- What is the most common intraabdominal tumor in children?
a.
hepatoma
b.
rhabdomyosarcoma
c.
ewing tumor
d.
Wilms tumor
Answer: D
Reference: http://www.ncbi.nlm.nih.gov/pubmed/15321038
96- A child presented with recurrent nasal congestion, rhinorrhea, sneezing and tearing eyes, what is the most likely
diagnosis?
a.
allergic rhinitis
Answer: A
97- What is the single most important risk factor for cerebral palsy?
a. prematurity
b. birth weight less than 1.5 kg
c. prenatal asphyxia
d. genetic mutations.
Answer: A
Prematurity is the single most important risk factor for cerebral palsy
Reference:https://books.google.com.sa/books?id=bonlLHarTFAC&pg=RA5-PT6753&lpg=RA5PT6753&dq=#v=onepage&q&f=false
98- A child with flat buttocks. What investigation will you do (celiac)?
a. Antibody
b. Intestine biopsy
Answer: A
Antibody testing, especially immunoglobulin A anti-tissue transglutaminase antibody (IgA TTG), is the best first test, although
biopsies are needed for confirmation; in children younger than 2 years, the IgA TTG test should be combined with testing for
IgG-deamidated gliadin peptides. Reference: http://emedicine.medscape.com/article/171805-workup

200

Reference: Toronto Notes

201

Vaccinations

Reference: Ministry of Health


Comparison between Croup, Tracheitis and Epiglottitis:

Reference: Toronto Notes

202

99-child with bad smell and tooth is good :::: tonsillitis with crept
Answer:
Tonsils The tonsils may be involved in the pathogenesis of bad breath in a small percentage of cases (perhaps 3
percent) [10]. Tonsillectomy based solely upon a complaint of bad breath should be avoided [33].
Some patients complain of small stones on their tongue or tonsils when they cough that have a foul odor (and often
lead patients to assume that they must have terrible breath). These stones are "tonsilloliths" that form in crypts of the
tonsils.
Ref : uptodate

100 -"2q about developmental milestone" 2 times


baby can smile at which age : 2monthe ,4 or 6
Answer:
social Smile 2 months
101 - child with chronic diarrhea , endoscopy showed sickle shaped parasite adherent to the bowl wall , what is it?
a:Giardia
b:Entemebea histolytica
Answer: A
Biopsy specimens from duodenum are often teeming with sickle-shaped Giardiatrophozoites,which are tightly
bound by the concave attachment disc to the villus surface of the intestinal epithelial cells
refrance : medescape
102 -child w fever, coryza, diarrhea:
Answer: Adenovrius
Typical symptoms and signs among children with rotavirus infection include vomiting, nonbloody diarrhea, and
fever .
Ref: uptodate
Fever, rhinorrhea, cough, and sore throat, usually lasting 3-5 days, are typical symptoms of adenoviral ARD
medescape
103 -scenario a child didn't take HBV VACCINE what u will give him
Answer: If the vaccine is not given after birth the baby may start the course of hepatitis B vaccines (in
combination with other childhood vaccines) beginning at six weeks, then at four and six months of age.

104 -2Q about coarctation of Aorta ( straight forward)


Coarctation of aorta: radio femoral delay
Baby with Deference in the Bp in upper and lower Extremities
Answer:
a clinical diagnosis of coarctation of Aorta is made if there is an absent or delayed
femoral pulse (when compared with the brachial pulse). if theres a murmur may be
associated with other cardiac defects, such as PDA, aortic stenosis, or ventricular
septal defect (VSD).
http://www.uptodate.com/contents/clinical-manifestations-and-diagnosis-of-coarctation-of-theaorta?source=search_result&search=coarctation+of+Aorta&selectedTitle=1%7E126
105 -Formula milk comparing to Breast milk contain more ..what
Answer:
Its contain more protein 1.5-1.9 g , Carbohydrate 7-8.6 g , sodium 0.65-1.1 mmol , Calcium 0.88-2.1 mmol ,
phosphorus 0.9-1.8 mmol and Iron 8-12.5 umol
Ref: illustrated textbook of paediaterics page 206 - Table 12.2

203

106 -breast feeding mother known history of seizure with phyntoin ask about breast feed ?
reassuranse
feeding after 8 hrs
Answer: reassuranse
Breast-Feeding Considerations
Phenytoin is excreted in breast milk; however, the amount to which the infant is exposed is considered small. The
manufacturers of phenytoin do not recommend breast-feeding during therapy.
phenytoin, carbamazepine and valproate are probably safe
http://www.uptodate.com/contents/phenytoin-drug-information?source=see_link&utdPopup=true
107 -child with croup what is the best initial investigation to diagnosis?
a X-Ray
b.pharyngeal swab
?
Answer: AP neck film show classic steeple sign from subglottic narowing
The diagnosis of croup is clinical, based upon the presence of a barking cough and stridor. Neither radiographs
nor laboratory tests are necessary to make the diagnosis. However, radiographs may be helpful in excluding
other causes if the diagnosis is in question.
Confirmation of etiologic diagnosis is not necessary for most children with croup, since croup is a self-limited
illness that usually requires only symptomatic therapy. When an etiologic diagnosis is necessary, viral culture
and/or rapid diagnostic tests that detect viral antigens are performed on secretions from the nasopharynx or
throat.
http://www.uptodate.com/contents/croup-clinical-features-evaluation-anddiagnosis?source=search_result&search=croup&selectedTitle=2%7E59#H23
108 -treatment of choice for kawasaki
Answer: Guidelines by the American Heart Association (AHA) and the American Academy of Pediatrics (AAP)
are available for the treatment of patients who fulfill the diagnostic criteria for Kawasaki disease (KD) and for
those who do not
the recommended initial therapy includes intravenous immune globulin (IVIG; 2 g/kg) administered as a single
infusion over 8 to 12 hours and aspirin (initial dose of 30 to 50 mg/kg daily divided into four doses).
IVIG : to reduce risk of coronary artery aneurysm
Aspirin : reduce risk of thrombosis
at risk of coronary arteries aneurysm within the first week of illness in about one third of affected children
http://www.uptodate.com/contents/kawasaki-disease-initial-treatment-andprognosis?source=search_result&search=kawasaki&selectedTitle=2%7E150
109 -child with jaundice, elevated direct bilirubin
Answer:
1-torontto note P72 (check the picture below)

204

2- http://www.pathophys.org/neonatal-hyperbilirubinemia/
110 -Case of gonorrhea eye infection in a newborn.. What to give
Answer: Infants with gonococcal ophthalmic disease should be hospitalized and observed for response to
therapy and for disseminated disease. Presumptive treatment should be started after obtaining cultures in
infants with organisms seen on Gram stain or in those with negative Gram stain, but who are considered to be
at high risk (eg, mother with no prenatal care, history of STDs, or substance abuse).
Treatment consists of a single dose of ceftriaxone (25 to 50 mg/kg, not to exceed 125 mg, intravenously or
intramuscularly) . A single dose of cefotaxime (100 mg/kg, intravenously or intramuscularly) is an alternative
option and is preferred for neonates with hyperbilirubinemia and those receiving calcium-containing
intravenous (IV) solutions (eg, parenteral nutrition) .
Topical antibiotic therapy alone is inadequate and is not necessary when systemic treatment is provided. The
eyes should be irrigated frequently with saline until the discharge clears.
Asymptomatic infants of untreated mothers Asymptomatic infants whose mothers have untreated
gonococcal infection are at high risk for acquiring infection. These infants also should receive systemic
treatment with a single dose of ceftriaxone (25 to 50 mg/kg, up to a total dose of 125 mg, administered
intravenously or intramuscularly) or cefotaxime (100 mg/kg, administered intravenously or intramuscularly)
and should be evaluated for chlamydial infection .
Prevention The most effective measure to prevent both gonococcal and chlamydial infections is to diagnosis
and treat these infections in pregnant women. In addition, prophylactic antibiotic eye therapy reduces the risk
of gonococcal conjunctivitis; however, it is not effective in preventing C. trachomatis conjunctivitis.
.
Neonatal prophylaxis . Prophylaxis can be administered up to one hour after birth to facilitate infant-family
attachment. The following are regimens recommended by the American Academy of Pediatrics.
Erythromycin (0.5 percent) ophthalmic ointment
Tetracycline (1 percent) ophthalmic ointment; tetracycline ophthalmic ointment is not available in the United
States
http://www.uptodate.com/contents/gonococcal-infection-in-the-newborn

205

111 -A child who came for 6 mo vaccination appt, his family report he had an anaphylaxis shock at 4 months
vaccination: what vaccine to give and what not to give? Or do an allergy test first? Or reassure and give them all?
Answer: http://www.uptodate.com/contents/allergic-reactions-to-vaccines
112 -2 month boy present with 2 cm of hemangioma in the back .. Wt is the ttt
close f/u **
excision
bblocker
]
Answer: The vast majority of infantile hemangiomas do not require any medical or surgical intervention.
Treatment options for clinically significant hemangiomas include the following:
Laser surgery
Surgical excision
Medication
http://emedicine.medscape.com/article/1083849-overview
113 -mother came to you that her child compline of spells for seconds
Answer: absence (petit mal): usually only seen in children, unresponsive for 5-10 s with arrest of
activity, staring, blinking or eye-rolling, no post-ictal confusion; 3 Hz spike and slow wave
activity on EEG
Seizure Mimics
Benign paroxysmal vertigo
Breath holding
Hypoglycemia
Narcolepsy
Night terror
Pseudoseizure
Syncope
TIA
Tic
refrance: torontto note
114 -20 days infant diagnosis as meningitis, his culture show gram negative bacilli. Which of following could be the
organism?
hemophiles influenza
E.coli
neisseria meningitides
Answer: E.coli
Note: group B streptococci (GBS) are the most commonly identified causes of bacterial meningitis, implicated in roughly
50% of all cases. Escherichia coli accounts for another 20%. Thus, identification and treatment of maternal
genitourinary infections is an important prevention strategy.[5] Listeria monocytogenes is the third most common
pathogen, accounting for 5-10% of cases; it is unique in that it exhibits transplacental transmission.[6]
- N . Meningitides it is gram negative diplococcus
- H . Influenza it is gram negative coccobacili
Ref : medescape

206

115 -young patient compline of scrotal pain, examination is normal, US normal , urine analysis show pyuria . what is
the next step :
Refer to surgery
Give him azithromycine and cefxime
Answer: Give him azithromycine and cefxime
This is a case of epididydimitis
This patient has epididymitis. In males 14-35 years of age, the most common causes are Neisseria gonorrhoeae and
Chlamydia trachomatis. The recommended treatment in this age group is ceftriaxone, 250 mg intramuscularly, and
doxycycline, 100 mg twice daily for 10 days. A single 1-g dose of azithromycin may be substituted for doxycycline. In
those under age 14 or over age 35, the infection is usually caused by one of the common urinary tract pathogens, and
levofloxacin, 500 mg once daily for 10 days, would be the appropriate treatment.
But testicular torsion cause severe pain without urinary symptoms diagnosis by Color Doppler ultrasonography will
show a normal-appearing testis with decreased blood flow. And it need ergent surgical intervention
116 -Patient with Ostium secundum atrial septal defect What you will see in his ECG
prolonged PR interval
left axis deviation
right axis deviation
Answer: http://emedicine.medscape.com/article/890991-workup#c6 An ECG demonstrates sinus rhythm,
often with evidence of right atrial enlargement manifested by tall, peaked P waves (usually best seen in leads II
and V2) and prolongation of the PR interval. The QRS axis is slightly directed to the right (+100), and the
precordial leads reveal right ventricular enlargement of the so-called volume overload type that is
characterized by an rSR' pattern in leads V3 R and V1 with normal T waves.
The QRS duration may be mildly prolonged because of right ventricular dilation. This mimics the finding in right
ventricular conduction delay.
A significant proportion (20-40%) of children with secundum atrial septal defect may not have abnormal ECG
[14]
findings.
Uncommonly, a patient with a secundum atrial septal defect may demonstrate a superior QRS axis with right
ventricular enlargement, mimicking findings observed in the ECG of a patient with an ostium primum atrial septal
defect

117 -best way to take urine sample:::


midstream
foley catheter
bag
Answer: Midstream
mid-stream urine sample means that you dont collect the first or last part of urine that comes out. This reduces the risk
of the sample being contaminated with bacteria from:
your hands
the skin around the urethra (tube that carries urine out of the body)
118 -Picture of growth chart all parameters was low what is the Dx?
A-Genetic
B- GH Def
Answer: depends on the chart
If the weight and hight proportionally small >> Chromosomal genetic
If the wight fall more that the hight FTT >> due to chronic illness , lack of intake
if hight fall more than the wight shrt stature >> endocrine .
Ref : uptodate

207

119
7 month baby with you discover that the baby has VSD and Asymptomatic otherwise healthy what are you
going to do?
a.close observation
b.Surgery
c.f/u after 6 month
Answer: f/u after 6 months
- No intervention is usually required for patients with small defects. These patients are typically asymptomatic and have
a reasonable expectation of spontaneous closure or decrease in the size of the defect over time.
-Patients who continue to have a murmur, but are otherwise asymptomatic and growing well at the 8- to 10-week visit,
are seen again by the pediatric cardiologist at approximately 12 months of age.
-If the murmur persists at the 12-month and the patient remains asymptomatic and clinically stable, no further
intervention is required. Echo follow-up is typically performed at three years of age for patients with membranous
defects. In those with a muscular defect, no echo is required if the patient remains asymptomatic.
-Asymptomatic patients with residual small defects are usually followed every two to five years for overall assessment.
http://www.uptodate.com/contents/management-of-isolated-ventricular-septal-defects-in-infants-and-children
120 patient with Barky cough ......what is the organism?
A.parainflunza
B.h.infelunza
Answer:
Croup (laryngotracheitis) characterized by inspiratory stridor, barking cough, and hoarseness. Is chiefly caused
by parainfluenza virus.
http://www.uptodate.com/contents/croup-approach-to-management
121 -pt with bronchiolitis ..what is the organism ?
Adenovirus
RSV
Parainfelunza
Answer: RSV
Bronchiolitis is typically caused by a virus. Respiratory syncytial virus (RSV) is the most common cause.
http://www.uptodate.com/contents/bronchiolitis-and-rsv-in-infants-and-children-beyond-the-basics
122 pt with respiratory sx..between attack ..he looks apathetic. .what u to investigate?
A.sputum culture
b.sweet chloride.
Answer:?
123 -Which of the following influenza vaccines is given intranasally
Answer: http://www.cdc.gov/flu/about/qa/nasalspray.htm

208

124 -Croup (parainfluenza)


Answer: Look for a child 1 to 2 years of age. Croup usually occurs in the fall or winter. Fifty percent to 75% of
cases are caused by infection with parainfluenza virus; the other common causative agent is influenza virus.
The disease begins with symptoms of viral upper respiratory infection (e.g.rhinorrhea, cough, fever). Roughly 1
to 2 days later patients develop a barking cough, hoarseness, and inspiratory stridor. The steeple sign
(describes subglottic narrowing of the trachea; is classic on a frontal radiograph of the
chest or neck.
Treatment
n Mild cases: Outpatient management with cool mist therapy and fluids.
n Moderate cases: May require supplemental O2, oral or IM corticosteroids,
and nebulized racemic epinephrine.
n Severe cases (eg, respiratory distress at rest, inspiratory stridor): Hospitalize
and give nebulized racemic epinephrine
Reference: usmle secrete- first aid
125 -Young patient swallowed pins, what are you going to do. ( serial xrays)
Answer:The peak incidence of foreign body ingestion is between the ages of 6 months and 3 years.1 2 Most
ingested foreign bodies pass through the gastrointestinal tract without difficulty, especially once they have
reached the stomach.
Children with foreign body ingestion typically do not require laboratory testing.
Laboratory studies may be indicated for workup of specific complications, such as potential infection
Chest/abdominal radiography
Most foreign bodies ingested by children are radiopaque (in contrast to inhalation, in which most are
radiolucent).
http://adc.bmj.com/content/84/2/165.full
http://emedicine.medscape.com/article/801821-workup
126 -apgar score :
Answer:
rapid scoring system that helps evaluate the need for neonatal resuscitation.
Each of 5 parameters:Appearance (blue/pale, pink trunk, all
pink) ,Pulse (0, < 100, > 100),Grimace with stimulation (0, grimace,grimace and cough), Activity (limp, some,
active), Respiratory effort (0, irregular, regular) is assigned a score of 02
at 1 and 5 minutes after birth.
n Scores of 810: Typically reflect good cardiopulmonary adaptation.
n Scores of 47: Indicate the possible need for resuscitation. Infants should
be observed, stimulated, and possibly given ventilatory support.
n Scores of 03: Indicate the need for immediate resuscitation.
Reference: first aid
127 -8 month child breastfeed for 6 month normally ,after he drinked fruit juice
He became sick and jaundice what should he avoid?
a-fructose https://www.nlm.nih.gov/medlineplus/ency/article/000359.htm
b-galactose
c- phenylalanine
Answer:(b)
Hereditary galactosemia is among the most common carbohydrate metabolism disorders and can be a life[1, 2, 3]
threatening illness during the newborn period.
First described in a variant patient in 1935 by Mason and
Turner, galactose-1-phosphate uridyltransferase (GALT) deficiency is the most common enzyme deficiency that
[4]
causes hypergalactosemia. Removing lactose largely eliminates the toxicity associated with newborn disease,
but long-term complications routinely occur.
http://emedicine.medscape.com/article/944069-overview

209

128 -Pnuemcocoal conjugat 13 what type of viccination ??


Answer:
Pneumococcal conjugate vaccine (called PCV13) protects against 13 types of pneumococcal bacteria.
PCV13 is routinely given to children at 2, 4, 6, and 1215 months of age. It is also recommended for children
and adults 2 to 64 years of age with certain health conditions, and for all adults 65 years of age and older. Your
doctor can give you details.
http://www.cdc.gov/vaccines/hcp/vis/vis-statements/pcv13.html
129 -Q down syndrome have bilateral
ventricular enlargment
Answer:
Children with Down syndrome are at a much higher risk for congenital heart disease. As a comparison: the incidence of
congenital heart disease in the general population is 0.8 percent. The incidence of congenital heart disease in children
with Down syndrome is between 40-60 percent.
Some heart defects can be left alone with careful monitoring while others require surgery to correct the problem.
The following types of heart defects in children with Down syndrome are discussed below.
Atrioventricular Septal Defects (AVSDs) These are the most common in children with Down syndrome.
Ventricular Septal Defects (VSDs)
Atrial Septal Defects
Patent Ductus Arteriosus
Tetralogy of Fallot

http://downsyndrome.nacd.org/heart_disease.php

130 -Q about CF chromosome


Answer: chromosome 7 called CFTR **

ADDED QUESTION: ( need to be more checked )


1.baby with recurrent infection tb, aspergillosis all type of infection with history of brothers death at 3 y
ear with same pr give? repeated
A. Influenza
B. BCG
C. Varicella
D. Polio
Answer:
2-2 year old complain of papule on the foot no itching pink pale not respond for antifungal?
A. Granuloma
Answer:
not clear question
3.A 7 y child and an U & L respiratory tract infection since birth with generalized joint pain aches his
uncle and brother have the same condition labs show high creatinine and BUN and +ve urine protein.
repeated
A.idiopathic fibrosis
B.Autoimmune
Answer:
4-a child have came to take the 6
month vaccination the mother mention on 4th month vaccination he developed an anaphylactic
What to do now ? repeated
A.Do allergen test
B.Give vaccine without DTP
C.Give vaccine with reassurance
D.Give vaccine and observe for 1h
Answer:

210

5-infant paraumbilical hernia , what you will do ?


A.put plastic in mid abdomen
B.ressurance
C.do hernia repari befour start school
Answer: Reassurance
http://pedsurg.ucsf.edu/conditions--procedures/umblical-hernia.aspx
6-Nine month child .. Take all vaccine regularly what vaccine should be given ?
Answer: 9 months vaccines : Measles , Meningococcal conjugate quadrivalent (MCV4)
Ref: according to saudi national vaccination schedule ->
7-Neonate with deafness , rash .. What the cause ?
A.Rubella
B.measles
C.CMV
Answer: A
Ref: https://pedclerk.bsd.uchicago.edu/page/torch-infections
Rubella infection Manifestation:
1.Blueberry Muffin rash due to extramedullary hematopoiesis
2. cataracts
3. Salt and Pepper retinopathy
4. Radiolucent bone disease (long bones)
5. UGR, glaucoma, hearing loss, pulmonic stenosis, patent ductus arteriosus, lymphadenopathy, jaundice,
hepatosplenomegaly, thrombocytopenia, interstitial pneumonitis, diabetes mellitus
8-6 month baby can't sit , hypotonia , crossed lower limb .. Which vaccine should be modified ?
A.change opv vaccine to ipv
Answer
9-12 yo , mild jaundice, splenomegaly , echogenic shadow of gall bladder
A.sca
B.thalassemia
Answer: hereditary spherocytosis
10-15 kg child .. What is the daily water requirement ?
Answer: 15Kg = 1250 ml.
Source: http://faculty.olin.edu/~jcrisman/Service/KWTWebNews/Nutrition/fluid.htm
Holliday Segar fluid requirement calculation:
1 to 10 kg: 100ml\kg
11 to 20 kg : 1000 ml + 50 ml\kg, for each Kg over 10kg.
over 20Kg: 1500 ml + 20 ml\kg for each kg over 20kg.
1Kg = 2.2lbs.
12-What is characterised about x fragile syndrome?
A.Obesity
B.Macrogonadisim
C.Smal
Answer: Fragile X syndrome (FRAX)
1 in 200 in frequency.
Most common cause inherited mental retardation.
FEATURES :
A)Characteristic craniofacial finding (large head,prominent forehead, jaw,ear)
B)Characteristic neurobehavioral profile including (mental retardation, autism spectrum disorder, pervasive
developmental disorder )
C)Macro-orchidism
D)Mild connective tissue disorder including (joint laxity,patulous eustachian tubes ,mitral valve prolapse)
Reference : ESSENTIAL NELSON OF PEDIATRICS

211

13-typical case of turner what another finding you find?


A.Cardiovascular abnormality .
Answer:A
Turner syndrome
FEATURES :
A)Characteristic facial appearance (low set mildly malformed ears,triangular face,flattened nasal bridge epicanthal fold )
B)Webbing of the neck with or without cystic hygroma
C)shield like chest with widened internipple distance
D)Internal malformations may include congenital heart defect (coarctation of the aorta is most common anomaly
Followed by bicuspid aortic valve
,post stenotic aortic dilation with aneurysm may develop )and renal anomaly (horseshoe kidney )
E)short stature is cardinal feature
F)hypothyroidism
Reference :ESSENTIAL NELSON OF PEDIATRICS
14-NF1 gene responsible for ?
A.Nureofibromatosis
Answer:A
Nf1 Gene located on chromosome 17q11.2
And coding for a tumor suppressor gene
Responsible for Neurofibromatosis type 1
Reference : ESSENTIAL NELSON OF PEDIATRICS
15-Autosomal recessive disease Both parents are carrier and phenotype normal what the chance they have a kid
with a disease ?
A.25
Answer:A
Ref: WIKIPEDIA
16-What come with Turner syndrome ?
A.Hypothyroid
B.DM
C.Addison's
Answer: A
(reference is nelson essential of pediatric
17-A baby 6 month show regurgitation after every meal he esophagus ph is low he is normally developing what is
the Rx?
A.Close follow up
B.Surgical fundal
C.Esophageal manometry
Answer:(no treatment is required may be A because I think this is a case of normal physiological gastroesophageal
reflux which happen in baby younger than 8months & presented with effortless regurgitation but otherwise the baby is
normal
reference: nelson
18-Child ingested a caustic material he present to ER craying drooling what to do 1st ?
A.Maintain airway
B.Activated charcoal
Answer:
Answer: ( A i think because activated charcol is contraindicated in causatic material ingestion
refernce: http://emedicine.medscape.com/article/813772-treatment#d10
19-What is the most common congenital abnormally cause infective endocarditis ?
A.ASD
B.VSD
C.Tetralogy of fallout
Answer:: (may be C because according to the references the cause of IE is unrepaired cyanotic congenital heart disease
referece: nelson essential + webs)

212

20-A baby with bilateral renal agenesis: (oligohydramnios)


Answer:
with bilateral renal agenesis: (oligohydramnios which is a sign for the disease during prenatal diagnosis step
reference:: http://www.ncbi.nlm.nih.gov/m/pubmed/264077/
21-A child alert, anterior fontanelle depressed, how much dehydration:
A.5 - 9
B.>9
Answer: A
Mild 5% : normal fontanelle
moderate 6-10% : Sunken slightly
severe >10 % : Sunken significantly
http://www.utmb.edu/pedi_ed/CORE/Fluids&Electyrolytes/page_09.htm
23-A child with hepatosplenomegaly, current infection. Brother died at 3 years with septic shock. How to give
vaccination?
A. Give all.
B. Dont give until 3 years.
C. Dont give live vaccines.
D.Dont give killed vaccines.
Answer: C
24-patient can't take BCG vaccine Because he deficiency in
A.IL
B.TNF gama
C. INF gama
Answer:C
25-child ride tricycle can't copy square what is the age
Answer:3 years

26-trauma to the fibular head what is the nerve injured?


A.Common peroneal nerve
B.presbycusis
C.Otosclerosis
D.otitis media
Answer: A
27-adolescent male with swollen parotid and salivary gland with dry eye and dry mouth, labs HLA, ANA and RF
are positive which of the following is appropriate treatment? repeated
A.Physostigmine
B.Artificial eye and saliva drops
Answer: B this is case of sjogrens syndrome
up to date
http://www.uptodate.com/contents/treatment-of-dry-eye-in-sjogrenssyndrome?source=outline_link&view=text&anchor=H3377156#H3377156
28-9 month old baby cannot sit by himself he is fisting his hand and crossing his leg Most likely his presentation
of? repeated
A.Normal child
B.CP
C.Down syndrome
Answer:

213

29-child can know color but with difficulty in making square


A.2 y
B.3 y
C.4 y
D.5 y
Answer:3 year
30-child can support his head when sit and loving when stare to him or cooing
A.4wk
B.8wk
C.12month
Answer: b
31-child with high fever 2 wk and abdominal distention and wt loss
A.bm
Answer:
32-what is the age of child should be know few word ?
A.6 month
B.8 month
C.12 month
D.24 month
Answer:
24 month
33-Child with typical symptoms of epiglottitis. (Tripod position) Diagnosis?

Answer:
http://www.uptodate.com/contents/epiglottitis-supraglottitis-clinical-features-and-diagnosis#H28
34-Child typical symptoms of croup.( laryngotracheobronchitis ) what is the Organism?
A.Parainfluenza virus
Answer: A

214

35-Developmental milestones Q: 5 words, hop on one leg ?


A.48 m
B.36 m
C.24 m
Answer:A
35-16 years old female . Fever and Chronic diarrhea for 10 months, Post meal periumbilical pain, Sometimes
blood mixed with stool?
A.crohn
B.chronic pancreatitis
Answer:
36-baby was playing with his father which suddenly his father looks the watch is not working baby become
agitated and refuse food what you will do : repeated
A.ichose upper Gi endo
Answer:
37-the most part in impaction of foreign body in the :
A.left bronch
B.right bronch
C.bifurcation
Answer: b
38-boy with glomerulonephritis after week he developed hemoptysis :
A.heno choinlin purpra
B.good pasture syndr.
C.rapid deterotion
Answer: b
39-Qs about osteosarcoma
Answer: http://www.uptodate.com/contents/osteosarcoma-epidemiology-pathogenesis-clinical-presentationdiagnosis-and-histology?source=search_result&search=osteosarcoma&selectedTitle=1~105
40-Qs about child development
Answer: http://www.uptodate.com/contents/developmental-behavioral-surveillance-and-screening-in-primarycare?source=search_result&search=child+development&selectedTitle=1~97
41. bacterial meningitis in 14 month child , Gram positive cocci, what is the management?
-amoxicillin
-amoxicillin and gentamicin
-ceftriaxone and vancomycin
-vancomycin
Answer: Ceftrixone and vancomycine
http://www.fpnotebook.com/mobile/neuro/ID/BctrlMngtsMngmnt.htm
42.3 year old child with UTI admitted what investigation to be done ?
-US
-Cystoscope
Answer: US
http://www.uptodate.com/contents/urinary-tract-infections-in-infants-older-than-one-month-and-young-children-acutemanagement-imaging-and-prognosis

215

Added Qs 7th update

1-child with fever, general swelling and dark colored of urine which best evaluate for this pt:
a- US
b-RFT
c-urine culture
d-urine specimen
Answer: D
Source: Urinalysis and sediment examination are crucial in the evaluation of patients with acute nephritic syndrome.
Look for the following:

Protein
Blood
Red blood cells (RBCs)
White blood cells (WBCs)
Dysmorphic RBCs
Acanthocytes
Cellular (ie, RBC, WBC) casts
Granular casts
Oval fat bodies

http://emedicine.medscape.com/article/239278-workup
2-Child history of hip pain x-ray of hip shows effusion What is the next step of management ?
a-Aspiration
b-Antibiotic
c-US
Answer:A
2 Case about neonate algorathium for resusuitation very long case i couldn't remember
Answer;
3-Baby wave his hand bye bye which developmental milstone reflect
???
abcAnswer 9 months
4-breast feeding mother known history of seizure with phyntoin ask about breast feed ?
a-reassuranse
b-feeding after 8 hrs
c-?
d-?
Answer:continous feeding
5-child with croup what is the best initail investigation to diagnosis?
a-c x-ray
b-pharyngeal swab
c-?
d-?
Answer;xray

216

6-Mother and her child visited the pediatrician for 6 months vaccination, however the mother stated
that her child was hospitalized after receiving the 4 months vaccination he develop anaphylaxis ,
what is the right thing to do?
A-Test the child for which antigen is allergic from
B-Give him steroid/antihistamine post the vaccination
C-Vaccinate him and discharge home
D-Vaccinate him and hospitalize the child for 1 hour
Answer : A
7-Children diagnosed to be hypertensive :
A-BP Above 90th percentile
B-BP Above 95th percentile
Answer : B
sBP and/or dBP>= 95th percentile for sex , age , height on >= 3 occasions
(Toronto note )
8-What is the injection that is routinely given to new-born to inhibit haemorrhage:
A-Vitamin K
B-VitaminC
C-VitaminD
D-VitaminE
Answer : A
(toronto note )
9-Child with URTI is complaining of bleeding from nose, gum and bruising the treatment is:
A-Prednisolone
B-IVIG
Answer : A
Explanation: ITP (idiopathic thrombocytopenic purpura) can occur in anyone at almost any age, but
these factors increase your risk:
1. Women are about twice as likely to develop ITP as men are.
2. Recent viral infection. Many children with ITP develop the disorder after a viral illness, such as
mumps, measles or a respiratory infection.
- Common medications used to treat idiopathic thrombocytopenic purpura include:
Corticosteroids. The first line of therapy for ITP is a corticosteroid, usually prednisone,
which can help raise your platelet count by decreasing the activity of your immune system.
Once your platelet count is back to a safe level, you can gradually discontinue taking the
drug..
Intravenous immune globulin (IVIG). If you have critical bleeding or need to quickly
increase your blood count before surgery, you may receive medications, such as immune
globulin, given intravenously.
Thrombopoietin receptor agonists. The newest medications approved to treat ITP are
romiplostim (Nplate) and eltrombopag (Promacta). These drugs help your bone marrow
produce more platelets, which helps prevent bruising and bleeding. Possible side effects
include headache, joint or muscle pain, dizziness, nausea or vomiting, and an increased risk
of blood clots.
Biologic therapy. Rituximab (Rituxan) helps reduce the immune system response. It's
generally used for people with severe ITP, and in those who corticosteroids don't help.
Possible side effects include low blood pressure, fever, sore throat and rash
Soure:http://www.mayoclinic.org/diseases-conditions/idiopathic-thrombocytopenicpurpura/basics/treatment/con-20034239

217

10-Cellulitis occurring about the face in young children (6-24 months) and associated with fever and
purple skin discoloration is MOST often caused by
A- group A beta hemolytic streptococci
B- Haemophilus influenzae type B
C- streptococcus pneumonie
D- staphylococcus aureus
E- pseudomonas
Answer : B ( Hib can lead to a violaceous or blue-purple color but it is not diagnostic )

.
Note : the most common organism can cause cellulitis at 6-24 month old is streptococcus
11- Child came with wheezing and cough and diagnosed with asthma and his dr. prescribe to him
beclomethasone nebulizer what most worried side effect of using it:
A- Growth retardation
B- Extaoriculer problem
Answer : Growth retardation
The most worrying potential systemic effects are adrenal insufficiency, growth suppression,
glaucoma and osteoporosis
reference
http://www.ncbi.nlm.nih.gov/pubmed/17602347
12- Attention Deficit Hyperactivity Disorder ( they give me the symptom not the diagnosis ) child
what is the manegment?
A- Ecitalpram
B- Atomoxetine
C- Olanzapine
D- Clonazepam
Answer : B
referene >>> uptodate

218

13- 4 years old brought by his parents, height < 5th percentile, they ask if he will remain short. what
you will do initially :
Parental height
Answer :
Parental height

219

14- 8 years old girl presented with fever, numerous bruises over the entire body and pain in both
legs. Physical examination reveals pallor and ecchymosis and petechiae on the face,trunk and
extremities. Findings on complete blood count includes a haemoglobin of 6.3 g/dl, white cell count
of2800/mm3 and platelet count of 29,000/mm3. Which of the following would be the MOST
appropriate treatment?
Answer :Dx>> Acute lymphoblastic leukemia. Treatment from Torronto>>

220

15- 3 years old his parents has TB as a pediatrician you did PPD test after 72 hr you find a10mm
enduration in the child this suggest
A- Inconclusive result
B- Weak positive result
C- Strong positive result
Answer : C

reference patient.info website


16- wich syndrome is associated with coactation of aorta:
A- down
B- patau
C- edwards
D- turner
answer: D
source: uptodate

221

17- Child with aspirin intake overdose what kind of acid-base balance:
A- Metabolic alkalosis
B- Metabolic acidosis
C- Respiratory alkalosis
D- Respiratory acidosis
Answer : B
The net effect of these changes in most adults is respiratory alkalosis or a mixed respiratory
alkalosis-metabolic acidosis. Pure metabolic acidosis is unusual in adults, but is more common in
children . Children with mild to moderate acute salicylate poisoning lose the respiratory drive and
are more likely to present with mixed metabolic and respiratory acidosis .
respiratory alkalosis first in oncent at first then metabolic acidosis.

18- Child with high-grade fever for 5 days and sore throat ,on examination there was tonsillitis and
white patches on the gingiva. No LN enlargement, ASO is negative. The most likely causative
organism is:
A- Coxsackievirus.
B- Herpes simplex virus.
C- EBV.
Answer : B

Acute herpetic gingivostomatitis


This is a manifestation of primary HSV-1 infection that occurs in children aged 6 months to 5 years.
Adults may also develop acute gingivostomatitis, but it is less severe and is associated more often
with a posterior pharyngitis.
Infected saliva from an adult or another child is the mode of infection. The incubation period is 3-6
days.Clinical features include the following:
Abrupt onset
High temperature (102-104F)
Anorexia and listlessness
Gingivitis (This is the most striking feature, with markedly swollen, erythematous, friable
gums.)
Vesicular lesions (These develop on the oral mucosa, tongue, and lips and later rupture and
coalesce, leaving ulcerated plaques.)
Tender regional lymphadenopathy
Perioral skin involvement due to contamination with infected saliva
Course: Acute herpetic gingivostomatitis lasts 5-7 days, and the symptoms subside in 2 weeks. Viral
shedding from the saliva may continue for 3 weeks or more.
medescape
[5]

19- 3 months baby with history of bronchiolitis, what is the cause?


A- RSV
Answer : A
USMLE FIRST AID STEP 2
20- Child presented with Croup presentation (barking cough etc.), what is the management?
A- Inhaled steroid
B- Inhaled epinephrine and oral steroid
C- Oral steroid with antibiotics
D- Empirical antibiotics
Answer : B
USMLE FIRST AID STEP 2

222

21- During otoscopy examination of a child, pulling the ear at which direction is going to help to see
tympanic membrane?
A- Anterior and inferior
B- Posterior and inferior
C- Anterior and superior
D- Posterior and superior
Answer :
B
straighten the patients ear canal by pulling the pinna up and back in children 3 years of age and
older and down and back in children younger than 3 years of age.

In general: In children, the auricle should be pulled downward and backward.


http://www.atitesting.com/ati_next_gen/skillsmodules/content/physical-assessmentchild/equipment/ap_ear_nose_throat.html
http://www.meddean.luc.edu/lumen/meded/medicine/pulmonar/pd/pstep18.htm
22- What is the daily fluid requirement for 15kg baby?
a) 1000
b) 1200
c) 1400
d) 1600
Answer : B
Explanation:
1. for the first 10 Kg we give 100 ml per each kilo
2. for the second 10 kg (from 10 to 20 kg) we give 50 ml per each kilo
3. for every kilo after 20 kg we give 20 ml per each kilo
So this baby is 15 kg which will be (1000 ml from the first 10 kg) and (5 kg multiplied by 50 = 250 ml)
so the babys daily requirement will be 1250 ml per day.
23- what is the most common abdominal tumor in children?
Answer : Wilms tumor (master the boards)
24- a case of a child with delayed developmental milestones flat nasal bridge wide forehead short
hand and feet what is the cause
A- genetic
B- dietry insufeciency
C- metabolism disorder
Answer :I am not sure
Genetic
https://quizlet.com/17898748/paediatric-syndromes-flash-cards/
25- child with septic arthritis how to manage
A -drainage and Abx
B -broad spectrum IV Abx
answer :
A
Medical management of infective arthritis focuses on adequate and timely drainage of the infected synovial fluid,
administration of appropriate antimicrobial therapy, and immobilization of the joint to control pain.
http://emedicine.medscape.com/article/236299-treatment

223

26-mumps in child what is the most common complication ?


A-orchitis
B -meningitis
answer : B
ref : uptodate
27- 15 month old child brought by the parent with peripheral and central cyanosis
He was diagnosed with cardiac problem but the parent doesn't know what exactly, most likely
diagnosis?
A- tetralogy of fallot
B- PDA
C- coaratacation of the aorta
answer : A
Explanation: This is a baby with a Cyanotic heart disease which includes most commonly tetralogy
of Fallot, tricuspid atresia, pulmonary atresia, and severe forms of Ebstein's anomaly of the tricuspid
valve. Acyanotic heart diseases include ASD, VSD, and PDA.
Source: http://pediatricheartspecialists.com/articles/detail/cyanotic_heart_defects

28- child abused sexually, in vaginal examination hymen rupture in which of the following indicate
rape?
A -6 o'clock
B-8 o'clock
C-3 o'clock
answer: A
source: forensic book..
https://books.google.com.sa/books?id=cLemGip2794C&pg=PA210&lpg=PA210&dq=position+of+hymen+rup
ture&source=bl&ots=gS4SIgf2Xj&sig=tmyZQiaLhEwUEOhptXVMA_BAdl0&hl=en&sa=X&ved=0ahUKEwi7t4f9
6tbJAhWKcBoKHWGwApcQ6AEIODAE#v=onepage&q&f=false
29- adolescent male with swelled parotid and salivary gland with dry eye and dry mouth, labs HLA,
ANA and RF are positive which of the following is appropriate treatment?
A-Physostegmine
B-Artificial tears and saliva drops
answer : B
- Sjgrens syndrome (SS) is a chronic autoimmune inflammatory disorder characterized by
diminished lacrimal and salivary gland function. The diminished exocrine gland function leads to the
sicca complex, a combination of dry eyes and dry mouth.
-SS is most common in women in their 50s and 60s but can affect adolescents and young adults, as
well as men.
-prominent parotid and lacrimal glandular enlargement, may result from SS.
-anti-Ro/SSA and/or anti-La/SSB antibodies, rheumatoid factor and ANA might be +ve but not
might be -ve not specific.
-Treatment of dry mouth due to salivary gland hypofunction aims to alleviate symptoms and prevent
complications with artificial tears and saliva.
http://www.uptodate.com/contents/diagnosis-and-classification-of-sjogrens-syndrome

224

30- 9 month old baby cannot sit by himself he is fisting his hand and crossing his leg
Most likely his presentation of?
A-Normal child
B-CP
C-Down syndrome
answer : B Signs of hypertonia include keeping the hands fisted, keeping the legs extended and
crossing the legs or ankles.
-Down syndrome >> hypotonia
31-Which of the is mostly associated sign with croup:
1- dysphonia
2- Cyanosis
answer : A hoarssness
uptodate
32- Child ate a number of iron tablets presented with severe symptoms including constipation and
bloody stool nausea and vomiting and drowsiness how would you treat him:
A- IV desferoxamine.
B- Dialysis
answer: A
explanation: Answer may be desfroxamine which is an antidote to Iron (chelating agent) used to
bring down iron levels, it is also used as treatment of Hemochromatosis (disease of iron overabsorption in the gut), another example to chelating agents is Penicilliamine used for the copper
overload in Wilsons disease.
33- 2 month old has diarrhea and his mother is worried from dehydration what will you advice the
mother:
A- Change milk
B- Oral rehydration solution
answer :B
-

Read about kawasaki disease


34- A child presented with diaper rash with sattalie lesion he was given local creams and steroid but
didnt work:
A- Local antifungal
answer: A
35-What is the most common cause of facial cellulitis in pediatrics?
answer :group A Streptococus beta hemolitic
36- Which of the following is a feature of Tetralogy of fallot?
A- Pulmonary stenosis
answer: A is correct.
Explanation: teratology of fallot consists of 4 pillers:
1. Pulmonary stenosis
2. VSD
3. over-riding of the aorta
4. Right ventricular hypertrophy

225

37-Medical director discovered cretinism in 90% of children in his village, when he analyzed the
water he found that it is deficient in iodine. The director wants to prevent and manage (the question
was asking about how to treat them) cretinism. What he is going to do initially?
A- Iodine supplementation
B- Thyroxin supplement (Levothyroxine).
C- TSH and T4 in 2 weeks.
answer: B
Explanation: INITIAL management should be supplementation of thyroxin, if the question was
asking about a definitive solution then we may choose to supplement water sources with iodine,
however they are asking about initial management.
38-What is the most common intra-abdominal tumor in children?
A) hepatoma (SCC)
B ) rhabdomyosarcoma
C) ewing tumor
D) Wilms tumor
answer: D
Explanation: although other tumors may occur at such age, Wilms tumor of the kidney is the most
common intraabdominal malignancy of childhood.
39-What is the single most important risk factor for cerebral palsy?
A) prematurity
B ) birth weight less than 1.5 kg
C) prenatal asphyxia
D) genetic mutations.
answer: A
ref : uptodate
40-What is the most common cause of hearing loss in children?
A) eustachian tube dysfunction
B) prenatal maternal infection
C) otitis media with secretion
Answer : most probably C
Source: Genetic defects are the most common cause of hearing impairment in newborns.

Ear infections, including secretory otitis media , and accumulation of earwax are the most
common causes of hearing impairment in infants and older children.
In older children, other causes include head injury, loud noise (including loud music), use of
aminoglycoside antibiotics (such as gentamicin) or thiazide diuretics, certain viral infections
(such as mumps), tumors or injuries that damage the auditory nerve, injury by pencils or
other foreign objects that become stuck deep in the ear, and, rarely, autoimmune disorders.
http://www.merckmanuals.com/home/children's-health-issues/ear,-nose,-and-throat-disorders-inchildren/hearing-impairment-in-children

41- Child came to the clinic with his mother was having ball, the doctor asked him to throw the ball
to him , he through it to the doctor and he went away to take the ball back
What is the developmental milestone of this child ?
Answer : 2 years = 24 Month

Movement/Physical Development
Stands on tiptoe
Kicks a ball
226

Begins to run
Climbs onto and down from furniture without help
Walks up and down stairs holding on
Throws ball overhand
Makes or copies straight lines and circles
http://www.cdc.gov/ncbddd/actearly/milestones/milestones-2yr.html
42- Baby with rash in diaper area was going to different private hospitals which they gave him 3
different steroid with no improvement, when you examine him you noticed that he has sattallite
spots in the thigh and buttucks .what you will give
A) Antifungal
B)Topical Steroid
C)Topical Antibiotics
Answer : A
Source: If candidiasis is suspected or proven by potassium hydroxide (KOH) preparation or culture, an antifungal
agent effective against yeast is indicated. The author has good experience in using hydrocortisone cream (1%)
twice daily and antifungal (nystatin cream, powder, or ointment; clotrimazole 1% cream; econazole nitrate cream;
miconazole 2% ointment; or amphotericin cream or ointment) cream after every diaper change or at least 4 times
per day.

http://emedicine.medscape.com/article/911985-treatment
43- 12 year boy came to the clinic complaining that he is short comparing to his colleagues, his
height was 155 and his weight is similar to 10 years boys.
His mother height is 145, father is 178, what is the expected height of the pt.
A)140_155
B)155_160
C)160_175
D)175_195
Answer : C - closest-

to calculate Mid-Parental Height:


For boys: [paternal height + (maternal height + 5 inches or 13 centimeters)] / 2
178 + (145+13) /2 = 336/2=168 +- 8 = 160-176 cm (Range)
For girls: [maternal height + (paternal height 5 inches or 13 centimeters)] / 2
44- 2 weeks infant came with history of SOB , sweating in forehead, what cardiac anomaly he has?
A)VSD
B)ASD or TOF (ONE OF THEM NOT SURE) ( ASD, Asymptomatic, TOF : Immediately after birth, severe
cyanosis is seen in patients with tetralogy of Fallot and pulmonary atresia or severe pulmonary stenosis)

C)trans-position of great vessels usually born at term, with cyanosis apparent within hours of birth.
D)PDA 3 week to 6-week-old infants can present with tachypnea, diaphoresis, inability or difficulty with feeding,
and weight loss or no weight gain.

Answer : Qs need more details, most probably A

Moderate VSDs
Babies may have excessive sweating as a consequence of increased sympathetic tone. This sweating is especially
notable during feeds. An important symptom is fatigue with feeding. Because feeding results in a need for increased
cardiac output, this activity may unmask exercise intolerance in a baby. Rapid breathing (tachypnea) at rest or with
feedings is usually present.

http://emedicine.medscape.com/article/892980-clinical
all information from MedScape website.
45- baby was playing with his father which suddenly his father looks the watch is not working baby
become agitated and refuse food what you will do :
i chose upper Gi endo
Answer : Repeated

227

46- boy with glumerilonephritis after week he develop hemoptysis :


A) heno choinlin purpra
B)good pasture syndr.
C)rapid deterotion
Answer : B
Explanation: If not Good-pasture syndrome it could be wegners granulomatosis vasculitis
47-best diagnosis tool for thalassemia is .
A)electrophoresis
Answer :A
48- baby full term flax..irritable.. give
A)ca
B)mg
C)ringer
D)glucose
Answer : A
http://emedicine.medscape.com/article/241893-treatment
49-what is the commonest cause of HTN in adolcent :
A- idipatic
B- renal
answer : A
explanation: disease or renal artery stenosi (secondary to renal involvement) .. Most childhood
hypertension, particularly in preadolescents, is secondary to an underlying disorder (Table 27). Renal
parenchymal disease is the most common (60 to 70 percent) cause of hypertension.23
Adolescents usually have primary or essential hypertension, making up 85 to 95 percent of cases.23
Table 32325 shows causes of childhood hypertension according to age.
source: http://www.aafp.org/afp/2006/0501/p1558.html

50- baby was playing with his father watch suddenly his father looks the watch is not working baby
become agitated abd refuse food what you will do :
A- i choose upper Gi endo
answer :
51- baby born full term flax-....enlarge labia the cause is :
A-estrogen
B-hcg
C- progestrone
.
answer : A
Newborn girls temporary changes in the vaginal area:
-The labia, may look puffy as a result of estrogen exposure in neonate.
-There may be a white fluid (discharge) from the vagina. This is called physiologic leukorrhea.
-There may also be a small amount of bleeding from the vagina.
-These changes are common and should slowly go away over the first 2 months of life.
https://www.nlm.nih.gov/medlineplus/ency/article/001911.htm

52-boy with asymptomatic hematuria the most important investigation in this case is :
cbc,or......
answer :urin analysis
hematuria cause

228

229

53-boy received vaccine in 4th month and he became febrile in 6th same vaccine what you will do :
A- give ...
B- dont give ...
C- allrgy test .
answer: -repeated- Q6
54-boy with asympatomitic hematurea the most important investigation in this case is:
A-cbc
B- .....
answer: reoeated
* uptodate pic
55- Rx of neonate with APGAR score 3
A- chest expansion
B- ventilation
C- IV fluids
answer: start resuscitation:
Initial stabilization (provide warmth, clear Airway if necessary, dry, and stimulate)
Breathing (ventilation and oxygenate)
Chest compressions
Administration of epinephrine and/or volume expansion
http://www.uptodate.com/contents/neonatal-resuscitation-in-the-delivery-room

230

56- child with eye itching for one month (no other symptoms in scenario), and have Hx of asthma,
what's the Dx?
a- bacteria conjunctivitis
b- viral conjunctivitis
c- venral conjunctivitis
answer: C
https://www.nlm.nih.gov/medlineplus/ency/article/001390.htm
57- tttx of de queverian syndrome??
Medical: first line of treatment is corticosteroids, ibuprofen can be used to relieve pain. surgery: for
unsuccessful medical treatment
Wikipedia + myoclinic
58- Child had bee sting. No symptoms other than edema and erythema at the site of the bee sting.
Mangment will be
A- oral steroid
B- antihistamine
C- admitte and observation
Answer :B
Antihistamines are used to treat mild urticarial symptoms. Catecholamines are needed in extreme
cases (eg, anaphylaxis).
59- 11 y.o ( typical bacterial maningitis case ) wich Ab will given :
A_ ceftriaxon and gentamycin
B_ Ampecillin and gentamycin
C_penicillin and gentamycin
D_vancomycin
Answer :D
MedScape
60- 5M child : mildestone !!
Table
61- Young girl came with s/s of anemia ,, there was splenomegaly ,, investigation : both direct and
indirect coomb's test were positive ,, most likely diagnosis ..?
A- autoimmune hemolytic anemia.
B- spherocytosis.
C- thalassemia.
Answer: A
Explanation: RBCs), or erythrocytes, that are sphere-shaped, rather than bi-concave disk shaped.
Spherocytes are found in hereditary spherocytosis and autoimmune hemolytic anemia. they have a high
osmotic fragility--when placed into water, they are more likely to burst than normal red blood cells.
These cells are more prone to physical degradation.They are most commonly found in
immunologically-mediated hemolytic anemias and in hereditary spherocytosis, but the former would
have a positive direct Coombs test and the latter would not. The misshapen but otherwise healthy red
blood cells are mistaken by the spleen for old or damaged red blood cells and it thus constantly
breaks them down, causing a cycle whereby the body destroys its own blood supply (autohemolysis).
62- girl came with upper arm bone pain investigation was given with anemia and high bilirubin and
high reticulocyte count what most appropriate next step:
A- Hb electrophoresis
B- arm x-ray
Answer: A
Explanation: We are suspecting Sickle cell anemia in this patient due to anemia (from sickle shaped
RBC rupture), High bilirubin (from RBC hemolysis) and high reticulocytes (young RBC trying to

231

compensate RBC loss), and most importantly Bone pains that can be explained by the occurance of
a Vaso-occlusive crisis known to happen to sicklers.
63- Which of the following used in SCA to increase production of HbF :
A- folic acid
B- hydroxycarbamide
C- forget other choices
Answer: B
Explanation: Hydroxyurea is used to elevate fetal hemoglobin.. (Hydroxycarbamide)
Folic acid replenishes the depleted folate stores necessary for erythropoiesis. Folic acid
supplementation is well established in the treatment of chronic hemolytic anemia. Although it is
proposed that folate in anemia raises hemoglobin levels and helps provide a healthy reticulocyte
response, the use of folic acid in patients with SCD is not well supported by the primary literature
64- infant with mother for routine check up , when the mother put the baby he was laughing and
when ha saw the Dr he tried to reach his mother what is most likely his age
A- 2 month
B- 4 month
C- 6 month
D- 8 month
answer : 6
65- boy came for routine check up , there was a murmur ,, echo done with only 2mm VSD what is
your management??
A- surgical repair
B- watchful waiting
C- medication can't remember the name
Answer: will depend on the age given in the Q, however a VSD this small will most likely be
monitored without intervention as long as it remains asymptomatic.
66- an infant came with cyanosis during feeding and crying ( there was many thing in scenario
) what is the management:
A- prostaglandin
B- surgical repair
Answer: Depend in the full history with which Cyanotic Congenital Heart Disease
Tetralogy of Fallot (Most likely the scenario about it and the treatment is surgical)
10% of all CHD, most common cyanotic heart defect diagnosed beyond infancy
embryologically, a single defect with hypoplasia of the conus causing:
VSD + right ventricle (RV) outflow tract obstruction (RVOTO) (e.g. pulmonary stenosis) + overriding
aorta + RVH
infants may initially have a L ~ R shunt and therefore are not cyanotic but the RVOTO is
progressive, resulting in increasing R ~ L shunting with hypoxemia and cyanosis
history: hypoxic "tet" spells
primary pathophysiology is hypoxia, leading to increased pulmonary vascular resistance
(PVR) and decreased systemic resistance, occurring in exertional states (e.g. crying, exercise)
paroxysm of rapid and deep breathing, irritability and crying
hyperpnea, increasing cyanosis often leading to deep sleep and decreased intensity of
murmur (decreased flow across RVOTO)
peak incidence at 2-4 months of age
if severe may lead to seizures, loss of consciousness, death (rare)
management: 0 2, knee-chest position, fluid bolus, morphine sulfate, propanolol
physical exam: single loud S2 due to severe pulmonary stenosis (i.e. RVOTO)
investigations
ECG: RAD, RVH

232

CXR: boot shaped heart (small PA, RVH), decreased pulmonary vasculature, right aortic
arch (in 20%)
treatment: surgical repair within first two years of life, or earlier if marked cyanosis, "tet"
spells, or severe RV outflow tract obstruction
Ebstein's Anomaly
congenital defect of the tricuspid valve in which the septal and posterior leaflets are malformed and
displaced into the RV leading to variable degrees of RV dysfunction, TS, TR or functional pulmonary
atresia ifRV unable to open pulmonic valves
RA massively enlarged, interatrial communication and patent foramen ovale (PFO) often exists
allowing R ~ L shunting
TR and accessory conduction pathways (WPW) are often present - often associated with
arrhythmia
cause: unknown, associated with maternal lithium and benzodiazepine use in 1st trimester
treatment: in newborns, consider closure of tricuspid valve + aortopulmonary shunt, or
transplantation in older children, tricuspid valve repair or valve replacement + ASD closure
Transposition of the Great Arteries (TGA)
3-5% of all congenital cardiac lesions, most common cyanotic CHD in neonate parallel
pulmonary and systemic circulations
systemic: body ~ RA ~ RV ~ aorta ~ body
pulmonary: lungs ~ LA ~ LV ~ pulmonary artery ~ lungs physical exam
no murmur if no VSD
newborn presents with progressive cyanosis unresponsive to oxygen therapy as the ductus
arteriosus closes and mixing between the two circulations diminishes; severe hypoxemia,
acidosis, and death can occur rapidly
if VSD present, cyanosis is not prominent and infant presents with CHF after a few weeks of
life
investigations
ECG: RAD, RVH
CXR: egg-shaped heart with narrow mediastinum ("egg on a string") treatment
prostaglandin E1 (Prostin VR'") infusion to keep ductus open until septostomy or surgery (arterial
switch procedure)
infants without VSD must be repaired within 2 wks to avoid weak LV muscle
Toronto Notes: Pediatrics P23
67- Long history (case of meningitis) child receive IV penicillin G, LP showed gram ve diplococcal,
family concerned about his young brother:
A- Admitted him to the hospital + observation
B- Give him IV penicillin
C- Oral Rifampicin
Answer: C
explanation: Rifampicin is the antibiotic of choice in cases of meningitis prophylaxis
68- Case of 12 years old boy on skate downstairs he felt and had perineum trauma, with bruises
over the scrotum, perineum, lower abdomen. Retrograde cystourethrogram show Extravasation of
the dye. Where is the injury:
A- Penile urethra
B- Urinary Bladder
C- Prostatic urethra
D- Ureters.
Answer: A
Anterior Urethral (composed of the penile and bulbar urethra) Injuries. This type of injury is seen
most commonly in blunt trauma, but is not usually associated with pelvic fractures. It results from a
strong blow to the perineum that causes the bulbar urethra to be crushed against the inferior border
of the pubic symphysis. This typically occurs in a fall astride, a straddle injury from a vehicle
accident, an assault, or from bicycle handlebars.

233

69- New born totally healthy with left thigh bruise all examination normal.
Prolonged PT , PTT
Your Dx:
A.
Hemophilia
B.
Factor 10 deficiency
C.
Idiopathic thrombocytopenic purpera
Answer: B
Vitamin K deficiency is factor 10, 9, 7, 2 prolonged PT, PTT

There were also missing questions about the following:


EBV
SCA (Including prevention, vaccination and vaso occlusive crisis)
Varicella virus
Developmental milestones
Read about vaccines: immune compromised pts, when to switch Dtap to Tdap

234

Obstetrics
and
Gynecology

235

1- Which of the following muscles will be affected in perineal tear during normal vaginal delivery?
Answer: ? (it depends on the degree of the perineal tear)
1st degree: vaginal mucosa affected only
2nd degree: involvement of perineal body muscles which includes:
bulbocavernous
superficial transverse perineal muscle
pubococcygeus muscles
3rd degree: involvement of the external anal sphincter and/ internal anal sphincter.
4th degree: extent through the anal mucosa.
Reference: UpToDate

2- Lactating mother complaining of breast tenderness, hotness and redness, diagnosed to have bacterial mastitis.
What will you recommend for her?
a.
Continue breast feeding, hot compressor and antibiotic.
b.
Discontinue breast feeding and give antibiotic to mother and baby.
Answer: A
Heat or ice packs, continued nursing/pumping, antibiotics (dicloxacillin/cephalexin) (erythromycin if pen-allergic).
Reference: Toronto notes and 3rd Edition UQU > Obstetrics and Gynecology > Q 385
3- Multigravida in labor with 60% effacement and dilated cervix (5 cm). After 1 hour she still has 60% effacement but
the cervix dilates to 6 cm. What will you do for her?
a.
expectant management
b.
oxytocin
c.
cervix ripening
d.
artificial rupture of membranes
Answer: D?
Artificial rupture of membranes (amniotomy) to stimulate PG synthesis and secretion; may try this as initial measure if
cervix is dilated. Toronto Notes.
4- Swelling at the labia majora for 6 month. It was aspirated and now it relapsed. What is your management?
a.
Aspiration
b.
Marsupialization
Answer: B
It is a Bartholin cyst, it needs to be drained with a simple incision and drainage but if it continued to recur, then
Marsupialization should be done. Reference: Master the board.

236

5- Primigravida week 16. She is RH negative. What is your next step?


a.
US
b.
Anti-D Rh immunoglobulin
c.
Rh antibody titer
Answer: C
Rh antibody titer during the initial prenatal visit if shes RH Unsensitized patients do not yet have antibodies to Rh positive blood. The goal is to keep it that way:
so any time that fetal blood cells may cross the placenta, anti-D Rh immunoglobulin (RhoGAM) are given.
Prenatal antibody screening is done at 28 and 35 weeks. Patients who continue to be unsensitized at 28 weeks
should receive anti-D Rh immunoglobulin prophylaxis.
At delivery, if the baby is Rh positive, the mother should be given anti-D Rh immunoglobulin again.
The patient is considered sensitized if she has a titer level more than 1:4.
If the titer is less than 1:16, no further treatment is necessary.
If it reaches 1:16 at any point during the pregnancy, serial amniocentesis should be done. Serial amniocentesis
allows for evaluation of the fetal bilirubin level. Reference: Master the board. 2nd edition P461
6- 42 years old female complaining of amenorrhea, night sweat and flushing for the last 6 months. What is the most
likely diagnosis?
a.
Hypothyroid
b.
Hyperprolactinemia
c.
Congenital adrenal Hyperplasia
d.
Pheochromocytoma
Answer: ?
7- Multigravida 34 week, her baby is breech, what you will do for her?
a.
expectant delivery
b.
CS
c.
External Cephalic Version
Answer: A
Expectant delivery until 36 week.
You should not perform ECV before 36 weeks, because the baby can turn into cephalic spontaneously.
Reference: Master the board
8- What is the most common cause of vaginal bleed?
Answer:
The most common specific causes in adult women who are not known to be pregnant are:
Anovulatory (dysfunctional uterine) bleeding > most common
Complications of an early, undiagnosed pregnancy
Submucous myoma
9- Which of the following non hormonal supplements will decrease the hot flashes in postmenopausal women?
a.
Black Cohosh
b.
Paroxetine
c.
Bromocriptine
Answer: B
SSRIs, venlafaxine, gabapentin, propranolol, clonidine. Reference: Toronto Notes
10- Why postmenopausal women develop osteoporosis?
a.
decrease progestin
b.
increase FSH
c.
decrease Estrogen
Answer: C

237

11- In polycystic ovarian syndrome, which of the following will be found on blood test?
a.
FSH:LH 1:3
Answer: A
12- Mom wants to know if her baby is having thalassemia or not. How you will investigate her antenatally?
Answer:
Screening tests for high risk population: CBC (MCV and MCH), Hb electrophoresis or high performance liquid
chromatography (HPLC)
Confirmatory tests: Chorionic villus sampling (CVS): between 10-12 weeks, Amniocentesis: between 15-16 weeks to
term. Reference: Toronto Notes
13- Young female complaining of whitish grey vaginal discharge. KOH test and clue test were positive. What is the
diagnosis?
a.
Gonorrhea
b.
Bacterial Vaginosis
c.
Trichomonas Vaginalis
Answer: B
Amsel criteria 3 out of 4 is diagnostic
ph >4.5
positive clue cells
discharge is thin, grey and homogenous
whiff test positive (KOH mount)
Reference: Amsel and 3rd Edition UQU > Obstetrics and Gynecology > Q 61.
14- 21 years old Female with negative pap smear. You should advise her to repeat pap smear every:
a.
6 months
b.
12 months
c.
18 months
d.
no repeat
Answer: B
Annual pap smear screening starting at age 21 years or no more than 3 years after becoming sexually active. Then
every 3 years after 3 normal consecutive test.
Women > 30 years old who have three consecutive normal tests screening (1 / 3 years).
Screening should be discontinuing for women > 60-70 years who have had 3 or more normal Pap smear.
Reference: 3rd Edition UQU > Obstetrics and Gynecology > Q 201 and Essential of obstetrics and gynecology
15- Woman with endometriosis. What is the best way to diagnose?
a.
laparoscopy
Answer: A
16- How ectopic pregnancy occurs at the cellular level?
a.
Disappearance of zona pellucida.
b.
Fertilization at ampulla tube.
c.
Persistence of Zona pellucida.
d.
Fast division of blastomere.
Answer: A
As cilia degenerate the amount of time it takes for the fertilized egg to reach the uterus will increase. The fertilized egg,
if it doesn't reach the uterus in time, will hatch from the non-adhesive zona pellucida and implant itself inside the
fallopian tube, thus causing the pregnancy.
Reference: Wikipedia and Clinical reproductive medicine & surgery book - textbook of clinical embryology

238

17- Nursing mom want to conceive but not at the coming two years. What will you recommend for her?
a.
vaginal ring
b.
combined OCP
c.
progestin injection
d.
patch
Answer: C
18- 20 years old sedentary female complaining of amenorrhea for the last 6 months and her BMI is 20.
a.
Prolactinoma
b.
Anorexia
c.
depression
Answer: ?
19- Lactating lady who didn't take the MMR. What will you advise her to do?
a.
Take the vaccine and stop feeding for 72 hour
b.
It is harmful for the baby
c.
She can take the vaccine
Answer: C
Reference: 3rd Edition UQU > Obstetrics and Gynecology > Q 39
20- A patient with ectopic pregnancy of 2.5*3.0 size. hCG is 5000. The patient is stable. What will you do?
a.
wait and watch
b.
laparotomy
c.
laparoscopy
d.
D&C
Answer: C
Reference: Toronto Notes OB24 + 25
21- A couple came to your clinic. They are trying to conceive for the last 3 months with no success. The girl had
appendectomy before marriage. She also has an aunt who is her uncles wife (not blood related) with down
syndrome. What should be done?
a.
try some more
b.
clomiphene
c.
laparoscopy
d.
semen analysis
Answer: A
22- A girl who hit puberty few months back and complains of spotting in between her periods. What will you tell her?
a.
She has PCOS
b.
She needs to take OCPs
c.
If tests were normal its not a disease
Answer: C
23- A 50 years old lady came with signs and symptoms of menopause. What picture describes his report best?
a.
increased LH and FSH
b.
decreased FSH and LH
c.
increased FSH decrease LH
d.
increase LH decrease FSH
Answer: A

239

24- A patient with cervical carcinoma. What viruses are thought to be major culprits?
a.
HPV 43 and 44
b.
HPV 16 and 18
c.
HPV 6 and 11
Answer: B
25- A long scenario of a lady with vaginal infection, has strawberry cervix. What is the organism?
a.
Trachomatis
b.
Bacterial vaginosis
c.
Gonorrhea
d.
Trichomonas vaginalis
Answer: D
26- A woman with vaginal infection that grows gram negative diplococci. What is the organism involved?
a.
N.gonorrhoeae
b.
HSV
c.
Candida
Answer: A
Reference: Toronto Note
27- What is the role of metformin in PCOS?
a.
Decrease glucose level
b.
Decrease insulin resistance
c.
Anti-androgenic
d.
Menstrual regulation
Answer: B
The hallmark mark of PCOS is insulin resistance.
28- Pregnant with monochorionic twins in week 27. One of them died, what to do?
Answer:
If fetal assessment after 26 weeks of gestation suggests impending death rather than demise of one twin of a
monochorionic pair, we suggest prompt delivery of both twins rather than expectant management given the high risk
of neurologic impairment in the surviving co-twin. Reference: http://www.uptodate.com/contents/twin-pregnancyprenatal-issues?source=search_result&search=monochorionic+twins+one+f+them+die&selectedTitle=1%7E150#H24
29- Pregnant lady with UTI, which Antibiotic is contraindicated?
a.
Tetracycline
Answer: A
Certain antibiotics should be avoided during pregnancy. For example, tetracyclines such as doxycycline and
minocycline can damage a pregnant woman's liver and discolor a developing baby's teeth.
Antibiotics generally considered safe during pregnancy:
Amoxicillin, Ampicillin, Clindamycin, Erythromycin, Penicillin and Nitrofurantoin.

240

30- Hormonal replacement therapy prevents which of the following?


a.
Postmenopausal symptoms
b.
Osteoporosis
c.
Coronary artery disease
d.
Stroke
Answer: ?
Indications of HRT: primary indication is treatment of menopausal symptoms (short-term).
It can be use to prevent/treat osteoporosis (long-term) and premature ovarian failure. Reference:
http://www.guidelines.co.uk/obstetrics_gynaecology_urology_mm_hrt#.VkoeS8p-_qM
31- Patient known case of PCOS and wants to get pregnant.
a.
clomiphene citrate
Answer: A
Reference: Toronto Notes
32- A patient with premature rupture of membranes for more than 18 hours (long scenario with a lot of details).
Which of the following give the patient high risk for GBS infection?
a.
Rupture of membranes for more than 18 hours.
b.
Family hx of GBS infection.
Answer: A
33- What is the best way to know the date of pregnancy?
a.
LMP
b.
Ultrasound
c.
Fundal height
Answer: B
The 3 basic methods used to help estimate gestational age (GA) are menstrual history, clinical examination, and
ultrasonography. The first 2 are subject to considerable error and should only be used when ultrasonography facilities
are not available. Reference: Medscape.
34- A pregnant woman who has a child with down syndrome. Shes concerned about having another child with down
syndrome. What is the best test to rule out down syndrome in the second trimester?
a.
Amniotic fluid sample
b.
Chorionic villous sampling
c.
Triple investigation
Answer: ?
Second trimester screening tests include:
Triple investigation is done between 15- 20 weeks. Its sensitivity is about 65% for trisomy 21. Patients with
positive screen should be offered U/S or amniocentesis for confirmation.
Quadruple screen can improve the detection rate for Down syndrome to about 81%.
Second trimester confirmatory test: Amniotic fluid sample.
*See the tables at the end of OB/GYN section.
References: Toronto Notes and American Family Physician Journals http://www.aafp.org/afp/2007/0901/p712.html
35- 46 years old woman comes with amenorrhea for 6 months and flushes at night that disturbs her sleep. What is
the best investigation to make your diagnosis?
a.
LH
b.
FSH
c.
Estrogen
d.
Progesterone
Answer: B

241

36- Premature menopause starts before...?


a.
30
b.
35
c.
40
d.
45
Answer: C
37- Which of the following is equally effective to laparoscopy in a patient with unruptured small ectopic pregnancy?
a.
Methotrexate
Answer: A
Reference: Toronto Notes
38- Megaloblastic anemia in pregnant.
a.
give folate
Answer: A
Reference: http://www.ncbi.nlm.nih.gov/pubmed/8612357
39- Which of the following viruses can cross the placenta?
a.
rubella
b.
mumps
c.
HBV
Answer: A or C ?
*HBV: depends if the disease is active or not.
Diseases that can cross the placenta:
All TORCH; Toxoplasmosis Others: e.g. syphilis Rubella, CMV, HSV
HIV, Chicken pox, CMV, Erythema Infectiosum (Fifth Disease), Hepatitis B . Reference: Toronto Notes
40-A pregnant lady with gestational diabetes. What medication will you prescribe for her?
a.
Metformin
b.
...zide
c.
...zone
d.
insulin
Answer: D
41- A pregnant lady had a child with 3500 grams with the use of forceps, presented to you 20 days postpartum with
whitish vaginal discharge but with no itching or cervical tenderness. On examination cervix is pink. Microscopic
examination reveals epithelial cells with leukocytes. What would you do for your patient?
a.
Dipstick urinalysis
b.
Pelvic ultrasound
c.
Reassure
d.
Metronidazole
e.
Culture discharge
Answer: C
Reference: http://www.ncbi.nlm.nih.gov/books/NBK288/
42- Female patient came with signs and symptoms of PCOS. Lab results: FSH= 1.5, LH= 10. What is the most
likely Diagnosis?
a.
PCOS
Answer: A

242

43- Female came to infertility clinic because she cannot conceive. She has a history of three elective abortion and
D&C in the past. She refused to be examined. What is the most likely diagnosis?
a.
Sheehan Syndrome
b.
Asherman Syndrome
Answer: B

44- Which of the following is an absolute contraindication for breastfeeding?


a.
active HIV
Answer: A
Contraindicated if mother:
is receiving chemotherapy or radioactive compounds:
has HIV/AIDS, active untreated TB, herpes in breast region:
is using >0.5 g/kg/d alcohol or illicit drugs
is taking medications known to cross to breast milk
Reference: Toronto Notes
45- (long scenario) Pregnant lady in 1st trimester (12 weeks) on iron trial, complaining of fatigue and shortness of
breath. CBC show: Hb = low , MCV = 70 , hematocrit = normal , reticulocyte = 10% . What is the most likely diagnosis?
a.
Physiological.
b.
Iron deficiency.
c.
Thalassemia.
Answer: B
46- 28 weeks pregnant (nuli), presents with generalized fatigue, BP 162/95, 3+ protein in urine. What is your next
step?
a.
Mgso4
b.
Labetalol
c.
MethylDopa
Answer: A

We recommend administration of antenatal corticosteroids for all pregnant women at 23 to 34 weeks


Betamethasone is given if < 34 weeks who are at increased risk of preterm delivery within the next seven days.
Reference: Master the Board + http://www.uptodate.com/contents/antenatal-corticosteroid-therapy-for-reduction-ofneonatal-morbidity-and-mortality-from-preterm-delivery
47- 45 years old gravida 4 para 3, week 8 pregnant. Last pregnancy she had a down syndrome baby so shes asking
for checkout regarding Down syndrome. What are the complications that you are going to tell her when you take
consent?
a.
rupture of amniotic sac
b.
unintended miscarriage
Answer: B
Chorionic villus sampling (10 - 12 wk):1-2% risk of spontaneous abortion. Reference: Toronto Notes

243

48- A patient presented to the ER with severe RLQ pain and positive B-HCG. What is the Diagnosis?
a.
ruptured ectopic pregnancy.
Answer: A
49- Which infection has high mortality rate in pregnant?
a.
Toxoplasmosis
b.
Syphilis
c.
CMV
Answer: C, Answerd by OBS/GYN consultant.
50- 63 years old female. Pap smear showed atypical squamous cells of undetermined significance (ASCUS). You
gave her local estrogen and after one week pap smear results still showing ASCUS. What will be your next step?
Answer: ?
Refer to the chart at the end of OB/GYN section.
51- What antibiotic is safe in pregnancy?
a.
Ciprofloxacin
b.
Amoxicillin
c.
Chloramphenicol
Answer: B
Some of the antibiotics that may be prescribed safely during pregnancy include: Amoxicillin, Ampicillin, Clindamycin,
Erythromycin, Penicillin, Gentamicin, Ampicillin-Sulbactam, Cefoxitin, Cefotetan and Cefazolin
52- What is the most common cause of secondary amenorrhea?
a.
Pregnancy
Answer: A
53- Mild Preeclampsia patient (34 weeks of gestation). What will you do next?
a.
Immediate C/S (culture and sensitive)
b.
Observe BP
Answer: B
+ betamethasone for lung maturation and mgso4 for seizure prophylaxis. Reference: Master the boards.
54- A 10 weeks pregnant lady, known case DM and HTN. Her BP 160/95. How will you manage this case?
a.
Observation
b.
Termination of pregnancy
c.
ACEI
Answer: ?
Methyldopa, nifedipine or labetalol. Reference: Master the boards.
The available data are insufficient to rule out unrecognised adverse effects of early and prolonged use of -blockers in
pregnancy. Methyldopa is a drug of first choice for control of mild to moderate hypertension in pregnancy. Oral
hydralazine, a direct vasodilator, is effective as monotherapy or as add-on therapy to methyldopa in the long term
management of chronic hypertension in pregnancy. Reference: http://www.medscape.com/viewarticle/406535_6
55- Typical case of PCOS. Biopsy showed Endometrial hyperplasia. What is the cause?
a.
Unopposed estrogen
Answer: A
Reference: Medscape

244

56- Multipara, 38 weeks pregnant. Cervical os is 7 cm with cord prolapse?


a.
CS
Answer: A

57- Diabetic female complaining of itchy vaginal discharge?


a.
Candidiasis
Answer: A
Refer to the table at the end of OB/GYN section.
58- Old woman, atrophic vaginitis, low mood, osteoporosis, .etc. what is your management?
a.
Estrogen
b.
SSRI
Answer: A
local estrogen replacement (ideal): Premarin cream, VagiFem tablets, or Estring
oral or transdermal hormone replacement therapy (if treatment for systemic symptoms is desired)
good hygiene Reference: Toronto Notes
59- Pregnant women (30 Something but definitely less than 38) with BP baseline 80 now present with 140 or 160
(Not sure), LL edema. +2 proteinuria, no change in LFT (not sure). Admitted to the hospital. What will you give her?
a.
Betamethasone
b.
Labetalol
c.
MgSO4
Answer: ?
Refer to Q46
60- A female patient presented with green vaginal discharge and pruritus?
a.
Trichomonas
Answer: A
Refer to the table at the end of OB/GYN section.
61- What is the treatment of choice for Trichomoniasis?
a.
Fluconazole
b.
Metronidazole
Answer: B
Check the table at the end of OB/GYN section.
62- What is the drug of choice for eclamptic seizure?
a.
Phenytoin.
b.
Diazepam.
c.
Magnesium Sulfate.
Answer: C
Although the definitive treatment is delivery, and the seizure should be controlled with magnesium sulfate.
Reference: Toronto Notes.

245

63- Pregnant woman in 3rd trimester, with vaginal infection (discharge), after delivery the baby got eye infection
(conjunctivitis and discharge). What is the most likely cause?
a.
chlamydia
b.
gonorrhea
Answer: B
Gonococcal conjunctivitis tends to occur 2-7 days after birth but can present later.
Chlamydial conjunctivitis usually has a later onset than gonococcal conjunctivitis; the incubation period is 5-14 days.
Reference: http://emedicine.medscape.com/article/1192190-clinical

64-Pregnant at 5 weeks of gestation. Cervical incompetence was diagnosed. what will you do?
a.
cerclage now
b.
cerclage at 12-13 weeks
Answer: B
Usually at the end of the 1rst trimester and removed in the third trimester.
Emerging evidence indicates that progesterone suppositories are superior to cerclage in preventing preterm labour late
in pregnancy. Reference: Toronto Notes.
65- A female patient cant get pregnant for 3 years. Recently she developed breast milk. What is the most likely
diagnosis?
a.
Hyperprolactinemia.
Answer: A
66- A pregnant lady presented with flank pain. On examination there was tenderness. Labs showed leukocytosis and
positive nitrate, what is the management?
a.
Admission to treat pyelonephritis
b.
Drink plenty of fluids
c.
Start antibiotics
Answer: A
Reference: Kaplan Obstetrics and Gynecology USMLE step 2.
67- A pregnant lady with a positive OGTT, what is your action?
a.
Repeat the test
b.
Check HgA1c
c.
Start insulin
d.
Do a random blood glucose
Answer: C
Reference: Master the Boards.
68- Which of the following is a side effect of OCPs?
a.
Breast cancer
b. Cervical cancer
c.
DVT
Answer: C
Reference: Master the Boards.

246

69- Pregnant G1P0 who has a history of travelling 1 year ago, came for check up. Result shows HIV +ve. What is the
action in this case ?
a.
Acyclovir for the mother during 1 week.
b.
(something) given to the baby after delivery.
c.
(something) given to the mother and baby after delivery.
d.
Acyclovir is contraindicated.
Answer: ?
In an HIV-infected pregnant woman who has never been exposed to antiretroviral medication, HAART should be started
as soon as possible, including during the first trimester. Combination antiretroviral therapy should be offered in all
cases. As zidovudine (ZDV) is the only agent specifically shown to reduce perinatal transmission, it should be used
whenever possible as part of the highly active antiretroviral therapy (HAART) regimen.
All HIV-exposed infants should receive zidovudine.
Reference: http://emedicine.medscape.com/article/1385488-overview#showall

70- Female G2P2 complaining of irregular menstruation for 6 months, history reveal normal babies with normal
deliveries but she did D&C after the second delivery for retained part of placenta, investigations (I cant remember),
what is your diagnosis ?
a.
Ashermans syndrome.
b.
Polycystic ovary.
Answer: A
Intrauterine adhesion (or intrauterine synechiae) is a condition in which scar tissue develops within the uterine cavity.
Intrauterine adhesion accompanied by symptoms (eg, infertility, amenorrhea or hypomenorrhea) is also referred to as
Asherman syndrome. Its primarily caused by curettage for pregnancy complications.
Reference: Uptodate
71- Postmenopausal women complaining of itchy vulva and erythema of the labia majora and sometimes bleed. On
examination there is a pea shaped mass. What is the diagnosis?
a.
Bartholin gland cyst.
b.
Bartholin gland carcinoma.
c.
Bartholin abscess.
Answer: B ?
It is generally recommended that women over age 40 with a Bartholin cyst or abscess undergo drainage and biopsy of
the gland to exclude the possibility of an underlying carcinoma.
https://quizlet.com/53890288/obgyn-neoplasia-flash-cards/
72- What is Adenomyosis?
a.
Presence of endometrial tissue and gland in Uterine Ligament.
b.
Presence of endometrial tissue and gland in Uterine Muscle.
c.
Presence of endometrial tissue and gland in Cervix.
d.
Presence of endometrial tissue and gland out Uterus.
Answer: B
It occurs when endometrial tissue, which normally lines the uterus, exists within and grows into the muscular wall of
the uterus.
73- Woman pregnant 42 Gestation was given gel for induction of labor. Later she was found to have 4 cm dilated
cervix and 70% effaced. However her contractions lasted 2 minutes and fetal HR dropped from 140 to 80. What to
do?
a. Give Oxygen
b. Immediate c-section
c. Give some medication
Answer: B
Reference: Toronto Notes. OB47

247

74- What is the most common sign and symptom in placental abruption ?
a. Vaginal bleeding
b. Uterine tenderness
c. Uterine contractions
d. Fetal distress
Answer: A
Placental abruption is mainly a clinical diagnosis with all the above findings. the most common symptom is dark red
vaginal bleeding with pain during the third trimester of pregnancy (80%) and abdominal or uterine tenderness (70%).
Bleeding may occur at various times in pregnancy:
Bleeding in the first trimester of pregnancy is quite common and may be due to the following: miscarriage (pregnancy
loss) ectopic pregnancy (pregnancy in the fallopian tube)
Bleeding in late pregnancy (after about 20 weeks) may be due to the following: placenta previa or placental abruption.
Reference: AlQassim Booklet. Q84

75-A patient did a PAP smear and the result showed high grade intraepithelial cells. what will you do next?
a.
Cone biopsy
b.
Total hysterectomy
c.
Colposcopy
Answer: C
Refer to the chart at the end of OB/GYN section
76-What is the best treatment for premenstrual dysmorphic syndrome?
a.
OCP
b.
Fluoxetine
c.
Bromocriptine
Answer: B
Reference: Master the Boards.
77- (long scenario) female patient with bacterial vaginosis. What is the most appropriate treatment?
a.
Ceftriaxone.
b.
Clindamycin.
c.
Ampicillin.
d.
Fluconazole.
Answer: B
Refer to the table at the end of OB/GYN section
78- 30 year old lady having whitish vaginal discharge, odorless and labial erythema. What is the most likely cause ?
a.
Candida infection.
Answer: A
Candida vulvovaginitis: Erythematous, excoriated vulva/vagina with thick white discharge without odor.
79- 34 year old lady pregnant, complaining of amenorrhea, bleeding, and abdominal pain. B-hcg done showed levels
of 1600, she was given methotrexate. One week later she still has abdominal pain despite analgesia. B-hCG done
showed 6000 units. What is the best management?
a.
Continue methotrexate.
b.
Exploratory laparoscopy.
c.
Salpingectomy
d.
Salpingostomy
Answer: B

248

80- 15 years old girl, presented with pain during menses only. What is the nerve and the muscle involved?
Answer: ?
Dysmenorrhea is due to prolonged uterine contractions and decreased blood flow to the myometrium.
Nerve: uterine nerve. Reference: Medscape
81- Sexually abused child vaginally, the hymen tear will be in which position?
a.
2 o'clock
b.
4 o'clock
c.
6 o'clock
Answer: C
Blunt penetrating trauma to the vaginal orifice produces a characteristic pattern of injury; bruising, lacerations
and/or abrasions are typically seen between the 4 and 8 oclock positions of the hymen.
Reference: WHO -Child sexual abuse

82- What is the best time to estimate the chorionicity of the twins on ultrasound?
Answer: ?
Assessment of chorionicity: Ultrasonography is an effective prenatal tool for determining amnionicity and chorionicity.
The optimal time for performing the ultrasound examination is in the first trimester after 7 weeks (sensitivity 98
percent), with lower but acceptable accuracy in the early second trimester.
Reference: http://www.uptodate.com/contents/twin-pregnancy-prenatal-issues
83- A 19 years old white female. Weight: 52 kg & Height: 145 cm with no history of twins in the family, got
spontaneously pregnant by twins. What is the risk factor in this case?
a.
Age
b.
Race
c.
Weight
Answer: ?
Dizygotic twins are the most common. Identifiable risk factors include IVF, newly discontinued OCP, race (e.g. certain
African regions), increased maternal age, geography, family history, or ovulation induction.
References: Kaplan Lecture Note; Ob/Gyn and Toronto Notes
84- The most common cause of postpartum hemorrhage(PPH) is..?
a.
Uterine atony
Answer: A
Uterine atony is the most common cause of PPH. Reference: Toronto Notes.
85- 20 years old female, presented with amenorrhea, short stature and webbed neck. Which hormone will be
affected?
a.
Decreased estrogen
Answer: A
Reference: Uptodate.
86- A mother delivered her first baby with cleft lip and palate. What is the percentage of recurrence for her next
pregnancy?
a.
1%
b.
4%
c.
15%
d.
20%
Answer: B

249

87-Pregnant lady with a history of 2 SVD and 1 CS. How will you manage?
a.
Natural vaginal delivery trial
b.
Admit at 38 weeks for CS
Answer: A
88- A pregnant lady 34 weeks gestation presents with headache, epigastric pain and blurred vision with a BP of
163/89. What is the best course of management?
a.
Stabilize the general condition, magnesium sulphate, .
b.
Deliver immediately
c.
Give magnesium sulphate, stabilize the general condition.
Answer: ?
ABC > MgSO4 + hydralazine + Delivery
Refer to Q46

89- Pregnant female is HIV positive. What is the most likely mode of transmission to the baby?
a.
Through the placental
b.
Through the blood cord
c.
By breast feeding
d.
Through hand contamination of mother
Answer: C
90- What is the best place to take a cervical sample for Pap smear?
a.
Endocervix Cancer,
b.
Exocervix
c.
Transformation
d.
Vaginal vault
Answer: C
The transformation zone is the site of origin for most cervical neoplasia and should be the focus of cytology specimen
collection. References: http://www.cytopathology.org/specimen-collection-adequacy-requisition/
https://books.google.com.sa/books?id=0flWgd3OJLEC&pg=PA11&lpg=PA11&dq=-#v=onepage&q&f=false
91- What to do after a Pap smear show atypical changes ?
a.
Hysterectomy
b.
Guided colposcopy biopsy
c.
Excisional biopsy
Answer: B
Refer to the chart at the end of OB/GYN section.

250

92- Uterine Myoma was found incidentally by US. What will you tell the mother?
a.
Regress normally
b.
May cause abortion
Answer: ?
Almost 90 percent of women with fibroids detected in the first trimester will have regression in total fibroid volume
when re-evaluated three to six months postpartum, but 10 percent will have an increase in volume. Regression may be
less in women who use progestin-only contraception.
Reference:http://www.uptodate.com/contents/pregnancy-in-women-with-uterine-leiomyomasfibroids?source=outline_link&view=text&anchor=H26#H26 + http://www.ncbi.nlm.nih.gov/pmc/articles/PMC3136622/
However, it's possible that fibroids could cause infertility or pregnancy loss. Submucosal fibroids may prevent
implantation and growth of an embryo. In such cases, doctors often recommend removing these fibroids before
attempting pregnancy or if you've had multiple miscarriages.
Reference: http://www.mayoclinic.org/diseases-conditions/uterine-fibroids/basics/complications/con-20037901
Similar Q was found in AlQassim Booklet (Q509) and UQU (Q64):
Pregnant women has fibroid with of the following is True?
a.
Presented with severe anemia .
b.
Likely to regress after delivery .
c.
Surgery immediately.
d.
Presented with antepartum hemorrhage.
Answer: B
Fibroids may also be the result of hormones. Reproductive hormones like estrogen and progesterone can stimulate
cell growth, causing fibroids to form. During pregnancy, your influx of hormones may cause your fibroids to grow in
size. After pregnancy and during menopause most fibroids begin to shrink, due to a lack of hormones.
93- A female tried to get pregnant for one year. She is healthy and her husband is known to be healthy as well. What
to do 1st?
a.
Hysterosalpingography
b.
Serum prolactin level of the woman
c.
Semen analysis
d.
Serum progesterone
Answer: C
94- What are the steps in staging cervical cancer?
a.
Colposcopy, cystoscopy, hysteroscopy
b.
Colposcopy, Colonoscopy, Hysteroscopy
c.
Laparoscopy
Answer: A
95- A pregnant during labor. Her cervical opening is 6 cm. Which stage?
Answer: ?

251

96- Pregnant lady, everything was normal except hemoglobin was low. What is the next step?
a.
Iron
b. Nothing
c.
Folate
d. B12
Answer: A
Iron supplementation is almost universally recommended during pregnancy to correct or prevent iron deficiency.
Reference: http://www.ncbi.nlm.nih.gov/pmc/articles/PMC1447059/
97- What is the best investigation to establish ectopic pregnancy?
a.
HCG
b.
Laparoscopy
c.
Pelvic U/S
Answer: B
-HCG: 85% of ectopic pregnancies demonstrate abnormal -HCG doubling
U/S: is only definitive if fetal cardiac activity is detected in the tube or uterus
Laparoscopy: for definitive diagnosis/ invasive
Reference: Toronto Notes + Master the boards
98- 40 y/o woman G3P1, history of 2 months pregnancy. Upon examination her uterus is large for gestational age.
HCG: very high, U/S: no fetus or heart sounds. She was diagnosed to have cancer which is sensitive to chemotherapy
and easily treated. What does she have? (She had molar or ectopic pregnancy before)
a.
Endometriosis
b.
Gestational Trophoblastic Disease.
c.
Ovarian CA
Answer: B
99- Pregnant lady known case of DM 1 presented with persistent hyperglycemia even after adjustment of her insulin
dose. What is the most likely cause?
a.
Maternal hyperglycemia
b.
Maternal hypoglycemia
c.
Fetal Hyperglycemia
d.
Fetal Hypoglycemia
Answer: ?
Similar question in AlQassim Booklet but they were asking about the complication (Answer: Fetal Hypoglycemia
100- 40 weeks of gestation primigravida presents with hypoxia, drowsiness and agitation for 6 hours, LL edema (Long
scenario with blood tests). What does she have?
a.
Amniotic Emboli
b.
PE
Answer: B
101- 35 weeks of gestation, come mention she doesnt feel the baby movement. Fetus is dead. She is hypoxic with
decreased DLCO (75% Normal 80%) Blood test: Low Hct, prolonged PT and PTT. What does she have?
a.
Amniotic embolism
b.
DIC
c.
ITP
Answer: A
Amniotic fluid embolism causes DIC and ARDS together.

252

102- Women 34 weeks pregnant, however fundal height is 28cm. What is the most likely cause of IUGR?
a.
GDM
b.
Oligohydramnios
c.
Polyhydramnios
Answer: ?
http://emedicine.medscape.com/article/261226-overview#a2

103- Treatment of community acquired pneumonia in pregnancy?


Answer: ?
For pregnant women:
Community acquired pneumonia and no features of severe disease: antipneumococcal beta-lactam (ceftriaxone,
cefotaxime, ampicillin-sulbactam) plus azithromycin
Allergic reactions to cephalosporins: clindamycin plus aztreonam, unless they have severe pneumonia.
severe pneumonia and past reactions to cephalosporins: vancomycin plus azithromycin plus aztreonam.
Reference:http://www.uptodate.com/contents/treatment-of-respiratory-infections-in-pregnant-women
104- Women with itchy, whitish vaginal discharge. KOH shows pseudohyphae. What is the treatment?
a.
Ointment Miconazole
Answer: A
Refer to the table at the end of OB/GYN section.
105- Women developed itchy rash with whitish vaginal discharge after nitrofurantoin course for UTI. How will you
treat this patient?
a.
Miconazole
Answer: A
Refer to the table at the end of OB/GYN section.

106- Anovulatory female. What will you give to induce ovulation?


a.
Clomiphene
b.
Danzo
c.
Pulsatile push of LH
Answer: A
107- Calculate day of delivery.
Answer: ?
EDC using Naegles Rule: 1st day of LMP + 7 d 3 mo e.g. LMP = 1 Apr 2013, EDC = 8 Jan 2014 (modify if cycle >28 d by
adding number of d >28) Reference: Toronto Notes.

108- Woman with bilateral ovarian abscess what dx


Answer:
Pelvic inflammatory disease
The major complication of pelvic inflammatory disease are tuboovarian abscess,chronic pelvic pain,infertility and
ectopic pregnancy

253

109- Vaginal infection lead to neonatal conjunctivitis


Answer:
The most common bacteria that can cause serious eye damage are gonorrhea and chlamydia. These can be passed
from mother to child during birth.
Time of onset:
Neisseria gonorrhoeae: Delivery of the baby until 5 days post-birth (Early onset)
Chlamydia trachomatis: 5 days post-birth to 2 weeks (Late onset - C.trachomatis has longer incubation period)
Wiki, Medline pluse
110-female with clear presentation of UTI. history of URTI. urine analysis showed nitrate : + Esterase : + what is the
organism:
A.
Klebsiella pneumoniae
B.
E.coli
C.
Pseudomonas
Answer: E. coli
-nitrate test is commonly used in diagnosing urinary tract infections (UTI). A positive nitrite test indicates that the
cause of the UTI is a gram negative organism, most commonly Escherichia coli
-A leukocyte esterase test (LE test) is a urine test for the presence of white blood cells and other abnormalities
associated with infection.
White blood cells in the urine usually indicate a urinary tract infection.also used to screen for gonorrhea and for
amniotic fluid infections.
-The combination of the LE test with the urinary nitrite test provides an excellent screen for establishing the presence
of a urinary tract infection (UTI).

111- A 17 yrs old girl vaginal delivery at home with perineum tear what the injured ?
A.
coccygeal
B.
pubococcygeal
C.
ischial...
Answer: pubococcygeus
The muscles of the anus (corrugator cutis ani, the internal anal sphincter and the external anal sphincter)
* The medial muscles of the urogenital region (the superficial transverse perineal muscle, the deep transverse perineal
muscle and bulbocavernosus)
* The medial levator ani muscles (puborectalis and pubococcygeus)
* The fascia of perineum, which covers these muscles
* The overlying skin and subcutaneous tissue.
112-What the treatment of eclamptic seizure?
A.
magnesium sulphate
B.
diazolam
C.
carbamazepine
Answer: MgSo4

254

113-A girl with bilateral ovarian abscess with fever ?


A.
immediate laparotomy
B.
laparoscopic
C.
trans us drain
D.
antibiotic
Answer:
hospitalization, IV antibiotics, if not improved after 48 hr
first-line therapy typically uses a second generation cephalosporin with anti-anaerobic activity and it is still
inconclusive whether additional anti-anaerobic coverage is needed above and beyond the second-generation
cephalosporing, cefoxitin or cefotetan, plus doxycycline , proceed to trans US drain.
up to date...
114- Trichomonas vaginosis read about treatment?
Answer: metronidazole
115- Diagnosis of trichomonas vaginosis?
A.
yellow discharge
B.
white discharge
C.
clue cell
Answer: yellow-green, and frothy
clue cells will be in bacterial vaginosis not in trichomonas
115 -Yellow secretion pv with pmn >10 ?
Answer: trichomonas

116- Self breast examination decrease breast cancer by years ?


A.
1 year
B.
2year
C.
3 year
D.
4year
Answer:
recommend CBE be performed at least every 3 years starting between ages 20 and 39 and annually starting at age 40.
The ACOG, ACR, and AMA recommend starting CBE at age 40 and annually thereafter.
Ref: medscape
[14]

[28]

[29]

117- Adenomyosis treatment?


A.
hysterectomy
B.
ocp
C.
gonadotropin analogue
Answer: hysterectomy is the definitive surgical treatment
first IUD (coil) may be offered, since it has been found that a low level slow release of progesterone can ease the
symptoms of adenomyosis
adenomyosis advice association..

255

118-pregnant with flank pain and tenderness, +nitrate and


leukocytosis what is the plan
A.
admission to treat pyelonephritis
B.
drink plenty of fluid
C.
start antibiotics
Answer:addmission and start ttt
119- Pt has a 3 years infertility and have breast milk the lab show
Hight wbc and hight prolactine what visual field will be
affected:
Upper outer
Di ....
???
???
Answer:
if its a pituitary adenoma it will present with bitemporal hemianopsia.loss in peripheral visual field
120- Cervical insufficiency: the canal is less than
A- 10 mm
B- 20
c-30
d- 40 mm
Answer: B
We make a diagnosis of cervical insufficiency in women with one or two prior second-trimester pregnancy losses or
preterm births and cervical length <25 mm on TVU examination or advanced cervical changes on physical examination
before 24 weeks of gestation. Risk factors for cervical insufficiency support the diagnosis.
Reference: http://www.uptodate.com/contents/cervicalinsufficiency?source=search_result&search=cervical+insufficiency&selectedTitle=1%7E36
121- while giving birth she lost sensation in the medial thigh: what nerve?
A- Pudendal,
B-obturator
Answer:
-Lithotomy positioning during delivery or in gynecologic/urologic procedures also has been associated with
compressive femoral neuropathy.
The sensory branch of the femoral nerve, the saphenous nerve, innervates skin of the medial thigh and the anterior
and medial aspects of the calf.
-The cutaneous branch of the obturator nerve supplies the skin of the middle part of the medial thigh .
Ref : http://emedicine.medscape.com/article/1141793-overview#a7
122- Pregnant early, low appetite nausea fatigue. Blood: low hb high mvc mcvc high tibc: whats the reason
Answer:macrocytic anemi , b12 or folate deficincy not sure
123- Woman with IUD and came with vaginal pain and discharge (what organism)
Answer:
Actinomyces infections in association with IUD use have been reported.
and PID

256

124- Sever symptoms of preclampsia


A.
abd pain
B.
high urea
C.
high blood pressure
Answer:
Severe preeclampsia

Blood pressure: 160 mm Hg or higher systolic or 110 mm Hg or higher diastolic on two occasions at least six hours
apart in a woman on bed rest
Proteinuria: 5 g or more of protein in a 24-hour urine collection or 3+ or greater on urine dipstick testing of two
random urine samples collected at least four hours apart
Other features: oliguria (less than 500 mL of urine in 24 hours), cerebral or visual disturbances, pulmonary edema
or cyanosis, epigastric or right upper quadrant pain, impaired liver function, thrombocytopenia, intrauterine
growth restriction

125- Pregnant women has GGT diagnostic what is your action ?


A.
repeat
B.
do HgA1c
C.
start insulin
D.
do Random blood Glucose
Answer: The increase at GGT level is an independent risk factor for GDM and identified as high-risk women for
diagnosis of GDM
*repeated question answered before
126- Pt in 30th week G2P2 came to usual checkup.
The previous pregnancy was emergent c/s ..by examination everything is normal ..and there is low transver
se section in the abdomen with double suture of uterus or something like that ..wt the plan for here?
A....
B....
C.c/s
D.natural vaginal delivery.
Answer:We generally perform a two-layer rather than a single-layer uterine closure so is not an issue if she had only
one caesarean section, and there were complications during the procedure CS is indecated . If there were no
complications, a vaginal birth after CS is possible but if previous 2 CS the 3rd should be CS
127-DVT prophylaxis for pregnant woman ?
Answer:
Enoxaparin
Up to date

257

128-Pregnant in 38th week ..by examination ..the fetus in breech position ..the the cervix is closed.
what is the next step?
C. Deliver her by c/s
Answer:

129- Risk factor of recurrent UTI in females


A- (whipping from back to front)
Answer:

258

130- Tubal ligation then c/o vginal spotting after 6 wk of amenorrhea


Answer:
Delayed complications of laparoscopic tubal ligation include the following:
Failure
Filshie clip complications
Regret
Ectopic pregnancy
Menstrual changes
Hysterectomy
Sexual function
ref: http://emedicine.medscape.com/article/1848429-overview#a6
131- Pregnant lady with lobar pneumonia wt is the type of immune the baby will have
Answer: there is no choices but the answer is passive immunity

ADDED QUESTION: ( need to be more checked )


1. Typical case of PCOS what is skin manifestation associated with?
Answer:
A Acanthosis nigricans
http://www.ncbi.nlm.nih.gov/m/pubmed/17645376/
2. G8P7 in operation room she tell you that she had after all previous
pregnancies severe postpartum hemorrhage what you will do you do to pt:
1 . give her crystalloid I.V during C/S or labour.
2 . active third stage.
Answer: Routine oxytocin administration in third stage of labor can reduce the risk of PPH by > 40%
(Toronto notes)
3. 40 year old lady early pregnant, what is useful for her:
1.urine dip steak.
2.blood group and rh factor
3.ultrasonography
Answer:
8-12 weeks GA> blood group and Rh
(Toronto notes)
4. What is the best drug given to prevent postoperative thromboembolism?
1-LW heparin
2-Uf heparin
3-Warfarin
4-eno.,,
Answer: 2
Unfractionated heparin (UFH) may be preferred if the patient is likely to have immediate surgery because of its shorter
half-life and reversibility with protamine compared with LMWH. (Medscape)
5. Lady with metromenorrhagia, from 6 month ago and abdominal pain interfere with her activity , what is the
best drug ?
1- hysterectomy
2- OCCP
3-estrogen analogous
Answer:
-NSAID used for relief of mild to moderate pain. Inhibits inflammatory reactions and pain by decreasing activity of
cyclooxygenase, which is responsible for prostaglandin synthesis.
Acute bleeding(stabilize and Iv estrogen or d&c)
Chronic bleeding
- (anatomical or organic problem>>Iv estrogen or d&c if no response after 24 hrs.
-(ocps -long progestin -NSAID)
http://emedicine.medscape.com/article/255540-medication#7

259

6. What is the best ttt of endometriosis?


Answer:depends on certainty of the diagnosis, severity of symptoms, extent of disease, desire for future fertility, and
impact to GI/GU systems (e.g. intestinal obstruction)
medical
-NSAIDs (e.g. naproxen sodium Anaprox)
-pseudopregnancy:
1-cyclic/continuous estrogen-progestin (OCP)
2-medroxyprogesterone (Depo-Provera)
3-dienogest (Natazia)
-pseudomenopause
1-2nd line: only short-term (<6 mo) due to osteoporotic potential with prolonged use,
unless combined with add-back therapy (e.g. estrogen/progesterone or SERM); if long-term use required, add-back
estrogen+progesterone
2-danazol (Danocrine): weak androgen
3-leuprolide (Lupron): GnRH agonist (suppresses pituitary)
can use 12 mo with add-back progestin or estrogen
surgical
conservative laparoscopy using laser, electrocautery laparotomy
ablation/resection of implants, lysis of adhesions, ovarian cystectomy of endometriomas
definitive: bilateral salpingo-oophorectomy hysterectomy
Ref: torento note
7. Missed period 2 months , high BhCG , examination show 16 weeks GA ,U/S show fetus small for data ? Dx
1- choriocarcinoma(raising or plateau HCG )
2- hydatidiform (large for date and BHCG>100,000)
3- placenta in site trophoblastic tumor(low BHCG)
Answer:
8. 30 years old female has 1 child ,Want to delay pregnancy 3 years later, she didn't want OCCP nor intravaginal
Device Doctor advice her for transdermal patch ,
What is the best advice to tell the patient about the patch ?
1- decrease compliance
2- increase blood clot
3- less effective than OCCP (same effect )
4- less skin complication(skin irritation )
Answer: 2
http://www.mayoclinic.org/tests-procedures/ortho-evra/basics/risks/prc-20013014
9. Patient complain of abdominal pain , Missed period ,Ultrasound show douglas pouch fluid and dark blood , dx
?
1- reputed ovarian cyst
2- ruptured ectopic pregnancy
Answer:2
http://radiopaedia.org/articles/ectopic-pregnancy
10. What to do after a Pap smear show atypical changers ?
1. Hysterectomy
2.Guided calposcopy biopsy
3.Exsional biopsy
Answer 2
http://www.mayoclinic.org/tests-procedures/pap-smear/basics/results/prc-20013038
11. Patient present at 10 week with painless bleeding not part of conception cervix closed what the diagnosis
Threatened abortion
Answer please see attached table

260

12. Female try to get pregnant for one year she is healthy and her husband is know to be healthy what to do 1st ?
Semen analysis
Answer :
since the woman is healthy we should investigate man, most common factor for them is semen abnormalities, therefore,
semen analysis and culture.
Reference: http://www.cdc.gov/reproductivehealth/infertility/
13. when do you do US for screening of the foetus?
1.early 2end trimester,
2. late 2end,
3.early 3rd and late 3red
Answer : please see attached table

261

14. A female postpartum with upper lateral quadrant mass, redness, tenderness,with +ve lymph nodes:
Answer:
During lactation, enlarged intramammary and/or axillary lymph nodes may be seen. The hyperplastic nodes are felt to be
related to the bacterial seeding of the nipple by the infant during breastfeeding. These nodes are typically seen in the
upper outer quadrant of the breast and axilla.
Benign entities include galactocele, fibroadenoma, obstructed milk duct, mastitis with or without abscess, hyperplastic
intramammary and/or axillary lymph nodes, and granulomatous mastitis. Malignant diseases include pregnancyassociated breast cancer and metastatic disease.
Reference: http://www.ncbi.nlm.nih.gov/pmc/articles/PMC3781252/
15. pt complain of scanty pubic hair and primary amenorrhea & secondary sexual character & develop breast
with bilateral growing swelling what diagnosis ?
ANDROGEN INSUFFICIENCY ?
Answer: please see table

262

263

16. pt known cervical carcinoma & her baby 8 years what to do for her baby ?
1.GIVE HER CONTRACEPTION
2.NOT related to family history
Answer : ?
Explanation :
Cervical cancer may run in some families. If the mother or sister had cervical cancer, the chances of developing the
disease are 2 to 3 times higher than if no one in the family had it. Some researchers suspect that some instances of this
familial tendency are caused by an inherited condition that makes some women less able to fight off HPV infection than
others
-Two vaccines against HPV are licensed in most countries.
-vaccines prevent over 95% of HPV infections caused by HPV types 16 and 18
-The WHO recommended target group for vaccination is 913 year old girls who have not yet become sexually active.
Ref : http://m.cancer.org/cancer/cervicalcancer/detailedguide/cervical-cancer-risk-factors + WHO
17. Most common complication in hysterectomy?
1.Bladder injury
2.Ureteral injury
Answer:
Ureteral injury
Explanation :Ureteral injuries are common, owing to the size and location of the ureter, and generally are the result of
excessive electrocautery and lasering adjacent to the ureter during surgery.
Ref : Medscape
18. Pap smear the way of doing it?
From the OS
Answer:
http://emedicine.medscape.com/article/1947979-overview#a5
19. pregnant in labor cervical opening 6 cm Which stage
Answer:
Stage 1 active phase
Explanation : Labour stages :
-First stage: The time of the onset of true labor until the cervix is completely dilated to 10 cm.
* Early Labor Phase The time of the onset of labor until the cervix is dilated to 3 cm.
* Active Labor Phase Continues from 3 cm. until the cervix is dilated to 7 cm.
* Transition Phase Continues from 7 cm. until the cervix is fully dilated to 10 cm.
-Second stage: The period after the cervix is dilated to 10 cm until the baby is delivered.
-Third stage: Delivery of the placenta.
Ref : http://americanpregnancy.org/labor-and-birth/first-stage-of-labor/
20. epileptic breastfeeding mother on phenobarbital
A- stop breastfeeding
B- continue breastfeeding
Answer:
Continue Breast Feeding
Explanation : the American Academy of Neurology and the American Academy of Pediatrics advise that women with
epilepsy taking AEDs can breastfeed. If mothers receiving ethosuximide, phenobarbital or primidone choose to
breastfeed, they should exercise caution and closely monitor the infant for sedation, lethargy and any significant clinical
findings.
Ref : Medscape
21. complicated labor switch to c-section when to give antibiotics
A- before c-section
B- after
C- during
Answer:A
a single intravenous dose of a narrow spectrum antibiotic should be administered preoperatively to all women undergoing
cesarean delivery
http://www.uptodate.com/contents/cesarean-delivery-preoperative-issues#H9

264

22. Hormonal changes at menopause in estrogen, LH, FSH.


Answer:
Follicle-stimulating hormone (FSH) levels are higher than luteinizing hormone (LH) levels, and both rise to even higher
values than those seen in the surge during the menstrual cycle. FSH is the diagnostic marker for ovarian failure. Estrogen
levels begin to fall.
23. A pregnant in 32 weeks of gestation, she is in true labor, what to do:
A.Call neonatologist, give corticosteroids, strict bed rest
B.Call neonatologist, give corticosteroids, give fluids o
C.Call neonatologist, give antibiotics, bed rest
Answer: B,or A

24. Best way to diagnose bacterial vaginosis:


A. Gram stain
B. Dark field microscopy
C. PCR
D.culture
Answer: A
Gram's stain Gram's stain of vaginal discharge is the gold standard for diagnosis of BV
http://www.uptodate.com/contents/bacterial-vaginosis#H5
25. pregnant women in 1st trimester never got chickenpox her antibody titer is zero what is the best management
?
A-avoid exposure
B-acyclovir 3
C-Varicella vaccine
Answer:A
http://www.cdc.gov/vaccines/pubs/preg-guide.htm
http://www.uptodate.com/contents/vaccination-during-pregnancy-beyond-the-basics#H3
Its contraindicated to take varicella vaccine during pregnancy if pt. got contact there is varicella zoster immune globulin
(VariZIG) that can help to reduce the risk of becoming infected with chickenpox.
non immune pregnant woman can take the vaccine one month prior to being pregnant or after delivery.
26. lactating women with positive HEBs what to do
1-continue breastfeeding
2-stop breastfeeding
Answer: 1
These data support the recommendation of the American Academy of Pediatrics that HBV infection not be considered a
contraindication to breastfeeding of infants who receive the HBIG and HBV vaccine as advised.
Medscape

265

27. pregnant lady no symptoms UTI, no frequency no urgency no dysuria Positive urine culture, Diagnosis?
1-Asymptomatic bacteriuria Answer: 1
http://www.uptodate.com/contents/approach-to-the-adult-with-asymptomaticbacteriuria?source=outline_link&view=text&anchor=H1#H1
Uptodate
28. tamoxifen for breast cancer has metrorrhagia, US showed thick endometrium what to do next ?
1-endometrial biopsy
2-CA 125 Answer:1
The risk of developing endometrial cancer from tamoxifen is low (less than 1% per year). Women taking tamoxifen must
balance this risk against the benefits of this drug in treating and preventing breast cancer. This is an issue women should
discuss with their doctors. If you are taking tamoxifen, you should have yearly gynecologic exams and should be sure to
report any abnormal bleeding, as this could be a sign of endometrial cancer
An endometrial biopsy is the most commonly performed test for endometrial cancer.
(American cancer society)
29. mother in labor you did vaginal examination, touched orbit ridge, nasal and
chin,What is the presentation?
1-cephalic
2-brow
3-face
Answer: 3
In a face presentation, the fetal head and neck are hyperextended, causing the occiput to come in contact with the upper
back of the fetus while lying in a longitudinal axis. The presenting portion of the fetus is the fetal face between the orbital
ridges and the chin.( medscape)
30. with heavy menses each 2 wk what to do?
A.endometrial bx
Answer: Investigations
CBC, serum ferritin
-hCG
TSH, free T4
coagulation profile (especially in adolescents): rule out von Willebrands disease
prolactin if amenorrheic
FSH, LH
serum androgens (especially free testosterone)
day 21 (luteal phase) progesterone to confirm ovulation
Pap test
pelvic U/S: detect polyps, fibroids; measure endometrial thickness (postmenopausal)
SHG: very sensitive for intrauterine pathology (polyps, submucous fibroids)
HSG
endometrial biopsy: consider biopsy in women >40 yr
must do endometrial biopsy in all women presenting with postmenopausal bleeding to

266

exclude endometrial cancer


D&C: not for treatment; diagnosis only (usually with hysteroscopy)
Ref: (toronto notes)

31. the safe vaccine in pregnancy


A.varicella
B. influenza
Answer:B
http://www.mayoclinic.org/healthy-lifestyle/pregnancy-week-by-week/expert-answers/vaccines-during-pregnancy/faq20057799
32. which OCP cause hyper k :
A. levo..
B. nor..
Answer:
Yasmin (ethinyl estradiol+drospirenone progestin) and Yaz causes hyperkalemia (rare side effect, but contraindicated
in renal and adrenal insufficiency)
Ref: (Toronto notes)
33.Female in 12 wk gestation develop uti treated then at 27 wk again had uti what you will do ?
a. you will treat if asymptomatic
Answer: A
Any asymptomatic UTI in pregnant women should be treated
34.Pregnant woman with significant edema in his hand and foot what will you do ? ( bp :# 160 /
a. give him diuretic
b. low diet salt
c. labtolol
b. observation in hospital
Answer: C
If a pregnant woman's blood pressure is sustained greater than 160 mm Hg systolic and/or 110 mm Hg diastolic at any
time, lowering the blood pressure quickly with rapid-acting agents is indicated for maternal safety.
Labetalol has a more rapid onset of action, may be given orally or parenterally, and is generally preferred as a first-line
agent.
(Medscape)
35. pregnant with repeated infection wbc 100000, treated and come agin with same symptom what?
Answer:

267

36.smoker pregnant women the effect to baby


A. macrosomia
B. low birth weight
C. transient tachypnea
Answer:B
The negative impact of cigarette smoking on fetal health is well established. Cigarette smoking has been associated with
numerous adverse outcomes, including spontaneous pregnancy loss, placental abruption, preterm premature rupture of
membranes (PPROM), placenta previa, preterm labor and delivery, low birth weight (LBW), and ectopic pregnancy. .
While the pathophysiology is not completely understood, as discussed above, several possible mechanisms related to
impaired gas exchange, direct toxicity, and sympathetic activation have been proposed. UpToDate
http://www.babycenter.com/0_how-smoking-during-pregnancy-affects-you-and-your-baby_1405720.bc
37.Pregnant woman in labor room , when she delivered her baby, sudden onset of bleeding from vagina , the baby
is not infected , after 2 hours mother onset bleeding from
A. DIC
B. deficiency in factor llx
Answer:?
PPH is described as primary or secondary: Primary PPH occurs in the first 24 hours after delivery (also called early PPH)
and secondary PPH occurs 24 hours to 12 weeks after delivery (also called late or delayed PPH). Coagulopathy is both a
cause and result of PPH since persistent heavy bleeding, irrespective of the cause, leads to consumption of clotting
factors and hemodilution of remaining clotting factors. UpToDate
Hemophilia A (factor VIII deficiency) is the most common X-linked genetic disease and the second most common factor
deficiency after von Willebrand disease (vWD) (Females usually are asymptomatic carriers). Medscape
38. pregnant lady everything normal except hemoglobin low, next step ?
A. iron
B. nothing
C. folate
D. b12
Answer: A
Iron deficiency anemia accounts for 75-95% of the cases of anemia in pregnant women. while folate deficiency is much
less common than iron deficiency. A woman who is pregnant often has insufficient iron stores to meet the demands of
pregnancy. Encourage pregnant women to supplement their diet with 60 mg of elemental iron daily. The clinical
consequences of iron deficiency anemia include preterm delivery, perinatal mortality, and postpartum depression. Fetal
and neonatal consequences include low birth weight and poor mental and psychomotor performance. Medscape
39- positive culture of budding yeast in urine what is the management ?
A-Flucanazole
B-caspofungin
-Answer: A
increasing are the numbers of fungal UTI, particularly those caused by Candida spp, and, to a lesser extent, by
Aspergillus spp and Cryptococcus neoformans. Candiduria is a condition most often found in elderly, hospitalized, or
immunocompromised patients.81 Candida albicans is the most common species isolated, accounting for more than half
of all fungal infection cases. The mainstay of antibiotic treatment for candiduria is the azolic compounds, mainly
fluconazole 200 mg orally daily for 2 weeks

268

40. w pregnant BP: 170/120 management?


1.Mg sulphate and deliver
2.Mg sulphate and wait till 34w :/
3.Call anesthesia and deliver
Answer: 2
When hypertension is first identified during a woman's pregnancy and she is at less than 20 weeks' gestation, blood
pressure elevations usually represent chronic hypertension. In contrast, new onset of elevated blood pressure readings
after 20 weeks' gestation mandates the consideration and exclusion of preeclampsia.
If a pregnant woman's blood pressure is sustained greater than 160 mm Hg systolic and/or 110 mm Hg diastolic at any
time, lowering the blood pressure quickly with rapid-acting agents is indicated for maternal safety. Anticonvulsant
therapy may be undertaken in the setting of severe preeclampsia (primary prophylaxis) or in the setting of eclamptic
seizures (secondary prophylaxis). The most effective agent is IV magnesium sulfate; phenytoin is an alternative, although
less effective, therapy.
Women with suspected, mild, or diagnosed preeclampsia remote from term or labile blood pressures due to chronic
hypertension and/or gestational hypertension should be hospitalized for close observation, bed rest, and frequent fetal
monitoring.
When preeclampsia develops remote from term (ie, < 34-36 weeks' gestation), attempts are often made to prolong the
pregnancy to allow for further fetal growth and maturation. Medscape
41. w pregnant cervix closed, baby HR 120 after one hour of induction baby's HR 80 and contractions last 2 mins.
Management?
1- CS
2-Oxygen
3-Oxytocin
4-Observe
Answer
42.Prolong labor. She might have post partum hmrg. How to asses?
1.Visual blood loss
2.Hematocrit count
3. Pulse
4.Hemoglobin
Answer We make the diagnosis of PPH in postpartum women with bleeding that is greater than expected and causes
symptoms (eg, pallor, lightheadedness, weakness, palpitations, diaphoresis, restlessness, confusion, air hunger, syncope)
and/or results in signs of hypovolemia (eg, hypotension, tachycardia, oliguria, oxygen saturation <95 percent) (table 1).
Diagnosis may be delayed in symptomatic women without heavy vaginal bleeding who are bleeding internally, such as
intra-abdominal bleeding related to a cesarean delivery or a broad ligament or vaginal hematoma due to a sulcus
laceration.
43.Best way to avoid transmission of tetanus in pregnant lady to her baby?
1.Newborn tetanus toxoid
2.Neonate anti tetanus
3.Giver the mother early tetanus toxoid
Answer 3
http://www.uptodate.com/contents/tetanus?source=machineLearning&search=neonatal+tetanus&selectedTitle=1~4&sect
ionRank=3&anchor=H1917252#H1917252
44. pregnant lady miss pregnant symptom since 1week and started complin of
spot bleeding the most valuable investigation in this condition is :
1.hcg-alpha
2.feto-us .....
answer 2
http://www.uptodate.com/contents/overview-of-the-etiology-and-evaluation-of-vaginal-bleeding-in-pregnantwomen?source=search_result&search=pregnancy+bleeding&selectedTitle=1~150#H3

45.increase in frequency of menstruation


1.polymenorrhea
2-hyper
3-metro..
answer 1

269

46- 3q. on contraception .


answer http://www.uptodate.com/contents/overview-ofcontraception?source=search_result&search=contraception&selectedTitle=1~150
48. Pregnant 40 g.a did not follow up ,, examination and ultrasound reflect breech presentation ,,, in progressive
labour pain.. cervical full dilation and full effacement , intact membrane .. Engagement zero ,, what to do ?
1- amniotomy
2- CS
Answer: non !!
http://www.uptodate.com/contents/delivery-of-the-fetus-in-breechpresentation?source=search_result&search=breech+delivery&selectedTitle=1~150
49. Pregnant lady with control DM there is hx of IUFD now she is in 32 wk management:
1-Induce labor in 36 wk
2-CS AT 38
3-OBSERVE
Answer: 2. CS at 38 wks
A cesarean delivery (C-section) may be recommended for obstetric indications such as severe preeclampsia with an
unfavorable cervix, estimated fetal weight >4500 grams, history of a C-section, or fetal distress.
Reference: http://www.endotext.org/
50. Safest antibiotic in breastfeeding?
Answer: 1.Cephalosporins (i.e. Keflex or cephalexin)
2.Erythromycin.
3.Trimethoprim-sulfamethoxazole (Bactrim, Septra) is compatible with breast-feeding,but its use should be avoided
when nursing infants are younger than two months because of its potential for causing increased bilirubin levels.3
4.Metronidazole (Flagyl): cessation of breastfeeding for 12 to 24 hours is recommended. Topical preparations of
metronidazole (MetroGel-Vaginal) is safe
5.Fluconazole (Diflucan) is commonly prescribed for yeast infections of the nipple in breast-feeding mothers.
51.PT with Vaginal discharge no other symptoms and normal investigation?
Answer: non infective Physiological During the reproductive years the fluctuating levels of oestrogen and progesterone
throughout the menstrual cycle affect the quality and quantity of cervical mucus which is perceived by women as a
change in their vaginal discharge. Initially, when oestrogen is low, the mucus is thick and sticky. As oestrogen levels rise,
the mucus gets progressively clearer, wetter and more stretchy. After ovulation, there is an increase in the thickness and
stickiness of the mucus once more.
52. Acute Salpingitis case?
Answer: http://emedicine.medscape.com/article/275463-overview#a2

There were also missing questions about the following:


Pap test chart

270

ASCUS = abnormal squamous cells of unknown significance; LSIL = low grade squamous intraepithelial lesion; HSIL = high grade squamous
intraepithelial lesion; ASC-H = abnormal squamous cells cannot rule out HSIL; AGUS = atypical glandular cells of unknown significance.

Infectious vulvovaginitis

271

Prenatal Screening and Diagnostic Tests

*CVS chorionic villus sampling, FTS first trimester screen, IPS integrated prenatal screen, PAPP-a pregnancy-associated plasma protein a, MSAFP maternal serum afetoprotein, OGCT oral glucose challenge test, GBS Group B Streptococcus.

Endometriosis (Presentation, investigations, treatment)

272

Added Qs 7th update


1- Female 3 children , doesn't want to get pregnant anymore ,has a hx of endometriosis in
ovary, She removed it, now she has another one in the right ovary ..how to manage ?
a.
Hysterectomy bilateral salbingo oophrectomy ,
b.
aspiration of the mass ..?
Answer: A
Definitive: bilateral salpingo-oophorectomy +/- hysterectomy
Reference (Toronto notes GY14)
2- Patient with pelvic inflammatory disease,didn't respond to abx after 3 days examination
revealed fluctuating mass, how to manage?
a.
Laparoscopy .
Answer:
A case of tubo-ovarian abscess?

3-Pregnant lady ,had an outbreak asking for all vaccination can be given what you will give
?
a.
Influenza
b.
MMR ,
c.
rubella ,
d.
varicella
Answer: A
4- Pregnant lady has only low hp , mcv ,mchc are normal ..? What to do ?
a.
Give iron ,
b.
do nothing
c.
answer: A
5- Patient in 3rd trimester have high blood blood glucose despite close observation What is
the suspect cause?
a.
neonate hyperglycemia
b.
neonate hypoglycemia
c.
mother hyper
d.
mother hypoglycemia
Answer: b
273

6- patient have obesity , hirsutism , HTN , insulin resistance What is the dx? In options
there's no pcos
a.
kallman syndrome
b.
kleinfilter syndrome
c.
I don't know the answer but I think there is another name of pcos
d.
Answer:
Polycystic ovarian syndrome also called chronic ovarian androgenism, hyperandrogenic
anovulation (HA), or SteinLeventhal syndrome
7- Lady with metromenorrhagia, from 6 month ago .. And abdominal pain interfere with her
activity , what is the best drug ?
a.
hysterectomy
b.
Occp
c.
"" i think "" estrogen analogous
Answer:
Management of dysfunctional uterine bleeding:
(acute, severe DUB)
replace fluid losses, consider admission
a) estrogen (Premarin) with dimenhydrinate or anti-fibrinolytic (e.g. Cyklokapron)
b) any OCP with minimum 50 g estradiol with dimenhydrinate
*after (a) or (b), maintain patient on monophasic OCP for next several months or consider
alternative medical treatment
*clomiphene citrate: consider in patients who are anovulatory and who wish to get
pregnant
*surgical: endometrial ablation; consider pre treatment with danazol or GnRH agonists
if finished childbearing
*repeat procedure may be required if symptom reoccur especially if <40 yr hysterectomy:
definitive treatment
8- Pregnant 40 g.a did not follow up ,, examination and ultrasound reflect breech
presentation ,,, in progressive labour pain.. cervical full dilation and full effacement , intact
membrane .. Engagement zero ,, what to do ?
a.
amniotomy
b.
CS
c.
Answer: repeated
9-Patient complain of abdominal pain .. Missed period .. Ultrasound show dougles pouch
fluid and dark blood , dx ?
a.
reputed ovarian cyst
b.
ruptured ectopic pregnancy
Answer : B

274

10-Lady with epithelial cell on UA, what's the cause?


a.
valvular contamination
b.
cervix lesion
Answer: A
http://www.mayoclinic.org/tests-procedures/urinalysis/basics/results/prc-20020390
11- case of typical presentation of syphilis
Answer :Toronto notes

12-Snowstorm appearance in pregnant what's the Dx?


Answer:
Complete mole(GTD)
Toronto notes

13-Postmenopausal lady taking tamoxifen, which of the following u will carefully assess?
a.
.vaginal bleeding
Answer:A
http://www.uptodate.com/contents/medications-for-the-prevention-of-breast-cancer-beyond-thebasics

275

14-Pregnant on labour, on PV u fell the orbital margin and the nose, what is the
presentation of the fetus?
a.
mento anterior < the most common
b.
mento posterior
c.
lateral mento transvers
d.
medial mento transvers
Answer:
Face presentations are classified according to the position of the chin (mentum):
Left Mento-Anterior (LMA), Left Mento-Posterior (LMP), Left Mento-Transverse (LMT);
Right Mento-Anterior (RMA), Right Mento-Posterior (RMP), Right Mento-Transverse (RMT);

A mento-posterior face presentation,


http://www.slideshare.net/mobile/addisuga/63fa20a2b64d4e6
15- a 39 weeks pregnant female .. Came with BP 160/95 ..no proteinuria.. The previous
visits the BP was normal .. What is your diagnosis?
a.
gestational hypertension
b.
chronic HTN
c.
preeclampsia
I think the answer is A because no proteinuria
Answer: A
GESTATIONAL HYPERTENSION
Definition
sBP >140 or dBP >90 developing after 20th wk GA in a woman known to be normotensive
before pregnancy
(Toronto notes)
16-A G3P0 female with 3 recurrent abortions, on investigating the last abortus you found an
aneuploidy 45X. What are the chances of having this abnormality in her next pregnancies?
a.
30%
b.
40%
c.
50%
d.
60%
Answer:
Likelihood of another affected child for parents is under 1%!

276

17-A female had an IUD inserted 2 years ago, now she's complaining of lower abdominal
pain and vaginal discharge which was foul smelling few days ago. On examination you
found a right 9-adnexal mass. A gram stain of the cervical discharge showed a gram
positive beading bacillus, what is the most likely causative organism?
a.
b.
Perfringens
c.
Bacteroides fragilis
Answer:
Repeated
Gram-positive branching filaments of Actinomyces species.
18-g8p7 in operation room she tell you , she have after all previous pregnant severe
postpartum hemorrhage what you will do to prevent bleeding ?
a.
give him crystalloid I.V during c.s or labour
b.
ACTIVE third stage
answer:B
Routine oxytocin administration in third stage of labor can reduce the risk of PPH by > 40%
19-early pregnant , what is useful for her
a.
urine dip steak
b.
blood group and rh factor
c.
ultrasonography
answer: C
Repeated
20- Pregnant woman in third trimester he as in airline about 18 hours and developed of
sudden chest pain , with dyspnea, what is the cause ?
a.
pulmonary hypertension
b.
.pulmonary embolism
c.
myocardial ischemia
answer: B
SoB and chest pain symptoms of PE
21- pregnant woman in labour room , when she delivered her baby, sudden onset of
bleeding from vagina , the baby is not infected , after 2 hours mother onset bleeding from
mouth and nose , what is the cause ? a.
Dic
b.
deficiency in factor llX
answer:repeated
22- Pregnant lady everything normal except hemoglobin low, next step ?
a.
Iron Nothing
b.
Folate B12
answer :iron
23- Pregnant with repeated infection wbc 100000, treated and come again with same
symptom what to do ? Treat her even she is asymptomatic ?
answer :treat even asymptomatic
Uptodate

277

24- Patient present at 10 week with painless bleeding not part of conception cervix closed
what is ?
answer : Threatened abortion
25-Female in 12 wk gestation develop UTI treated then at 27 wk again had uti what you will
do ? E. coli >100000 -you will
a- treat if asymptomatic
answer : treat even if asymptotic
26-pt with Hx of lower transverse incision and double uterine fold suture and she is at 37 wk
and doing fine
A-CS
B-SVD
answer :studies suggested that rupture less likely if double so SVD safe unless there is
permanent cause such CPD)
27- 42 with heavy menses each 2 wk what to do
answer
Repeated
28- Young pt with 4 weeks amenorrhea , presented with abdominal pain and
severe vaginal bleeding , the bleeding from ?
A-Ovary
B-Fallopian tube
C-Cervix
answer:Fallopian tube(webmed)
29-complicated labor switch to c-section when to give antibiotics ?
a.
before c-section
b.
after
c.
during
Answer: a
Single dose prophylactic antibiotic should be used (e.g. Cefazolin 1-2g)
30-Pregnant women at 14 weeks with splenomegaly low platelet what is diagnosis ?
Answer:?
*In normal pregnancy there will be mild asymptomatic thrombocytopenia >70.000
Which will return to normal within 2-12 weeks postpartum!
31-

Women during delivery developed dyspnea and hypotension seizure diagnosis :

Answer:
amniotic fluid embolism
32Pregnant Women with hypotension and low platelet what is type of anesthesia you
prefer ?
a.
Epidural
b.
Pudendal block
c.
SpinaL
d.
General
Answer: b
278

33- Follicular cell of ovary what it is orign embryologically???


Answer:
34- patient with BPH+HTN what TTT?
best management
Answer: alpha blocker
35- MMR vaccine in breastfeeding : A-harm to baby
B-Safe to baby
C-Delayed feeding 72 hours
D- Live attended vaccine
Answer D
36-female was dx with ovarian cancer , she haven't ever used OCP , what will you tell her
daughters ?
A- OCP can protect you from ovarian cancer
B- Bilateral oophorectomy is recommended for you
Answer A
37-in mastectomy what you will cut ? Pectorals major muscle
Answer
38-female pregnant with twins , in her 34 week ( she had some complication I forget it ) the
doctor said that she needs emergency CS cuc the presentation of twin A might result in
fetal complications , what is the presentation of twin
A-twin Cephalic- breech
B.Transverse-cephalic
C.Breech-cephalic
D.Cephalic-cephalic
Answer C ?
39-pregnant lady miss pregnant symptom since 1 week and started complain of spot
bleeding the most valuable investigation in this condition is :
a- hcg
b- alpha feto
c- us
Answer: C?
40- 3 questions on contraception

279

41-You performed a pudendal nerve block on a woman in labor, which of the following
structures will be fully sensitive and not blocked by the anesthesias?
A. Perineal body
B. Urogenital diaphragm
C. Rectum
Answer:C
The sensory and motor innervation of the perineum is derived from the pudendal nerve, which is composed of the
anterior primary divisions of the second, third, and fourth sacral nerves. The pudendal nerves 3 branches include
the following:
1.
2.
3.

Dorsal nerve of clitoris, which innervates the clitoris


Perineal branch, which innervates the muscles of the perineum, the skin of the labia majora and labia
minora, and the vestibule
Inferior hemorrhoidal nerve, which innervates the external anal sphincter and the perianal skin

http://emedicine.medscape.com/article/83078-overview#a1
The anal canal also has differing nervous innervations above and below the line. Above the pectinate line,
the nerve supply is visceral, coming from the inferior hypogastric plexus. As is it visceral, this part of the anal canal
is only sensitive to stretch. Below the pectinate line, the nerve supply is somatic, receiving its supply from the
inferior rectal nerves (branches of the pudendal). As it is somatically innervated, it is sensitive to pain, temperature,
and touch.

http://fitsweb.uchc.edu/student/selectives/Luzietti/Painful_anus_anorectal_anatomy.
htm
42-What you should look for in case of gonorrhea?
A.
Chlamydia trachomatis
Answer: A
The uterine cervix is the most common site of mucosal infection with N. gonorrhoeae
in women.
Symptomatic infection typically manifests as vaginal pruritus and/or a mucopurulent
discharge. Some women may complain of intermenstrual bleeding or menorrhagia.
Pain is atypical in the absence of upper tract infection. On examination, the cervix
may appear normal or show signs of frank discharge. The cervical mucosa is
often friable.Importantly, these symptoms and signs of gonococcal cervical
infection, when present, are indistinguishable from those observed with acute
cervicitis of other causes.
Other sexually transmitted pathogens, including Chlamydia trachomatis,
Trichomonas vaginalis, and Mycoplasma genitalium, as well as herpes simplex virus
(HSV) and syphilis, can cause similar symptoms caused by N. gonorrhoeae.
Use of nucleic acid amplification testing (NAAT) is recommended as the optimal
method for the diagnosis of genital and extragenital infections caused by N.
gonorrhoeae.
N. gonorrhoeae not only causes similar clinical syndromes as C. trachomatis but also
coexists in a significant proportion of patients with chlamydial infection. Thus, any
testing for N. gonorrhoeae should also prompt testing for C. trachomatis

280

43-pregnant in 40week suddenly become drowsy seizure what is the cause :


a- PE
b- amniotic Emboli.
Answer: B
A woman in the late stages of labor becomes acutely dyspneic with hypotension; she may experience seizures
quickly followed by cardiac arrest. Massive DIC-associated hemorrhage follows and then death. Most patients die
within an hour of onset.
Currently no definitive diagnostic test exists. The United States and United Kingdom AFE registries recommend the
following 4 criteria, all of which must be present to make the diagnosis of AFE [1, 13, 14] :
1.
2.
3.
4.

Acute hypotension or cardiac arrest


Acute hypoxia
Coagulopathy or severe hemorrhage in the absence of other explanations
All of these occurring during labor, cesarean delivery, dilation and evacuation, or within 30 minutes
postpartum with no other explanation of findings

http://emedicine.medscape.com/article/253068-overview#a5

44-Questions about the contraindications of external cephalic version


Answer:
Barring contraindications, ECV is recommended by several national organizations for all women with an
uncomplicated singleton fetus in breech presentation at term to improve their chances of having a cephalic vaginal
birth.
Contraindications to ECV exist either when the procedure may put the fetus in jeopardy or when the
procedure is very unlikely to succeed
Clearly, if cesarean delivery is indicated for reasons other than breech presentation, ECV is contraindicated.[23]
Placenta previa or abruptio placentae, nonreassuring fetal status, intrauterine growth restriction in association with
abnormal umbilical artery Doppler index, isoimmunization, severe preeclampsia, recent vaginal bleeding, and
significant fetal or uterine anomalies are also contraindications for ECV.
Other contraindications to ECV include ruptured membranes, fetus with a hyperextended head, and multiple
gestations, although ECV may be considered for a second twin after delivery of the first.
Relative contraindications include maternal obesity, small for gestational age fetus (less than 10%), and
oligohydramnios because they make successful ECV less likely.[24] Previous uterine scar from cesarean delivery or
myomectomy may also be a relative contraindication for ECV.
http://emedicine.medscape.com/article/1848353-overview

45-about Endometriosis best management ? And treatment ..


Answer:
Clinical manifestations of endometriosis fall into three general categories: pelvic pain,
infertility, and pelvic mass. The goal of therapy is to relieve these symptoms. There is no
high quality evidence that one medical therapy is superior to another for managing pelvic
pain due to endometriosis, or that any type of medical treatment will affect future fertility.
Therefore, treatment decisions are individualized, taking into account the severity of
symptoms, the extent and location of disease, whether there is a desire for pregnancy, the
age of the patient, medication side effects, surgical complication rates, and cost.
Treatment options include:
Expectant management
Analgesia
Hormonal medical therapy
Estrogen-progestin oral contraceptives, cyclic or continuous
281

Gonadotropin-releasing hormone (GnRH) agonists


Progestins, given by an oral, parenteral, or intrauterine route
Danazol
Aromatase inhibitors
Surgical intervention, which may be conservative (retain uterus and ovarian tissue) or
definitive (removal of the uterus and possibly the ovaries)
Combination therapy in which medical therapy is given before and/or after surgery
http://www.uptodate.com/contents/overview-of-the-treatment-ofendometriosis?source=search_result&search=Endometriosis&selectedTitle=1~150

46-Pregnant in the third trimester with history of recurrent herpes simplex, she is in labour,
during exam no lesions what to do?
A. CS
B. IV acyclovir
C. Do specular exam before
Answer: B
Although treatment during primary infection lessens morbidity, it does not eradicate
latent virus, which can subsequently reactivate. Clinical recurrences are common
and can be treated episodically or prevented with continual antiviral suppression.
Recurrent genital infection refers to the occurrence of genital HSV infection in a
patient seropositive for the HSV type recovered from the lesion. The risk of neonatal
transmission at delivery is much lower than in patients with primary or first episode
genital infection.
Transmission of herpes simplex virus (HSV) to neonates usually occurs during labor
and delivery as a result of direct contact with virus shed from infected sites (vulva,
vagina, cervix, perianal area). Importantly, viral shedding can occur when
maternal symptoms and lesions are absent
Suppressive antiviral therapy is suggested at 36 weeks of gestation through delivery
for women with a history of recurrent genital herpes to reduce the risk of lesions at
the time of delivery
Suppressive therapy reduces the risk of clinical recurrence of HSV and
asymptomatic viral shedding at delivery, and thus the need for cesarean
delivery. However, the clinical impact on neonatal HSV is unknown.
http://www.uptodate.com/contents/genital-herpes-simplex-virus-infection-andpregnancy?source=see_link
47-about pregnant lady with twins on 24 weeks and you discover one fetal death what to
do?
- Delivery + dexamethasone
- wait until 34 weeks
- wait until 37 weeks
48-Pregnant women has GGT diagnostic what is your action ?
- do HgA1c
- start insulin
- do Random blood Glucose
Answer:
282

49-about old female with recurrent fracture ..


- Estrogen Def
Answer:
50-about the side effect of OCP ?
- breast cancer
- cervical cancer
- DVT
Answer: DVT
51-target HA1C in type 1 DM ?
Answer: < 6.5
52-Female with no sexual hx presented with amenorrhea and thyroid disease has been
excluded what first investigation will you do ?
Pregnancy test
TSH
Prolactin
Answer:
First rule out pregnancy
Then hyperprolactinemia.
53-Questions about cervical cancer (staging, biopsy)
53-Group B strep positive pregnancy and she is in her 24 weeks , when to give prophylactic
antibiotic ?
Answer:
54-Primary amenorrhea , what is the next investigation ?
Answer:

55-You are inserting catheter in male, u r afraid to injure :


Answer:
a- membranous urethra
b- prostatic urethra
answer A ( narrowest part)
283

56-pregnant women in 1st trimester never got chickenpox her antibody titer is zero what is
the best management ?
a-avoid exposure
b-acyclovir
c -Varicella vaccine
Answer: A
57-what cross the placenta?
a-rubella
b mumps
c -HBV
Answer:
58-pregnant in labour with 6 cm dilation and 1+ effacement and spontaneous rupture of
membrane, which analgesic to give
1- Pudendal
2- Cervical
3- General
Answer: anesthesia section
58-pregnant lady no symptoms UTI, no frequency no urgency no dysuria Positive urine
culture? Diagnosis?
Answer: Asymptomatic bacteriuria
59-positive culture of budding yeast in urine what is the management ?
a -Fluconazole
b -caspofungin answer: A
60-tamoxifen for breast cancer has metrohagia, US showed thick endometrium what to do
next ?
a -endometrial biopsy
b-CA 125
answer:repeated
61- mother in labor you did vaginal examination, touched orbit ridge, nasal and chin What
is the presentation?
a cephalic
b -brow
c -face
answer:repeated
62-multiple something? -bone marrow>>not sure 38)isotretinoin most feared
complication -birth defect -( I don't know about this question is it correct or not )
Answer:
Isotretinoin is a teratogen highly likely to cause birth defects if taken by women during
pregnancy or even a short time before conception. A few of the more common birth defects
this drug can cause are hearing and visual impairment, missing or malformed earlobes,
facial dysmorphism, and mental retardation. Isotretinoin is classified as FDA Pregnancy
Category X and ADEC Category X, and use is contraindicated in pregnancy
(Wikipedia)
284

63-Married female Patient with white vaginal discharge, odorless, Wet test was negative,
KOH test showed Psudohyphae, what is the diagnosis?
Vaginal candidiasis.
Ans: Vaginal candidiasis
Answer:

64- early pregnant , what is useful for her


1 - urine dip steak
2 - blood group and rh factor
3 - ultrasonography
answer: c ?

65-female pregnant in her 24 week , came to you for her first prenatal visit , lab are
provided all were normal except that the Hgb is slightly low ( I think it was 10) , what will you
do ?
A.Nothing
B.Folate
C.Vit B
D.Iron
Answer d
Iron deficiency is the most common cause of anemia in pregnancy.
66-pregnant in 40week suddenly become drowsy seizure what is the cause :
a- PE
b- amniotic Emboli.
Answer b
http://emedicine.medscape.com/article/253068-clinical#b1
67-Pregnant in the third trimester with history of recurrent herpes simplex, she is in labour,
during exam no lesions what to do?
Answer : 1) CS IV acyclovir 2) Do specular exam before
http://emedicine.medscape.com/article/274874-overview#a1
68-Pregnant with HIV on medication, used to have 400 copies and now 200 copies on labor
A.Spontaneous Vaginal delivery
B.Forceps delivery
C.CS
Answer: A
285

69-Q about Ocp side effects on the liver ?


a.- hepatocellular carcinoma
b. - sinusoidal diletation
c.- hepatic adenoma All but i think more C
Answer:c
Hepatocellular adenomas occur mostly in women of childbearing age and are strongly
associated with the use of oral contraceptive pills (OCPs) and other estrogens( medscape)
70- Women with negative pregnancy test have vaginal bleeding, Hgb 9 mangement?
A. Blood transfusion
B. Progesterone
C. Conjugated Estrogen
Answer: b
Its DUB the drug of choice progesterone
( Wikipedia)
71-Contraindicated antibiotic in pregnancy?
A.Nitrofurantoin
B. Erythromycin
C. Tetracycline
answer: C
Tetracycline use should be avoided in pregnant or lactating women ( Wikipedia)
*Antibiotics contraindicated in pregnancyMCAT
M Metronidazole

C Chloramphenicol

A Aminoglycoside

T Tetracyclines

286

72-Pregnant lady with positive nitrite and leukocyte esterase and E.coli?
A. Penicillin !!!
B. Advise her to drink a lot of fluid.
answer: A
uncomplicated UTI
first line: amoxicillin (250-500 mg PO q8h x 7 d)
alternatives: nitrofurantoin (100 mg PO bid x 7 d)
Ref. Toronto notes
73- Lady with metromenorrhagia , from 6 month ago .. And abdominal pain interfere with
her activity , what is the best drug ?
hysterectomy
A.Occp
B. i think "" estrogen analogous
Answer: A
medical treatment of endometriosis
NSAIDs (e.g. naproxen sodium Anaprox)
-pseudopregnancy:
cyclic/continuous estrogen-progestin (OCP)
medroxyprogesterone (Depo-Provera)
dienogest (Natazia)
- pseudomenopause
2nd line: only short-term (<6 mo) due to osteoporotic potential with prolonged use,
unless combined with add-back therapy (e.g. estrogen/progesterone or SERM); if
long-term use required, add-back estrogen+progesterone
danazol (Danocrine): weak androgen
side effects: weight gain, fluid retention, acne, hirsutism, voice change
leuprolide (Lupron): GnRH agonist (suppresses pituitary)
side effects: hot flashes, vaginal dryness, reduced libido
can use 12 mo with add-back progestin or estrogen
surgical
conservative laparoscopy using laser, electrocautery laparotomy
ablation/resection of implants, lysis of adhesions, ovarian cystectomy of endometriomas
definitive: bilateral salpingo-oophorectomy hysterectomy
follow-up with medical treatment for pain control not shown to impact on preservation of
fertility best time to become pregnant is immediately after conservative surgery
Ref:Toronto notes
74-Pregnant 40 g.a did not follow up ,, examination and ultrasound reflect breech
presentation ,,, in progressive labour pain.. cervical full dilation and full effacement , intact
membrane .. Engagement zero ,, what to do ?
amniotomy
CS
Answer: Repeated Q
287

75-Missed period 2 months , high ah BCG , examination show 16 weeks GA .. US show


fetus small for data ? Dx:
A.Choriocarcinoma
B.Hydatidiform
C.Placenta in site trophoblastic tumor
Answer:
Repeated Q
76- Patient complain of abdominal pain .. Missed period .. Ultrasound show douglas pouch
fluid and dark blood , dx ?
Reputed ovarian cyst
A.ruptured ectopic pregnancy
Answer:A
Repeated
77-Patient with Invasive Cervical Cancer, you want to stage her cancer, which of the
following tests you should perform?
Proctoscopy, colonoscopy, hysteroscopy
Proctoscopy, cystoscopy, hysteroscopy
Proctoscopy, gastroscopy, hysteroscopy
Proctoscopy, laparoscopy, hysteroscopy
Answer: B. Proctoscopy, cystoscopy, hysteroscopy
Repeated
78-Single Female came to your clinic one day after condom rupture during vaginal
intercourse, she is worried about becoming pregnant. What you will do?
A.Pregnancy test
B.Wait and arrange for appointment after one week
C.Give post-coital contraception
D.Give progesterone only contraception
Answer: C or D
EMERGENCY CONTRACEPTION
hormonal EC (Yuzpe or Plan B, usually 2 doses taken 12 h apart) or post-coital
IUD insertion
hormonal EC is effective if taken within 72 h of unprotected intercourse (reduces
chance
of pregnancy by 75-85%), most effective if taken within 24 h, does not affect an
established
pregnancy
post-coital IUDs inserted within 5 d of unprotected intercourse are significantly more
effective
than hormonal EC (reduces chance of pregnancy by ~99%)
*Yuzpe method = 98% (within 24 h), decreases by 30% at 72 h
*Plan B levonorgestrel only= 98% (within 24 h), decreases by 70% at 72 h

288

79-35 year old female P3+0 with amenorrhea for 6 months, thinning and dryness of vaginal
mucosa, she underwent D & C 3 years ago due to retained placental tissue following one of
his deliveries. on examination: normal cervical canal, normal uterus with non-palpable
ovaries. Her hormonal profile is: (numbers was given with the normal range) FSH: high LH:
high TSH: normal Estradiol: low
What is the diagnosis?
Ashermans syndrome.
Turner syndrome.
Ovarian failure.
Answer: C. Ovarian failure
80-Postpartum patient with bilateral breast engorgement and tenderness, what is the
management?
Hot compression and continue breastfeed
Cold compression and stop breastfeed
Oral Dicloxacillin and continue breastfeed
Oral Dicloxacillin and pump the milk into bag and discard it
Answer: A. Hot compression and continue breastfeed

81-best prenatal screening in the first trimester for Down syndrome ?


A.
Chorionic villous biopsy
B.
Amniocentesis
C.
Answer:
82- 64) postmenopausal women has hot flushes what altered enzyme is the reason behind
her symptoms ?
A. TSH
B. estrogen
C. Progenstron
Answer: B
83-postmenopausal lady came with vaginal spotting , on examination there was cystic
nodule in her labia majora , what is the dx ?
A.
Bartholin cyst
B.
Bartholin carcinoma
C.
Answer:
Repeated
84-female pt obese with regular menstrual cycle , on PE/ she had acne , other examination
is normal , what investigation will you order ?
A.
TSH
B.
ACTH
C.
Answer:

289

85-clear scenario abou PCOS and they gave the other name of it (repeated)
when a couple should seek help regarding infertility ?
A.
12 months
B.
6 months
C.
Answer:

86-pregnant lady come to you with splenomegaly and platelet = 50 and uterus on the level
of symphysis pubic , what is Dx :
a- idiopathic thrombocytopenic purpura
b- gestational thrombocytopenia
c- thromboembolic disease
Answer:
87pregnant lady with hypotension , what type of anesthesia you will given :
a- pudendal
b- epidural
c- general
Answer: anesthesia section
88pregnant lady with vaginal discharge and + KOH , what is Dx :
a- bacterial vaginisis
answer: A
89pregnant lady with vaginal discharge caused by n. Gonorrhea , which of the
following is associated with this case :
a- chlamydia
b- HSV
answer:A
90- pregnant lady with whitish vaginal discharge :
a- trichomonas
b- bacterial vaginosis
answer: question is not complete , could be normal vaginal discharge

290

91vaginal infection which organism can causes :


a-HSV 1
b- HSV 2
answer: b
HSV 2
92- female with 3 months meses loss what is best action :
a- reassurance
answer:
93female last delivery with forceps what is complicate :
a- primary postpartum hemorrhage
b- secondary postpartum hemorrhage
c- uterine inversion
answer:
A primary PPH which occurs within the first 24 PP
94-female with ovarian mass, hysterectomy was done, specimen showed thecal cell tumor,
what other things you would find in the specimen?
a. Moles in the uterus
b.Endometrial hyperplasia
c.Others
answer:B
95-pregnent lady miss pregnant symptom since 1 week and started complain of spot
bleeding the most valuable investigation in this condition is :
A)HCG
B)alpha feto
C)us .....
D)
answer:C
96-increase in frequency of muenst:
A) Metrorrhagia
B) hypermenorrhea
C) metrorrhagia
D)Polymenorrhea
answer:
D
hypermenorrhea : abnormally heavy or prolonged menstruation; can be a symptom of
uterine tumors and can lead to anemia if prolonged
Metrorrhagia: is uterine bleeding at irregular intervals, particularly between the expected menstrual
periods
Polymenorrhea is the medical term for cycles with intervals of 21 days or fewer.
Oligomenorrhea is the medical term for infrequent, often light menstrual periods (intervals exceeding 35
days).
menometrorrhagia (meno = prolonged, metro = uterine, rrhagia = excessive flow/discharge) may be
diagnosed. Causes may be due to abnormal blood clotting, disruption of normal hormonal regulation of
periods or disorders of the endometrial lining of the uterus

291

97-post menpose women pallor vagina and week what is the TTT :
A)estrogen
B)proges
C)diet
D).....
answer:A
e therapeutic standard for moderate to severe vaginal atrophy is estrogen therapy, administered either
vaginally at a low dose or systemically. There has been a relative lack of randomized controlled trials
performed to date, but they have shown that low-dose, local vaginal estrogen delivery is effective and
well tolerated for treating vaginal atrophy.
http://www.medscape.org/viewarticle/556471

98-during pv exam you found cervical mass 1012mm what you will do :
A) reassure ..
B) biobsy..
C)test for human pappiloma virus
D)
answer:
https://my.clevelandclinic.org/health/diseases_conditions/hic_Genital_Warts/hic_Understanding_
HPV
99-Type of fibroid cause abortion :
A)
B)
C)
D)
answer:
Fibroids that bulge into the uterine cavity (submucous) or are within the cavity (intracavitary) may sometimes cause
miscarriages. The fertilized egg comes down the fallopian tube and takes hold in the lining of the uterus. If a submucosal
fibroid happens to be nearby, it can thin out the lining and decreases the blood supply to the developing embryo. The
fibroid may also cause some inflammation in the lining directly above it. The fetus cannot develop properly, and
miscarriage may result.
http://www.fibroidsecondopinion.com/fibroids-and-pregnancy/

100-girl 15 never had mensis Examination breast bed ,fine hair >n After 1 year come e
increase height >6cm And gain kilograms:
A) Primary amenorrhea
B) amenorrhea Precocious puberty
C) Constitutional
D)
answer:http://emedicine.medscape.com/article/252928-overview
101-Postmenopausal with osteoporosis and high Alkaline phosphatase ,What will you give :
A)
answer:
Bisphosphonate
DISCUSSION: Bisphosphonate treatment lowered ALP levels, and this decrease was
strongly correlated with a decrease in BAP. Among blood test data, the decrease in BAP
had the strongest correlation with the ALP decrease.
CONCLUSION: For treatment of osteoporosis, ALP is an acceptable alternative to BAP.
Elevated ALP in postmenopausal women is mainly caused by high bone turnover.
http://www.ncbi.nlm.nih.gov/m/pubmed/25534961/
292

102- pt have obesity , hirsutism , htn, insulin resistance What is the dx? In options there's
no pcos?
kallmann syndrome
klinefelter syndrome
I don't know
the answer but I think there is another name of pcos
answer:
Repeated
103- female with ductal carcinoma Doctor want treat her, what is the gene responsible for
that cancer? (No BRCA1 in options)
tp53
I don't remember the remaining
answer:
104- what is the antithyroid used in pregnancy?
propylthiouracil
answer:
PTU
105-17 year old deliver her baby in the home with help of her friend ,, what type of perineal
muscle tear ?
pubococcygeus
ischiocavernosus
answer:
Repeted
106- Postpartum patient with bilateral breast engorgement and tenderness, what is the
management?
A)Hot compression and continue breastfeed.
B) Cold compression and stop breastfeed.
C) Oral Dicloxacillin and continue breastfeed.
D) Oral Dicloxacillin and pump the milk into bag and discard it.
Answer: A
107- 42-year-old patient with PCOS, nulligravida, she never took any medication to regulate
her period, endometrial biopsy showed endometrial hyperplasia, what is the cause?
Old age.
Unopposed estrogen.
Ans: unopposed estrogen.
108- Old female with endometrial biopsy showing: high grade hyperplasia with atypia. What
is the best management?
Trial of OCP.
Total abdominal hysterectomy.
Cauterization.
Answer:

293

109- Pregnant female in 24 weeks gestation, known case of chronic DM type 2 and chronic
HTN, fundal height is 25 cm, which one of the following is a complication of her pregnancy?
a. Preeclampsia.
b. Shoulder dystocia.
c. Large infant for gestational age.
Answer: A
Hypertension/preeclampsia (especially if pre-existing nephropathy/ proteinuria): insulin
resistance is implicated in etiology of hypertension

110- On examination: her cervix is dilated by 3 cm and effaced by 70% and fetal presenting
part at 0 station. After 6 hours or so, her cervix is dilated by 7 cm and effaced by 80%, but
she is complaining of tenderness when palpated the uterus, her temperature is 38. what is
the best management?
Give intrapartum antibiotics.
Emergency CS.
Wait for spontaneous delivery.
Answer:A
ref :uptodate
111- Pregnant in 3rd trimester with pain and bleeding, CTG showed late deceleration.
Uterus is distended. What is the diagnosis?
Placenta previa.
Vasa previa.
Abruptio placenta.
Answer:
Causes of late deceleration:
fetal hypoxia and acidemia, maternal hypotension or uterine hypertonus (Toronto notes
OB35)

294

Emergency Medicine

295

1- Patient after road traffic accident developed chest pain. On examination: no lung sound and hyperresonance on
one side. What is the 1st thing to do?
a.
Needle decompression
Answer: A
Tension pneumothorax
2- Patient ingested 20 gram of acetaminophen 8 hours ago, what is your next step?
a.
Charcoal
b.
N acetylcysteine
c.
Gastric lavage
Answer: B
gastric lavage/emesis (if <2 h after ingestion)
oral activated charcoal (if the patient has a stable mental and clinical status and presents to the emergency
department within 1 hour of ingestion)
N-acetylcysteine (NAC, Mucomyst) can be given PO or IV (most effective within 8-10 h of ingestion, but should
be given no matter when time of ingestion) > promotes hepatic glutathione regeneration
Draw a 4-hour serum acetaminophen concentration to determine the risk for hepatotoxicity, plot this value using the
Rumack-Matthew nomogram. Patients with acetaminophen levels below the possible line for hepatotoxicity on the
Rumack-Matthew nomogram may be discharged home after they are medically cleared. If the ingestion occurred with
intent to do self-harm, a thorough psychosocial, psychological and/or psychiatric evaluation is indicated before the
patient can be discharged safely from the medical care facility.
Admit patients with acetaminophen plasma levels above the possible line on
the Rumack-Matthew nomogram for treatment with N -acetylcysteine (NAC).
Treat patients with evidence of hepatic failure, metabolic acidosis, coagulopathy,
and/or encephalopathy in an intensive care unit (ICU). Transfer patients with
evidence of clinically significant hepatotoxicity to a medical facility with intensive
care support and organ transplant services.
Reference:
Toronto Notes
http://emedicine.medscape.com/article/820200-treatment
3- Patient ate from a restaurant. 2 days after that started to complain of diarrhea, vomiting and urticaria.
a.
Food poisoning
b.
Food allergy
Answer: ?
Food poisoning: http://emedicine.medscape.com/article/175569-overview
Food Allergy: http://emedicine.medscape.com/article/135959-overview
4- Treatment of bee sting.
Answer: ?
ABC management, epinephrine 0.1 mg IV over 5 min if shock, antihistamines, cimetidine 300 mg IV/IM/PO, steroids,
-agonists for SOB/wheezing 3 mg in 5 mL NS via nebulizer, local site management (ice packs). Reference: Toronto
Notes and Medscape.
5- Aspirin toxicity treatment
a.
Alkalinization of urine
Answer: A
Reference: Toronto Notes

296

6- What is the antidote for organophosphate poisoning?


a. Atropine
Answer: A
Reference: Master the Boards.
Organophosphate poisoning signs and symptoms in the mnemonic DUMBBELS: Diarrhea, Urination, Miosis,
Bradycardia, Bronchorrhea, Emesis (NV), Lacrimation, Salivation/Sweating.
7- A patient comes to the emergency with sudden dypsnea. X ray picture was attached showing dark area
without vascular marking. What is the diagnosis?
a.
Pneumonia
b.
Pulmonary edema
c.
Pneumothorax
d.
Pulmonary embolism
Answer: C
8- A guy was standing at a bus stop then fell. No previous history and no cardiac anomalies. What is the diagnosis?
a.
Cardiac syncope
b.
Fainting??!
c.
Seizure
d.
Vasovagal
Answer: D
Vasovagal cause fainting/ syncope. I dont think Fainting was included in the choices provided in the exam.
9- A guy presents in emergency with decreased oxygen carrying capacity of blood cells. Suicidal attempt is suspected.
what is the culprit?
a.
Carbon monoxide
b.
Cyanide
Answer: A
10- Patient came to the ER with status asthmaticus. What will you use to intubate him?
a.
Propofol
b.
Ketamine
c.
Midazolam
Answer: B
Studies suggest that ketamine and propofol have bronchodilatory properties and thus are suitable induction agents for
the severe asthmatic. Ketamine is preferable because it aids bronchodilation through both direct and indirect
mechanisms and helps to maintain blood pressure. Barbiturates, such as thiopental, should be avoided because they
can exacerbate bronchospasm through histamine release. Reference: http://www.uptodate.com/contents/emergencyairway-management-in-acute-severe-asthma
11- Trauma due to MVA. The patient has spinal injury with hypotension and bradycardia. What is the type of shock?
a.
Neurogenic
Answer: A
12- Soldier tries to have an excuse from the military, presented to the ER with symptoms of tremors, hypoglycemia.
What is he using?
a.
Factitious injection of insulin.
b.
Metformin
Answer: A

297

13- Acute attack of cluster headache, what is the abortive treatment?


a.
Oxygen 100%
b.
Subcutaneous Sumatriptan
Answer: A
Both Oxygen 100% and Subcutaneous Sumatriptan are effective. Reference: UpToDate.
Abortive Therapy for Cluster Headache: ergotamine, triptans (sumaptriptan, eletriptan, almotriptan, zolmitriptan),
100% O2, prednisone, lithium. Prophylaxis: Verapamil
Abortive Therapy for Migraine Headache: ergotamine, triptans (sumaptriptan, eletriptan, almotriptan, zolmitriptan).
Prophylaxis (Given when there is 3 or more episodes per month): Propranolol (Best), CCB, TCA, SSRI, Topiramate,
Botulinum toxins injections. Reference: Master the Boards
14- Semiconscious polytrauma patient, has difficulty breathing on bag ventilator (Ambu bag), prepared for
intubation. What is the next step?
a.
Go directly and intubate
b.
Cricoid pressure
c.
O2 supplements
d.
Jaw thrust
Answer: C
Preoxygenation/Denitrogenation with 100% O2 for 3 min or 4 vital capacity breaths. Reference: Toronto Notes A9
15- A patient ate a wild mushroom. Which of the following will be inhibited?
a.
RNA polymerase I
b.
RNA polymerase II
c.
RNA polymerase III
d.
DNA Gyrase
Answer: B
Reference: Wikipedia.
16- Contraindication for gastric lavage?
Answer: ?
Initial resuscitation incomplete
Risk assessment indicates good outcome with supportive care and antidote therapy alone
Unprotected airway where there is a decreased level of consciousness or risk assessment indicates potential for
these complications during the procedure
Small children
Corrosive ingestion
Hydrocarbon ingestion
Reference: http://lifeinthefastlane.com/ccc/gastric-lavage/+
https://www.clintox.org/documents/positionpapers/GastricLavage.pdf
17- Post-traumatic amnesia, vitally stable but hes complaining of pain in all of his 4 extremities. Which type of
shock might be developed?
a.
Neuro
b.
Cardiac
c.
Hypovolemic
d.
Reversible
Answer: A
Reference: Toronto Notes

298

18- Person escaped from fire in his flat from 3rd floor and jumped through the window. What will be the most life
saving in this case?
a.
Maintain Airway.
b.
Call for help.
Answer: B
19- 4 members in the family developed nausea and diarrhea 8 hours after eating at a restaurant. Then after 48 hours
they improved. Stool test showed oxidase positive gram +ve bacilli. What is organism?
a.
Shigella
b.
Salmonella
Answer: ?
Shigella (gram ve)
Salmonella (gram ve)
Bacillus cereus is a Gram-positive aerobic or facultatively anaerobic, motile, spore-forming, rod-shaped bacterium that
is widely distributed environmentally. B. cereus is associated mainly with food poisoning. Reference:
http://www.ncbi.nlm.nih.gov/pmc/articles/PMC2863360/

Reference: Toronto Notes

20- What is the degree of shock in this patient?


Answer: ?

299

Reference: Toronto Notes


21- Aspirin toxicity resulting in which of the following?
a.
Respiratory alkalosis followed by metabolic acidosis
Answer: A
Phase I: Characterized by hyperventilation resulting from direct respiratory center stimulation, leading to respiratory
alkalosis and compensatory alkaluria. Potassium and sodium bicarbonate are excreted in the urine. This phase may
last as long as 12 hours.
Phase II: Characterized by paradoxic aciduria in the presence of continued respiratory alkalosis occurs when
sufficient potassium has been lost from the kidneys. This phase may begin within hours and may last 1224 hours.
Phase III: Characterized by dehydration, hypokalemia, and progressive metabolic acidosis. This phase may begin 46
hours after ingestion in a young infant or 24 hours or more after ingestion in an adolescent or adult. Reference:
Wikipedia.
22- Which of the following babies needs immediate action?
a.
Cough and wheezing.
b.
Shallow spinal skin.
c.
Absent lower pulse.
d.
Yellowish discoloration.
Answer: A

23- 36 year old male having retrosternal chest pain that radiates to jaw and left arm for 20 minutes (vitals
included). What is the first thing to give him?
a.
Aspirin.
b.
Morphine.
c.
Oxygen.
d.
IV nitroglycerin.
Answer: A

300

Reference: Master the boards: USMLE step 2 CK


24- Male patient presented to the ER after MVA, he has tachycardia, tachypnea, hypotension and cardiac
arrhythmia. What is your most correct next step is ?
a.
FAST to check abdominal hemorrhage
b.
Needle decompression
c.
Pericardiocentesis
Answer: A
25- You are working as an emergency doctor and you got a call that there was an accident at the nuclear plant and 2
men were exposed to nuclear radiation. They dont have any complaints right now. What will you advise them?
a.
Cover them up with dry thick blankets
b.
Give them aspirin and ask them to keep their legs moving
c.
Isolate them and advise them to stay calm
d.
Give them antibiotics
Answer: C or D ?
In hospital casualty will be finally decontaminated and kept in a clean special ward. The decontamination room
must be at the entry of the hospital and should be sealed off from other premises and should have a separate
ventilation system. References: http://www.apiindia.org/pdf/pg_med_2008/Chapter-17.pdf
Prevention and treatment of infections. References: http://emergency.cdc.gov/radiation/arsphysicianfactsheet.asp

26 -Young pt come to ER with palpitation, euphoria, visual hallucinations,


a.

Amphetamin poisining

Answer: A
CLINICAL FEATURES Clinicians should consider the diagnosis of methamphetamine intoxication in any diaphoretic
patient with hypertension, tachycardia, severe agitation, and psychosis. Acutely intoxicated patients may become
extremely agitated and pose a danger to themselves, other patients, and medical staff
Ref :uptodate

301

27 - Unconscious pt after ingestion of overdose sleep pills, breathing was reflex response breathing , you give him 2
breathing by mask , then you check pulse it was rapid and weak , what is
next step:
A.
wait code blue team
B.
put pt in recovery position
C.
intubat and ventilation
D.
do CPR 5 cycle 30:2
Answer: C
Airway protection Airway protection by endotracheal intubation should be performed early in the poisoned patient
with depressed mental status, unless the cause is easily reversible (eg, opioid intoxication or hypoglycemia), because of
the high risk for aspiration and its associated complications, particularly when gastric decontamination procedures
need to be undertaken [40]. Tracheal intubation with mechanical ventilation is also indicated in the presence of severe
acid-base disturbances or acute respiratory failure. Particularly when intubating a severely acidemic patient, it is
important to prevent the development of a respiratory acidosis through inadequate minute ventilation. Occasionally,
the management of high-grade physiologic stimulation may require sedation and/or paralysis with mechanical
ventilation to limit the extent of complications such as hyperthermia, acidosis, and rhabdomyolysis. One rare exception
to this important principle of aggressive airway management is salicylate poisoning, in which mechanical ventilation
should be avoided unless absolutely necessary.
Ref: uptodate
28 - child girl obese try to suicide and eat alot of drug because of her friend and came to the ER?
A.
referral immediate to the psychatry
B.
treatmen for acute depression
C.
something
D.
another something
Answer:A ?
29 - after intubation still low in spo2 wt u will do
Answer:
causes of hypoxia occurring soon after intubation: Think DOPES:
Displacement of the endotracheal tube (ETT)
Obstruction of the ETT
Patient especially pneumothorax; also: pulmonary embolism, pulmonary edema, collapse, bronchospasm
Equipment ventilator problems
Stacked breaths a reminder about bronchospasm and ventilator settings.
first step in managing an intubated and ventilated patient who is hypoxic: Disconnect the ventilator and administer
high-flow 100% oxygen using a bag-valve-mask.
30 - patient brought to ER with head trauma and loss of consciousness, what should you do first?
A) check for pupillary reflex
B ) intubate and hyperventilate
C) glascow coma scale
D) check the airway.
answer : d

ADDED QUESTION: ( need to be more checked )


1.anterior abdominal stab wound omuntam come through the wound ?
A. Fast
B. CT
C. Exploratory laparotomy
Answer: C?

302

2.patient came from RTA and the staff preparing him for intubation he devolved low ventilation on bival
ve what to do ? repeated
A.Proceed to intubation
B. jaw thrust
C. More head tilt
D. Cricoid pressure
Answer:
3.Pt work outdoor in temp 42 c .. Tired and Complain of crampy abdominal
pain .. Lower limb pain and fever .. Otherwise normal ,, what to do ?
1- electrolytes and oral replacement fluid
2- warming
3- cooling
Answer:
electrolytes and oral replacement fluid
4.Patient is bleeding the baro receptor activated result in increased tachycardia and decrease of ?
1.HR
2.Ventricular rate
3.Coronary artery flow
Answer 3?
5.Atropine side effect ? Dry mouth
Answer : Dry mouth
6.Patient came to you with history of bee sting with light heeded and shortness of breath what is the Rx ?
1.Oral histamine reassurance
2.Fluids and elevate the leg
3.Sc epinephrine , IV histamine
Answer
7.Which type of insulin used in DKA ?
Regular ( short acting )
Answer: short acting
8.man got a bee sting then his wife trying look for the epinephrine what it gonna inhibit?
1-leukotriene release from macrophages
2-cross reactivity with the cardiac..
3-inhibit immunocomplex formation
Answer:
9.Pt conscious with multiple injuries. How do you maintain airway?
1.Mask
2,Oro pharyngeal airway
3,Nasopharyngeal
4, Endotracheal Intubation
Answer : mask
10.Depressed pt with HTN Brought by family to ER for drug overdose palpitation diaphoresis and ECG shows
arrhythmia. What is the possible drug ?
1.Ssri
2.Digoxin
Answer. Ssr
11.Old pt with high cholesterol level measured before 4 months and he is on a diet. Came to ER complaining of
chest pain. What will concern you?
1.Current symptoms
2.Cholesterol level before 4 m
Answer. Current symptoms

303

12.Pt in ER MVA. First thing you do?


ABC
Answer abc
13.gunshot with wound bowel perforation
What antibiotics you should give
answer :
14. patient presented to ER with history of drug overdose and coma for the
last 8 hours on examination absent gag reflex best management is :
1.iv naloxone
2-gastric lavage
3-immediate endotracheal intubation
4-coracol
answer immediate endotracheal intubation
15.Case of RTA and they found difficulty in intubation what should they do :
1.just proceed to intubation whatever.
2.increase chit tilt .
3.press on cricoid
answer
16- Patient has acute MI. which of the following enzymes will be elevated?
Creatine kinase.
alanine aminotransferase.
Alkaline phosphatase.
Answer: ck
17- Patient had bee sting on the hand yesterday, he is presenting to you with redness and itching on the bite site.
What is the treatment?
Oral steroid.
Antihistamine.
Oral Antibiotic.
Ans: Antihistamines
Answer: antihistamines
18 -Patient had bee sting and presented to you immediately with shortness of breath and lightheadedness. What is
the treatment?
SC epinephrine.
Ans: SC epinephrine
Answer: im epinephrine
19- Patient with ischemic stroke presented after 6 hours. What is the treatment?
Tissue plasminogen activator.
Aspirin.
Clopedogril.
answer : aspirin
20- patient came from RTA and the staff preparing him for intubation he devolved low ventilation on bivalve
what to do ?
1-Proceed to intubation
2- jaw thrust More head tilt
3-Cricoid pressure
answer : Proceed to intubation
21-Unconscious patient after RTA .. Ventilation with bag mask was difficult .. What to do ? 1- exaggerate jaw
thrust
2- increase head tilt
3- coricoid pressure
4- intubation
answer : Intubation

304

22-Man got bee sting ,, his wife gave him epinephrine .. What is the Immunologic action of epinephrine ?
Iterlukine..... Ect
Tryptanase .....ect
Answer: tryptan

305

Anesthesia

306

1- Earliest sign of local anesthetic toxicity:


a.
Tongue and circumoral numbness
Answer: A
Reference: Toronoto Notes.
2- What is the fasting duration for non-breast milk?
a.
8 hours
b.
6 hours
c.
4 hours
d.
2 hours
Answer: B
8 hours (solid)
6 hours (formula)
4 hours (breast milk)
2 hours (clear liquid)
Reference: Toronto notes.
3- Which of these analgesia has x10 the strength of morphine?
a.
Fentanyl
b.
Tramadol
c.
NSAID
Answer: A
Reference: Toronto Notes.
Extra notes:
Common Side Effects of Opioids: Nausea and vomiting, Constipation, Sedation, Pruritus, Abdominal pain, Urinary
retention, Respiratory depression..
When prescribing opioids, consider: Breakthrough dose, Anti-emetics, Laxative.
4- What is the sign that indicate inadequate general anesthesia?
a.
Dry skin.
b.
Hypotension.
c.
Bradycardia.
d.
Dilated Pupils.
Answer: D
Reference: Toronto Notes
5- Pregnant on delivery she has hypotension and dyspnea. Which type of anesthesia will be given?
a.
Pudendal n
b.
Local cervical
c.
General anesthesia
d.
epidural ( Newly added )
Answer: A
Reference: UpToDate
6- What is the anesthetic drug that can be used as an analgesic in mild doses?
a.
Midazolam
b.
Thiopental
Answer: ?
ketamine (Answered by Anesthesiologist consultant)
Reference: http://www.ncbi.nlm.nih.gov/pubmed/25197290

307

7- Mallampati class 3 what you will see?


a.
Soft palate and uvula
b.
Soft palate and base of uvula
c.
Soft palate,uvula and tonsillar pillars
Answer: B

8- During labor, the anesthesiologist inject analgesic drug at L3-L4, after 2 days the mother still have pain in the site
of injection. Which ligament is affected ?
a.
Anterior longitudinal ligament.
b.
Posterior longitudinal ligament.
c.
Ligamentum flavum.
d.
Interspinous ligament.
Answer: D
In this Q: we don`t know if the doctor used spinal or epidural but in general pain at site of injection = inflammation of
supraspinous lig or interspinous lig. (Answered by an anesthesia consultant at KFMC)
Note: The associations between back pain and epidural analgesia are unclear.
- Local tenderness at the site of epidural or spinal placement are relatively common & usually clears within several days
to 3 weeks and may be related to superficial irritation of the skin or periosteal irritation or damage.
- Although short-term back pain is common, it does not appear to be related to the use of regional analgesia. Similarly,
no causal relationship exists between the use of epidural analgesia and the development of long-term postpartum
backache.
9- How to prevent a high level of anesthesia?
a.
Anti trendelenburg
b. Trendelenburg
Answer: A?
If the level of spinal anesthesia is not fixed, the Trendelenburg position can alter the level of spinal anesthesia and
cause a high level of spinal anesthesia in patients receiving hyperbaric local anesthetic solutions. This can be minimized
by raising the upper part of the body with a pillow under the shoulders while keeping the lower part of the body
elevated above heart level. Reference: http://www.nysora.com/index.php?news=3424
10 - pt with local anesthesia cant remember
how to prevent high level anesthesia?
A.
anti trendlendburg
B.
trendlenburg
Answer:a??
11 - Station +1 80% effacement and 4 cm dilatation and RUpture of membrane with clear fluids, : what anesthesia to
give?
A- VS
B-Pudendal,
C- paracervical,
D- GA
Answer: D

308

12 - Same of common about breastfeeding


wt is indicative of poor anasthsia ??
Answer:
"awareness with recall" (AWR) refers to both intraoperative consciousness and explicit recall of intraoperative events.
Other terms, such as "intraoperative awareness during general anesthesia," "anesthesia awareness," or simply
"awareness," are used as synonyms.
Anesthetic underdosing The most important contributing factor for AWR is underdosing of anesthesia relative to a
given patient's specific requirements.
This can occur for the following reasons [21-23]:
There is a mistake or failure in the delivery of anesthesia
The anesthetic technique results in inadequate anesthesia
It is judged unsafe to administer sufficient anesthesia
A specific patient's needs are underappreciated
Ref :uptodate
13 - pregnant with late deceleration what type of anesthesia u will give
her
Answer:?

ADDED QUESTION: ( need to be more checked )


1.anesthesiologist put the needle lateral to spinous process, which ligament is perforated? 2 times
a.
interspinous
b.
ligamentum flavum
c.
Anterior ligament
Answer: B
The needle should be inserted 1 cm lateral to this point and directed toward the middle of the interspace. The
ligamentum flavum is usually the first resistance identified
http://www.nysora.com/index.php?news=3425

2. -to increase hyperbaric in intrathecal analgesia put patient in :


A) lateral
B) supin
C)trundlberg
D)antitrendulberg
Answer: C
Hyperbaric solutions goes with gravity
Hypobaric solutions goes against gravity
So to increase hyperbaric solutions spread the patient is put in antitrendulenberg position

309

Family Medicine
and
Statistics

310

1- What is most common serious chronic infection found in expatriates coming to Saudi Arabia?
a.
Hepatitis A
b.
Hepatitis B
c.
Hepatitis C
d.
HIV
Answer: B
Reference: http://applications.emro.who.int/emhj/v19/07/EMHJ_2013_19_7_664_670.pdf?ua=1
2- The most difficult method to prevent transmission:
a.
Person to person
b.
Vector
c.
Droplet
d.
Airborne
Answer: D
Reference: 3rd Edition UQU last touch > Family medicine and statistics > Q 3.
3- At a daycare center 10 out of 50 had red eye in the first week, another 30 developed the same condition in the
next 2 weeks. What is The attack rate (cumulative incidence)?
a.
40%
b.
60%
c.
80%
d.
20%
Answer: C
Attack Rate = cases due to a specific cause in a short period of time divided by the population at risk, often associated
with an epidemic situation such as food borne disease (cumulative incidence)
Reference: 3rd Edition UQU > Family medicine and statistics > Q 15.
4- In a study they are selecting every 10th family in the city, what is the type of study
a.
systematic study
b.
stratified study
c.
non randomised study
Answer: A
It differs from the one written in 3rd Edition UQU > Family medicine and statistics > Q 19
Random sampling: preferred way of sampling, it is often difficult to do. It requires that a complete list of every element
in the population be obtained. Computer generated lists are often used with random sampling.
Systematic sampling: every kth element is taken. This is similar to lining everyone up and numbering off "1,2,3,4;
1,2,3,4; etc". When done numbering, all people numbered 4 would be used.
Convenience sampling: readily available data is used. That is, the first people the surveyor runs into.
Cluster sampling: accomplished by dividing the population into groups called clusters -- usually geographically.
The clusters are randomly selected, and each element in the selected clusters are used.
Stratified sampling: divides the population into groups called strata. For instance, the population might be separated into
males and females. A sample is taken from each of these strata using either random, systematic, or convenience
sampling.

311

5- Epidemiological study want to see the effect of smoking in lung cancer: 90% of lung cancer patients are
smokers while 30% of those who dont have the disease are smokers. The specificity of smoking as a risk factor is:
a.
10%
b.
40%
c.
30%
d.
70%
e.
90%
Answer: D (The answer was confirmed by an intern who had the same question in the exam and got 100% in
statistics section)
Explanation: Specificity = 70/ (30+70) = 70%
Lung Cancer

No Lung Cancer

Smoker

A (90)

B (30)

Non smoker

C (10)

D (70)

Important equations:
Prevalence of Disease = People who have the disease / Total 100
Sensitivity = A/(A+C) 100
Specificity = D/(D+B) 100
Positive Predictive Value = A/(A+B) 100
Negative Predictive Value = D/(D+C) 100
6- You have devised a new test to diagnose a disease and you want to check it against a gold
standard test. There were figures showing the number of those who actually have the disease according
to the gold standard test and the number of those who were tested positive with your test. Also figures showing the
number of true negatives with your test. What is the specificity of your new test?
a.
40%
b.
50%
c.
60%
d.
80%
Answer: ?
Similar to question 5
7- You conducted a study in which a group of epileptic patients using carbamazepine for 10 years. Now you
compare them with their age equivalent healthy individuals. What is this type of study called?
a.
case control study
b.
retrospective cohort study
c.
cross sectional
d.
prospective cohort study
Answer: D
Cohort study: compares a group with exposure (carbamazepine in this Q) to a group without such exposure. Done in
two ways; either Prospective (you give the exposure and follow the subjects for a specific time) or retrospective (the
exposure occurred already in the past, you look back in the history).
Case control study: compares a group with disease to a group without disease.

312

8- The chairman of the public health wants to reduce the incidence of stroke. He is reading the literature but he is
confused. What is the best intervention?
a.
Cholesterol level test for all population
b.
Anti Smoking campaign
c.
Hypertension booth in the malls
d.
Obesity booth in the mall
Answer: C
9- An adult came to you for varicella vaccine. How will you give it to him?
a.
one dose now and one after 2 weeks.
b.
one dose now and one after 3 months
c.
one dose now and one after 6 weeks
d.
just give one dose now
Answer: C
For adult; two doses of varicella vaccine at least 4 weeks apart (the 2nd dose can be given after 4 weeks or more)
For children; two doses of varicella vaccine at least 3 months apart
10- Least harmful vaccine in immunocompromised patient.
a.
BCG
b.
pneumococcal vaccine
c.
measles
d.
Mumps
Answer: B
Pneumococcal vaccine (both conjugate and polysaccharide) classified as a subunit vaccine, which is like inactivated
vaccine can be given to immunocompromised patients.
BCG, measles and mumps are a live attenuated vaccine which should not be given to immunocompromised patient.
MMR (measles, mumps and rubella) vaccine is the only exception of live attenuated vaccine type can be administered
to asymptomatic HIV/AIDS patient (not all immunocompromised patients)
11- There is a new outbreak of TB. What will you do?
a.
Give Ethambutol chemoprophylaxis.
b.
Give Rifampicin chemoprophylaxis.
Other 2 irrelevant choices.
Answer: B
Rifampicin: Prophylaxis for those exposed to people with N. meningitidis or HiB meningitis. Reference: Toronto Notes
12- Non response bias. What is this study?
a.
Cross sectional.
b.
Cohort.
c.
Case-control
Answer: A
Non response bias (example of selection bias) occurrs with cross sectional study.
13- A man who wants to travel. What is your advice?
a.
Eat fruits and vegetables.
b.
Eat vegetables that can be peeled.
c.
Drink iced water.
Answer: B
Reference: Oxford handbook of General practice, 3rd edition, Prevention of travel-related illness (P174)

313

14- Exposed odds ratio of 3 and non exposed odds ratio 2. Which of the following is true?
a.
Number needed to treat is ...
b.
Number needed to harm is ...
c.
Relative risk is ...
Answer: ?
15- Cohort study, calculate the Relative Risk.
Diseased
Healthy
Exposed
A
B
Not exposed
C
D
Answer: ?
Relative Risk = (A / (A+B)) / (C / (C+D)).
16- Case-control study, calculate the odds ratio.
Answer: ?
Odds ratio: (A/B)/ (C/D) or A*D/ B*C

17- What is the most common problem faced by community medicine.


a.
HTN
b.
Coryza
c.
UTI
Answer: C
Reference: Oxford handbook of General practice, 3rd edition, UTI (P456)
This question was answered by one of the doctors presenting in VMA lectures. He said that Coryza is the most
common.

18- What questionnaire to ask about alcohol intake?


Answer:
http://www.niaaa.nih.gov/research/guidelines-and-resources/recommended-alcohol-questions

314

19- What is the comment cause of hepatitis in KSA?


a.
HBV
Answer: B
Reference: http://www.alliedacademies.org/articles/profile-of-viral-hepatits-in-saudi-arabia.pdf
20- An example of secondary prevention is:
a.
Detection of asymptomatic diabetic patient
b.
Coronary bypass graft
c.
Measles vaccination
d.
Rubella vaccination
Answer: A
Primordial prevention: consists of actions to minimize future hazards to health and hence inhibit the establishment
factors (environmental, economic, social, behavioural, cultural) known to increase the risk of disease, e.g. improving
sanitation. It addresses broad health determinants rather than preventing personal exposure to risk factors, which is
the goal of primary prevention.
Primary prevention: Action to protect against disease as immunization and to promote health as healthy lifestyle.
Secondary prevention (screening): Identifying & detecting a disease in the earliest stage before symptoms appears.
Tertiary prevention: Improves the quality of life of people with various diseases by limiting the complications.
Reference: 3rd Edition UQU > Family medicine and statistics > Q 98.
21- Healthy young adult with high cholesterol level. When will you follow him up again for dyslipidemia?
a.
6 months
b.
12 months
c.
24 months
d.
36 months
Answer: A?
Fasting lipid levels should be obtained 6-8 weeks following the initiation or alteration of therapy. For patients at goal,
lipid profiles should be performed annually.
Reference: https://ghcscw.com/SiteCollectionDocuments/Clinical_Practice_Guidelines/8_CPG_Dyslipidemia.pdf.
Less than two Cardiac Risk Factors with elevated lipids: Repeat lipid panel in 3-6 months
Reference:http://www.fpnotebook.com/cv/Lipid/Hyprchlstrlm.htm
22- A patient is concerned about microwave radiation and its risk of cancer on their children. They are asking for
your advice, what will you tell them?
a.
Microwave cause cancer but not in children
b.
Microwave don't cause cancer
Answer: B
If you use a microwave oven in the correct way there is no known harmful effect on humans.
http://www.cancerresearchuk.org/about-cancer/cancers-in-general/cancer-questions/radiation-microwaves-and-cancer
23- Vegetarian patient having angular cheilitis:
a.
Vit B6 deficiency
Answer: A
Causes of angular chelitis: Vit B2, Vit B5, Vit B12, Vit B3, Vit B6 deficiencies, zinc and iron deficiency,
Vitamins deficiency among vegetarians: Vit B12, D, B6,
B12 deficiency is a common problem among people who follow a vegan diet.
Reference: Webmed + Wikipedia + Medscape.

315

24- What is the organism found in undercooked meat?


a.
Entameba histolytica
Answer: ?
Organisms that can be found in raw meat:
Beaf: E. coli O157:H7, Salmonella, Shigella, Staphylococcus aureus and Listeria monocytogenes
Poultry: Salmonella and Campylobacter
Shellfish: Vibrio gastroenteritis, Salmonellas, Plesiomonas shigelloides, Staphylococcus and Bacillus cereus

Reference: Toronto Notes.


25- Best types of carbohydrate in DM.
a.
Monosaccharide
b.
Disaccharide
c.
Polysaccharide
Answer: C
Simple carbs have only one or two sugars, so they are digested quickly, making blood glucose rise rapidly to a high peak,
which is what diabetics need to avoid. Examples of simple carbs include the sugars found in fruits and milk, the added
sugars in processed foods, and table sugar.
Complex carbs contain three or more sugars, so these take longer to digest and thus they cause a less rapid rise in blood
glucose and a lower peak. Examples of complex carbs include the fibers in spinach, watercress, buckwheat, barley, wild
or brown rice, beans, and some fruits.
26- Children who are living in a poor country with poor hygiene will have a high risk to hepatitis:
a.
A
b.
D
c.
E
Answer: A
27- A school did a screening test for their students and they found that there is a good number of obese students.
The school doctor wants to know more about these students before educating their parents. What should you
provide him with?
a.
HDL\LDL
b.
Girth measurement
c.
Dietary habits
Answer: C?
Reference: Oxford General practice, 4th edition, obesity (P:174)
28- Medical director discovered cretinism in 90% of children in his village, when he analyzed the water he found
that it is deficient in iodine. The director wants to prevent and manage cretinism. What he is going to do initially?
a.
Iodine supplementation
b.
Thyroxin supplement (Levothyroxine).
c.
TSH and T4 in 2 weeks.
Answer: B
The mainstay in the treatment of congenital hypothyroidism is early diagnosis and thyroid hormone replacement.
Reference: Medscape.

316

29- What is the mineral found in water prevents teeth cavities?


a.
Folic acid
b.
Iron
c.
Fluoride
Answer: C
30- The targeted glycated hemoglobin in a patient with type 1 DM should be:
a.
0.065
b.
0.08
c.
0.095
d.
0.11
Answer: A
Therapy in most individuals with type 1 or type 2 diabetes should be targeted to achieve a HbA1c 7.0% in
order to reduce the risk of microvascular and if implemented early in the course of disease, macrovascular
complications.
More intensive glucose control, HbA1c <6.5%, may be targeted in patients with a shorter duration of diabetes
with no evidence of significant CVD and longer life expectancy, to further reduce risk of nephropathy and
retinopathy, provided this does not result in a significant increase in hypoglycemia.
A HbA1c target <8.5% may be more appropriate in type 1 and type 2 patients with limited life expectancy,
higher level of functional dependency, a history of recurrent severe hypoglycemia, multiple co-morbidities,
extensive CAD, and a failure to attain established glucose targets despite treatment intensification. Reference:
Toronto notes.
31- (long scenario) old man with DM and HTN, came for routine checkup. ( CBC, FBS, Lipid profile .. etc.).
What is the targeted glycosylated hemoglobin?
a.
0.065
b.
0.070
Answer: B
See the explanation of previous question
32- 80 persons found to have Typhoid Fever in a population of 20,000 during the last 4 years. If we measure it
on a population of 100,000, what will be the incidence in one year?
a.
20
b.
100
c.
150
d.
200
Answer: B
Incidence in a population of 20,000 for 1 year is 20/20,000.
Therefor, the incidence in a population of 100,000 for 1 year is 100/100,000
33- 32-year-old athlete man, who has a family history of HTN and DM, came for check up. He has good
musculature and doing weight-lifting exercises. His blood glucose level within normal range also his lipid
profile except for total serum cholesterol 210 mg/dl. What is your action?
a.
Start statin.
b.
Change diet.
c.
Check up in next few months.
d.
Reassure him.
Answer: B
Borderline high cholesterol treated with diet modification.

317

34- Young female with height of 167 cm and weight of 153 kg. In which class of BMI ?
a.
Overweight.
b.
Obesity class I.
c.
Obesity class II.
d.
Obesity class III.
Answer: D

Easy way to calculate BMI: Wt. 153 kg, Ht. 167 cm 1.67 meter.
Step 1: Wt./Ht. (in meters) = 153/1.67 = 91.6
Step 2: Result of step one divided again by Ht. (in meters) = 91.6/1.67 = 54.8
Answer is 54 Which is extreme obese.
35- 28-year-old girl came for checkup. She asks about when to do breast self examination?
a.
Not advised.
b.
Advise every 4 months.
c.
Advise every 12 months.
d.
Advise 3 years.
Answer: A
Reference: Oxford General practice, Edition 4 (P:686)

36- Saudi Arabia has a good screening for military job. what the infection he man can't take the job?
a.
Hepatitis A virus
b.
Hepatitis B virus
c.
Hepatitis C virus
d.
HIV
Answer: D?
Reference: http://www.beforejoiningthemilitary.com/military-medical-requirements-and-disqualifying-conditions/
37- What is the vaccine type of Hepatitis B?
Answer: ?
Subunit
Refer to the table at the end of FM section.
38- What is the vaccine type of Pneumococcal vaccine?
Answer: ?
Both conjugate and polysaccharide are classified as a subunit vaccine (which is like an inactivated vaccine)
Refer to the table at the end of FM section.

318

39- Patient G2P0 had two abortions, came to you in clinic asking to give her any vaccine to help her to get a baby,
HCG is positive. Which vaccine will you give her?
a.
Varicella
b.
Rubella
c.
Mumps
d.
Influenza
Answer: D
Varicella, Rubella and Mumps vaccines are all contraindicated in pregnancy. Reference: Uptodate.
40- Pregnant woman came to the clinic asking about tetanus risks and prevention. There is a history of contact
with patients who have tetanus. On examination: Normal and the measurement of uterine level from symphysis is
12 cm. What is the action to prevent baby from tetanus?
a.
Introduce Acyclovir to the mother at 18 weeks.
b.
Tetanus immunoglobulin and vaccine to the baby after delivery.
c.
Give Acyclovir to mother and baby after delivery.
d.
Give tetanus vaccine to the mother.
Reference: http://www.cdc.gov/vaccines/pubs/preg-guide.htm#tdap
41 - definition of epidemiology
Answer: the study of the distribution and determinants of health related events ( including diseases and application of
this study to control of diseases and the others health problems) (Om al Qura, family medicine )
42 - Most common problem in primary care?
A:uti
B:coryza ** rhinitis (OM al Qura, family medicine)
Answer:B
43 - Saudi arabia have good screening for military gob what the infection he man can't take the job:
A-hav
B-hbv
C-hcv
D-hiv
Answer:D
44 - You have an old pt he can't understand has answer you will take him to your clinic ask him open end
question:
Answer:?
45 - Whats the best approach in hx?
A- Open ended Q
Answer:
46 - best way of prevention :
A. Screening program
B. Genetic counseling
C. Increase individual health awareness
Answer: A-screening
47 - What is the best measure in health care ?
A-improve personal something ** I chose this one but im not sure
B-screening
C-individual education
d- behavioral modification of the person ( newly added )
Answer: 2 statistical
questions ..one about sensitivity and spectifity Another one about mortality rate

319

48 - THE MOST R.F OF STROKE?


A.DM
B.HTN
C.SMOKING
D.....
Answer: B
49 - THE MOST """REVERSIBLE"""" CAUSE OF STROKE IS ?
A.OBESITY
B.DM
C.HTN
D.HYPERLIPIEMIA
Answer:C
50 - WHAT IS THE MOST PREVENTIVE METHODS TO PREVENT HUMAN DISEASES
A.SCREENING
B.GENTIC COUNSELING
C.ENVITROMENT MODIFICATION

Answer:A

ADDED QUESTION: ( need to be more checked )


1.Ttt of traveler's diarrhea? (Repeated)
Answer: ciprofloxacin
2.patient with normal glucose level and HBA1c, he is 42 male that smokes for 20 years, when do you do these tests
again?
A.after 3 months
B.6 months
C. 12 months
D. 36 months
Answer
3. Collect information about BMI, what is the most imp additional information:
A. eating habits
B. HDL/LDL.
Answer: a?
4-.Doctor start research to comparing between people had risk factors to MERS-CoV and control group 100 had
risk factor 88 developed MERS-CoV 500 control group developed 6 what the ratio between risk group and control
group
A-(1:33
Answer:
5.Pt came with HTN and get controled by medication after 1 week she came with dry cough which medication is
prescribed ?
A- Perindoprill (ACE)
Answer:
6-.diabetic patient what type of carbohydrates is recommended
Answer:
7-.increase of which of the following prevalence cause reactivation of TB in developed countries? A-DM
B -HIV
Answer: B

320

8. with table of sensitivity, specificity, positive and negative predictive values


Answer
Sensitivity : the probability that a diseased pt will have a positive test result
Specificity : the probability that a non-diseased pt will have a negative test result
See the table :

source :UQU
9-.best prophylactic aginest travellar diarrhea :
A.fresh fruit and vegetable
B.daily antibiotic
C.peeld fruit
D.drinks with rice
Answer : c?
1072 old patient start to have memory loss gradually since 2 yrs back ..but he is capable of doing his daily
activity dressing himself but lately he start to forget the burner on.. and his personality changed from kind and
caring father to agg. And irritablewhat u will do
a) Do cost effective Ix
b) Refer to geriatric
c) TCA trial
d) Give him Risperidone ( antipsychotic ) e) Arrange to transfer him to caring facility true for severe case
Answer:.
B (UQU sle ) q151 p447 4th edition
11.Attributable risk meaning
Answer :
AR = the amount of disease incidence that can be attributed to a specific exposure
Difference in incidence of disease between exposed and non-exposed individuals
Incidence in non-exposed = background risk
Amount of risk that can be prevented
http://www.pitt.edu/~super7/30011-31001/30101-30111.ppt

321

12- CALCULATE THE ANUUALLY PREVELANCE


Answer:

322

13--COHERT STUDY ADVANTAGE


Answer:

323

http://www.ncbi.nlm.nih.gov/pmc/articles/PMC2998589/#!po=20.9677

There were also missing questions about the following:


Vaccination types
Different types of research sampling

324

325

Ethics

326

1- If you successfully treated your patient. Then the patient brought to you an expensive watch . What would be
your response?
a.
Accept the gift and say thank you
b.
Refuse the gift politely
c.
ask him for more gifts.
d.
Shout at him and say "I do not accept gifts"
Answer: B
2- (long scenario) lady complaining about work partner. She is agitated and mad. What is your action?
a.
Instruction to deal with him.
b.
Empathy during session.
c.
Avoid him.
Answer: ?
3- You have old patient who can't understand you. What will you do to get more information?
a.
you will take him to your clinic and ask him open ended question
Answer: ?

4- (long scenario) lady complaining about work partner, she is agitated and mad. What is your action ?
a. Instruction to deal with him.
b. Empathy during session.
c. Avoid him.
d. !!!!
Answer:?

327

Psychiatry

328

1- Obese boy is teased by his friends tells you that he wants to take pills and never wake up. What's your next step?
a.
Immediate psychiatry referral
Answer: A
2- What is the treatment of generalized anxiety disorder?
a.
Escitalopram
Answer: A
lifestyle: caffeine and EtOH avoidance, sleep hygiene
psychological: CBT including relaxation techniques, mindfulness
biological
SSRIs and SNRIs are 1st line (paroxetine, escitalopram, sertraline, venlfaxine XL)
2nd line: bupropion (caution due to stimulating effects ), buspirone (tid dosing)
add-on benzodiazepines (short term, low dose, regular schedule, long half-life, avoid prn)
Reference: Toronto Notes
3- Girl with BMI=16 says I'm fat.
a.
Depression
b.
Anorexia bulimia
c.
Anorexia nervosa
Answer: C
4- Elderly patient his wife died recently. Hes been depressed and having difficulty sleeping since that time. Which of
the following is a short term medication that is suitable for him?
a.
Diazepam
Answer: ?
Long acting benzodiazepines, with active metabolites should usually be avoided in the elderly e.g. diazepam,
chlordiazepoxide, flurazepam, nitrazepam. When benzodiazepines are prescribed in older people, short acting
benzodiazepines with few active metabolites are preferable e.g. lorazepam 0.5mg bd/tid, temazepam 10-20mg nocte,
alprazolam 0.25mg bd/tid. Reference: http://www.svhf.ie/documents/BenzoGoodPracticeGuide.pdf
5- Why SSRI is the drug of choice for depression?
a.
More tolerable and less side effects
b.
Cheap
Answer: A
Reference: http://www.bpac.org.nz/BPJ/2012/december/docs/bpj_49_nzf_pages_34-35.pdf
6- Patient turns to be erratic for 4 months, he said that people on TV know what hes thinking about and they are
talking about him. In the last 2 months he started to claim that he has special power that no one has. What is the
most likely diagnosis?
a.
Uni-polar...depression
b.
Bipolar...Mania
c.
Schizophrenia
Answer: C?
Reference: 3rd Edition UQU > Psychiatry > Q 30 and Toronto Notes

329

7- A man is behaving in a strange way after the death of his son. Hes approaching random people on the road and
ask them irrelevant questions. He wont listen when asked not to do that. He had no such behavior before in life.
Which of the following will be excluded from your differentials?
a.
Brief psychotic disorder
b.
Schizophrenia
c.
Schizophreniform
d.
Schizoaffective
Answer: D?
Reference: Toronto Notes PS 6 - 8
8- A patient presented with history of washing his hand a lot. What is the most likely diagnosis?
a. obsessive compulsive disorder
Answer: A
9- A guy who is showing an erratic behavior lately. He is drinking a lot, spending too much money and having
involved in sexual activities a lot. What is your diagnosis?
a.
drinking problem
b.
mania
c.
drug use
d.
schizophrenia
Answer: B
Reference: Toronto Notes PS 10
10- An alcoholic comes to you with symptoms of alcohol withdrawal. Last drink he consumed was 2 days back. What
drug will you give?
a.
Naloxone
b.
Diazepam
c.
Nicotine
d.
Disulfiram
Answer: B
diazepam 10-20 mg IV/PO or lorazepam 2-4 mg IV/PO q1hr until calm
thiamine 100 mg IM/IV then 50-100 mg/d
magnesium sulfate 4 g IV over 1-2 h (if hypomagnesemic)
admit patients with delirium tremens (DT), or multiple seizures.
Reference: Toronto Notes.
11- A male patient presented with depression. What is the first drug to be given in depression?
a.
TCA
b.
lithium
c.
CBT
d.
SSRI
Answer: D
12- What is the best factor for smoking cessation?
a.
The patient desire.
Answer: A

330

13- 40-50 Female presented with depression and she was given amitriptyline 30 mg (TCA). Later she came
complaining of dizziness. What will you do? (Similar Q: A patient with depression on TCA came with dizziness. How
will you manage?)
a.
Decrease dose to 10 mg
b.
Switch to SSRI
Answer: B
14- A depressed patient on medication, but he had weight gain and erectile dysfunction. What is the medication
responsible for these side effects?
a.
Sertraline.
b.
Venlafaxine
c.
TCA.
Answer: A
Sertraline is an antidepressant in a group of drugs called selective serotonin reuptake inhibitors (SSRIs) and most
common side effects is decreased sexual desire or ability, failure to discharge semen (in men) and weight gain.
15- Old man, lives in a nursery home, missed his wife recently. Also he had heart failure and osteoarthritis recently
which makes him more depressed. He is sad and having low mood. He doesnt eat that lead to loss of 4 kgs of his
weight in the last month. Sometimes he forgets. What is the most likely diagnosis?
a.
Alzheimers disease
b.
Depression
Answer: B
16- A patient moved to a new schoolWhat is the most likely diagnosis?
a.
Adjustment syndrome
Answer: A
17- Which of the following is considered good prognostic factor for schizophrenia.
a.
No identified cause
b.
Family history
c.
Insidious onset
d.
Affective symptoms
Answer: D
Good Prognostic Factors:
Acute onset, later age at onset, shorter duration of prodrome, female gender, good cognitive functioning, good
premorbid functioning, no family history, presence of affective symptoms, absence of structural brain abnormalities,
good response to drugs, good support system. Reference: Toronto Notes
18- What is an alternative medication for severe depression?
a.
triptyline
b.
psychotherapy
c.
electroconvulsive therapy
Answer: A (TCA), if you find RCT in the answers then choose it.
Reference: Toronto Notes

331

19- A female patient says her newborn is evil and wont live long. What is your diagnosis?
a.
Postpartum psychosis
Answer: A
Postpartum psychosis (or puerperal psychosis) It is a severe episode of mental illness which begins suddenly in the days
or weeks after having a baby. Symptoms vary and can change rapidly. They can include high mood (mania), depression,
confusion, hallucinations and delusions. Postpartum psychosis is a psychiatric emergency, the patient needs
hospitalization.
20- A woman is afraid that there is a snake in her child room. Last night she checked his room and opened his door
like 50 times and she is not relieved by that?
a.
Psychosis
b.
Obsession
Answer: B
21- Difference between delirium & dementia?
a.
impaired memory
b.
state of consciousness
c.
confusion
Answer: B

Reference: Toronto Notes

332

22- Clozapine is used in which disease in children


a.
Bipolar
b.
Depression
c.
Substance abuse
d.
Psychosis
Answer: D
Reference: http://www.jaacap.com/article/S0890-8567%2809%2961687-7/abstract
23- A patient presented with behavioral changes. His father died recently. He is walking around naked and saying
that his father asked him to do that then returns to his normal state. What is the most likely diagnosis?
a.
Brief psychotic disorder
b.
Schizophrenia
c.
Schizophreniform
d.
Schizoaffective
Answer: A
24- A guy who is heroin addict. You want to start rehabilitation. What drug will you prescribe?
a.
Diazepam
b.
Methadone
Answer: B
Withdrawal: long-acting oral opioids (methadone, buprenorphine), -adrenergic agonists (clonidine)
Treatment of Chronic Abuse:
psychosocial treatment (e.g. Narcotics Anonymous) usually emphasize total abstinence
naltrexone or naloxone (opioid antagonists) may also be used to extinguish drug-seeking behavior
long-term treatment may include withdrawal maintenance treatment with methadone or buprenorphine
Reference: Toronto Notes.
25- Male patient is concerned about his sexual performance. During intercourse he develops palpitations &
diaphoresis. Physical exam: normal. lab: within normal range. What is the diagnosis?
a. Anxiety
Answer: A
26- Patient came to you for check up and tells you that he diagnosed two years ago with pancreatic cancer and
asking you for the medication but he lost the prescriptions, when you have looked in the system there wasn't any
data about him. When you face him with what you find, he runs away. What is the diagnosis?
a.
Malingering.
b.
Somatization.
c.
Schizophrenia
d.
Drug addict.
Answer: A or D?

Reference: First aid USMLE Step 1

333

27- Old patient with Alzheimer's dementia, became agitated and have hallucinations and delusions. What is the
appropriate drug in his case ?
a.
Haloperidol.
Answer: A
Haloperidol: is a typical antipsychotic medication.It is used in the treatment of schizophrenia, tics in Tourette
syndrome, nausea and vomiting, delirium, agitation, acute psychosis, and hallucinations .

28- Which one of the antipsychotic drugs is most likely to cause weight gain ?
a.
Ziprasidone.
b.
Olanzapine.
c.
Quetiapine.
d.
Aripiprazole.
Answer: B however ( B,C,D causes weight gain but more common in 40% is B )
29- Old man is walking on the street and asking random people the same question. When asked to stop he doesnt
stop. What does he have?
a.
Loose of association
b.
Delusion of control
Answer: ?
30- A patient was brought by his family with 3 months history of delusions, hallucination, that are gone by
themselves now. No mood disorders. What is the diagnosis?
a.
Brief Psychotic disorder
b.
Schizophrenia
c.
Schinziform disorder
Answer: C
31- Which of the following is a criteria of Somatization disorder?
a.
intentionally symptoms
b.
at least 2 GI symptoms must be present
c.
more than 40 years old
d.
loss of occupational / life function
Answer: B
8 physical symptoms that have no organic pathology including each of:
four pain symptoms related to at least four di erent sites or functions
two gastrointestinal symptoms, not including pain
one sexual symptom, not including pain
one pseudo-neurological symptom, not including pain (e.g. numbness, paresthesia)
onset before age 30; extends over a period of years.
Reference: Toronto Notes

32 about male pt he cover the TV because he thinks that he got instruction from the TV and the government
Watching him
Mania
Depression
schizophrenia
..cant recall and im not sure about the answer
Answer: Schizophrenia

334

33 - 20s Female patient c/o Multiple GI symptoms such as Nausea, vomiting, diarrhoea, headache, fatigue, joint pain,
urine retention. The all lab result is normal. These symptoms for 2 year unchanged:
A. Somatisation disorder
B. Conversion disorder
C. GAD
D. Dissociative disorder
Answer: A
Somatisation disorder:
The essential feature of a somatization disorder is several symptoms that not lead to any medical sense, with a pattern
of many physical complaints in persons younger than 30 years that occurs over several years and results in unnecessary
medical treatment and/or causes significant impairment in functioning.
All the following historical criteria are required for a diagnosis
Four different pain sites (eg, head, abdomen, back, joints, extremities, chest, rectum) or painful functions (eg,
menstruation, sexual intercourse, urination)

Two gastrointestinal symptoms other than pain (eg, nausea, bloating, vomiting, or intolerance of several
different foods)

One sexual or reproductive symptom other than pain (eg, erectile or ejaculatory dysfunction, irregular menses,
excessive menstrual bleeding)

One pseudoneurological symptom (eg, impaired balance, paralysis, aphonia, urinary retention)
Reference: Medscape http://emedicine.medscape.com/article/918628-overview#a2
34 - The doctor asked the patient t: "do you think you are mentally ill?"
Its evaluating:
A. Insight
B. Judgment
Answer: A
Judgement
1.
2.

Judgment is an assessment of real life problem-solving skills


Testing
1. What should you do if you find a stamped (not canceled) and addressed envelope?

Insight
1.
2.

Insight is an understanding of their condition (including abnormal thoughts)


Testing
1. Tell me about your illness and if it is worse now?
2. What medications are you taking?
Reference: http://www.fpnotebook.com/neuro/Exam/InsghtAndJdgmnt.htm
35 - Which one is known to cause restlessness, insomnia and drowsiness ?
A. SSRI
B. MAOI
C. Tetracycline antidepressants
D. Tricycle antidepressants
Answer: A
The trick of this question is recall all drugs side effects
Which all of this can cause restlessness and drowsiness but SSRI cause insomnia
Reference: http://www.drugwatch.com/ssri/
http://www.medicinenet.com/mao_inhibitors-oral/page2.htm#what_are_the_side_effects_of_MAOIs
http://www.mayoclinic.org/diseases-conditions/depression/in-depth/antidepressants/art-20046983

335

36 - Middle age male presented in the ER with irritability, suspiciousness, overactive, and poor hygiene what is the
diagnosis:
A. Schizophrenia
B. Bipolar Disorder
C. Psychoses
D. Acute anxiety state
Answer: I'm not sure: C, We cannot judge by these symptoms ONLY its Schez or Bipolar

37 - Cardiac patient with depression on regular medications, started to have convulsion then coma , He took
overdose of some pills which his relative does not know it, What is the cause of this seizure?
A.SSRI
B. Digioxin
C. quinine
Answer: I'm not sure, its not SSRI, rare in digoxin

ADDED QUESTION: ( need to be more checked )


1.72 y male disoriented and hallucinating and disorganized thinking had aorto popletial graft and sympto
m fluctuates in the 2 days what the cause ?
A.multi infraction demntia
B.mania
C .demensia
D . delirium
Answer: D
2.Pt think neighbors want to kill him .. He said he can listen to them talking about him .. In Spit they are not
around ?
Answer: Delusion
3.Excessive daytime sleeping?
A.Narcolepsy
Answer : OSA
4.Panic disorders treatment:
Answer : SSri 1st line
Benzodiazepines 2nd line
5.Diazepam used to treat which psychotics disorder in children
Answer: diazepam doesn't treat psychosis !
6.patient see in cars and people in his plate food, What is the dx?
Answer: visual hallucinations ( if there is no food in his plate )
If there's food then it's illusion!
7. A 23 YO patient who sees her nose very big, but all doctors see its normal, already done two plastic surgerys
before for lips and eyelids. What is this condition called in psychiatry?
Answer: body dysmorphic disorder
8.A pt cannot sleep, goes to work exhausted becuz she check gas, door and kids stuff for school several times.
What is this condition called in psychiatry?
Answer:

OCD

9. ADHD case what is the treatment? - - - Answer:stimulants ( methylphenidate)

336

10 - case of poor hygiene in and hallucinating, what medication to give ?


Answer : -antipsychotic
11.Pt loss of interest, sucidal attempt?
A.Major depression
B.Minor depression
Answer : A
12.Mania ?
A. Bipolar
B.affective disorder
C.Schizophrenia
Answer : A
It depend on the symptoms, the question is incompleted. To be more specific its Bipolar type I which is include at least 1
episode of Manic episode (DSM-5 Page 126). Affective disorder is including depression, bipolar disorder, and anxiety
disorder.
13.case about addict person and ask which of the following quiestion is include in criteria of CAGE questionnaire
?.
Answer:
CAGE is a questionnaire indicated in alcoholism
1. Have you ever felt you needed to Cut down on your drinking?
2. Have people Annoyed you by criticizing your drinking?
3. Have you ever felt Guilty about drinking?
4. Have you ever felt you needed a drink first thing in the morning (Eye-opener) to steady your
nerves or to get rid of a hangover?
14.patient in clinic suddenly he said he looks to left and said he saw his mother
and no one can saw her except me his mother died when he was child ?
A.audible hallucination
B.visual hallucination
C.delirium
Answer : B
15.psycatric patient said Tv he talk to me ....
Answer : delusion of reference
16-Best antidepressants drug in adults:
Answer: floxatine ( SSRI )
SSRI is the best initial Rx in adult
http://www.uptodate.com/contents/depression-treatment-options-for-adults-beyond-the-basics

337

17-tttx of dementia?
A.edrophonium
B.revistagmine
C.Neostigmine
Answer: B
Neostigmine is a cholinesterase inhibitor indicated in Symptomatic control of myasthenia gravis; antidote for
nondepolarizing neuromuscular blocking agents after surgery; prevention and treatment of postoperative distention and
urinary retention
edrophonium is indicated in myasthenia gravis and respiratory depression
Rivastigmine is a cholinesterase inhibitor. It works by increasing the amount of a certain substance (acetylcholine) in the
brain, which may help reduce symptoms of dementia in patients with Alzheimer disease.
http://www.rxlist.com/enlon-drug/indications-dosage.htm
19 - An elderly male told you that he's always occupied by the idea that his backyard is invaded by aliens,
although he knows that aliens don't exist and no one is invading his back yard. He's afraid that he's "going
insane". He has:
A. Delusions
B. Obsessions
C. Compulsions
D. Hallucinations
Answer: B
Obsession is the inability of a person to stop thinking about a particular topic or feeling a certain emotion without a high
amount of anxiety
Delusion is a belief held with strong conviction despite superior evidence to the contrary.
http://www.alleydog.com/glossary/definition.php?term=Obsession
20- Son after death of his father express some psycosis for 3 days then return to his normal mental and behavioral
status , dx ?
Answer: brief psychotic disorder
The disorder is characterized by a sudden onset of psychotic symptoms, which may include delusions, hallucinations,
disorganized speech or behavior, or catatonic behavior.
Post Traumatic Stress Disorder is Posttraumatic stress disorder (PTSD) is an anxiety disorder that can develop after a
person is exposed to one or more traumatic events, such as sexual assault, warfare, traffic collisions, terrorism or other
threats on a person's life.[1] Symptoms include disturbing recurring flashbacks, avoidance or numbing of memories of the
event, and hyperarousal, continue for more than a month after the occurrence of a traumatic event.
https://en.wikipedia.org/wiki/Brief_psychotic_disorder
21- about lady scares of some places like streets and malls and she thinks that those places are dangerous
Answer:
Agoraphobia
22- about lady has neurological symptoms, such as numbness, blindness, paralysis,
-Answer:Conversion disorder

338

23-about male pt he cover the TV because he thinks that he got instruction from the TV and the government
Watching him
A.
Mania
B.
Depression
C.
Schizophrenia
Answer: C
Signs & Symptoms of schizophrenia:
Delusions are false beliefs that are not part of the person's culture and do not change. The person believes delusions even
after other people prove that the beliefs are not true or logical. People with schizophrenia can have delusions that seem
bizarre, such as believing that neighbors can control their behavior with magnetic waves. They may also believe that
people on television are directing special messages to them, or that radio stations are broadcasting their thoughts aloud to
others. Sometimes they believe they are someone else, such as a famous historical figure. They may have paranoid
delusions and believe that others are trying to harm them, such as by cheating, harassing, poisoning, spying on, or
plotting against them or the people they care about. These beliefs are called "delusions of persecution."
http://www.nimh.nih.gov/health/topics/schizophrenia/index.shtml

24- yo came with depression what is your assessment :


Answer: sucide
http://www.nimh.nih.gov/health/topics/depression/index.shtml

25- long scenario when you read it you think it about psychiatry but I think it about premenstrual
syndrome What is the treatment
ssri
other psych medication
combined oral contraceptive
psych medication
Answer: SSRI
26- Eroded enamel, enlarged glands, in a girl whats ur dx
** eats alot then self induce vomite
Answer: bulimia Nervousa
https://www.nationaleatingdisorders.org/bulimia-nervosa
27 -Whats the approach to a 40 yo lady with anxiety in work place because of conflicting with colleagues? Educate
about
how to deal with ppl? Empathy?
Answer:
CHICK IF she is psychotic or not first

There were also missing questions about the following:

Malingering
Bereavement
Dysthymia
Techmania
ADHD treatment
Bulimia
Adjustment disorder

339

Orthopedics

340

1- Child with painless limping.


a.
Perthes disease
Answer: ?
Differential diagnosis of painless limping in children:
Developmental dysplasia of the hip
Neuromuscular disease (Cerebral palsy and muscular dystrophy)
Lower limb length discrepancy
Perthes disease: Typically present with painless limp, but may be associated with groin or anterior thigh pain (It
become painful as it progresses)
2- Old lady presented with back pain increased when walking downhill but relieved when walking uphill with
normal neurovascular exam. What is the diagnosis?
a.
Spinal stenosis
b.
Vascular claudication
Answer: A

3- What is the most commonly injured Carpal bone?


a.
Scaphoid.
Answer: A
Important tips to remember:
Scaphoid (palpated in anatomic snuff box) is the most commonly fractured carpal bone and is prone to avascular
necrosis owing to retrograde blood supply.
Dislocation of lunate may cause acute carpal tunnel syndrome.
A fall on an outstretched hand that damages the hook of the hamate can cause ulnar nerve injury.
Reference: FA USMLE step 1
4- Knee Examination with positive lachman test indicates injury of:
a.
Anterior cruciate ligament
Answer: A
The anterior drawer test also can be used to assess ACL
Reference: 3rd Edition UQU > Orthopedics > Q 62.
5- A male patient running 20 km complaining of upper leg pain.
a.
stress fracture
Answer: A

341

6- Surgical neck of the humerus fracture. Which of the following nerves has high risk of
injury?
a.
radial
b.
median
c.
axillary
Answer: C
The surgical neck of the humerus is a constriction below the tubercles of the greater tubercle and lesser tubercle. It is
much more frequently fractured than the anatomical neck of the humerus. A fracture in this area is most likely to cause
damage to the axillary nerve and posterior circumflex humeral artery. Reference: Wikipedia.
7- Mid humerus shaft fracture:
a.
radial nerve injury (loss of wrist extension)
Answer: A
Reference: http://www.orthobullets.com/trauma/1016/humeral-shaft-fractures
Important tips to remember: Fracture of humerus at:
Surgical neck: axillary nerve and posterior humeral circumflex artery.
Mid-shaft: radial nerve (loss of wrist extension) and profunda brachii artery.
Supracondylar region: median Nerve and brachial artery.
Medial epicondyle: ulnar nerve.
8- Athlete presented with severe painful plantar flexion which prevents him from raising the foot. Which ligament
is affected?
a.
Plantar fascia (Plantar fasciitis)
Answer: A
9- septic arthritis organism?
a.
Staph aureus
Answer: A
Staphylococcus aureus is the cause of the vast majority of cases of acute bacterial arthritis in adults and in children
older than 2 years.
Neisseria gonorrhoeae is the most common pathogen (75% of cases) among younger sexually active individuals.
Reference: http://emedicine.medscape.com/article/236299-overview
10- constrictor with repeated hand use:
a.
Lateral epicondylitis
Answer: A
Lateral epicondylitis (tennis elbow): Repetitive extension (backhand shots) or idiopathicpain - near lateral epicondyle.
Medial epicondylitis (golfers elbow): Repetitive flexion (forehand shots) or idiopathic - pain near medial epicondyle.
11- A man arrived to ER after MVA the neck of the femur was fractured, Which of the following may happen in
sequence?
Answer: ?
Injury of sciatic nerve; blood supply to femoral headavascular necrosis (AVN)
Reference: Surgical recall 6th edition

342

12- A child with avascular necrosis of the head of femur. What should be done?
a.
surgical correction
b.
keep immobile for 6 weeks
c.
keep externally rotated
Answer: A? (but it depends on further details in the question)
AVN (Avascular Necrosis) or Osteonecrosis:
Commonest Presentation:
Pain (inguinal area then radiated to the buttocks and thigh).
Investigation:
Radionuclide bone scanning Technetium-99m bone scanning has been used for patients with suspected disease
who have negative radiographs, unilateral symptoms, and no risk factors.
MRI without contrast (Gold standard) is far more sensitive than plain radiographs or bone scanning, with reported
sensitivity of up to 100% Focal lesions are well-demarcated and inhomogeneous on T1-weighted images. The
earliest finding is a single-density line (low-intensity signal) that represents the separation of normal and ischemic
bone.
Management:
Nonoperative management essentially, with analgesics and pharmacological agents,
Restricted patient weight bearing with the use of a cane or crutches has not been shown to affect the natural
history of the disease and is useful only in controlling symptoms, poor outcome (only 15% resolved with
nonoperative intervention).
Joint-preserving procedures In early stage 0 to II lesions, in young active patients, core decompression is the
most conservative surgical procedure that offers the best chance at preserving the femoral head.
Joint replacement
13- What is the type of fracture that has several segments in the diaphysis?
a.
compound
b.
impacted
c.
comminuted
Answer: C

Reference: The pathophysiology of fractures Textbook.

14- A patient with Tibial fracture with abnormal ABI. What


will you do next?
a. MRI
b. CT
c. Angiography
Answer: C
- Tibial fracture with ABI/PVR abnormal, Needs vascular team involvement and urgent vascular imaging (CT
angiography)
Reference: Tibial Plateau Fractures Initial Management Guidelines. See:
http://www.medicine.virginia.edu/clinical/departments/orthopaedics/orthopaediceducation/residency%20programs/treatmentprotocols/Tibial_Plateau_Fractures_2013_Initial_Management_Guidelines.pdf

343

15- Female patient with vertebral compression fracture. Which on of the following will prevent osteoporosis?
a. Vit D supplement
Answer: A
16- Military soldier with flat foot. Which of the following will be sustained?
a.
Flexor retinaculum
b.
Extensor retinaculum
c.
Spring ligament
d.
Achilles tendon
Answer: C
Medial arch (navicular) injury: Injuries to the navicular bone and related structures jeopardize the structural integrity
of the medial foot arch and thus can lead to severe disability and chronic pain. Therefore, navicular injury is considered
high risk, particularly for runners. Such injuries include navicular stress fractures, tendinopathy of the posterior tibialis
tendon insertion onto the navicular, traumatic separation of an accessory navicular, and partial or complete tears of the
attachment of the plantar calcaneonavicular (spring) ligament. Unless the clinician has experience managing injuries of
the medial arch, consultation with a foot and ankle surgeon should be obtained for navicular injuries, including fractures
or separation of an accessory navicular, as well as for complete or partial ruptures of the posterior tibialis tendon or the
spring ligament, as such injuries often compromise the integrity of the longitudinal arch.
Reference: http://www.uptodate.com/contents/evaluation-and-diagnosis-of-common-causes-of-foot-pain-inadults#H13378296
17- Adult with mid-clavicular fracture. X-ray showed overlapped bone fragments. How will you treat?
a. internal fixation
b. external fixation
c. splint
d. figure 8
Answer: A
Nondisplaced midshaft clavicle fractures: arm sling or figure-of-eight dressing (arm sling is better)
Displaced mid-shaft fractures: Although nonoperative treatment is a viable option to treat displaced mid-shaft
fractures, operative repair should be considered in patients with multiple risk factors for nonunion, especially
significant fracture displacement or clavicle shortening. Reference: http://www.aafp.org/afp/2008/0101/p65.html
18- A patient with bone pain and pathological fracture. X Ray: lytic lesion. Labs: high ALP. What is the most
likely diagnosis?
a. Paget's disease
Answer: A
Pagets disease: Common, localized disorder of bone remodeling caused by increase in both osteoblastic and
osteoclastic activity. Usually asymptomatic, but may present with aching bone or joint pain, headaches, skull deformities,
fractures, or nerve entrapment (leads to loss of hearing in 3040% of cases). The investigations show increase in serum
alkaline phosphatase and lytic bone lesion in x-ray with normal calcium PTH and phosphate levels.
Reference: FA USMLE step1 and step2 CK
19- Patient came to the clinic with severe plantar flexion and heard a snap (No Achilles tendon in the answers).
Answer: ?
The usual mechanism of lateral ligament injury is inversion and plantarflexion. This may be accompanied by an audible
snap, crack or tear. Depending on the severity of the injury, the athlete may have been able to continue activity
immediately or have been forced to rest. Swelling usually appears soon after the injury, although occasionally it may be
delayed some hours.

344

20- Long scenario. What is the ligament that prevent overextension of hip?
Answer: ?
Iliofemoral ligament: Y-shaped. It prevents hyperextension of the hip joint during standing by screwing the femoral head
into the acetabulum.
21- A patient presented with Trendelenburg gait. Which muscle is responsible?
Answer: ?
Trendelenburg gait caused by weakness of the abductor muscles of the lower limb, gluteus medius and gluteus minimus.
http://www.uptodate.com/contents/overview-of-hip-pain-in-childhood
22- Old lady complaining of right femur pain when she goes upstairs and downstairs. Examination of the right
hip: flexion, extension and rotation are normal but abduction caused pain. What is the best investigation to do ?
a.
Radiological assessment.
b.
Blood testing.
c.
Rheumatoid factor.
Answer: A
(Pain while abduction of rt hip) Most probably to be posterior hip dislocation because all movements are intact (flexion
internal rotation...) except abduction so we need radiological views to confirm the diagnosis
Reference: Netters concise orthopedic anatomy
23- Football player he can't raise his big toe. what muscle affected:
a.
Deep muscle under foot
b.
Superficial muscle under foot
c.
Tibia
Answer: A
(Pain while raising up the big toe) Deep muscles under foot responsible for dorsiflexion of big toe most probably the
diagnosis will be turf toe, which is common in football players (it sprains to the ligaments around the big toe)
Reference: Millers review of orthopedic
24- Function of anterior forearm muscle:
Answer: ?
- Superficial group: 5 muscles; flexor carpi radialis, Flexor carpi ulnaris, flexor digitorum superficialis, palmaris longus
and pronator teres.
- Deep group: 3 muscles; flexor digitorum profundus, flexor pollicis longus and pronator quadratus.
- All responsible for flexion and pronation
Reference: Zuckerman handbook of fractures
25- Type of bone hunger.
Answer: ?
There are no types!
Hungry bone refer to severe and prolonged hypocalcemia which occurs after parathyroidectomy or less commonly
thyroidectomy despite normal or even elevated level of parathyroid hormone ( PTH) . The fall in serum calcium is
primarily due to functional or relative hypoparathyroidism leading to increase calcium influx of Ca+ into bone in patient
without end-stage renal disease .Reference: Uptodate and toronto notes
26- Child coming with fracture of the forearm (x-ray picture show open fracture of radius & ulna), what is the
treatment?
a.
Closed reduction
b.
Open reduction
c.
Surgical intervention
Answer: ?
O.R.I.F

345

27- Injury to the surgical neck of humerus cause injury to :


a. Radial A
b. Median
c. Ulnar
d. Brachial plexus
Answer: D
- This type is proximal humerus injury
- A, B and C go with distal and humeral shaft injury. Assessment: Perform a neurological examination, particularly
examining the axillary nerve Assess and brachial plexus injury through distal neurological examination.
28- A patient was diagnosed with scoliosis, based on the cobb angle, when to do an orthopedic referral?
a. > 5 degrees
b. > 10 degrees
c. > 15 degrees
d. > 20 degrees
Answer: D
Reference: Toronto notes.
29- (Long scenario) Patient with low back pain, loss of sensation, weakness and loss of reflexes, what will do
next?
Answer: ?
Urgent MRI
The most common indication for the use of imaging procedures (MRI or CT) is the clinical setting of Low back pain
complicated by radiating pain (radiculopathy, sciatica), as well as in cauda equina syndrome (bilateral leg weakness,
urinary retention, saddle anesthesia), neurogenic claudication and/or spinal stenosis. MRI of the lumbar spine has
become the initial imaging modality of choice in complicated LBP, displacing myelography and CT in recent years.
See: http://www.guideline.gov/content.aspx?id=35145
30 - child sitting always in w position what the change in bone:
a.metatarsus adductusb.
b. femoral anteversion (femoral torsion)
Answer:b

femoral anteversion (femoral torsion)


In Toeing : metatarsus Adducts, internal tibial torsion, femoral antiversion
Out toeing: temoral retroversion
For more info : http://www.wheelessonline.com/ortho/femoral_anteversion
Reference: VMA course

346

31- girl with Right knee swelling Aspirations results show WBCs and what best management:
Discussion:
- if the WBC 15,000 to 200,000 the diagnosis is (Septic Arthritis)
- Empiric intravenous antibiotic treatment of septic arthritis should be based on the organism found in the Gram stain of
the synovial fluid, or on the suspicion of a pathogen from the patient's clinical presentation.

References:
all about the knee : http://www.aafp.org/afp/2000/0415/p2391.html#abstract
management of septic arthritis according the gram stain
http://www.aafp.org/afp/2011/0915/p653.html#sec-3
32 - Boutonnire deformity (BD) ?
can manifest itself acutely after trauma, but most BDs are found weeks following the injury or as the result of
progressivearthritis. The proximal interphalangeal (PIP) joint of the finger is flexed, and the distal interphalangeal (DIP)
joint is hyperextended.
Reference: Medscape

347

ADDED QUESTION: ( need to be more checked )


33 - Blood supply of post. Leg compartment:
a. tibial
b. :Common fibula
c. :Superfacial fibual
d. :something
Answer: posterior tibial artery
Explanation:The posterior tibial artery is the larger of the terminal branches of the
popliteal artery. It descends deep to soleus, where it can be exposed by splitting
gastrocnemius and soleus in the midline, then becomes superficial in
the lower third of the leg and passes behind the medial malleolus between
the tendons of flexor digitorum longus and flexor hallucis longus.
posterior compartment has two parts ( superficial and deep )
Superficial contains : Gastrocnemius , sleus, plantaris
Deep contains : popliteus, flexor hallucis longus, flexor digitorum longus and tibialis posterior
Reference: Review of orthopedics by Miller + Gray's anatomy

34- In case of Mid humerus fracture, Which nerve is affected?


A- radial nerve.
B- axillary nerve.
C- median nerve.
D- ulnar nerve.
Answer is A
35 - a fracture of the distal one-third of the humeral shaft commonly associated with neuropraxia of the radial
nerve (22% incidence).
Reference: http://www.orthobullets.com/trauma/1016/humeral-shaft-fractures
36- Tx of perthes d (avascular necrosis)
a. surgery
b. without any treatment
Answer:
Until recently most children with Perthes disease were treated with a plaster cast or brace, or surgery. However, it is now
known that at least half of cases heal well without any treatment, particularly children aged five and under, and milder
cases
The aim of treatment is to promote the healing process and to ensure that the femoral head remains well seated in the hip
socket as it heals and remodels. Treatments advised can depend on the age of the child and the severity of the condition.
Treatments may include observation, bed rest and crutches, a plaster cast or special leg brace, or surgery\
Reference
http://www.perthes.org.uk/what-is-perthes-disease/
37 - "case of osteoarthritis. Dx?
Answer:
Symptoms of OA: joint pain or tenderness, Stiffness, loss of flexibility, Grating Sensation, & bone Spurs
Tests & Diagnosis: X-ray will show narrowing of the space between the bones in the joint
Reference: http://www.mayoclinic.org/diseases-conditions/osteoarthritis/basics/tests-diagnosis/con-20014749
http://emedicine.medscape.com/article/330487-differential
38- osgood:schlatter disease clear history very easy?
Answer:
http://orthoinfo.aaos.org/topic.cfm?topic=a00411

348

39- girl with high heel break the heel of RT foot with inversion of RT foot what the tendon injured?
Answer:
calcaneofibular ligament
MOST PROPABLY
40- ptn wear high heel ..then her Rt heel is broking ..which structure is injured?
Answer:
Might be Achillus tendon
41 - picture of tibial fracture proximal peripheral perfusion normal and normal sensation the best next to
investigate?
angiography
MRI
doppler us
42- patient with septic arthritis the labs showing resistance to the antibiotic what u will do ?
stop anti biotic
start vancomycin
add vancomycin
gentamicin
THIS QYESTION IS NOT COMPLATE, IF THERE IS SURGERY OPTIION IT WIL BE THE ANSWER. IF NOT WE
SHOULD KNOW THE CULTURE AND THE RESULTS OF INVESTIGATION READ :
http://emedicine.medscape.com/article/236299-medication#2
43- Type of bone hunge :
Answer:

44- Elbow type of bone :


http://www.teachpe.com/anatomy/joints.php
Scoliosis orthopedic referal degree: 5 or 10 or 15 or 20
Answer:

Management of congenital scoliosis consists of serial radiographs to determine whether the deformity
is worsening. If worsening of 5-10 degrees or more is documented, surgical fusion is necessary
regardless of child's age.
Controversy surrounds indications for bracing. Trends over the past 20 years have moved towards
bracing only the more significant curves (20-50 degrees).
Surgery, rod placement, and bone grafting may be necessary to achieve partial or complete correction
in adolescents with curves >45 degrees.

http://www.medscape.com/viewarticle/449233_5

349

45- Periorbital lesion: what to do


Answer:
This is not ortho.
46- Surgical neck fx of humerus what nerve
Answer:
Axillary nerve
http://www.orthobullets.com/trauma/1015/proximal-humerus-fractures

47- Mid shaft humerus fx? Wrist drop .. RADIAL NERVE

48- patient has left leg trauma, you did for him plaster of paris cast , after 5 hours come to ER complain of leg
pain. Which of following support your diagnosis :
A- Decrease pulse in L foot
B- Decrease sensation in L foot??
C- Pain in L foot
Answer: C, " pain out of proportion to clinical situation is usually first and the most sensitive symptom of compartment
syndrome "
http://www.orthobullets.com/trauma/1001/leg-compartment-syndrome
49- Women worn high shoes And then broken her feet evertd laterally which ligment injerd
A- deltoid ligment
..
Answer: A, " deltuid legament injury occurs with pronation (eversion) trauma leading to forced external rotation and
abduction of ankle "
http://www.orthobullets.com/foot-and-ankle/7005/ankle-ligaments
50- Osteoporosis t score > :2.5 osteoprosis
Answer : > -2.5
Reference: 3rd edition UQU last touch Q 61

350

51 - High density bone in dexa With scheduled total hip replacement


a. 1 osteoporosis
b. 2 osteomalacia
c. 3 osteoarthritis
d. 4 paget disease
Answer: c. 3 osteoarthritis
Bone density is actually HIGHER rather than LOWER in osteoarthritis. Low bone density is the telltale sign of
osteoporosis, a skeletal disorder characterized by weakened bones due to excessive loss of bone mass.
Osteoarthritis, on the other hand, is characterized by increased bone density and bony growths (osteophytes) in
conjunction with articular cartilage degeneration
Reference:
http://www.orthop.washington.edu/?q=patient-care/articles/arthritis/osteoarthritis.html
52 - What is the most specific sign for osteoporosis ?
bone density
subchondral cyst
compression fracture **
Answer: compression fracture **
Fractures are the most dangerous aspect of osteoporosis. Debilitating acute and chronic pain in the elderly is often
attributed to fractures from osteoporosis and can lead to further disability and early mortality. [14] These fractures may also
be asymptomatic. The most common osteoporotic fractures are of the wrist, spine, shoulder and hip. The symptoms of a
vertebral collapse ("compression fracture") are sudden back pain,
Reference: https://en.wikipedia.org/wiki/Osteoporosis
53 - Q about old female with recurrent fracture ..
Estrogen Def
cant recall others
Answer: Estrogen improves calcium absorption and reduces the amount of calcium lost in urine
so deficiency of estrogen will lead to osteoprosis
an osteoporosis-related fracture are at high risk of recurrent fractures.
54 pt had posterior
hip dislocatio?
a.internal rotation ..adduction
B.external rotation ..abduction.
C...
D...
Answer: a
http://slideplayer.com/slide/4311100/

351

55- pt with long scenario had trauma to knee..the knee displaced to anterior ..which structure is injured?
A.anterior cruciate ligaments
B.posteroir ========
C.medial meniscus
D.latral =========
Answer : a
Pivot shift test
The pivot shift test is performed by extending an ACL-deficient knee, which results in a small amount of anterior
translation of the tibia in relation to the femur. During flexion, the translation reduces, resulting in the "shifting or
pivoting" of the tibia into its proper alignment on the femur. It is performed with the leg extended and the foot in internal
rotation, and a valgus stress is applied to the tibia.
Ref :http://emedicine.medscape.com/article/89442-overview
56 - child 9 years with metaphysial femur or tibial a vascular necrosis ..what u want to do ?
A.surgery.
B.avoid weight bearing for 6 months or weeks
.c..
D..
Answer: surgery
Ref: http://www.ncbi.nlm.nih.gov/pubmed/3047257
57 - case of posterior dislocation of the shoulder
http://radiopaedia.org/articles/posterior-shoulder-dislocation
58 - ttt of medial epicondylitis
Answer: conservative physiotherapy
Ref : miller

352

Also , ADDED QUESTION: ( need to be more checked )


1.What type of radial nerve injury?
1-nuropraxia
2-nurotemesis
3-axontemesis
Answer: Nuropraxia
i think depend on scenario , if Q mean closed fracture most likely the answer will be nuropraxia
Ref: http://www.orthobullets.com/trauma/1015/proximal-humerus-fractures
to know what is daffiness btw these type look the link :https://jammyjasmine.wordpress.com/2015/04/06/dontpanic-a-laypersons-guide-to-nerves-and-nerve-damage/
Radial nerve injury association with humeral fracuter :
proximal humoral fracuter :
Nerve injury
axillary nerve injury (up to 58%)
increased risk with anterolateral acromial approach
axillary nerve is found 7cm distal to the tip of the acromion
suprascapular nerve (up to 48%)
humoral shaft
Radial nerve palsy
incidence
seen in 8-15% of closed fractures
increased incidence distal one-third fractures
neuropraxia most common injury in closed fractures and neurotomesis in open fractures
Holstein-Lewis fracture
a spiral fracture of the distal one-third of the humeral shaft commonly associated with neuropraxia of the radial nerve
(22% incidence)
distal humoral # :
Nerves
ulnar nerve
resides in cubital tunnel in a subcutaneous position below the medial condyle
radial nerve
resides in spiral groove 15cm proximal to distal humeral articular surface
between brachioradialis and brachialis proximal to elbow
divides into PIN and superficial radial nerve at level of radial head
2.Pt dose not complain of anything ,, has sudden knee swelling ? What is the best thing to do ?
Answer:
may be knee x ray to exclude any injury if case related to sport injury , rule out broken or dislocated bones

Reference: Rapid Swelling Without An Injury


Occasionally, a swollen knee develops rapidly without any injury. The most common causes of this are:
Fluid removed during aspiration of a swollen knee due to gout
1)
Infection: Infections increase in the amount of fluid produced in the joint resulting in a swollen knee. They
usually develop after surgery or a deep cut, but sometimes an infection in your body can spread to your joint. It is very
difficult for your body to fight an infection within a joint and sometimes surgery is required
2)
Gout: High levels of uric acid (produced as part of the digestive process) cause sharp, needle like crystals to
form in your joints leading to inflammation and water on the knee. It is usually treated with medication and proper
diet. Find out more in the Gout Knee section
- See more at: http://www.knee-pain-explained.com/swollen-knee.html#sthash.NlwrJagJ.dpuf

353

swollen knee that develops immediately after an injury, within minutes, is usually due to haemarthrosis, where blood
accumulates in the joint. Essentially what happens is that a structure inside the knee gets damaged and starts to bleed,
building up pressure in the joint. The swelling is normally profuse and the knee balloons up. It will feel tense and very
sore and is often accompanied by bruising, although that may take longer to develop. There are three main causes of a
swollen knee from a haemarthrosis:
Ligament tears are a common cause of knee swelling.
1)
Ligament Tear: Where a ligament ruptures (tears completely). This is the most common cause and usually
involves the ACL (Anterior Cruciate Ligament)
2) Meniscus Tear: A tear in the outer rim of the cartilage lining the knee
3)

Bone Fracture: A break in one of the knee bones

A swollen knee like this needs urgent medical attention. Visit the Knee Injuries section to find out more about these
common causes of knee swelling, including symptoms and treatment options.
- See more at: http://www.knee-pain-explained.com/swollen-knee.html#sthash.NlwrJagJ.dpuf
3.Fraction of head of tibia with drop feet which nerve?
A - Deep peroneal
B - Common peroneal
Answer: deep?
4.Patient complaint progressive wrist pain since 2 months and increased in the passed 1 month , give history of
cesarean delivery on exam there numbness and normal range of motion what is the Rx?
a.Thumb cast
b.Whole are cast
c.Surgical decompression
answer: c
it is pregnancy induced carpal tunnel syndrome
5. twisted ankle, What is the most common affected ligament ?
Answer :
anterior talofibular ligament,and calcaneal fibular ligament
If both in choices , choose talofibular
Reference :
https://www.aofas.org/footcaremd/conditions/ailments-of-the-ankle/Pages/Ankle-Sprain-.aspx
6. which nerve is affected in case of fibular fracture? repeated
A.common peroneal n
B. deep peroneal
C. superficial peroneal
Answer: common peroneal nerve
Reference: https://www.nlm.nih.gov/medlineplus/ency/article/000791.htm

354

7.trendelenburg gait features:


A. internal rotation with adduction
B. internal rotation with ab
C. external rotation with abduction
Answer: A
Since the trendelenburg gait develops as a result of weakness in Abductor muscle, the affected side tend to be in adduced
position with internal rotation, based on the pictures below

8. Mechanism of osgood schlatter disease


iliotibial band osgoodschlatter disease or syndrome (tibial tubercle apophyseal traction injury and epiphysitis of the
tibular tubercle) is an irritation of the patellar ligament at the tibial tuberosity. it is characterized by painful lumps just
below the knee and is most often seen in young adolescents. risk factors include excess weight and overzealous
conditioning (running and jumping). diagnosis is made clinically treatment is conservative with rice (rest, ice,
compression, and elevation), and if Required acetaminophen.

similar Q: Q 4 in uqu 4th edition


''boy after running for hours, has pain in knee and mass on upper surface of tibia, Dx? Osgood schlatter disease''
for more reading http://www.orthobullets.com/sports/3029/osgood-schlatters-disease-tibial-tubercle-apophysitis
9.trauma to the fibular head what is the nerve injured?
a-Common peroneal nerve
Answer:a
''Peroneal nerve injury - Often the result of direct contusion in proximal fibula Fx and has a variable prognosis''
Reference: http://www.orthopaedicsone.com/display/Main/Proximal+fibula+fractures

355

10.Osteoporosis, how to prevent spinal compression fx ?


a.Daily vit D
b.Arobic exercise
c. Weight bearing
d. Walking
Answer: d?
''Physical therapy is recommended for gait and core strengthening when the patient can tolerate this level of activity''
Reference: http://www.uptodate.com/contents/osteoporotic-thoracolumbar-vertebral-compression-fractures-clinicalmanifestations-andtreatment?source=search_result&search=osteoporotic+vertebral+compression+fracture&selectedTitle=1~150#H1565673
11.Best exercise for osteoporosis?
a.Low resistance exercise and conditioning
b.Low resistance and highly repetitive weight bearing
Answer: B
Increase weight bearing and muscle strength exercise
Reference: Medscape
http://emedicine.medscape.com/article/330598-overview#showall
12.Old pt with spinal stenosis, management?
Physical therapy?
Answer: oral medications, physical therapy, and corticosteroid injections
Reference : Orthobullet
http://www.orthobullets.com/spine/2037/lumbar-spinal-stenosis
13.Muscle that extends the knee?
Answer:
The main muscle for extension is the quadriceps femoris, which is the most important muscle in stabilizing the knee
joint. The quadriceps is made up of the vastus medialis and lateralis, rectus femoris, and vastus intermedius. The tensor
fasciae latae is a weak extensor.
Reference: http://emedicine.medscape.com/article/1898986-overview#a2
14.Muscle respondible of unLUCKING THE KNEE:
Answer: popliteus muscle
Reference:
https://books.google.com.sa/books?id=0h7FTCb02eUC&pg=PA412&lpg=PA412&dq=popliteus+muscle+unlock&
source=bl&ots=M0SwF6rNME&sig=-g9JvcwfhSLnDy9vJMRC8micow&hl=ar&sa=X&ved=0ahUKEwj08_2Py8zJAhUD_HIKHYq2DEI4KBDoAQgcMAE#
v=onepage&q=popliteus%20muscle%20unlock&f=false

356

Added Qs 7th update


1.patient had posterior hip dislocatio? /#surgery
A.internal rotation ..adduction
B.external rotation ..abduction.
Answer :Flexion and abduction, between 90 and 100
reference: http://www.hindawi.com/journals/aorth/2015/103580/
2- child 9 years with metaphysial femur or tibial a vas /#surgery
cular necrosis ..what u want to do ?
A.surgery.
B.avoid weight bearing for 6 months or weeks
answer : b
Asymptomatic lesions of the femur that involve less than 15 percent of the femoral head may resolve without surgical
intervention and may, therefore, be treated nonoperatively.
Asymptomatic lesions that involve more than 30 percent of the femoral head are likely to progress to collapse despite
surgical intervention; thus, these patients may also be nonoperatively managed at the
reference ; uptodate
http://www.uptodate.com/contents/osteonecrosis-avascular-necrosis-ofbone?source=outline_link&view=text&anchor=H18899908#H47
3-young man close the door on his nail color become blue
under nail what will u do ?
A- evacuate hematoma
B- remove nail
C-reassure it well resolved by it's self
Answer: C
by general surgery consultant
4- man make RTA he was coicious , oriented , alert , but his extremity ,type of shock ?
A-irreversible
B- neuro
C-cardio
D- hemorrhagic
Answer: Question is not clear , but if it's meaning cold extremity most likely Hemorrhagic
Reference : Toronto note
5-girl with high heel break the heel of RT foot with inversion of RT foot what the tendon4 injured?
a-calculotibialis
b-anterior tibialis
answer: Talofibular
Reference: http://www.iaaf.org/download/download?filename=5b2291a7-258c-4635-bbc08657eec73740.pdf&urlslug=Chapter%2010%3A%20Specific%20injuries%20by%20anatomic%20site
6- fracture to the humerus shaft "radial nerve injury" what type of nerve injury?
a-Neurotomesis
b-Axonetmesis
c-Neuropraxia
answer is: C
Reference: http://www.orthobullets.com/trauma/1016/humeral-shaft-fractures

357

7- young adult playing golf's came with Hx of atrophy of thener muscle and tenderness at medial epicondyl first
evalute :
a-x-ray
b-CT scan
c-reassure
Answer:
patient with posterior hip dislocation,, what is the position of the leg
adducted, internal rotation and shorter <<< I think
8-Fracture in mid shaft of humerus. What will cause ?
A. Waiter's Tip Hand
B. Wrist Drop * [NOT SURE 100%]
Answer : B
9- Physician in the clinic tell the child to bend forward and hang his both hands freely. This test is used in
detection of ?
A. Rectal Prolapse
B. Sexual Abuse
C. Scoliosis * [NOT SURE 100%]
Answer :c
Reference: http://www.physio-pedia.com/Adam's_forward_bend_test
10-trauma to the fibular head what is the nerve injured?
A-Common peroneal nerve
The answer is: A
Reference: https://www.nlm.nih.gov/medlineplus/ency/article/000791.htm
11-Chronich Gout ?
answer:
allopuriol

Reference:toronto note

358

12-Pyseodogout ?
answer:
pyrophysphate crystel

Reference :toronto note


13-Construction worker presented with pain in the lateral epicondyle of the elbow
Answer : Lateral Epicondylitis =temis elbow
patients history will often describe repetitive activities causing symptoms, yet onset of lateral epicondylitis commonly
occurs with no injury but more a gradual onset of symptoms. Patients complaints typically include, pain at the location
of the lateral epicondyle, sharp type pain that is exacerbated with gripping activities, pain in the morning and at times
when the wrist is held in flexion. Also, patients typically complain of an inability to grip objects.
On clinical examination, signs of lateral epicondylitis include the following: point tenderness and pain over origin of
common extensor tendon, and maximal tenderness anteriorly and just distal at the origin of the ECRB and ECD muscles.
Additional findings are reduced strength with resisted grip, supination, and extension of the wrist.
Reference:http://morphopedics.wikidot.com/lateral-epicondylitis
14. Signs of osteoporosis on X-ray?
Answer :Reference: http://emedicine.medscape.com/article/330598-workup#c9.
15- One Small thyroid nodule , we do investigation and its Increase in iodine uptake , what is the best treatment
?
A.
Conservative
B.
Antithyroid drug
C.
Iodine radiotherapy
Answer: B
Reference: http://emedicine.medscape.com/article/924550-overview#a1

359

16- minutes of Xray radiation?


- 25
-15
-5
-0
answer: Q is not clear but the following may help ,
AIRPORT PASSENGER SCREENING There has been increasing concern regarding radiation exposure from wholebody transmission or backscatter radiograph scanners at airport security checkpoints that have been deployed in some
airports across the United States. Reports calculate the dose emitted from backscatter radiograph scanners as being
equivalent to approximately three to nine minutes of radiation received from the environment through normal daily living
[79,80]. Put another way, among 100 million annual passengers, only six cancers may be attributed to airport radiograph
screening over the lifetime of these individuals. Current evidence thus suggests that there is negligible individual risk
from airport passenger screening [81]. Millimeter-wave scanning is a similar technology without the use of ionizing
radiation which may become a comparable technology and preferable alternative to radiograph airport passenger
screeners
Reference: Uptodate
17- carpal tunnel syndrome vs. thoracic outlet obstruction??
-Carpal tunnel syndrome:
Entrapment of the median nerve at wrist caused by decrease space of the carpal tunnel leading to parasthesia, pain,
paralysis.
Causes: overuse of wrist flexors, associated with DM , thyroid dysfunction ,pregnant , middle aged women.
Ref. Step 2 CK page 221
-Thoracic outlet obstruction :
Thoracic outlet syndrome (TOS) refers to a constellation of signs and symptoms arising from compression of the upper
extremity neurovascular bundle by various structures in the area just above the first rib and behind the clavicle, typically
within the confined space of the thoracic outlet. The term thoracic outlet syndrome is not a specific diagnosis and the
appropriate type of TOS, such as neurogenic (nTOS), arterial (aTOS), or venous (vTOS) thoracic outlet syndrome should
be used. (See 'Introduction' above.)
Thoracic outlet syndromes are due to rib anomalies, muscular anomalies, or a result of injury. Cervical ribs predispose
the patient to TOS after hyperextension-flexion (whiplash) injury. The absence of a rib anomaly makes the diagnosis of
arterial thoracic outlet syndrome less likely. Many patients with nTOS have a prior history of neck trauma or repetitive
occupational physical stress. Similarly, vTOS is highly associated with repetitive movements, particularly with repetitive
overhead upper extremity movements. (See 'Pathogenesis' above.)
Reference: uptodate
http://www.uptodate.com/contents/overview-of-thoracic-outletsyndromes?source=search_result&search=thoracic+outlet+syndrome&selectedTitle=1%7E36
18- case of tibial collateral ligament injury?
answer:.
Reference: Uptodate
http://www.uptodate.com/contents/lateral-collateral-ligament-injury-and-related-posterolateral-corner-injuries-of-theknee?source=search_result&search=tibial+collateral+ligament+injury&selectedTitle=4%7E150
19- Scoliosis what radiological method will you order:
a-Xray
b-CT scan
c-MRI
answer :A
"PA spine radiographs Radiographs are required to confirm the diagnosis of scoliosis, evaluate the etiology
(congenital, neuromuscular, idiopathic), determine the curve pattern and measure the magnitude (Cobb angle), and to
evaluate skeletal maturity (to determine the risk for progression).
MRI of the spine Magnetic resonance imaging (MRI) of the spine may be indicated in patients with scoliosis and
clinical or plain radiographic findings suggestive of intraspinal pathology (tumor, dysraphism, infection)"
Reference.uptodate http://www.uptodate.com/contents/adolescent-idiopathic-scoliosis-clinical-features-evaluation-anddiagnosis?source=machineLearning&search=scoliosis&selectedTitle=1~150&sectionRank=2&anchor=H15#H15

360

20-Patient presented with knee swelling and pain they did x ray and aspiration and found negative birefringence
needlelike what are you going to discharge the patient with?
a-Allopurinol
answer :A

21- Patient presented with distal thigh pain erythema and warmth seems inflamed but the knee is completely
normal theres limited range of motion with tenderness which of the following would be helpful in establishing the
diagnosis:
A)C-reactive protein (or ESR)
B)X ray
C)Blood culture
answer : B this is a case most probably osteoarthritis
Reference: http://www.uptodate.com/contents/osteoarthritis-symptoms-and-diagnosis-beyond-the-basics#H11

22- Ankle joint ? ( another same Q :


wich part of body bone is forming the ankel )
A)Distal tibia fabula talaus
B)Latral malloli medial malloli talaus
C)Calclunus tibia fibila
Answer: B

361

23- wich part of bone is firstly affected in hematogenous osteomilitis .


A-epiphysis.
B-metaphysis.
C-diaphysis.
answer is: B
Reference: http://www.uptodate.com/contents/overview-of-osteomyelitis-inadults?source=outline_link&view=text&anchor=H1#H1
24- Pt. Presented with deformity in the hand, xray showed fracture in the radius, they put picture of xray and the
hand. How you will fix it ?
A)Close reduction ... below elbow
B)Closed reduction. ... above elbow
C)Open reduction ... below elbow
D)Open reduction above elbow
answer: B or D
The Q is not clear as its saying fracture in the radius but didnt specify is it radial head, shaft or distal.
For the distal radius fracture the treatment depend :
- if extra-articular and/or <5mm shortening we will do closed reduction & above elbow cast
- if its intra-articular we can do either closed reduction & external fixation with above elbow back slab or ORIF with
above elbow back slab

Reference: http://www.orthobullets.com/trauma/1027/distal-radius-fractures

362

25- Retire farmer recently he develop pain in the left arm, which with time progress until he can't sleep on his left
side, by examination found to have sever decrease in motion, what he has ?
A)osteoporosis of the bone
B)spondylitis (arthritis that affects the spine only)
answer: both answers are wrong, the case scenario goes more with Bursitis as it mostly affect the shoulder joint, old age
and occupation ( any repetitive motion or pressure on particular bursa
Reference: http://www.mayoclinic.org/diseases-conditions/bursitis/basics/risk-factors/con-20015102
26- Male pt complaine of weakness in flixtion of both Rt knee and Rt hip which muscle affected ?
A)Sartoruis
answer:
http://www.orthobullets.com/anatomy/10055/sartorius
27- women complaining of left hand tingling mainly at thump and index on exam there was mild atrophy of
thinner muscle Tenil's test was positve which nerve may be affected??
A/ radial nerve
B/ median nerve
C/ musclcutenous nerve
answer is: B
Reference: http://www.medicinenet.com/script/main/mobileart.asp?articlekey=16687
28- boy was playing football with barefoot and was injured in his sole , and Dr found that it is only superficial
planter injury which structure may be affected:
1/ posterior tibialis tendon
2/ adductor halicous longus
3/ can't remember other choice
answer: abductor halicous longus , NO adductor halicous longus
answered by consultant .
29- Football player came with + anterior drawer test, where is the injury:
A- ACL
B- PCL
C- Medial collateral
D- Lateral collateral
answer is: A
Reference: http://physicaltherapyweb.com/anterior-drawer-test-orthopedic-examination-knee/
30- picture was provided for 4 ys old boy presented with distal ulnar and redial bone fracture the skin was slightly
injured by the bone what is the management1??
a- closed reduction with cast below elbow
b- closed reduction with cast above the elbow
c- open reduction with plaid
d- Aggressive debridement and irrigation and Fracture stabilization
answer:D
Reference:http://www.orthobullets.com/trauma/1004/open-fractures-management
31- Football player came with + anterior drawer test, where is the injury:
A- ACL
B- PCL
C- Medial collateral
D- Lateral collateral
Answer: A
32- Case of young boy with falling in outstretched hand MRI image attached
A.
shoulder dislocation http://www.orthobullets.com/sports/3051/posterior-instability-and-posterior-dislocation
B.
rotator tear http://www.orthobullets.com/sports/3043/rotator-cuff-tears

363

33-Active osteoarthritis in knee pain best excercise:


A.
Quadriceps Ms strengthen.
*other options also cant remember it*
best answer:"Quadriceps and Hamstring Ms lengthen+strengthen"
34- to Prevent fracture caused by osteoporosis in old:
B.
Vit D
C.
*other medication*
answer:?
"For older adults with inadequate dietary calcium and vitamin D intake, we suggest calcium and vitamin D
supplementation (Grade 2B). (See 'Calcium and vitamin D' above and "Calcium and vitamin D supplementation in
osteoporosis".)

For the majority of adults, we suggest not using pharmacological therapy for prevention of bone loss (Grade 2B). (See
'Candidates for pharmacologic therapy' above.)
Since pharmacological therapy has significant costs and associated risks, only patients with the highest risk of fracture
are candidates for preventive drugs. Thus, patients with low bone mass (T-score between -1.0 and -2.5) should be
considered for pharmacological intervention based upon fracture risk, as determined by a combination of BMD and
clinical risk factors. Fracture risk can be calculated using the World Health Organization (WHO) Fracture Risk
Assessment Tool (FRAX). A reasonable cut point that may be cost-effective in some settings is a 10-year probability of
hip fracture or major osteoporotic fracture of 3.0 or 20 percent, respectively.
Reference:http://www.uptodate.com/contents/prevention-ofosteoporosis?source=machineLearning&search=prevention+of+osteoporosis&selectedTitle=1~150&sectionRank=1&anc
hor=H28#H28

35-wich part of bone is firstly affected in hematogenous osteomilitis .


* epiphse
* metaphase
* diaphase
Answer: C
up to date
http://www.uptodate.com/contents/hematogenous-osteomyelitis-inadults?source=outline_link&view=text&anchor=H18562114#H18562114
36-What is the most accurate test for carpal tunnel syndrome:
* Tinel
* Compression test
* Durkan's carpal test
* Phallens test
Answer: B
uptodate
http://www.uptodate.com/contents/carpal-tunnel-syndrome-clinical-manifestations-anddiagnosis?source=outline_link&view=text&anchor=H5#H5
37- Old female with lytic bone lestion , high ALP whats the tx ?
* Bisphosphonate
* selective estrogen
Answer:A
http://www.uptodate.com/contents/overview-of-the-management-of-osteoporosis-in-postmenopausalwomen?source=outline_link&view=text&anchor=H40#H40

364

38-tttx of de queverian syndrome


Answer:
usually to start with conservative interventions, which include a forearm-based thumb spica splint with the
interphalangeal joint free along with short-term nonsteroidal antiinflammatory drugs (NSAIDs). We suggest a
local glucocorticoid injection for patients whose symptoms have not resolved with conservative management.
Most patients recover with this intervention.
Patients who present with severe symptoms may benefit from a glucocorticoid injection at the initial
presentation. Surgical release is generally reserved for patients who have not improved with conservative
therapy and one or two glucocorticoid injections .
up to date
http://www.uptodate.com/contents/de-quervaintendinopathy?source=outline_link&view=text&anchor=H660498#H660498
39-Case of tibial collateral ligament injury
A.
B.
C.
Answer:
-Mechanism of injury of lateral collateral ligament contact injury, such as a direct blow to the medial side of the
knee, or a noncontact injury, such as a hyperextension stress, may result in a varus force across the knee
injuring the LCL
LATERAL COLLATERAL LIGAMINT TESTED BY VARUS STRESS TEST
- MEDIAL COLLATERAL LIGAMENT TESTED BY VLULGAS STRESS TEST
40- Case of closed Fx of ulnar and radius, what's the tttx??
Answer:
open reduction internal fixation is treatment of choice in fracture radius and ulna together .
From Prof. Mohammed Jalaat Alfaisal
http://emedicine.medscape.com/article/1239187-treatment#d10
41-read about how to differentiate between carpal tunnel syndrome and thoracic outlet obstruction
Answer:
Read about it
42-trendelenburg gait feature :A.
internal rotation with adduction
B.
internal rotation with abduction
C.
external rotation with adduction
D.
external rotation with abduction
Answer:?
Watch video for more information
http://www.physio-pedia.com/Trendelenburg_Test

365

43-elderly patient with low back pain , what is the drug cause relaxation of muscle ?
A.
diazepam
B.
clorcarpine
Answer:
Answer: one of diazepam uses is to relieve muscle spasm
http://www.webmd.com/drugs/2/drug-6306/diazepam-oral/details
Mechanical low back pain treated with NSAID (master the board)
44- The most common cause of olecron bursitis is ?
A- Repeated injury
Answer: A
Reference: http://www.nhs.uk/Conditions/Bursitis/Pages/Causes.aspx
45- The nerve which works on arm extension is?
A- Radial nerve.
Answer: A
The radial nerve is the "great extensor" of the arm: it innervates all the extensor muscles in the upper and
lower arm.
Reference: https://informatics.med.nyu.edu/modules/pub/neurosurgery/motor.html
46-Patient complaint progressive wrist pain since 2 months and increased in the past 1 month, give
history of cesarean delivery on exam there numbness and normal range of motion what is the Rx?
Thumb cast Whole are cast
Surgical decompression
Answer:B its Carpal tunnel syndrom *Not Sure*
Reference: http://www.uptodate.com/contents/ulnar-neuropathy-at-the-elbow-andwrist?source=see_link&sectionName=Ulnar+neuropathy+at+the+wrist&anchor=H87963063#H87963063
47-Pt with +ve sign of Finkelstein test what is your management?
(No options)
+ve Finkelstein sign in case of De Quervain's tenosynovitis.
Answer: Most likely is thumb splint.
Reference: http://orthoinfo.aaos.org/topic.cfm?topic=a00007
48-Ankle joint is formed by:
A- Distal tibia fabula talaus Latral malloli
B- medial malloli talaus Calclunu
C- tibia fibila
Answer: A
5D6698AB-1FE8-4BF3-B847-25F93DC13D2E
49-Old patient with inability to abduct and externally rotate the shoulder What's the abnormality?
A. Acromio something
B. Rotator cuff injury
Answer: B Toronto Notes OR 13
50-Pt fall on outstretched hand , he can't flex the distal part of his second finger ? Muscle injury?
Answer: The flexor digitorum profundus acting on the proximal and distal joints, and the flexor digitorum
superficialis acting on the proximal joints
Reference: Wikipedia

51-What is the most specific sign for osteoporosis ?


A.
bone density
B.
subchondral cyst
C.
compression fracture
Answer: C?

366

52- 12 y/o Obese cant't bear weight with left hip extrnal rotation, x-ray provided?
A) Sliped captal femoral epiphyesis
B) Fracure of femor neck
Answer: B ?
- femoral neck Fx presentation :
Symptoms
o impacted and stress fractures
slight pain in the groin or pain referred along the medial side of the thigh and knee
o displaced fractures
pain in the entire hip region
Physical exam
o impacted and stress fractures
no obvious clinical deformity
minor discomfort with active or passive hip range of motion, muscle spasms at extremes of motion
pain with percussion over greater trochanter
o displaced fractures
leg in external rotation and abduction, with shortening
http://www.orthobullets.com/trauma/1037/femoral-neck-fractures
- Slipped capital femoral epiphyesis presentation :
Symptoms
o groin and thigh pain
most common presentation
o knee pain
can frequently present as knee pain (15-23%)

o motion
patients prefer to sit in a chair with affected leg crossed over the other
o duration
symptoms are usually present for weeks to several months before diagnosis is made
Physical exam
o abnormal gait
coxalgic, externally rotated gait or Trendelenburg gait
o decreased hip motion
obligatory external rotation during passive flexion of hip
loss of hip internal rotation, abduction, and flexion
o abnormal leg alignment
externally rotated foot progression angle
o weakness
thigh atrophy
http://www.orthobullets.com/pediatrics/4040/slipped-capital-femoral-epiphysis

'Peroneal nerve injury - Often the result of direct contusion in proximal fibula Fx and has a variable prognosis''
http://www.orthopaedicsone.com/display/Main/Proximal+fibula+fractures

53.basal skull fracture cavernous sinus affected>>not sure, which muscle is intact?
trapezius
Sternocliedmastoid
Answer: ?
Vernet syndrome or jugular foramen syndrome is involvement of the IX, X, and XI cranial nerves with the
fracture. Patients present with difficulty in phonation and aspiration and ipsilateral motor paralysis of the vocal
cord, soft palate (curtain sign), superior pharyngeal constrictor, sternocleidomastoid, and trapezius.
medscape
http://emedicine.medscape.com/article/248108-overview#a10

367

54-tttx of de quervain syndrome


Answer:
usually to start with conservative interventions, which include a forearm-based thumb spica splint with the
interphalangeal joint free along with short-term nonsteroidal antiinflammatory drugs (NSAIDs). We suggest a
local glucocorticoid injection for patients whose symptoms have not resolved with conservative management.
Most patients recover with this intervention.
Patients who present with severe symptoms may benefit from a glucocorticoid injection at the initial
presentation. Surgical release is generally reserved for patients who have not improved with conservative
therapy and one or two glucocorticoid injections .
up to date
http://www.uptodate.com/contents/de-quervaintendinopathy?source=outline_link&view=text&anchor=H660498#H660498
55-Muscle of knee extension?
Quadriceps
Biceps femoris
Answer: A. Quadriceps
from ; britannica
http://www.britannica.com/science/knee
56-Case of tibial collateral ligament injury
Answer:
Valgus stress testing of the MCL
Varus stress testing of the LCL
*
*
*
*

Injury severity
Grade I - Less than 5 cm laxity (partial tear)
Grade II - 5-10 cm laxity
Grade III - More than 10 cm laxity (complete tear)

medial collateral ligament (MCL) injury depends on the severity of the injury.[7, 14] Recommendations for
treatment include the following:
Grade I - Compression, elevation, and cryotherapy are recommended. Short-term use of crutches may be
indicated, with weight-bearingastolerated (WBAT) ambulation. Early ambulation is recommended.
Grade II - A short-hinged brace that blocks 20o of terminal extension but allows full flexion should be used.
The patient may ambulate, WBAT. Closed-chain exercises allow for strengthening of knee musculature
without putting stress on the ligaments.
Grade III - The patient initially should be nonweight-bearing (NWB) on the affected lower extremity. A hinged
braced should be used, with gradual progression to full weight-bearing (FWB) over 4 weeks. Grade III injuries
may require 8-12 weeks to heal.
All MCL injuries should be treated with early range of motion (ROM) and strengthening of musculature that
stabilizes the knee joint. Conservative measures usually are adequate, but, if the patient fails to progress with
treatment, a meniscal or cruciate ligament tear is suggested.
Lateral collateral ligament (LCL) injuries heal more slowly than do MCL injuries, due to the difference in
collagen density. Recommendations for the treatment of LCL injuries include the following:
Grades I and II - These injuries are treated according to a regimen similar to that for MCL injuries of the same
severity. A hinged brace is used for 4-6 weeks.
Grade III - Severe LCL injuries typically are treated surgically due to rotational instability, because they usually
involve the posterolateral corner of the knee. Patients may require bracing and physical therapy for up to 3
months in order to prevent later instability.
medscape
http://emedicine.medscape.com/article/307959-treatment#d9

368

57- Case of closed Fx of ulnar and radius, what's the tttx??


Answer:
Most closed fractures of the humeral shaft can be treated successfully with closed methods; union rates of
more than 90% are often reported.Multiple closed techniques are available, including the following ;
*
Traction
*
Hanging arm cast
*
Coaptation splint
*
Velpeau dressing
*
Abduction humeral/shoulder spica cast
*
Functional brace
All of these techniques have been used successfully, but closed humeral shaft fractures are usually treated
with a hanging arm cast or a coaptation splint for 1-3 weeks, after which they are placed in a functional brace.
medscape
http://emedicine.medscape.com/article/1239985-treatment#d9

? 58-Minutes of X-ray radiation?


25
15
5
0
Answer: B. 15
59-Read about how to differentiate between carpal tunnel syndrome and thoracic outlet obstruction
60-Elderly patient with low back pain , what is the drug cause relaxation of muscle ?
Diazepam
clorazepate
Answer: A
clorazepate Treatment of generalized anxiety disorder; management of ethanol withdrawal; adjunct
anticonvulsant in management of partial seizures
Diazepam : treatment of Muscle spasm , Rapid tranquilization of agitated patient , Sedation in the ICU patient
, Seizures , Status epilepticus , Skeletal muscle relaxant
Source : up to date
http://www.uptodate.com/contents/diazepam-druginformation?source=search_result&search=Diazepam&selectedTitle=1%7E150
? 61- Muscele spasm after run
Answer : ?

62-Patient complaint progressive wrist pain since 2 months and increased in the past 1 month , give history of
cesarean delivery on exam there numbness and normal range of motion what is the Rx?
* Thumb cast Whole arm cast
* Surgical decompression
Answer : ?
Wrist splints with the wrist joint in neutral or slight extension (to be worn at nighttime for a minimum of 3-4 wk)
sorce ;medscape
http://emedicine.medscape.com/article/327330-treatment#d9

369

63-Pt does not complain of anything ,, has sudden knee swelling ? What is the best thing to do ?
Answer: aspiration ?
General measures to relieve knee pain and swelling should be tailored to the individual patient. These may
include advice for partial or nonweight bearing, splints, cold packs, and prescription of simple analgesics and
nonsteroidal anti-inflammatory drugs if not contraindicated. However, antibiotics should not be started before
cultures are obtained from appropriate diagnostic sampling. Similarly, intraarticular steroids should not be
administered unless an appropriate diagnosis has been reached and contraindications have been ruled out.
sorce ; medscape
http://www.medscape.com/viewarticle/714757

64-Pt with + sign of finkelstein test what is your management


Answer:
dx: De Quervain
Rx: Thumb spica
De Quervain tendinopathy is a nonprogressive, albeit painful condition that is typically self-limited in the
majority of patients . Therefore, the goal of treatment is to relieve symptoms and limit disability.
therapy is usually to start with conservative interventions, which include a forearm-based thumb spica splint
with the interphalangeal joint free along with short-term nonsteroidal antiinflammatory drugs (NSAIDs). We
suggest a local glucocorticoid injection for patients whose symptoms have not resolved with conservative
management. Most patients recover with this intervention.
Patients who present with severe symptoms may benefit from a glucocorticoid injection at the initial
presentation. Surgical release is generally reserved for patients who have not improved with conservative
therapy and one or two glucocorticoid injections.
sorce : up to date
http://www.uptodate.com/contents/de-quervaintendinopathy?source=outline_link&view=text&anchor=H660498#H660498k
65-Old patient with inability to abduct and externally rotate the shoulder What's the abnormality?
A-Acromio something
http://www.orthobullets.com/sports/3047/acromio-clavicular-injuries-ac-separation
B-Rotator cuff injury
http://www.orthobullets.com/sports/3043/rotator-cuff-tears
Answer: B ?
Presentation of Rotator cuff tears:
Symptoms
o insidious onset of pain exacerbated by overhead activities
o night pain, which is a poor indicator for nonoperative management
Physical exam (complete exam of the shoulder
Overview of Physical Exam of Rotator Cuff
Cuff Muscle
Strength Testing
Special Tests
Supraspinatus
Weakness to resisted elevation in Jobe position
o Drop arm test
o Pain with Jobe test
Infraspinatus
ER weakness at 0 abduction

370

o ER lag sign
Teres minor
ER weakness at 90 abduction and 90 ER
o Hornblowers
Subscapularis
IR weakness at 0 abduction
o Excessive passive ER
o Belly press
o Lift off
o IR lag sign
66- Pt fall in outstretched hand , he can't flex the distal part of his second finger ?Muscle injury ?
1.
Answer: flexor digitorum profundus(for distal part only)

67- A 13 years old boy with right hip pain for 3 months x ray shows degenerative collapse of femoral
neck Wbc is 17 Esr normal Alp is normal
a.
Perthe diseases
b.
Hip arthritis
c.
Tumor
Answer: a
68- Pt after RTA , no abduction and lateral rotation of the arm.. . What is the origin of the affected nerve ?
* Medial plexus
* Lateral plexus
* Lower plexus
* Root
Answer: D
69- Old Patient with back pain. Spine DEXA results is provided below:
Cervical 0.05
Thoracic <-1.5
Lumbar <-2.8
Sacral <-1.3 What is the diagnosis?
* Osteopenia
* Severe osteopenia
* Osteoporosis
* Established osteoporosis
Answer:

up to date
http://www.uptodate.com/contents/image?imageKey=ENDO%2F53999&topicKey=ENDO%2F2056&source=s
ee_link

371

70- most common nerve injury in the humerus :


A.
B.
C.

Radial

Answer: A
Radial nerve palsy associated with fractures of the shaft of the humerus is the most common nerve lesion
complicating fractures of long bones
medscape
http://emedicine.medscape.com/article/825488-overview
71- which of the following is non traumatic fracture in osteoporosis ?
A.
B.

Vertebral fracture
statin induced myopathy in old lady

Answer: A
Vertebral fracture is the most common clinical manifestation of osteoporosis.
http://www.uptodate.com/contents/clinical-manifestations-diagnosis-and-evaluation-of-osteoporosis-inpostmenopausal-women?source=outline_link&view=text&anchor=H3#H3
72- Old male not known to any medical illness presented with RT knee swelling, no fever or
tenderness, what investigation will you order?
A. Arthrocentesis
B. Knee MRI
Answer: if these are the choices and no X-ray then its A
Reference: http://www.aafp.org/afp/2000/0415/p2391.html
73- wich part of body bone is forming the ankle?
Answer:
Ankle joint: the fibula, the tibia and the talus.
74- Examiner ask to stand on toes, What N examine?
Answer:
Toe Dorsifelxion by Deep perineal N
Toe Planter flexion by Tibial N
So By Tibial in this case.
Reference: http://www.orthobullets.com/spine/2002/lower-extremity-spine-and-neuro-exam
75-pt can't do dorsiflexion & eversion :
A)common peroneal N
B)Deep perineal N
Answer: B
Reference: http://www.getbodysmart.com/ap/muscularsystem/footmuscles/fibularistertius/tutorial.html
76- Pt with 1st metatarsal joint pain, redness and erythema,High temperatures? What is the cause?
A.staph arose
B.Mono sodium urate crystal
C.Ca pyrophosphate crystal
Answer: B its Gout.
Reference: PubMed

372

77- Tibial fracture, next step?


A.CT
B.MRI
C.Angiography
D.X-RAY
Answer: D
The standard protocol is to obtain anteroposterior and lateral radiographs of the injured leg.
Reference:PubMed
N.B: Tibial plateau fracture CT is Mandatory.
78-Patient is referred to Orthopedics because of abnormally high bone density scan. She has no history of
fractures. She takes analgesics and is waiting for hip replacement surgery. Her dietary calcium and serum
biochemistry are normal. What is the the cause of her high density result? (no numbers were provided).
Osteoarthritis.
Pagets disease.
Osteoporosis.
Osteopenia.
Answer: A
http://www.ncbi.nlm.nih.gov/pmc/articles/PMC3651616/
Applyes book orthopedics
79- bilateral shoulder and hip stiffness and pain what is Dx :
A) polymyalgia rheumatica
B) OA
answer: a
80-loss of adduction of fingers caused by injury to:
A) ulnar
B) median
answer: a

81-patient with numbness of index finger when he scissor .... What is Dx:
A) OA
B) ducyptus
answer:
119- ??
82- patient with OA which type of exercise is the best :
A) high repitition and .........
B) low repition and .......
answer:

83-picture of tipial fracture proximal perefral perfusion normal and normal sensation
the best next to investigate?
a-angiography
b-MRI
c-doppler us
Answer:

There were also missing questions about the following:


Osgood-Schlatter disease:
Mechanism of injury: Overuse apophysitis of the tibial tubercle. Causes localized pain, especially with quadriceps
contraction, in active young boys.
Treatment: Decrease activity for 23 months or until asymptomatic. A neoprene brace may provide symptomatic
relief.
10 years old boy with hip pain
Anterior cruciate ligament.
Medial collateral ligament strain.

373

Dermatology

374

1- A beach guard who stays most of the time under the sun, presented to you complaining of new papules over his
nose and cheeks. What is the most likely diagnosis?
a.
Melasma
b.
Hamartoma
c.
Freckling
d.
Actinic keratosis
Answer: D
Premalignant lesions caused by sun exposure. Small, rough, erythematous or brownish papules or plaques. Risk of
squamous cell carcinoma is proportional to degree of epithelial dysplasia.
Reference: FA USMLE 1
2- A patient with a typical history of herpes zoster infection with dermatomal involvement.
a.
HSV
b.
Herpes Zoster
c.
Bacterial vaginosis
Answer: B
3- Patient with psoriasis involving 15% of his body with nail involvement. What is the best treatment for him?
a.
Laser
b.
Topical steroid
c.
Topical vitamin D analogs
d.
Methotrexate
Answer: D
Methotrexate can be used in severe psoriasis (involving more than 10% of body surface).
4- Picture of a woman's face with comedones and papules on the cheek. Which of the following drugs should be
avoided?
a.
retinoic acid
b.
erythromycin
c.
tetracycline
d.
steroids
Answer: D
5- Description of a man with red nodules and papules on the face involving mainly the cheeks and nose. Few
telangiectasias are also present. What treatment will you suggest?
a.
cold compresses
b.
oral doxycycline
c.
topical retinoic acid
d.
topical steroids
Answer: B
Its a case of rosacea.
6- 18 years old girl with vitiligo on the face and arm that is symmetrical started on medical treatment 3 years ago.
She wants to get married soon and wants the lesions go away. What will you do for her?
a.
Split thickness graft
b.
Continue medical treatment
c.
Melanocyte transfer
d.
Stop med and observe
Answer: A or C?

375

7- A long scenario about a rash over the elbows, knees, and cheeks that is itchy, weeps and crusts which got better
with steroids. What is the diagnosis?
a.
HSV
b.
staph
c.
fungal
d.
eczema
Answer: D
8- A patient complaining of itchiness that increases at night, superficial linear burrows around 4th & 5th digits,
inflammatory papules and nodules.
a.
Atopic dermatitis
b.
Allergic dermatitis
c.
Scabies
d.
Tinea
Answer: C
9- Which of the following drugs is used for acne rosacea
a.
Erythromycin
b.
Clindamycin
c.
Cephalexin
Answer: ?
For mild rosacea: We can use topical metronidazole or azelaic acid creams and gels.
For severe rosacea: doxycycline (first line therapy), tetracycline and erythromycin.
Reference: http://www.nhs.uk/Conditions/Rosacea/Pages/Treatment.aspx
10- Rash in the dermatomal distribution of the trunk.
a.
Shingles.
Answer: A
11- Treatment of pyoderma gangrenosum?
a.
Methotrexate.
b.
Antibiotics.
c.
Systemic Steroids.
Answer: C
Reference: http://emedicine.medscape.com/article/1123821-treatment
12- Condyloma lata is caused by?
Answer: ?
A Smooth, moist, painless, wart-like white lesions on genitals caused by spirochete Treponema pallidum in secondary
syphilis.
13- Condyloma acuminata is caused by?
Answer: ?
Commonly HPV 6 and 11
14- 30 years-old man came with erythema of the nose and pain in the right eye, there is erythema and nodules in
left periorbital area and forehead. What is the most likely diagnosis?
a. Systemic lupus Erythematosus.
b. Herpes zoster.
c.
Rosacea.
d.
Measles.
Answer: B

376

15- Schoolboy brought by his mother, he have 2x2 hair loss in the temporal area, hair around this area-clubbed
hair. What is the most likely diagnosis?
a. Trichotillomania.
b. Alopecia areata.
c. Tinea infection.
Answer: A

16. (long scenario) 4 years old child presented with area of 3x3 hair loss, on examination of the area there is
multiple pustule. What is the most likely diagnosis?
a. Aplasia cutis congenita.
b. Staphylococcal infection.
c. Trichotillomania.
Answer: B
(staph infection alone won't cause hair loss , its only cause pustules so in this case it's look like infection on top of an
original Dx which had caused the alopecia which is missing in this scenario , if the child born with areas of alopecia the
dx would be Aplasia cutis congenita which later could be complicated by secondary infection )
Reference: http://dermnetnz.org/lesions/aplasia-cutis.html
17- A Picture of 1 month baby with rash and scale what is the most likely cause?
a. Eczema
Answer: ?
- Seborrhoeic dermatitis
18-What is the main treatment for non inflammatory acne?
a.
Azelaic acid
b. Isotretinoin
c.
Differin cream
Answer: C
19-Patient on long term use of topical steroids.What is the most likely side effect?
a.
Scaling
b.
Atrophy
Answer: B
20- What is the treatment for moderate to severe acne vulgaris?
a.
Isotretinoin
b.
Tetracycline
c.
Clindamycin
Answer: ?
Refer to the table at the end of Dermatology Section.
21-Patient with Positive HPV. What is associated with it ?
a.
Hyperkeratosis
b. Parakeratosis
c.
Apoptosis
Answer: A

377

22 - Pyoderma gangrenosum with which disease?


Answer: Ulcerative colitis
Ref: VMA videos
23 - Patient with single hypopigmentes lesion on the forarm, with Ulnar nerve thickness, what is the diagnosis:
a:Vitilligo
b:Amylodosis
c:TB
Answer: I think it's associated more with leprosy because leprosy can presents with Hypopigmented or reddish skin
lesion with Involvement of the peripheral nerves: Demonstrated by definite thickening of the nerve with/ without loss
of sensation and/or weakness of the muscles of the hands, feet or eyes supplied.
while vitiligo rarely to be associated with nerve thickness
Ref: http://nlep.nic.in/pdf/manual3.pdf
24 - First line treatment for non inflammatory acne: azolic acid, isoteritoin, diffirin cream
Answer: the best is isoteritoin but we should start with adapalene diffrin cream first
Ref: European Handbook of Dermatological Treatments
According to SLE made easy isotretinoin is the main treatment for non-inflammatory acne
25 - Black to brown lesion on sole of the foot
A)helmata
Others I don't remember
Answer: differential diagnosis includes benign naevus, a pigmented seborrhoeic wart, a squamous cell papilloma,
carcinoma or malignant melanoma. a capillary cavernous haemangioma and Kaposi's sarcoma. so the differential is so
broad and we need more details.
26 - Diaper rash that is resistant to topical steroids and has satellite lesions on thighs (well demarcated and red):
topical antifungal?
Answer: Antifungal agents such as nystatin, clotrimazole, miconazole, ketoconazole, and sertaconazole are effective
topical therapies for diaper rash
Ref: uptodate
27 - about pt on longe uses of topical steroids. What will happen ?
A.scalling
B.atrphy.
C...
D....
Answer: long use of steroid Super potent and potent topical corticosteroids may induce atrophy, telangiectasia, and
striae.
Ref: uptodate

28 - about baby with multiple plaques in his face, abdomen and feet what is the Dx?
A.
Basal cell carcinoma
B.
squamous cell carcinoma
Answer: most likely its eczema but im not sure

ADDED QUESTION: ( need to be more checked )


1.A patient with a typical history of headache, fever and then rash which type of herpes .
A-HS type 1
B-HS type 2
c-Vercella
Answer :C

378

2.patient with a localize patch of hair loss


male pattern
female pattern
something starting with T I dont know what is it but I chose it !!
Answer :
*male pattern hair loss: receding hairline and/or hair loss on the top and front of the head.
*Female pattern hair loss: diffuse thinning of hair on the scalp
*causes of localized patch of hair loss:
-Tinea capitis the most common cause of hair loss in children. patchy hair loss with some broken-off hairs visible just
above the surface of the scalp
-Alopecia areata appear literally overnight, or sometimes over a few days
-Trauma including trichotillomania
-Telogen effluvium
-traction alopecia thinning from tight braids or ponytails
3. Patient came with lobulated nose, erythema over cheeks and (other features indicating rosacea type 4), what is
the treatment?
Doxycycline. *
Acyclovir.
Answer Doxy :http://emedicine.medscape.com/article/1071429-treatment
4.First line of defense in the skin is..
1 mucous membranes
2 collagenous cell.
3 keratinocytes.
4 areolar connective tissue
answer:
5. First immunologic defense of skin:
A) Keratinocytes
B) Blast cells
C) melanocytes
6. scenario for pt with scabies what is the ttt of choice???
Answer:
1/treated overnight with 1-2 applications of 5% permethrin from neck down +their contacts should be treated
2-oral ivermectin
3-symptoms treatment pruritus
*FIRST AID USMLE STEP2 CK
7. A patient presented with macular papular rash and fever no vaccination before ?
A) Measles
B) Mumps
C) Rubella
Answer: it could be A or C.
Although the distribution of the rubella rash is similar to that of rubeolameasles, the spread is much more rapid, and
the rash does not darken or coalesce
also Rubella has low grade fever
Measles has a high grade fever
reference: Uptodate
Mumps: low-grade fever, malaise, headache, myalgias, and anorexia. These symptoms are generally followed within 48
hours by the development of parotitis

379

9.about patient on longe uses of topical steroids. .what will happen ?


A.scalling
B.atrphy.
C...
D....
Answer: Local side effects are the main side effects associated with topical corticosteroids. They usually affect the face,
folds of skin and areas that have been treated many times during the past months or years.
Local side effects can include:
burning or stinging of the skin this is a common side effect that usually occurs when you start treatment; it tends to
improve after a few days as your skin gets used to the medication
worsening of a pre-existing skin infection
folliculitis inflamed hair follicles
thinning of the skin this can make the affected skin more vulnerable to damage; for example, you may bruise more
easily
stretch marks
contact dermatitis skin irritation caused by a mild allergic reaction to the substances in a particular topical
corticosteroid
acne, or worsening of existing acne
rosacea a condition that causes the face to become red and flushed
changes in skin colour this is usually more noticeable in people with dark skin
excessive hair growth on the area of skin being treated
10. The best test for hypersensitivity type 1:
1- subdermal skin
Answer: subdermal skin test
11. Pregnant +ve hx of herpetic outbreak annually on examination everything is normal. What is the
management?
A Reassurance
B Acyclovir
Answer:?
12.How to decrease the keratosis by?
A Wound trauma
B Use AB
C Avoid sun exposure
Answer :C
13. Pt have nodulo-cystic acne how to management?
A Clindamycin
B Oral AB
C Erythromycin
D Isotrention
Answer :?
14.A case of a patient diagnosed to have cutaneous leishmania or bghlabar/ bhagdad type which type of
leishmania ? or whats the organism ?
A- Tropica ,
b- donovani
c- Post kala-azar dermal leishmaniasis
Answer: A
Reference:
http://www.uptodate.com/contents/image?imageKey=ID%2F86988&topicKey=ID%2F5689&rank=1%7E93&source=se
e_link&search=leishmania&utdPopup=true
http://www.uptodate.com/contents/clinical-manifestations-and-diagnosis-of-cutaneousleishmaniasis?source=search_result&search=leishmania&selectedTitle=1~93

380

15- Oropharyngeal maculopapular rash .. Also rash in palm and foot ..?
A CMV
B EBV
C coxsackievirus
D Vaccina virus
Answer: C
Reference:
http://www.uptodate.com/contents/hand-foot-and-mouth-disease-and-herpangina-an-overview

16 .2 year old complain of papule on the foot no itching pink pale not respond for antifungal?
A: Granuloma
Answer: A
Granuloma annulare is a common skin condition with raised, flesh-colored bumps that appear in a ring. It may occur on
any part of the body (though most commonly on the sides or backs of the hands or feet). The bumps may be red at the
beginning, but this disappears as the ring forms. There is no itching or scaling.
Reference:
http://www.uptodate.com/contents/granuloma-annulare
http://www.drgreene.com/qa-articles/ringworm-lookalikes/
17. Isotretinoin most feared complication (repeated)
A birth defect
Answer:
isotretinion (Accutane) (roaccutane) is teratogenic it can cause embryopathy in 20 - 30 % and spontaneous abortion is
approximately 20 %
Reference:
http://www.uptodate.com/contents/oral-isotretinoin-therapy-for-acne-vulgaris

18- Young female, loss of hair half of her head and the skin is normal?
A Alopecia
B Trichomania?
Answer no Enough details but it could be Trichomania
http://emedicine.medscape.com/article/1071854-overview#a6
19. Bee bit when you give him epinephrine what you will inhibit ?
a.
Interleukin ...... ?
b.
Systemic inflammatory .... ?
Answer:?
20. pt dx with sheglles around thoracic dermatome, what will you give him?
A. Topical acyclovir and topical steroid
B. Oral steroid
C.
Answer: Oral acyclovir
A.
B.

21- male with pustules and papules and telangiectasia , what is the dx ?
Rosacea
Answer:A

381

22. Q about description of scarlet fever rash?


A.
B.
Answer: The rash of scarlet fever is a diffuse erythema that blanches with pressure, with numerous small (1 to 2
mm) papular elevations, giving a "sandpaper" quality to the skin (picture 1). It usually starts in the groin and armpits
and is accompanied by circumoral pallor and a strawberry tongue (picture 2). Subsequently, the rash expands rapidly
to cover the trunk, followed by the extremities, and, ultimately, desquamates (picture 3); the palms and soles are
usually spared. The rash is most marked in the skin folds of the inguinal, axillary, antecubital, and abdominal areas
and about pressure points. It often exhibits a linear petechial character in the antecubital fossae and axillary folds,
known as Pastia's lines (picture 4).
23. patient present to the ER with erythema and peeling of the skin with fever and other things
A-Toxic epidermal syndrome
B-Nisseria medgitidis
C- septicemia
Answer: ?
24- Symmetrical hypo pigmentation with no scaling for 1 year and it's progressing with no previous lesion
a. Post inflammatory
b. Vitiligo
c.Leprosy
Answer: B
25. Scabies ,whats the organism ?
Answer Sarcoptes scabiei
http://www.uptodate.com/contents/scabies?source=search_result&search=scabies&selectedTitle=1~75
26. Purple,papule,polygonal rash on the flexors?
A- ..lichen planus .
Answer A
27. child with macupapular rash in face and nose ( not sure about vesicles ) Whites spot inside cheeks I think there
is fever
1. mumps
2. measles
3. hsv1
4. vercialla zoster virus
Answer:
The answer depends on vesicle if it present it would be vsv if absent would be measles.
28. DM pt with lesion papule on dorsum of right hand
A -granuloma annulare
B -lichen plants
C -fungal infection
Answer:A
29. Old pt. with diffuse thinning of hair and without eyelash, dx ?
Answer: alopecia areata
http://www.uptodate.com/contents/clinical-manifestations-and-diagnosis-of-alopecia-areata?source=see_link

382

30-what most commonly cause itching ?


a -bile salt retention
b -eczema
c-pregnancy
answer:-b
http://www.uptodate.com/contents/pruritus-etiology-and-patientevaluation?source=search_result&search=itching&selectedTitle=2~150#H690781

There were also missing questions about the following:


Hidradenitis suppurativa of the buttocks.
http://www.nhs.uk/conditions/hidradenitis-suppurativa/Pages/Introduction.aspx
Lichen planus (5 P's)
Management of Acne
Scabies

383

Ophthalmology

384

1- Painful vision loss.


a.
Acute angle closure glaucoma
Answer: A
Reference: Lecture note on ophthalmology, 9 th edition

2- Picture of snellen's test. What is the visual level of this patient?


Answer: ?

e.g. A patient can read until the 3rd line, what is the visual acuity? 20/70.
3- A known case COPD and DM, diagnosed to have primary open angle glaucoma. What is the optimal treatment
for glaucoma?
a.
Topical Timolol
b.
Topical Carteolol
c.
Systemic Acetazolamide
Answer: C

385

4- 24 years old female newly diagnosed type 2 DM, she is wearing glasses for 10 years, how frequent she should
follow with ophthalmologist?
a.
6 months
b.
12 months
c.
5 years
d.
10 years
Answer: B
For type 1 diabetic: retinal screening annually beginning 5 years after onset of diabetes, general not before onset of
puberty.
For type 2 diabetic: screening at the time of diagnosis then annually.
Reference: 3rd Edition UQU > Ophthalmology > Q 78.
5- Lacrimal gland tumor lead to proptosis in which direction?
a.
Down \in
Answer: A
Maxillary sinus growth: Superior
Lacrimal gland tumor: Inferomedial
Frontal or ethmoidal sinus lesion: Inferolateral
Reference: Textbook of Ophthalmology
6- A patient with unilateral painful vision loss. What is the cause?
a.
papilledema
b.
toxic amblyopia
c.
optic neuritis
Answer: C
Explained previously, see Q1
7- A patient with eye movement abnormalities and ptosis. What is the nerve is involved?
a.
3rd
b.
4th
c.
6th
Answer: A
Oculomotor (3rd cranial nerve): Eye movement (SR, IR, MR, IO), pupillary constriction (sphincter pupillae:
Edinger-Westphal nucleus, muscarinic receptors), accommodation, eyelid opening (levator palpebrae)
Loss of levator palpebrae function will cause ptosis
Trochlear (4th cranial nerve): Eye movement (SO)
Abducens (6th cranial nerve): Eye movement (LR)
Reference: FA USMLE step 1
8- Iris neovascularization caused by?
a.
Non-proliferative diabetic retinopathy.
b.
Central retinal vein occlusion
Answer: B
Abnormal iris blood vessels (neovascularization) may obstruct the angle (secondary glaucoma) and cause the iris to
adhere to the peripheral cornea, closing the angle (rubeosis iridis). This may accompany proliferative diabetic
retinopathy or central retinal vein occlusion due to the forward diffusion of vasoproliferative factors from the
ischaemic retina.
Reference: Lecture note on ophthalmology, 9 th edition
9- What is the treatment of chronic ptosis?
Answer: ?
http://eyewiki.aao.org/Aponeurotic_ptosis#General_treatment

386

10- Patient came for annual check up & found to have cupping of disk. What is the diagnosis?
a.
Retinal detachment
b.
Diabetic retinopathy
c.
Chronic open angle glaucoma
Answer: C
11- Ring lesion in eye.
a.
Acanthamoeba infection
Answer: A
12- Picture of a patient with corneal ulcer. What is the treatment?
a.
Contact lens
Answer: ?
The Contact lens is one of the causes of corneal ulcers.
The Question could be similar to the next (Q13)
13- Corneal epithelial defects (CED) (Another term of corneal abrasion or ulcer) due to fingernail injury. What is
the treatment?
a.
Double patch.
Answer: A
The answer confirmed by an intern who had the same Q in his exam and got the full mark in ophthalmology section.
14- Corneal epithelial defects (CED), in addition to this presentation, what else this patient might have?
a.
Photophobia
b.
Visual loss
Answer: A
15- Long term topical steroid drops can cause:
a.
Glaucoma.
b.
posteior subcapsular cataract
Answer: B
16- A patient known case of COPD and glaucoma complaining recently of intermittent cough.
a.
Pitoxolol
b.
Timolol
Answer: B
Timolol is A Nonselective B blocker used to treat glaucoma by decrease secretion of aqueous humor. Should be avoided
in asthma/COPD patient due to increase risk of exacerbation. Reference: FA USMLE step 1
17- How to administer eye drops and ointments?
a.
One drop in the lower fornix
Answer: A
18- A patient came with drooling and lacrimation (cholinergic reaction). What to give?
a.
Physostigmine
Answer: ?
NB. No atropine in the choices
*Physostigmine is NOT the right answer, it is an acetylcholinesterase inhibitor (It can be the cause of the cholinergic
reaction)

387

19- Trabeculectomy is an operation used for which of the following conditions?


a.
Open angle glaucoma
b.
Closed angle glaucoma
Answer: A
Trabeculectomy is effective for chronic angle-closure glaucoma. However, compared to primary open-angle
glaucoma, any aqueous-draining procedure in an eye with a shallow anterior chamber and a chronic closed angle poses
the risk of further shallowing the anterior chamber or precipitating malignant glaucoma.
Reference: http://emedicine.medscape.com/article/1205154-treatment#showall
20- A patient with painful swelling (dacryocystitis). What is the best management?
a.
Oral antibiotic
b.
Drain surgically
Answer: ?
In general, dacryocystitis is a surgical disease. Surgical success rates in the treatment of dacryocystitis are approximately
95%. Acute cases are best treated surgically after the infection has subsided with adequate antibiotic therapy.
For acute dacryocystitis, an external dacryocystorhinostomy is preferred after several days of initiating antibiotic therapy.
Rarely, dacryocystorhinostomy must be performed during the acute phase of the infection to facilitate clearing of the
infection. Reference: Medscape.
21- A patient has painless vision loss for 3 days.
a.
Retrobulbar optic neuritis
b.
Uveitis
Answer: ?
Explained previously, see Q1. Reference: Lecture note on ophthalmology, 9 th edition
Retrobulbar optic neuritis: Pain with eye movement in retrobulbar optic neuritis because rectus contraction pulls on
the optic nerve sheaf.
Uveitis: http://emedicine.medscape.com/article/798323-overview
22- A patient with follicular keratosis. What will you check?
a.
Eyes
Answer: A
Because of Vit. A deficiency
Keratosis pilaris is another name of follicular keratosis
Reference: http://disorders.eyes.arizona.edu/category/clinical-features/keratosis-pilaris
23- A patient presented with red eye and ciliary flush. What is the diagnosis?
Answer: ?
Redness of the eyes: In conjunctivitis the entire conjunctival surface including that covering the tarsal plates is involved.
If the redness is localized to the limbus ciliary flush the following should be considered:
1. Keratitis
2. Uveitis
3. Acute glaucoma.
Reference: Lecture note on ophthalmology, 9 th edition (P:65)
24- A patient presented with colour vision loss. What is the cause?
Answer: ?
http://www.webmd.com/eye-health/tc/color-blindness-topic-overview
25- A patient with increased ICP. What will you check?
a.
Papilledema
Answer: A

388

26- Cavernous sinus aneurysm leading to diplopia & blurred vision, what nerve affected?
a.
Trochlear
b.
Abducens
c.
Optic
d.
Ophthalmic (V1)
Answer: B
Cavernous sinus syndromepresents with variable ophthalmoplegia, decrease corneal sensation, Horner syndrome and
occasional decreased maxillary sensation. 2 to pituitary tumor mass effect, carotid-cavernous fistula, or cavernous sinus
thrombosis related to infection. CN VI is most susceptible to injury. Reference: FA USMLE step 1
27- Eye drops that is contraindicated in acute closed angle glaucoma?
Answer: ?
1-agonists (Epinephrine) because of Mydriasis side effect. Reference: FA USMLE step 1
28- A male patient presented with breast enlargement and decreased sexual desire. Labs: Hyperprolactinemia.
What visual deficiency this patient might have?
a.
Bitemporal hemianopsia
Answer: A
29- How to test visual acuity?
a.
Snellen chart
Answer: A
30- A patient comes with signs and symptoms of glaucoma. What is treatment?
Answer: ?
IV acetazolamide, topical pilocarpin and B blocker. Reference: UQU
31- Adult complaining of pain when moving the eye, fundoscopy reveal optic disc swelling. What is the most
likely diagnosis of this case?
a.
Optic neuritis.
b.
Central vein occlusion.
c.
Central artery occlusion.
Answer: A
32- Father came with his 6 years old daughter to the clinic, she has esotropia. What you will do?
a.
Glasses.
b.
Surgery.
Answer: ?
Nonsurgical treatments include patching, correction of full hyperopic refractive error (glasses) then if not
improved surgery.

33-A patient presented with pterygium. What is the possible complication?


Corneal scarring
Corneal perforation
Strabismus
Answer: A

389

34- 47 year old man with HTN & DM, past history of treatment of mycobacterial infection. He gave history of
blurred and decreased vision. On examination there is a flame shape on the retina, cotton wool spots and macular
edema (Other investigations were included as well). What is the cause?
a.
Retinal vein occlusion.
b.
Ethambutol.
c.
DM.
Answer: C
Diabetic retinopathy is common in patients with chronic poor glycemic control. Features include:
Microaneurysms: The earliest clinical sign of diabetic retinopathy
Dot and blot hemorrhages
Flame-shaped hemorrhages
Retinal edema and hard exudates
Cotton-wool spots
Venous loops and venous beading: Their occurrence is the most significant predictor of progression to
proliferative diabetic retinopathy (PDR).
Macular edema: Leading cause of visual impairment in patients with diabetes
Reference: http://emedicine.medscape.com/article/1225122-overview
35- What is the function of superior rectus muscle work ?
a. Up and out
b. Up and in
Answer: B
Contraction of the superior rectus results in elevation, intorsion, and adduction of the eye.
See: http://www.yale.edu/cnerves/cn3/cn3_3.html
36- Diabetic patient with high cup to disc ratio. What is the cause?
Answer: ?
Patients with diabetes were previously thought to have a greater risk of developing primary chronic glaucoma with loss
of visual field. However, more recent papers suggest that diabetes is not a greater risk factor, but simply that glaucoma
was found more readily.
Patients with PDR are at risk of developing secondary glaucoma, particularly neovascular (rubeotic) glaucoma
Rubeosis iridis is the growth of new vessels on the iris in eyes with advanced retinal ischaemia. Rubeosis
neovascularization of iris ( NVI) may induce a severe form of intractable glaucoma with growth of new vessels in the
anterior chamber angle (NVA). If uncontrolled, NVA leads to closure of the aqueous fluid drainage route in the anterior
chamber angle of the eye by fibrovascular tissue
Reference:http://www.icoph.org/dynamic/attachments/taskforce_documents/2012-sci267_diabetic_retinopathy_guidelines_december_2012.pdf

36 - pic . Eye lid inflammation what most associated symptoms


(A)secretion
(B) vision loss
Answer: A
37 - pic . Eye with entropian
Answer: Entropion is a malposition resulting in inversion of the eyelid margin.
38 - orbital cellulitis 2 times
Answer: Orbital cellulitis and preseptal cellulitis are the major infections of the ocular adnexal and orbital tissues.
Orbital cellulitis is an infection of the soft tissues of the orbit posterior to the orbital septum, differentiating it from
preseptal cellulitis, which is an infection of the soft tissue of the eyelids and periocular region anterior to the orbital
septum. Patients with orbital cellulitis frequently complain of fever, malaise, and a history of recent sinusitis or upper
respiratory tract infection. Other signs include ; Conjunctival chemosis, Decreased vision, Elevated intraocular
pressure, Pain on eye movement. The patient with orbital cellulitis should be promptly hospitalized for treatment, with
hospitalization continuing until the patient is afebrile and has clearly improved clinically. Medical management is
successful in many cases. Consider orbital surgery, with or without sinusotomy, in every case of subperiosteal or
intraorbital abscess formation.

390

39 - mucupurulent unilateral bacterial cojuctivitis very easy ?


Answer:
40 - What is the function of superior rectus muscle?
A.
in down
B.
out down
C.
up in
D.
up out
Answer: D, it elevated the abducted eye + internally rotate the adducted eye
Referance : Grays anatomy
41 - keratitis caused by parasites
Answer: Acanthamoeba
42 - For what test use distance eye :
Answer: visual acuity ?
43 - patient complaint of red eye and decreased vision in left eye since 3 days, in examination there are ciliary
flush, not fully reactive pupil and floaters in anterior chamber. What is the diagnosis:
A.
Uveitis
B.
Glaucoma
C.
Retinitis
D.
Keratitis
Answer: A
44 - Watery discharge , congested eye preauricular lymph node enlargement, diagnosis?
Answer: viral conjunctivitis ( adeno? )
45 - Which of the following cause loss of color vision?
A. acute angle closure glaucoma
B. mature cataract.
C. severe iridocyclitis.
D. optic atrophy.
Answer : D
46 - Pic about periorbital swelling what other symptoms could the pt has? Image result for eyelid swelling like
this pic
discharge ** and im not sure about the answer
vision
uveitis
Answer: decrease vision if they mean (eye cellulitis) in thi senario .
47 - Red eye pic after pharyngitis and fever what is ur dx?
Answer: adenovirus conjunctivitis
48 - Q about ttt of corneal ulcer ?
A.
Antibiotics
B.
Antifungal
Answer: A?
Bacterial ulcer is most common cause in contact lens wearer. Acanthamoeba ulcer is due to swimming while wearing
contact lens. Fungal ulcer is due to trauma with vegetable material. Treatment for corneal ulcers, regardless of cause,
begins with Antibiotics. Source; mercks manual.
49/Patient had laser for refractive correction ..one of post op complications of this procedure is dryness
of the eye..how to prevent this dryness?
A.
blockage of lacrimal duct
B.
blockage of lacrimal canal
C.
blockage of punctua
D.
blockage of lacrimal sac
Answer: c.

391

50- Pt wake up with mucopurulent discharge and red eye ..and thick mdre esh ..what is d?
A.
viral .
B.
bacterial
C.
allergy
D.
hypersensitive I think
Answer:B
51 - What is distichiasis
Answer: abnormal growth of lashes from the orifices of the meibomian glands on the posterior lamella of the tarsal
plate
52 - pic of disc cup ask for diagnosis
Answer:
53 - pic of cornea after nail injury ask diagnosis
Answer: corneal abration?
54 - stage of hypertensive or increase iop cant remember ask about third stage optic change?
A.
papilloedema
B.
nipping of vein
C.
somthing
D.
another something
Answer:
Grade I : mild arteriolar narrowing
Grade II : arteriovenous crossing
Grade III : retinal hemorrhage, exudate, cotton wool
Grade IV : papillodema
55 - Which layer remove excessive water from cornea
A. stroma
B. tear film
C. endothelium
D. epithelium
Answer : c
56 - 20 y.o had trauma to left eye, he has corneal ulcer, photophobia, and pain, how do you manage?
A. debridement with Burr and systemic antibiotics.
B. cotton swab debridement and local steroid.
C. irrigation, systemic antibiotics and cycloplegia.
D. local antibiotics, cycloplegia and referral.
Answer : D

ADDED QUESTION: ( need to be more checked )


1.
Bacterial keratitis occurs with contact lenses mc organism: (repeated)
a. S.aurea
b. pseudomonas areginosa
Answer: B
2. Man got new glass , after one week he can not see well by the new glass ,
after examination change the glass and referred to ophthalmologist.. What is
the cause of change the old glass ?
1-Glucose in lens
2-Cataract
3-Glaucoma
Answer:3
open angle GLaucoma should be suspected in pt > 35 y who need frequent lens change and have mil headach visual
disturbance and impaired adapation to darkness. reference : step 2 Ck

392

3. What is responsible for clear the cornea from water ? (Repeated)


1- epithelial
2- endothelium
3- stroma
Answer: b
4. 2 q about the eye parasitic infection one is the diagnosis ( acanthamoeba ) and the other q what other
manifestation for the disease
Answer : skin manifestations : ulcers, nodules, or subcutaneous abscesses.
CNS : meningeoencephalitis ;Mental status changes, Seizures, Hemiparesis, Fever, Headache, Meningismus, Visual
disturbances, Ataxia, Nausea and vomiting, Hallucinations, Personality change, Photophobia, Sleep disturbances.
Disseminated disease without CNS involvement may manifest as skin lesions, sinusitis, pneumonitis, or a combination.
Reference : medscape
5. Sudden eye swelling redness and pain, hazy cornea :
Answer:

Acute angle closure glucoma

Reference ; introduction to clinical emergency medicine text book. / uptodate

6. long case of eye trauma then repair then developed most likely endophthalmitis done enucleation, found in the
uvea lymphocyte and CD4 i think, what's is the cause? (Repeated)
a. cross reactivity
b. activation of lymphocytes
c. antigen release or something?
Answer:
antigen release
7. pt with bilateral eye myopia each 2 month change his glass
a. keratoconus
b. keratomalcia
Answer:A
8. Painless loss of vision in right eye with headache. First thing to do?
A Prednisolone
B CT
Answer
Prednisolone
9. commonest cause of iris neovascularization is :
A DM
B HTN
Answer A

10. pic of glaucomatous cupping


Answer:
normally 0,2- 0,4 Physiological cupping =0.5
But could be pathological if there is abnormalities by the other investigation ot there is difference btw two eyes
Pathological >0.5

393

12. A young male complaining of redness and discomfort in his eyes during the exam time, his visual acuity is 6/6
in both eyes unaided. Both the anterior and posterior chambers are normal. What is the most likely diagnosis:
A.Myopia
B.Anisometropia
C.Astigmatism
D.Hyperopia
Answer: D?
13. Dilated rt pupil and constricted Lt while light on rt
A.
Rt optic
B.
Lt optic
C.
Rt oculomotor
D.
Lt oculomotor
answer : C
Afferent ---> optic nerve ----> when light shone on 1 eye both eyes are stimulated
Efferent ---> oculomotor nerve ---> constrict the ipsilateral pupil
14. you did examination on elderly Hypertensive man you find increase in cupping he is not complaining of
anything what is the cause of cupping ?
A.
Acute angle glaucoma
B.
HTN-dm
C.
Retinal detach .
Answer: B?
Acute angle glaucoma ----> painful
retinal detach ------> curtain like vision loss or floaters
15. Case of chronic eye irritation and Watery secretion + eye Enotropia Enotropia
Answer:
conjunctivitis
16. Case of eye injury by fingernails Which stain is used ( blue eye stain)?
Answer:
fingernails it is corneal abrasion
de-epithelized area stains with fluorescein
Ref: toronto note 2015
17. Case of patient with progressive painless visual loss -8 6/18 is it
Physiological
Pathological
Curvature Index alized area sta
Answer:
pathological which could be keratoconus
18. Type of gonorrhea discharge in eye:
A. Purulent
B. Mucopurulent
Answer: A
Hyperacute bacterial conjunctivitis Neisseria species, particularly N. gonorrhoeae, can cause a hyperacute bacterial
conjunctivitis that is severe and sight-threatening, requiring immediate ophthalmologic referral [8]. The organism is
usually transmitted from the genitalia to the hands and then to the eyes. Concurrent urethritis is typically present.
The eye infection is characterized by a profuse purulent discharge present within 12 hours of inoculation [9]; the amount
of discharge is striking. Other symptoms are rapidly progressive and include redness, irritation, and tenderness to
palpation. There is typically marked chemosis, lid swelling, and tender preauricular adenopathy. Gram negative
diplococci can be identified on Gram stain of the discharge.
These patients require hospitalization for systemic and topical therapy and for monitoring of the ocular component.
Keratitis and perforation can occur.
uptodate

394

19. Pic about periorbital swelling what other symptoms could the pt has? Image result for eyelid swelling discharge
- vision
- uveitis
Answer:a
20. Q about ttt of corneal ulcer ?
A.
Antibiotics
B.
Antifungal
Answer:
Treatment
urgent referral to ophthalmology
culture prior to treatment
topical antibiotics every hour
must treat vigorously to avoid complications
toronto note
21. when to refer conjunctivitis to ophthalmologist :
a- photophobia
b- bilateral conjunctivitis
c- mucopurelent discharge
Answer: a
Ophthalmologist referral is indicated for any pt with conjunctivitis presented with one of the following Sx:
eye tenderness
Difficult seeing clearly
Difficult keeping the eyes open or sensitivity to light
Severe headache with nausea
Recent trauma to the eye
Use of contact lenses
Reference : uptodate
http://www.uptodate.com/contents/conjunctivitis-pinkeye-beyond-thebasics?source=outline_link&view=text&anchor=H8#H8

395

There were also missing questions about the following:


Orbital cellulitis.

th

Reference: Lecture note on ophthalmology, 9 edition


Proliferative diabetic retinopathy.

th

Reference: Lecture note on ophthalmology, 9 edition (P:139)

396

ENT

397

1- What is the most common site of malignancy in paranasal sinus?


a.
Maxillary
Answer: A
Reference: 3rd Edition UQU > ENT > Q 92.
2- Child complaining of painless unilateral hearing loss. On examination, tympanic membrane was opaque.
a.
serous otitis media
Answer: A
3- 40 years old male, complaining of sudden dizziness attack for 30-40 min, associated with nausea, vomiting,
unilateral tinnitus and hearing loss.
a.
Acoustic neuroma
b.
Neuritis
c.
Meniere disease
Answer: C
Menieres disease: a cause of recurrent vertigo with auditory symptoms more common among females.
Hx/PE: Presents with recurrent episodes of severe vertigo, hearing loss, tinnitus, or ear fullness, often lasting hours to
days. Nausea and vomiting are typical. Patients progressively lose low-frequency hearing over years and may become
deaf on the affected side.
Reference: 3rd Edition UQU > ENT > Q 104.
4- 40 years old man presents with decreased hearing but his own sound feels louder than before to him. What is
diagnosis?
a.
presbycusis
b.
otitis media
Answer: B, but if there is osteosclerosis it will be more aprroptiate.
5- Post tonsillectomy patient has loss of taste of the posterior 1/3 of tongue. What is the nerve injured?
a.
Glossopharyngeal nerve injury
Answer: A
6- In presbycusis (SNH loss). Which structure in the inner ear gets affected?
a.
Hair Cells.
Answer: A
7- Intracranial growth compressing the maxillary artery. Which one of the following is affected?
Answer: ?
The maxillary artery can be defined as one of the continuations of the external carotid artery, and distributes the blood
flow to the upper (maxilla) and lower (mandible) jaw bones, deep facial areas, cerebral dura mater and the nasal cavity.
Hence it is considered a blood vessel which supports both hard and soft tissues in the maxillofacial region.
NB Know the anatomy and branches.
8- Old guy cant tolerate loud noises which nerve is affected?
Answer: ?
Facial nerve due to loss of protective muscle (stapedius)
9- A patient with chronic tonsillitis complaining of fever, chills, left throat pain and uvula deviation. What is the
diagnosis?
Answer: ?
Peritonsillar abscess

398

10- What is the most sensitive part to the Linear Acceleration?


a.
Semicircular Canal.
b.
Organ of corti.
c.
Utricle.
d.
Saccule.

Answer: C&D?
The otolith organs sense gravity and linear acceleration such as from due to initiation of movement in a straight line.
Persons or animals without otolith organs are imbalanced.. A set of hair cells are coupled to a mass of stones. When the
stones accelerate, with respect to the hairs, they exert a shearing force on the hairs. This force is detected by the hair
cells and sent to the brain via branches of the vestibular nerve. The utricle sends input to the brain via the superior
division of the nerve, and the saccule, via the inferior division. There is considerably more complexity to the
organization of the utricle and saccule, including different types of hair cells and detail to the sensory macule (patch of
sensory cells) that we have omitted.
The otolithic organs sense motion according to their orientation. The utricle is largely horizontal in the head, and largely
registers accelerations acting in the horizontal plane of the head (called the axial plane by radiologists). The saccule is
largely vertical, actually parasagittal, in the head, and registers accelerations in the vertical plane (called parasaggital or
coronal plane).
otolithic organs= the saccule and utricle >> for linear acceleration
Reference: http://www.tchain.com/otoneurology/disorders/bppv/otoliths.html + medscape
11- Man complaining of ear pain. On examination he feel pain when moving ear pinna and there is erythema of
auditory canal and normal tympanic membrane. What is the most likely diagnosis ?
a.
Otitis Externa.
b.
Otitis Media.
c.
Squamous cell carcinoma.
Answer: A
12- Picture of tonsillitis, what is the best drug for treatment?
a.
Ceftriaxone.
b.
Acyclovir.
c.
Ampicillin.
d.
Doxycycline.

Answer:?
The best is penicillin or amoxicillin; penicillin allergic patient are treated with Cephalexin if the reaction only rash , if
allergy is anaphylactic use clindamycin or a macrolides
Reference: Master the Board step 2 CK
13- 60 year old male with unilateral parotid swelling tender but no facial nerve dysfunction?
Answer: ?
If there is fever along with the tenderness the answer would be bacterial parotitis , if no systematic Sx usually there will
be no tenderness where the answer would be Benign pleomorphic adenoma , facial nerve involvement would rise the
suspicion toward malignancy .
Reference: Medscape

399

14- 17 years old male presented with hearing loss in the left ear (picture of conductive hearing loss Audiogram).
What is your diagnosis?
a.
Presbycusis
b.
Otosclerosis
c.
Interosseous
d.
Otitis media
Answer: B
Otosclerosis is traditionally diagnosed by characteristic clinical findings, which include progressive conductive hearing
loss, a normal tympanic membrane, and no evidence of middle ear inflammation.

15 - sore throat, palpable neck mass, hoarseness voice, hearing loss Ask about tx?
a. surgery and radiotherapy
(I thought about nasopharyngeal ca)
Answer: I dont know
16 - "Otitis media case + child with acute ear pain , redness and bulging of tympanic membrane, whats the Dx
a.
otitis media
Answer:
Clinical Features:
triad of otalgia, fever (especially in younger children), and conductive hearing loss
rarely tinnitus, vertigo, and/or facial nerve paralysis
otorrhea if tympanic membrane perforated
infants/toddlers ear-tugging (this alone is not a good indicator of pathology) hearing loss, balance disturbances
(rare) irritable, poor sleeping vomiting and diarrhea anorexia
otoscopy of TM hyperemia bulging, pus may be seen behind TM loss of landmarks: handle and long process of
malleus not visible
Ref:Toronto note

17- this a case of Otitis Media with Effusion


child with middle ear fluid and bulging in ear drum the RX?
tymbanosomy
grommet
Answer: surgery: myringotomy ventilation tubes adenoidectomy (if enlarged or on insertion of second set of tubes
after first set falls out)
ventilation tubes to equalize pressure and drain ear
Ref: Toronto note
18 - about OM ttt straight forward?
Amoxicillin
Answer: Amoxicillin
Antimicrobial agents for AOM 1st line treatment (no penicillin allergy) amoxicillin: 75 mg/kg/d to 90 mg/kg/d divided
3x/d
Ref: Toronto note

400

19 - patient with nasal sx. What is the "best" treatment for it ?


A.steroid
B.antihistamin.
C. decongestants
D.ab
Answer: A (im sure because I got 100% in ENT)
1. Prevention with avoidance of the precipitating allergen:
Close the windows and use air conditioning to avoid pollen.
Get rid of animals to which the patient is allergic.
Cover mattresses and pillows.
Use air purifiers and dust filters.
2. Intranasal corticosteroid sprays
3. Antihistamines: loratidine, clemastine, fexofenadine, brompheniramine
4. Intranasal anticholinergic medications: ipratropium
5. Desensitization to allergens that cannot be avoided
Ref: Master the boar

ADDED QUESTION: ( need to be more checked )


1- Nasal obstruction + rhinorrhea + pale swelling
Answer: Allergic Rhinitis
2- PT complaining of euphonia & doing laryngoscope it was normal & ask him to
coughing he is cough what diagnosis?
A Functional euphonia
Answer:?
3- PT have sleep apnea & on pacemaker not responsing to CPAP
(continuous positive airway pressure) what you do ?
NOSE SURGERY
CHEST SURGEY
DO CPAP AGAIN
Answer:?
4- Most sensitive test for maxillary sinus:
A.
CT
B.
X-ray
Answer: A
5- 55 years with unilateral nasal chronic obstruction and epistaxis what you suspect
A. polyp
B. malignancy
C. fibroangioma
Answer: B

401

6- 18 years old ear examination is normal and there is audiogram picture,


what's the diagnosis?
A presbycusis
B Otosclerosis
C otitis media
Answer:?
7- How to examine child ear? Pull pinna?
A Inf backwards
B Sup backward
C Inf forward
D Sup forward
Answer A?
B for adult
8- 24-Most common cause of deafness in children is
A-recurrent Otitis media
Answer: A
9- What is the artery supply posterior inferior nasal septum ???
Answer: sphenopalatine artery
10- A 17 years old male complains of sore throat, cough, fever, ear pain. On examination you found hemorrhagic
vesicles over his tympanic membrane, what's the causative organism:
A.
Pseudomonas
B.
Strept. pyogenes
Answer:
This is called Myringitis
(particularly Streptococcus pneumoniae), or mycoplasma otitis media.
http://www.merckmanuals.com/home/ear,-nose,-and-throat-disorders/middle-ear-disorders/infectious-myringitis
11- Otitis media pt. Not taking antibiotic probably then after week came back by tenderness and swelling behind
the ear,dx?
a.
Acute mastoiditis
Answer: A

12- Thick White plaques in throat associated with gingivitis , dx?


Answer: ????

402

13- Patient came to ER with inability to breath from one nostrils and on examination the nostril is edematous and
swollen What's the initial management?
A) Local decongestant
B) Steroid
Answer: B

http://www.aafp.org/afp/2010/0615/p1440.html
14- Q about OM ttt?
- amoxicillin
Answer: A
15- Elderly patient with sudden onset hearing loss, loss of balance, and loss of eye blinking, where is the tumor?
A.
Foramen Ovale
B.
Foramen Secundum
C.
Acoustic
Answer: C ; Acoustic neuroma or called Schwannoma
http://www.nidcd.nih.gov/health/hearing/pages/acoustic_neuroma.aspx
16- pt with recurrent episodes of vertigo , she feels of ear fullness , SNHL and nausea , what is the dx?
A. Menere's diseases
B. Benign Progressive vertigo
Answer: A
http://www.nidcd.nih.gov/health/balance/pages/meniere.aspx
A.
B.

17- which of the following medication causes vertigo ?


Ethambutol
Streptomycin

Answer: B
http://www.drugs.com/sfx/streptomycin-side-effects.html

403

18scenario about adult male complaining of severe ear pain with discharge inflamed tympanic membrane (
missed some points) What is the Dx?
A.
otitis externa
B.
cholesteatoma
Answer: B
The majority (98%) of patients with cholesteatoma have ear discharge or hearing loss or both in the affected ear.
https://en.wikipedia.org/wiki/Cholesteatoma
O.E is not associated with tympanic membrane infection

404

Basic Science

405

Embryology
1- Failure of joining renal collecting duct with distal tubules.
Answer: ?
Multicystic Renal Dysplasia?
Reference: http://emedicine.medscape.com/article/982560-overview#showall
Polycystic Kidneys? Reference: Maternal, Fetal, & Neonatal Physiology

2- What is the origin of smooth part of right atrium?


a.
Right sinus venosus
b.
Left sinus venosus
Answer: A
Sinus venosus (R horn): smooth part of right atrium (sinus venarum) and the "valve" of the superior vena cava. The
sino-atrial node.
Sinus venosus (L horn): coronary sinus, valve of coronary sinus.
Reference: http://www.nervenet.org/embryo/hderiv.html
3- Lung Embryology (about alveolar?)
Answer: ?
Development of the lung can be divided into two phases,
lung growth (structural development) and lung
maturation (functional development). Lung growth can
be influenced by a host of physical factors. Lung
maturation and the achievement of functionality is
primarily a biochemical process and is under the control
of a number of different hormones. Lung growth
proceeds through gestation. There is progressive
branching of the airways and finally development of
alveolar spaces capable of gas exchange in the last
trimester. The surfactant system, composed of
phospholipids that decrease surface tension within the
alveoli and prevent alveolar collapse during exhalation,
develops in the last trimester, and reaches maturity by
approximately 36 weeks. Lung growth continues after
birth as alveolar number continues to increase. The end
result of the development of the lung is an organ with a
tremendously large surface area that is approximately
50-100 m2, capable of exchanging oxygen and carbon
dioxide across a very thin membrane.
Reference: http://www.columbia.edu/itc/hs/medical/humandev/2004/Chpt12-LungDev.pdf
4- Non keratinized stratified squamous epithelium:
a. Small bowel
b. Large bowel
c. Esophagus
Answer: C

406

Histology
1- What is the strongest part of the vein?
a.
Tunica intima
b.
Tunica media
c.
Tunica adventitia
d.
Serosa (Not sure if it was there)
Answer: C
2- Whats the origin of juxtaglomerular apparatus?
a.
Macula densa cell
b.
Bowman capsule
Answer: A
The juxtaglomerular apparatus consists of three cells:

The macula densa, a part of the distal convoluted tubule of the same nephron.

Juxtaglomerular cells, which secrete renin.

Extraglomerular mesangial cells.

3- In which phase the endothelial lining of alveoli came in contact with blood vessels ?
a.
Alveolar phase.
b.
Canalicular phase.
c.
Pseudoglandular phase.
d.
Saccular phase.
Answer: D
Saccular phase (terminal sac period - 26 weeks to birth)
4- calcitonin secreted from ?
A Parafollicular
Answer: parafollicular cell ( C cell )

407

Physiology
1 - the corrected sodium of a patient is 138, what is the anion gap?(no Additional information)
Answer: ?
2 ABG question ( compensated vs non compensated, respiratory acidosis vs metabolic acidosis )
Answer:

3 - history of metabolic acidosis and lab showing decrease of ph and Hco3 how to compensate ?
A-co2 wash out
Answer:A
4- Which system or organ will work in stress ?
A.repiratory
B.renal
C.sympathatic
D. parasympathetic
Answer:Sympathetic system

Microbiology
1-parasite in soil contamination:
A tenia saginatm
B ascaris-bancrofti ..
Answer b
2-Gram ve bacilli, lactose fermenting & non lactose fermenting in urine culture?
Answer E. Coli?
3- A child with chronic diarrhea, endoscopy with biopsy showed Sickled shaped parasite. Whats the dx?
A. Entamyba histolytica
B. Giardia
C. Ancylostoma
Answer: B

408

Anatomy
1- Dorsalis pedis pulse location?
a.
Beside extensor hallucis longus.
Answer: A (lateral)
2- Internal carotid artery branches.
Answer: ?
http://www.bartleby.com/107/146.html
3- Which part of the Spermatic Cord originate from Internal Oblique Abdominal Muscle ?
a. Internal Spermatic Sheath.
b. External Spermatic Sheath
c. Tunica Vaginalis.
d. Cremaster Muscle.
Answer: D
4- The radial pulse can be palpated lateral to which tendon:
a.
flexor carpi ulnaris
b.
flexor carpi radialis
c.
flexor digitorum profundus
Answer: B
Proximally, the radial pulse is best appreciated within the inverted V formed by the biceps tendon laterally and the
bicipital aponeurosis medially. This inverted V also defines the site where the radial recurrent artery (RRA) branches
off from the RA. Distally, the RA can be palpated between the radial styloid laterally and the tendon of the flexor carpi
radialis medially. Reference: Annals of Cardiothoracic Surgery.

5- Posterior vagal trunk supplies:


a.
esophagus
b.
jejunum
c.
descending colon
Answer: A
The posterior vagal trunk is a branch of the vagus nerve which contributes to the esophageal plexus. It consists primarily
of fibers from the right vagus, but also contains a few fibers from the left vagus. Reference: Wikipedia.
6- What is the position of the femoral vein to the artery:
a.
Medial to anterior
b.
Lateral
c.
Posterior
d.
Anterior
Answer: A

409

7- Which of the following muscles initiate unlocking of the knee during walking?
a.
Plantaris
b.
Tibialis anterior
c.
Sartorius
d.
Popliteus
Answer: D
The popliteus muscle in the leg is used for unlocking the knees during walking/standing by laterally rotating the femur on
the tibia during a closed chain movement. Reference: Clinical Kinesiology and Anatomy.
8- Which of the following nerves is responsible for adduction of fingers?
Answer:
Ulnar nerve. Reference: Textbook of Anatomy - Upper Limb and Thorax
9- Peroneal artery is a branch of which artery?
a.
femoral
b.
popliteal
Answer: B
Popliteal artery > Posterior tibial artery > Peroneal artery.
10- Which nerve supplies the frontal belly of the occipitofrontalis muscle?
a.
Temporal branch of the facial nerve?
Answer: A
Reference: Endoscopic Plastic Surgery.
11- Whats the narrowest part of the urethra?
a.
Prostatic urethra
b.
Membranous urethra
Answer: B
12- Where does the lymph drain from the Testicles?
a.
Superficial inguinal
b.
Deep inguinal
c.
Para-aortic
d.
Thoracic duct
Answer: C

410

13- What is the type of joint between the bodies of the vertebra?
a.
Synovial
b.
Cartilaginous
c.
Suture
Answer: B
14- What is the blood supply to posterior compartment of leg?

a.
b.
c.

Tibial
Common fibular
Superficial fibular

Answer: A
15- Which nerve is responsible for the gag reflex:
a.
Glossopharyngeal
Answer: A
Afferent: Glossopharyngeal IX.
Efferent: Vagus X
16- Which nerve is responsible for tongue movement:
a.
Hypoglossal
Answer: A
17- Patient with medial loss of sensation of one and half finger (Palmar and dorsal surfaces). Which nerve is
affected?
a.
Median N
b.
Ulnar N
c.
Axillary N
d.
Radial N
Answer: B
18- Muscle passing through the lesser sciatic foramen:
Answer: ?
The following pass through the foramen:
- The tendon of the Obturator internus
- Internal pudendal vessels
- Pudendal nerve
- Nerve to the obturator internus
Referance: https://en.wikipedia.org/wiki/Lesser_sciatic_foramen
19- In which scalp layer does vessels run:
- Connective tissue (superficial fascia)

20 - Origin of gluteal artery ( scenario: ischemia in gluteaus area then ask for origin of gluteal artery)
a. internal iliac artery
b. Read about:
c. Internal iliac branches
d. External iliac branches
Answer : C-Internal Iliac artery
Ref: greys Anatomy

411

21 - Which ligament pass inside inguinal canal


A) broad ligament
B ) round ligament
C) uterosacral ligament
D) transverse cervical ligament
Answer: B: round ligament
The inguinal canal is a tubular structure that runs inferomedially and contains the spermatic cord in males and the round
ligament in females
Ref: medscape

22 - inferior alveolar n from facial nerve injury, during dental procedure?


What's the manifestation?
a. read about its function
Answer:
The main symptoms for the inferior alveolar nerve injury are: sensory paralysis of the lower lip on the affected side, the
mental region and the gingivae; stiffness, persistent pain; neuropathic pain such as allodynia; and pain and discomfort
with occlusion.
Ref:http://www.aqb.jp/english/file/ClinicalpracticePart3-6.pdf

23 - Surgery in posterior triangle then develop loss of sensation in lower mandible ipsilateral which nerve is
affected
A- lesser occipital nerve
B- grater occipital never
C- great auricle nerve
D- Dont remembers
Answer: C

24 - Knee trauma then can not fully extend the knee so which muscle is affected
A-Quadriceps femoris
B-Biceps femoris
Others I don't remember
Answer: Extensors: quadriceps femoris
Ref: Greys anatomy
25 - knee dermatome?
L1:L2
L3:L4
L4:L5
L5:S1
Answer: L3:L4
Ref: medscape
26 - Gluteal muscle what the nerve intervention :
Answer:
Gluteus Maximus >> inferior gluteal nerve .
Gluteus Medius >> Superior gluteal nerve.
Gluteus monimus >> Superior gluteal nerve.
Ref:http://teachmeanatomy.info/lower-limb/muscles/gluteal-region/
27 - brain tumor compressed third portion of maxillary artery. Which of following artery will affect it:
A-Temporal artery
B-sphenopalatine artery
Answer:Sphenopalatine artery
Ref :Greys Anatomy

28- Pt complain of lower back pain after lifting heavy objects


Answer: ?

412

29- Question about the thyroid gland and fascia related..


A-deep cervical fascia **
Answer:A
30- which muscle pass through lesser sciatic?
A.pectenious m.
B....
C...
D...
Answer: The following pass through the foramen:

the tendon of the Obturator internus


internal pudendal vessels
pudendal nerve
nerve to the obturator internus

31Patient has trouble dorsi flexing his ankle joint which structure is affected (1 question nerve and 1 muscl
e)
Answer: muscles: 1. Ant tibialis. 2. Extensor digitorum longus. 3. Extensor hallucis longus 4. Peronus tertius
Nerve affected: Deep peroneal n.

32- Which of the following is not supplied by the inf. Mesenteric


artery:
( splenic flexure, descending colon, transverse, cecum)
Answer:Cecum
Ref:greys anatomy book

33- artery in prepare inguinal hernia


Answer: testicular, cremasteric, and deferential A

34- 3rd layer of scalp


Answer: Epicranial Aponeurosis
The scalp consists of 5 layers (seen in the image below):
1- the skin,
2- connective tissue,
3- epicranial aponeurosis,
4- loose areolar tissue
5-pericranium.
Ref: Medscape
35- tabia move for femur which ligament
Answer: ACL?
36- medial 1/3 loss of sensation in hand, where is the deformity
Answer: ulnar n
37- unstable gate which artery response
Answer: if the gait hes talking about is waddling gluteal medius and minimus affected superior gluteal A.

38- which ligament will be tearing during LP


Answer: Interspinous ligament

413

1- Skin
2- Facia and SC fat
3- Surpaspinous ligament
4- Interspinous ligament
5- Ligamentum flavum
6- Epidural space and fat (epidural anesthesia needle stops here)
7- Dura
39- accessory nerve present in which triangle?
A. Posterior
B.Mental
C, Mandibular
D. Muscular
Answer:
40-

Posterior cervical triangle

Phranic nerve paralysis:


A. increased heart rate
answer:
Clinical manifestations Patients with unilateral diaphragmatic paralysis are usually asymptomatic at rest,
but may have exertional dyspnea and decreased exercise performance [9,14]. However, patients with
underlying or intercurrent lung disease may experience dyspnea at rest. Orthopnea can also occur, but is
not as intense as with bilateral diaphragmatic paralysis [7,15]. Unilateral diaphragm paralysis may also
be associated with sleep-disordered breathing during rapid eye movement sleep
Ref :upodate

41- . during laparoscopic surgery of inguinal hernia you find artery superficial going upward ?
A. Inferior epigastric artery.
answer:A
41-

Popliteal artery branch of?


A. Peroneal
answer: Femoral external illiac common iliac abdominal aorta
42MVA diagnosed with fracture of base of skull and injury to nerve pass through jugular foremen
which muscle affected?
Answer:
-9th cranial n >>Motor: Innervates the stylopharyngeus muscle of the pharynx.
-10th cranial n >> Motor: Provides motor innervation to the majority of the muscles of the pharynx, soft palate and
larynx.
-11th cranial n >>motor :the sternocleidomastoid and trapezius.
43. Fraction of head of tibia with drop feet which nerve?
A Deep peroneal
B Common poronial
Answer: deep peroneal

44. injury to jugular Forman which structure will not be affected all of the are muscle :
A. sternocleidomastoid
B. sphenopalatine
C. 2 more option

414

Answer:
-9th cranial n >>Motor: Innervates the stylopharyngeus muscle of the pharynx.
-10th cranial n >> Motor: Provides motor innervation to the majority of the muscles of the pharynx, soft palate and
larynx.
-11th cranial n >>motor :the sternocleidomastoid and trapezius.
45.

Injury to temporal area and superficial temporal artery bleed which layer of the skull where ve
ssels are ?
1. peri cranial
2. epi cranial aponeurosis
Answer: Subgaleal hemorrhage? galeal aponeurosis (epicarnial aponeurosis)
46. upper outer mass in breast skin look like bakkering what is the cause ?
1 pectoralis major
2 crourp ligament
Answer: Cooperligment
47. Pt after RTA , no abduction and lateral rotation of the arm.. . What is the origin of the affected nerve ?
1- medial plexus
2- lateral plexus
3- lower plexus
4- root
Answer: Negative Apleys scratch test - Suprascapular nerve (innervate supraspinatus and infraspinatus) and it is a
branch of the upper trunk.
Ref: Snells Clinical Anatomy

48. what is the level of cardiac notch?


A 3rd
B 4th
C 5th,
D 6th rip
Answer: B. 4th rib. ref:Handbook of Cardiac Anatomy, Physiology, and Devices, page 58
49. where do feel the pulse of factual artery?
Answer: located on the mandible (lower jawbone) on a line with the corners of the
mouth (wiki)
50. Action of Anterior compartment of the forearm muscle :
Answer: A a flex rest and flex finger

52. Organ that originate from midgut and hindgut?


A- duodenum
Answer : transverse colon

415

53. Pt with right arm numbness and tingling in thumb and index fingers, symptoms increased with hands raised
up. The cause:
a. Thoracic inlet.
b. Thrombus
Answer: A??
54. trauma to the fibular head what is the nerve injured?
A Common peroneal nerve
Answer: common peroneal n
55. from where peroneal artery come
Answer: post. Tibial a. > popliteal a > femoral a > external iliac
56.After surgery, pt loss of sensation of medial thigh, which nerve is affected?
A Femoral
B Obturator
Answer: obturator ( L2 - L4 )
57.Which ligament comes before you reach epidural space?
A Flavum
Answer A

416

58. Indirect inguinal hernia weakness occur in which muscle


A transverse facialis
Answer: Probably question meant direct inguinal hernia?
59.stab wound lateral to sternocleidomastoid. Patient unable to do adduction of the shoulder and elevation over
his head . What is the narve injured
A- long thoracic
Answer : long thoracic
60. fecal incontinence which level is affected
A-above c2
B- below c2
C-above t12
D-bolow t12
Answer: D. Below T12
61.A 24 years old male was jogging and suddenly he had a muscle strain what is the most commonly affected
muscle?
Answer: calf muscle

417

418

Biochemistry
1- How does the protein enter the peroxisomes?
a.
Folded with the help of C-terminal
b.
Unfolded with the help of T-terminal
c.
Folded with the help of T-terminal
d.
Unfolded with the help of C-terminal
Answer: A
Peroxisomal proteins are fully folded in the cytoplasm and enter the organelle in folded form. The most common signal
sequence which directs proteins to peroxisomes is a C-terminal Ser-Lys-Leu tripeptide. Reference: Biochemistry and
Molecular Biology Textbook.
http://global.oup.com/uk/orc/biosciences/molbiol/snape_biochemistry5e/student/mcqs/ch27/ Q5

2- How does the body get rid of Lactic acidosis?


a.
Excretion by kidney
b.
Liver metabolism
Answer: B
In basic terms, lactic acid is the normal endpoint of the anaerobic breakdown of glucose in the tissues. The lactate exits
the cells and is transported to the liver, where it is oxidized back to glucose.
Reference: http://emedicine.medscape.com/article/167027-overview
3 - from rest muscle status to sudden contractions. What change in cellular level.
Answer:

419

4- which of the following made in nucleolus:


DNA
mRNA
rRNA
Answer: rRNA
5- Protein after transfer sequence what is last destination
A. endoplasmic retuculum **
B. Cytosol
Answer:A
6- ALP which enzyme confirms that elevation from liver
Answer: gamma-glutamyl transpeptidase (GGT) or 5'-nucleotidase (5'-NT),
Ref: http://www.liver.ca/liver-disease/diagnosing-liver-disease/liver-function-tests.aspx

7- Mendles Laws:
Answer: MENDEL'S LAWS OF INHERITANCE Mendel noted that many physical characteristics of seeds and
flowers were transmitted from parental strains to offspring in a predictable and reproducible manner. From these
observations, Mendel proposed that traits manifest through the joint effects of two paired elements (or genes), with each
parent contributing one element (allele) to each offspring at conception
Law of segregation The law of segregation states that paired parental copies of genes are separated from each other
during gamete formation, with each copy (ie, allele) segregating into separate gametes. This process of gamete formation
is now known as meiosis. (See "Principles of molecular genetics", section on 'Meiosis and sustained genetic diversity'.)
Law of independent assortment The law of independent assortment states that genes segregate into gametes
independent of other genes such that the allelic status at one locus does not determine segregation of alleles at other loci.
This is only true for genes that are not linked to each other.
Law of dominance The law of dominance distinguishes dominant, recessive, and co-dominant traits.
A trait is considered dominant when it is manifest in heterozygous carriers (termed heterozygotes).
A trait is considered recessive when it is observed only among those homozygous for the disease allele or with two
different mutations in the same disease-causing gene (compound heterozygotes). Heterozygotes for a recessive trait
(carriers) are typically phenotypically indistinguishable from non-carrier homozygotes. In X-linked disorders, however,
carrier females can sometimes be affected.
A co-dominant or semi-dominant trait is one where both alleles contribute equally to phenotypic expression.
Heterozygotes manifest an intermediate phenotype between those of the two homozygous classes.
Ref : uptodate

8- Athletic come for check up all thing normal except Xanthelasma on achilles tendon and cholesterol?
A.Ldl resptor
B. Apo ll
C. Apo c
Answer: A

9- organic phosphate , found in or effect of :


A. liver
B. kidney
C. lung
Answer: C
Mnemonic devices used to remember the muscarinic effects of organophosphates are SLUDGE (salivation, lacrimation,
urination, diarrhea, GI upset, emesis) and DUMBELS (diaphoresis and diarrhea; urination; miosis; bradycardia,
bronchospasm, bronchorrhea; emesis; excess lacrimation; and salivation). Muscarinic effects by organ system include the
following:
Cardiovascular - Bradycardia, hypotension

420

Respiratory - Rhinorrhea, bronchorrhea, bronchospasm, cough, severe respiratory distress


Gastrointestinal - Hypersalivation, nausea and vomiting, abdominal pain, diarrhea, fecal incontinence
Genitourinary - Incontinence
Ocular - Blurred vision, miosis
Glands - Increased lacrimation, diaphoresis
Ref : Medscape
10. Bond between DNA and tRNA:
A Covalent
B Ionic
C Hydrophiclic
Answer: A
11. Which is acidophilic:
A Lace
Answer:?

Immunology
1- (long scenario) man traveled for business, went to his flat to relax, second day he noticed mold on his flat, he
developed rash all over his body. We took sample of mold and injected him, inflammation occurs after 30 minutes.
What does it indicate?
a.
Immediate immunity.
b.
Late immunity.

2- Someone with cancer, took cells from him inject with cancer, then re inject to patient so they can be antigen for
cancer. What type of immunity?
A Passive
B Active immunotherapy.
Answer:
3- man done vasectomy, change his mind want to reproduce again, they found antisperm antibodies what is the
cause ?
A antigen release or something like that?
B cross reactivity with viral infection
C inappropriate response of MHC II to antigen presenting

421

Answer: Anti sperm antigen has been described as three immunoglobulin isotopes (IgG, IgA, IgM) each of which targets
different part of the spermatozoa. If more than 10% of the sperm are bound to anti-sperm antibodies (ASA), then
infertility is suspected. The blood-testis barrier separates the immune system and the developing spermatozoa. The tight
junction between the Sertoli cells form the blood-testis barrier but it is usually breached by physiological leakage. Not all
sperms are protected by the barrier because spermatogonia and early spermatocytes are located below the junction. They
are protected by other means like immunologic tolerance and immunomodulation.
Infertility after anti-sperm antibody binding can be caused by autoagglutination, sperm cytotoxicity, blockage of spermovum interaction, and inadequate motility. Each presents itself depending on the binding site of ASA.

Pathology:
1- Nephro about minimal change GN..u should see in the biopsy ?
Answer: LM: normal glomeruli
2- Fragile x syndrome associated with
Answer: Large protruding ear, long face, high arched palate, hyperextensible finger joint, double joint thumbs, flat feet,
soft skin, post pubescent macroorchidism , hypotonia, language deficit, autism, ADHA, strabismus, seizures, fertility.

Pharmacology
1- Mode of action of aspirin:
a.
inhibit platelet aggregation
b.
acts on antithrombin
Answer: A
Aspirin inhibits platelet aggregation by irreversibly inactivating cyclooxygenase, a key enzyme in platelet prostaglandin
metabolism. Reference: Pubmed.
2- Clonidine decrease the effect of which of the following drugs?
Answer: ?
Clonidine may decrease the effectiveness of levodopa to control the symptoms of Parkinson disease.

3- What is the mechanism of action of denosumab anti-resorptive properties:


a.
cases osteoclastic apoptosis
b.
inhibit RANKL
Answer: B
Denosumab is a fully human monoclonal antibody that binds the cytokine RANKL (receptor activator of NFB ligand),
an essential factor initiating bone turnover. RANKL inhibition blocks osteoclast maturation, function and survival, thus
reducing bone resorption.
In contrast, bisphosphonates bind bone mineral, where they are absorbed by mature osteoclasts, inducing osteoclast
apoptosis and suppressing resorption. These differences in mechanism influence both the onset and reversibility of
treatment. Reference: Pubmed.
4- What is the side effect of thalidomide in pregnancy?
Answer: ?
Thalidomide can cause embryofetal harm when given to a pregnant female; it is a human teratogen. The use of
thalidomide in humans has produced a high rate of severe and life-threatening birth defects such as phocomelia, amelia,
hypoplasticity of the bones, absence of bones, external ear abnormalities (including anotia, micropinna, small or absent
external auditory canals), facial palsy, eye abnormalities (anophthalmos, microphthalmos), and congenital heart defects.
5- What is the mechanism of polio vaccine (Both types; oral and IV)?
Answer: ?

422

Infection with poliovirus causes IgM and IgG responses in the blood, but mucosal IgA is vital for blocking infection.
This antibody can neutralize poliovirus in the intestine, the site of primary infection. The live attenuated Sabin
poliovirus vaccine is effective because it elicits a strong mucosal IgA response and provide intestinal immunity. In
contrast, the injected (Salk) polio vaccine does not produce intestinal immunity, and therefore is less effective at
preventing spread of poliovirus in a population. Reference: Pubmed.
6- What is the action of Atropine?
a.
Increase heart rate.
Answer: A
7- What is the mechanism of action of Orlistat?
a.
decrease cholesterol
b.
increase satiety
c.
decrease pancreatic enzymes
Answer: C?
Orlistat: gastrointestinal lipase inhibitor, reduces fat absorption by 30% by inhibition of pancreatic lipase.
Reference: Toronto Notes.

8- Side effect of anticholinergic drugs?


a.
Diarrhea
b.
Excess Salivation
Answer: ?
Dry mouth, urinary retention, constipation, blurred vision, toxic-confusional states. Reference: Toronto Notes.
9- Which of the following medications used in treating osteoporosis and can cause heartburn sensation?
a.
Denosumab.
b.
Risedronate.
c.
Raloxifene.
Answer: B
Reference: Kaplan Pharmacology for Step 1.
10- Regarding mechanism of action of Aspirin on enzymes?
a.
Stimulate cyclooxygenase.
b.
irreversible inactivation of the cyclooxygenase (COX) enzyme
Answer: B
Irreversibly inhibits cyclooxygenase (both COX-1 and COX-2) enzyme by covalent acetylation. Reference: FA STEP1
11. Mechanism of action of inhaled antiviral ?
a.
Inhibit DNA
b.
Inhibit polysaccharide capsule.
c.
Inhibit replication
Answer: C
Inhaled antiviral (Zanamivir; A neuraminidase inhibitors) is used for prevention and treatment of influenza A and B. It
inhibits influenza neuraminidase lead to decrease the release of progeny virus and inhibit virus replication.
Reference: FA STEP1
12- Which one of the following can be given to a patient with beta-lactam-resistant organisms?
a.
Azithromycin
b.
Vancomycin
c.
Gentamicin
Answer: B

423

13- What are the antibiotics can be given safely during breast feeding?
Answer: ?

14- Which medication decreases the effect of OCPs?


a. Antiepileptic
b. Anticoagulant
Answer: A
Antiepileptics (barbiturates, phenytoin, and carbamazepine) are general inducers, they increase the metabolism of OCPs
thereby decreasing their effect. Reference: kaplan Pharmacology for Step 1.
15- Which antidepressant can cause constipation:
Answer: ?
TCAs
16- Aspirin inhibit which product formation?
a.
Thromboxane A2
Answer: A
Aspirin irreversibly inhibits platelet cyclooxygenase 1 preventing the formation of prostaglandin H2, and therefore
thromboxane A2. Reference: https://en.wikipedia.org/wiki/Thromboxane_A2
17- Aspirin can be used as an antipyretic but why does it cause hyperthermia if given in high doses?
Answer: ?
Salicylates also uncouple cellular oxidative phosphorylation, resulting in increased oxygen consumption, increased heat
production, and fever with simultaneous decreased ATP production.
Reference: Critical Care Medicine - Perioperative Management
18Long scenario of patients labs show low hg low platelet with normal reticulocyte which Antibiotic cause t
his ?
A Tetracycline
B cloroampincoal
C cefepime
answer: chloramphenicol
19- treatment of positive oxidase bacteria ?
Answer:
*Antipseudomonal penicillins include ticarcillin (3 g every four hours) and piperacillin (3 g every four hours or 4 g
every six hours).
-*Cephalosporins with antipseudomonal activity include:
Ceftazidime 2 g every eight hours
Cefoperazone 2 g every 12 hours (not available in the United States)

424

Cefepime 2 g every eight hours


*Monobactam:
Aztreonam 2 g every eight hours
*Fluoroquinolones:
Ciprofloxacin
Ref : uptodate
20- Idiopathic anovulatory drug to enhancing ovulation ?
Answer:
Clomiphene is a weak estrogen-like hormone that acts on the hypothalamus, pituitary gland, and ovary to increase levels
of follicle-stimulating hormone (FSH) and luteinizing hormone (LH, which is also important in the process of ovulation).
Ref : uptodate

21- Osteoporosis drug causes retrosternal pain or heart pain :


A Ralixofin
B Densomub
Answer: A
it causes chest pain
Ref : uptodate
Mode of action monotuksit ?
a-Inhibit lukotrine
Answer
Montelukast is a selective leukotriene receptor antagonist that inhibits the cysteinyl leukotriene CysLT 1 receptor. This
activity produces inhibition of the effects of this leukotriene on bronchial smooth muscle resulting in the attenuation of
bronchoconstriction and decreased vascular permeability, mucosal edema, and mucus production.
Ref : uptodate
22- What antihypertensive medication increase uric acid and glucose:
A thiazide
Answer: Thiazide
23-Aspirin side effect
-dry mouth
-constipation
-Diarrhea
Answer C
Ref : uptodate
24-What anticoagulant can distract the clot?
A EnaxbarenB heparinC asprin
Answer : fibrinolytic ( streptokinase, urokinase, tissue plasminogen activator altplase )
25-which medication decrease effect of OCP? A anti epileptic B anticoagulant - Answer: antiepileptic
26-man got a bee sting then his wife trying look for the epinephrine what it gonna inhibit?
a.
luektrine release from macrophages
b.
cross reactivity with the cardiac..

425

c.
inhibit immunocomplex formation
Answer:
27- eldery patient with low back pain , what is the drug cause relaxation of muscle ?
A. diazepam
B. clorcarpine
Answer: A
28-Mechanism of action of combined oral contraception & progesterone?
Answer: inhibit midcycle LH surge so that ovulation doesnt occur

Genetics
1-What is the longest cell cycle?
a.
Prophase.
b.
Cytokinesis.
c.
Interphase
d.
Metaphase
Answer: C
Interphase is usually longest, followed by prophase and telophase; metaphase/anaphase is usually shortest.
2-what is genetic beta thalassemia?
A. Inseration
B. Mutation
C. Depleting
D. Fharm shaft
Answer: B
3-Mode of transmission of polycystic kidney in adults:
Answer: AD

There were also missing questions about the following:

Digitalis side effects


Ca channel blockers side effects
Warfarin MOA
Nerve dermatomes of lower and upper limbs + Cranial Nerves
Major arteries and their branches

426

Newly added Qs 7th update


Renin secreted from ?
Renal afferent
Renal efferent
Distal tubules
Proximal tubule
Answer: A ( by juxtaglomerular cells in afferent arteriole )
Reference: USMLE step 1
A patient had a dislocation in his jaw, the doctor wanted to retract the mandible to his position which
of the following retracts the mandible in his normal position?
a) temporalis
b) lateral pterygoid,
c) medial ptergypoid,
d) masseter
Answer: a
Muscle responsible for plantar flexion?
a) rectus femoris,
b) quads,
c) plantaris
Answer: C
Reference : BRS gross anatomy
Patient who cannot flex his knee, absent ankle reflex, where is the level of injury?
a) L4-5,
b) L5-S1
Answer: B
Reference: BRS gross anatomy
Pain or numbness of 4 first fingers of hand. Which nerve affected?
a) Ulnar
b) Median
c) Radial
d) brachial plexus
Answer: B -Median. If the last finger and a half (pinky) then it would be ulnar.
Main action of anterior muscles of forearm?
Answer: Flexion and pronation
Male pt complain of weakness in flexion of both Rt knee and Rt hip which muscle affected?
a) Sartorius
Answer: a
Reference: BRS gross anatomy
Patient complains of severe chronic constipation. Which muscle is important to be relaxed?
a) Puborectalis
b) Pubococcygeus
c) Iliococcygeus
Answer: A

427

Patient was running and then felt pain in his left leg. The pain gets better with stretching the leg What
is the muscle affected:
a) Posterior tibial
b) Gastrocnemius
c) Soleus
Answer: B

What is the type of the pelvic bone:


a) Pivot
b) Ball and socket
Answer: B
12- Picture of sole hand shows
Answer:
knee dermatome
a) L1-L2
b) L3-L4
c) L4-L5
d) L5-S1
Answer: b
The Ankle joint consists of what bones?
a) Distal tibia, fabula, and talaus.
b) Latral malloli, medial malloli, and talaus.
c) Calclunus, tibia, and fibila.
Answer: A
Reference: BRS gross anatomy
Stab wound lateral to sternocleidomastoid, Patient unable to do adduction of the shoulder and
elevation over his head. What is the nerve injured?
a) long thoracic
Answer: A
pts injury at gluteal area affect walking which nerve affected:
a) sup. gluteal n
b) b-infer. " "
c) obturator n
Answer: gluteal area ( gluteus medius and minimus) by Superior gluteal n :
Which ligament prevent uterine prolapse?
a) Round ligament
b) Broad ligament
Answer: Prolapse happens when the ligaments supporting the uterus become so weak that the
uterus cannot stay in place and slips down from its normal position. These ligaments are the round
ligament, uterosacral ligaments, broad ligament and the ovarian ligament. The uterosacral
ligaments are by far the most important ligaments in preventing uterine prolapse.
Patient have blunt in right 4 intercostal space affected:
a) upper loop of lung
b) lower loop of lung
c) horizontal
d) ?
Answer: C
Nerve to gluteus:
a) Femoral
b) Popliteal
c) Obturator
d) ..Obturator
Answer:
Patellar knee jerk reflex maintained by:
a) L1-L2
b) L2-L3
c) L3-L4
d) L5-S1
Answer: C
Reference: USMLE step 1

428

Waiter at a restaurant tripped on his knee and fractured his neck of the fibula while serving
customers, sometime after he noted dropping of his big toe while walking, which nerve got
affected?
a) Tibial n.
b) Deep peroneal n.
c) Common peroneal n
Answer: B
Explanation: Common peroneal nerve gives 2 branches (superficial and deep) if the superficial
gets injured the clinical significance is that you cannot evert the foot and you will have sensation
loss over some parts, while if the deep peroneal is injured you will have a foot drop (similar to wrist
drop in radial injury), you can claim its a
common peroneal injury if all these symptoms present in the same case
.
what is the position of the femoral vein to the artery:
a) Medial
b) Lateral
c) Posterior
d) Anterior
Answer: A
Explanation: They are VAN from medial to lateral (vein then artery then nerve) so if we are asked
about its relation regarding the artery it will be MEDIAL.
in examination doctor touch posterior pharynx he want to test which nerve :
a) vagus
b) hypoglossal
Answer: A
Explanation: Testing the Gag reflex, Afferent part by the glossopharyngeal cranial nerve nine
(sensory part) and Efferent part by the Vagus cranial nerve ten (motor part)
Patient in dental clinic received local anesthesia and give numbness below eye , maxilla and part of
the nose most likely nerve blocked is :
A) Sphenopalatine
B) Infraorbital
Answer: B
Explanation: Infraorbital beneath the orbital rim, and supraorbital for sensation above the orbital rim.
Patient with lesion above the left eye brows , first lymph node to be examined is :
a) Parotid
b) Mental
c) Submandibular
Answer: A
Injury to deep peroneal nerve??
Answer: result in foot drop
Reference: BRS gross anatomy

a)
b)

Muscle of knee extension?


Quadriceps
biceps femoris
Answer: A
Reference: RBS gross anatomy

What is the name of the muscle that pass below lesser sciatic nerve canal?
pectenious m.
Answer:
femoral neck # his leg was rotated laterally which muscle responsible
a) Rectus femoris
b) Gluteas maximus
a)

429

Answer: B , Bot hBices femoris , obturar externus and Gluteus maximus >> Lateral leg rotation
Big and 2ed toe cannot dorsiflexion
a) deep pernoal n
b) post tibial n
c) sural n
Answer: A
Inferior horn of lateral ventricle which affect
a) Hiccup
b) Patman
c) cudate nucl.
Answer:
I think truma >> clear nasal discharge which affect
a) optic n
b) olfactory n
c) ophtham- ... )
Answer: B
Explanation: Clear nasal discharge is a hint of CSF due to fracture of the cribriform plate bone which contains
nerve fibers of the olfactory nerve.
Testicular lymph node drainage? ( I am not sure is it testicular or scrotum)
a) Left aortic lymph node
b) Superficial inguinal lymph node
c) Deep inguinal lymph node
Answer:A
pt when walking and lifting his Rt leg , his left hip drops , the doctor told the or that he has
Trendelenburg sign , which muscle is affected ?
a) Left gluteus medius
b) Rt gluteus Magnus
Answer: , righ gluteus medius
what is posterior to the mid part of inguinal ligament or canal not sure
Answer:femoral a
origin of gluteal artery ( scenario: ischemia in gluteus area then ask for origin of gluteal artery)
A) internal iliac artery
Answer: A
51- Occlusion to internal iliac artery, what area could be affected?
Answer: Bladder

A)
B)
C)
D)

Patient lost sensation at the level of umbilicus after MVA. the lesion will be at the level of
T4
T10
T11
T 12
Answer: B
What is the artery supply posterior inferior nasal septum?
Answer: external carotid internal maxillary sphenopalatine A. nasopalatine

Case head trauma on parietal lobe subdural hematoma which artery is injured?
a) superficial temporal .
b) mid cerebral
c) Rt.cerebral .......
answer : B

a)

58- Flexion and pronationlling worse at morning. Dx?


answer :

430

A woman complaining of lt hand tingling mainly at thumb and index on exam there was mild atrophy
of thinner muscle Tenil's test was positve which nerve may be affected?
A) radial nerve
B) median nerve
C) musculocutaneous nerve
Answer: B
2 Q's about the anatomy of the arteries in the brain.
Answer:
Loss of sensation over the maxilla and mandible:
A) trigeminal
Answer:A

After endoplasmic reticulum protein moves to :


A golgi apparatus
B mitochondria
C neocleas
D
answer: A
Reference: USMLE step 1
Type of protein form in neocleas:
A mrna
B trna
answer: similar to question 4 page 286
Q-5-What is the source of energy to transfers molecules in and out of the nucleus ?
ATP within cytoplasm
ATP within nuclus
Dtp or something else!!
answer :

What give the cell energy...


A) ATP from cytoplasm
B) ATP beoclyes
C) GTP from cytoplasm
D) GTP neuc
Answer:

Patient irriagate , and weakness in lower and upper extremities , what is the cause ?
- deficincy in vit D
- deficincy in vit A
- deficincy in B1
- deficincy in B3
answer : C
Q - Which one has powerful molecular energy in the cell?
A:a - ATP hydrolysis move components from Cytoplasm to Nucleus
b - ATP hydrolysis move components from Nucleus to Cytoplasm *
c - GTP hydrolysis move components from Cytoplasm to Nucleus
d - GTP hydrolysis move components from Nucleus to Cytoplasm
Answer :
-rejection of transportation liver mechanism
answer:

431

a complain for breast cancer screening for women under 52 years and older to decrease unnecessary
anxiety to the public. Youll include those under 52 if they have which gene mutation?
BRCA2
Answer :A
At which temperature does the blood bank preserve the blood?
A-37 C
B-4 C
C-22 C
Answer : B
Cold agglutination test use which antibodies?
A-IgA
B-IgM
C-IgG
D-IgE
Answer: B
what most commonly cause itching ?
A-bile salt retention
B-eczema
C-pregnancy
answer :B
-Read about epstein bar virus??
-Read about mononucleosis ??

Patient with high Ca and low Iga Igm what is the diagnosis (this Q was already submitted by one of our
colleagues but the answer in the (gathered smle) had multiple myeloma and an explanation to a subtype of it.
In the exam they offer you both multiple myeloma and it's subtype as choices and you have to choose
between the two
answer :
-

Organism from animal bit?? no rabies in answers


A) Streptococcus mutalis<< not sure
Answer:
- A woman has +ve Hep B surface antigen delivered a baby in a hospital and resaved immunogloben and B
vaccine with in the first 12 hour ,, what are the recommendation for breast feeding ??
A) no breast feeding
B) breast feeding is okay
C) breast feeding after 12 hours
Answer: B ?
What is the minimum DTaP side effect ??
A) Redness at the injection site
B) generalize muscle pain
C) fever 38.3
p.s. No low grade fever
Answer: A
long case of eye trauma then repair then developed most likely endophthalmitis done
enucleation, found in the uvea lymphocyte and CD4 i think, what's is the cause?
A-cross reactivity
B-activation of lymphocytes
C-antigen release or something?
answer :C?
Patient with ventilator associated pneumonia. Culture showed lactose non-fermenting, gram negative
motile not the same Q but they asked about an organism!!!
Answer:

432

bacilli producing greenish colony + Oxidase positive. What is the organism?


A) Haemophilus Influenzae
B) Streptococcus pneumoniae
C) Klebsiella or other gram negative bacteria
D) Pseudomonas aeruginosa
Answer:D
positive culture of budding yeast in urine what is the management ?
A-Flucanazole
B-caspofungin
answer :A
man done vasectomy, change his mind want to reproduce again, they found antisperm
antibodies what is the cause ?
A-antigen release or something like that?
B-cross reactivity with viral infection
C-inappropriate response of MHC II to antigen presenting cell
answer : A ?
HIV patient, presented with SOB and productive cough bronchoalviolar lavage showing ((something)) . Similar Q
but no Pnemocystis in answers !!
A) Pneumocystis jiroveci
B) Aspergillusfumigatus
C) Cryptococcus neoformans
Answer:
- something produced by the nucleus
A) Mrna
B) tRNA
Answer: mrna

-C.perf >>
A-cruptis under skin
Answer:A

Pt dx with cutaneous leishmanial what is the organism?


A) Lishmenia aka aza
Answer: tropica
the best test for hypersensitivity type 1 :
1- subdermal skin injection ( my answer)
Answer:A
HIV patient test to confirm :
A- PCR
B- western blot
C- elisa
answer: B
test for Sickle cell anemia :
A- Hb electrophoresis
B- bone marrow aspiration
Answer:A

-ventilator associated pneumonia in icu patient Gram negative oxidase postive What is the organism?
Answer: Psuomonas agina

Q - Child diagnose to have Giardiasis (Giardia Lamblia). What is the best investigation that confirm the
diagnosis ?

433

A:- 3 stool analysis in consecutive days


B- 3 stool analysis in separated days
* Answer :A

Which of the following medications is safe during breastfeeding?


A- chlormphenicol,
B- cimitidine,
C- erythromycin,
D- tetracycline
answer: C

DM pt. with necrotizing fasciitis .. they asked about antibiotic combination


A)penicillin/gentamycin
B)amoxil/erythromycin..
C)pipracillin/tazobactam
NB..read about antibiotic and its combinations
Answer: carbapenem+clindmycin +antiMRSA vancomycin
pipracillin/tazobactam instead of carbapenem

Patient is taking Sublingual Isosorbide Dinitrate for myocardial infarction. What will be the side effects
of the drug?
a- Hypotension *
b- Hypokalemia
c- Heart Block
Answer:A is a nitrate

Pregnant with past history of depression on Paroxitine for long time. She is asking the physician if she can use
this medication or not while she is pregnant. What you have to tell her ?
- It is not safe because the risk of cardiac congenital malformation *
- It is not safe...
- It is safe...
- It is safe...
Answer:A
pt diagnosed with MI and aspirin was giving what is mode of action
A. anti platelet aggregation
B. anti thromban III
C. forget other choices
Answer: A
-Ciprofloxacin MOA ?
A-Inhibits RNA
B-Inhibits DNA
C-Inhibits cell wall synthesis
D-Other
Answer:B

434

What is the rule of metformin in PCOS?


a)
Decrease glucose level
b)
Decrease insulin resistance
c)
Anti-androgenic
d)
Menstrual regulation
Answer :b
Which of the following well break acute coronary thrombi, causing MI?
a)
ASA
b)
Heparin
c)
Altepase
d)
Enoxparen
Answer : B? Is it HIT?
Which medication break down an acute coronary thrombus ?
A-Alteplas
B- aspirin
C-enxaparen
Answer: A
Q1/ 40 y.o lady diagnosed with GERD 2 months ago on antiacids, developed RHD 2 weeks ago on Aspirin.
What is the side effect of Aspirin:
A. Diarrhea
B. Constipation
C. Dry mouth
Answer :A
61. A patient presented to the ER with respiratory depression and pinpointed
the drug is
Answer : morphine
- What is the treatment of shigella?
A-Cefuxime.
B-Ampicillin.
C-Metronidazole
Answer:: ciproflocxacin ,ceftriaxon, TMP-SMZ , azithromycin
What is the mechanism of warfarin?
Answer : block function of vit K epoxide reductase in the liver
Anticholinergic Side effects??
A-Diarrhea.
B-Urine Incontinence.
C-Blurred vision
Answer : C
- Which one of these drugs will cause hyperpigmentation?
(the answers were either anti-HTN or anticonvulsant, cant remember)
Answer : Phenytoin

patient with asthma exacerbation, Which drug will decrease the mucous secretion more the broncho
dilation
A-oral steroids
B-ipratropuime
B-luekot
answer :B
ADHD case what is the treatment?
answer : Atomoxitine

435

-patient HTN he complin of cugh he want to change the drug to other one drug with same effect but no cough
?
answer:
** probably he is using ACEI .. We give ARB instead
parient presented to ER with history of drug over dose and coma for the last 8 hours on examination absent
gag reflex best manegment is :
A-iv naloxone
B-gastric lavage
C-immediate endotracheal intubation
D-coracol
answer : C
- side effect of atropin include :
A- diaphragm paralysis
B-stimulate sweat gland
or......or
Answer:non of them
- Case of Absent seizure, whats the effect of injecting Phenytoin intrathecal:
A- inhibit secretion of substance P
B- seizure secondary of toxic metabolite
Answer : increase in sizure frequency
Medication for osteoporosis that causes epigastric and heartburn sensation:
a- Dunosumab
b- Risidronate
c- Raloxifine
Answer:B
Medication for acidity and heartburn that causes constipation .
Answer:Alamonium hydroxide
bacterial meningitis in 14 month child I think? Gram positive cocci, what is the management?
A-amoxicillin
B-amoxicillin and gentamicin
C-ceftriaxone and vancomycin
D-vancomycin
answer : C
-

which of the following will cause sedation in low dose ;


A) midazolam
B) lorazipam
Answer: A??
What antibiotic is contraindicated to use in a patient with culture showing sensitivity to many antibiotics
(vancomycin, Fluoroqinilones...) but resistant to Beta lactase??
A) Gentamycin
B) Azthromycion
Answer:
-Tx for fever, cough, bilat infiltration ?
A- Cipro
B- azithro
Answer: B..IF they meant legionella in the question.
-Treat of hyperTG only ?
A- NA
B- fibrate
C- pyt

436

Answer:B

-Nitate sublingual side effect ??


A- headach
B-hypokalm
C- bronchospasm
Answer:if no hypotension in the choices >> then choose Headache .

side effect of Ribvrin ??


A... Lactat..
B- liver damage
C- renal damage
D- anemia
Answer: D
Drug for bladder ot prostate oxybutynin
A-block muscarinic cholinergic
B- block b 2
C- activate a adrenergic
D- activat
Answer: A
Hyper pigmentation caused by ??
A- ccb
B- amiodarone
C- digoxin
Answer: B
enterococcus faecalis antibiotic and the pt is allergic to ampicillin , what to give VancomycinAnswer: B
statin induced myopathy in old ladyAnswer:

pt had MI , he was given sublingual nitrate and analgesia , the MOA of the analgesic that was given ?
Answer: if they mean morphine in the Q >> its MOA >> bind to Mu receptor .
Which of the following antibiotics is DNA gyrase and it works on what organism ?
Answer: pseudomonas
Old pt with recent memory loss and poor self-care and social withdrawal, what to give him?
A) Neostigmen
B) Rivastigmen
Answer:B

Which of the following medication causes vertigo?


A) Ethambutol
B) Streptomycin
Answer:B

what medication that cause hypertensive crisis with Tyramine


A) MAOI
B) TCA
Answer:A

437

What is the best drug given to prevent postoperative thromboembolism?


A) LW heparine
B) Uf heparine
C) Warfarin
D) eno.,
Answer: A
Ref : uptodate

-hich of the following is side effect of atropine :


1- vasoconstriction
2- decrease IOP
3- decrease urine output
4- dry of mouth
Answer:D
Which of the following is side effect of morphine :
1- dry cough
2- nausea , vomiting
3- tachypnea
4- anxiety
Answer: B *
??. Aspirin toxicity resulting in a- metabolic acidosis followed respiratory alkalosis
Answer: Respiratory alkalosis followed by Metabolic acidosis
Which of this drug cause seizure ?
A- isoniazid
B- ethambutol
C- ripaficin
D- Pyrazinamide
Answer :A

40 y.o lady diagnosed with GERD 2 months ago on antacids, developed RHD 2 weeks ago on Aspirin. What
is the side effects of Aspirin:
A- Diarrhea
B- Constipation
C- Dry mouth
Answer : A

Autosomal recessive disease (


)
Both parents are carrier and phenotype normal what the chance they have a kid with a disease?
a-25%
answer:A
what is the gene for ductal carcinoma of breast?
a)
P53
b)
Bcl2
c)
C-myth
Answer: A ??
Qs about the anatomy of the arteries in the brain.
Answer:
-Ionised radiation what will happen ?
A- deamination
B- de purantion

438

C- DNA break strin


Answer: C
Sx of hydrop fetalis or HB h
-normal 2 beta abnormal 4 alfa
- oppiste above
Answer: abnormal 4 alpha

Pneumococcal 13 conj type


-Toxoid
conjogoid
- inactivated
-live
Answer:B

they want to stop screening for breast cancer for women under 47 to decrease unnecessary anxiety to the
public , you didn't agree cus there is gene that cause cancer in young women , what is the gene ?
BRCA2
Answer:A

Q-16-Long scenario.. What is the gene mutation for it? (( gilbert syndrome))
Answer : UGT1A1 on Chromosome 2q37.

What is the artery supply posterior inferior nasal septum ???


Answer: Sphenopalatine artery

Follicular cell of ovary what it is orign embryologically ???


Answer:
Cortical Cords , which split into isolated cell clusters ,with each surrounding one or more primitive
germ cells . Germ cells subsequently develop into oogonia , and the surrounding epithelial cells ,
descenants of the surface epithelium , form Folicular cells .
Ref: Langmans Medical Embryology .

embryogenic origin of tongue muscle?


a lingual
b occipital
Answer: B , Originating in occipital somites .
Ref : Langmans Medical Embryology

which hepatic cell produce extra matrix component ?


A.Stellate cell
B.Hepatocytes

439

Answer: a ( Stellate cell = Ito cell )

Pt with vit b 12 deficiency what gastric cell type will be affected


A.Chief cell
B.Parietal cell
Answer: b
Reference: USMLE step 1
What part of the liver is affected during ( I think hypoxia) Zone 2 and 3 Central part of portal something
Answer: zone 3
Reference: USMLE step 1

Accessory N pass through which triangle


A- Posterior triangle
Answer : A
Reference ( snell )
Pt with forehead multiple fractures which nerve is involved ?
A-optic ,
B-olfactory ,
C-opthalmic ,
answer C
Pt with breast cancer and axillary LN involvement ..what muscle might be involved ?
A-Pectoralis major ,
B-pecoralis minor ,
C-latismus dorsi
Answer B?
Stand on toes ..which nerve ?
A-Tibial n .
Answer A
Ref: Greys Anatomy
about kidney vein that drain to IVC (Part of kidney)
answer:
both right and left renal vains drain into IVC
right supra renal vain drains into IVC
Left supra renal vain dains into Left renal vain
Ref: greys anatomy
Pt after RTA , no abduction and lateral rotation of the arm.. . What is the origin of the affected nerve ?
A. medial plexus
B. lateral plexus
C. lower plexus
D. root
Answer : B
Deltoid M ..action : abducts arm ( 18 - 90 degree) teres minor M .. Action :- lateral rotate of arm
Both supply by axillary nerve from posterior it is a part of the brachial plexus . It consists of contributions from
all of the roots of the brachial plexus ( trunks )
Also, the upper trunk give suprascapular N which supply supraspinatus and infraspinatus
Supraspinatus action :- abducts of arm
Infraspinatus action :- lateral rotation of the arm

440

Patient when walking and lifting his Rt leg , his left hip drops , the doctor told the or that he has
trendelenburg sign , which muscle is affected ?
A.
Left gluteus medius
B.
Rt gluteus Magnus
Answer: A
Several dysfunctions can produce a positive Trendelenburg Test, including [6]
Superior Gluteal Nerve Palsy
Lumbar disk herniation
Weakness of gluteus medius
Advanced degeneration of the hip
Legg-Calv-Perthes Disease
Ref :http://www.physio-pedia.com/Trendelenburg_Test
which part of body bone is forming the ankle ?
Answer : the tibia, the fibula, and the talus.
Superior rictus muscle >> eye movement
A.
In down
B.
Out down
C.
Up in
D.
Up out
Answer: D
action of SR at the eye ball >> Up + In
movement of the eye when testing the muscle >> Up + out
Ref:Greys anatomy

Femoral neck his leg was rotated laterally which muscle responsible:
A.
Rectus femoris
B.
Gluteus maximus
Answer: Biceps Femoris
Ref: Greys anatomy

Patient with pain in hands and fingers during typing, decrease of blood flow to which of the following
will cause this pain?
A.
Radial
B.
Ulnar
C.
Posterior interosseous
D.
Anterior interosseous
Answer: B. Ulnar (superficial palmar arch)
If there was inferior mesenteric artery thrombosis. Which artery will not be affected! a. descending colon b.sigmoid c.splenic d.cecum
Answer: D
What type of radial nerve injury ?
a-nuropraxia
b-nurotemesis
c-axontemesis
Answer: A

A-facial
B-trigeminal

loss of sensation of anterior 2/3 of tongue which nerve affected?

441

C-glossopharyngeal
D- hypoglossal
Answer: B
Sensory innervation by Trigeminal , Taste inttervation by Facial
pt with excessive tearing and they gave a picture of enotripion and they asked about cause of
tearing ?
A.
B.
C.
Answer:?
External laryngeal nerve action :
A.abduct cord
B.adduct cord
C.upper cord area sensation
Answer: B
-The cricothyroid muscle produces tension and elongation of the vocal folds by drawing up the arch of the cricoid
cartilage and tilting back the upper border of the cricoid cartilage lamina; the distance between the vocal
processes and the angle of the thyroid is thus increased, and the folds are consequently elongated, resulting in
higher pitch phonation.
- This muscle is the only laryngeal muscle supplied by the branch of the vagus nerve known as the external branch
of the superior laryngeal nerve

A patient with a stabbed wound to the Gluteus. Examination: The patient tilt to the unaffected side
while walking. Which nerve is affected?
Answer:
a. Femoral N
b. Obturator N
c. Superior Gluteal N
d. Inferior Gluteal N
e. Peroneal N
Answer : C
The Trendelenburg gait pattern (or gluteus medius lurch) is an abnormal gait (as with walking)
caused by weakness of the abductor muscles of the lower limb, gluteus medius and gluteus
minimus. People with a lesion of superior gluteal nerve have weakness of abducting the thigh at
the hip.

Palmar arterial arch is:


a- radial
b- ulnar
c- anterior interosseous
d- posterior interosseous
Answer :
- deep palmar arch >> by Radial artery ,,

442

- Superficial palmar arch>> Ulnar a.


Ref: Greys anatomy

Which in lipid profile

is most important risk for coronary heart disease ?


LDL,
HDL,
Triglyceride ,
total cholesterol

answer A
Infection that is more severe in immunocompromised rather than immunocompetent:
a. crypto.... can't recall the rest
Answer :
It is now well known that people who are immunosuppressed secondary to HIV infection are at higher
risk for Cryptosporidium infection and that carriage of the parasite is associated with diarrheal
disease in most cases. Furthermore, in those with diarrhea, the disease is much more severe and
prolonged than in otherwise healthy individuals
Ref: http://www.ncbi.nlm.nih.gov/pmc/articles/PMC118064/

Boy swimming in river , after one day , onset of fever and weakness , what the most common organism cause
infection ?
Answer: (Katayama's fever) Acute schistosomiasis
Monospot test +ve what is the diagnosis
Answer : EBV

Organism undercooked beef ?


A.Entamoeba
B.Tinea
Answer : b
Organisms that can be found in raw meat:
Beaf: E. coli O157:H7, Salmonella, Shigella, Staphylococcus aureus and Listeria monocytogenes
Poultry: Salmonella and Campylobacter
Shellfish: Vibrio gastroenteritis, Salmonellas, Plesiomonas shigelloides, Staphylococcus and Bacillus cereus
Bacteria sexual like behavior ?
A-Conjunction
Answer : A
Ref: http://www.ncbi.nlm.nih.gov/books/NBK21942/
Which of the following vaccines is the least likely to harm an immunocompromised patient?
A.
Measles vaccine
B.
Mumps vaccine
C.
Pneumococcal vaccine
Answer: C. Pneumococcal vaccine
You perform a skin hypersensitivity test for molds which came strongly positive within 30 minutes,
what is the type of his hypersensitivity reaction?
A.
Immediate hypersensitivity reaction
B.
Delayed hypersensitivity reaction
C.
Contact dermatitis
D.
immune-complex reactions (serum sickness)
Answer: A. Immediate
female pt ate from seafood restaurant , in the second day she developed diarrhea and nausea and
urticaria , what does she have ?

443

A. Food poisoning
B. Food allergy
c.
Answer: B?

A.
B.
C.

male just moved to new apartment in humidified area and there was dust , he was injected with
mites and 30 mints later , he developed allergy , what type of hypersensitive he has ?
Immediate
Delayed
Answer:
Pt with MI and take morphine (not sure about route of administration ) What is the side effect of morphine
1-dry cough
2-tachypnea
3- anxiety .....
Answer :
Nausea, vomiting, constipation, lightheadedness, dizziness, drowsiness, increased sweating, or dry mouth may occur.
Pain, redness, or swelling at the injection site may occur if this medication is given into a muscle or under the skin.
Ref : http://www.webmd.com/drugs/2/drug-18436/morphine-intravenous/details#side-effects
prophylactic of rheumatic disease Normal pt
A- I think the answer penicillin ( not sure)
Answer : A
what is the antithyroid used in pregnancy?
A- propylthiouracil
Answer : A
On patient with gout you will not use:
A.

spironolactone can't recall the choices

Answer : Thiazied
Patient on statin for his HTN, lab showed high triglyceride: what will you add:
a- fibrate
Answer : A
2-dexamethasone toxicity is reverse by : ....
. Answer:Case of digoxin toxicity
Answer:
Symptoms
Confusion.
Irregular pulse.
Loss of appetite.
Nausea, vomiting, diarrhea.
Palpitations.
Vision changes (unusual), including blind spots, blurred vision, changes in how colors look, or
seeing spots)

444

Ref :https://www.nlm.nih.gov/medlineplus/ency/article/000165.htm
Female pt came with excessive facial and abdominal hair growth.. Which antiepileptic can cause this
side effect?
A.
Phenytoin
B.
Carbamazepine
C.
Phenobarbital
D.
Valproic acid
Answer: A. Phenytoin
Which antiparkinson cause hepatotoxicity?
Answer:
tolcapone (Tasmar), used for Parkinson disease;
( medscape )
Which of those medication cause anxiety
a.
TCA
answer:
SSRI ( USMLE step 2 ck )
Which of bisphosphonate causes inhibition of osteoclast activity?
Answer:
Alendronate inhibits osteoclasts -mediated bone-resorption. Like all bisphosphonates, it is chemically related to
inorganic pyrophosphate, the endogenous regulator of bone turnover. But while pyrophosphate inhibits both
osteoclastic bone resorption and the mineralization of the bone newly formed by osteoblasts, alendronate
specifically inhibits bone resorption without any effect on mineralization at pharmacologically achievable doses. (
ref. wiki )

Which of the following should be avoided during pregnancy?


A.
Amoxicillin
B.
Cephalosporins
C.
Fluoroquinolones
Answer: C
-Phenobarbital in lactating women :
A)Stop immediately
B) Continues
C) Fed after 8 hr
D)
Answer: B?
Inter- and intrapatient variability in excretion of phenobarbital into breastmilk is extensive.
Phenobarbital in breastmilk apparently can decrease withdrawal symptoms in infants who were
exposed in utero, but it can also cause drowsiness in some infants, especially when used with other
sedating drugs. Monitor the infant for drowsiness, adequate weight gain, and developmental
milestones, especially in younger, exclusively breastfed infants and when using combinations of
psychotropic drugs. Sometimes breastfeeding might have to be limited or discontinued because of
excessive drowsiness and poor weight gain. If there is concern, measurement of the infant's serum
phenobarbital concentration might help rule out toxicity.
Ref: http://www.drugs.com/breastfeeding/phenobarbital.html

-phenytoin side effect StevensJohnson syndrome ? Read about its clinical presentation:
A)
B)
C)
D)
Answer:
Typical prodromal symptoms of Stevens-Johnson syndrome are as follows:

445

Cough productive of a thick, purulent sputum


Headache
Malaise
Arthralgia
Patients may complain of a burning rash that begins symmetrically on the face and the upper part of
the torso. The cutaneous lesions are characterized as follows:
The rash can begin as macules that develop into papules, vesicles, bullae, urticarial plaques,
or confluent erythema
The typical lesion has the appearance of a target; this is considered pathognomonic
In contrast to the typical lesions of erythema multiforme, these lesions have only 2 zones of
color
The lesions core may be vesicular, purpuric, or necrotic; that zone is surrounded by macular
erythema
Lesions may become bullous and later rupture, leaving denuded skin; the skin becomes
susceptible to secondary infection
Urticarial lesions typically are not pruritic
Infection may be responsible for the scarring associated with morbidity
Although lesions may occur anywhere, the palms, soles, dorsum of the hands, and extensor
surfaces are most commonly affected
The rash may be confined to any one area of the body, most often the trunk

Signs of mucosal involvement can include the following:


Erythema
Edema
Sloughing
Blistering
Ulceration
Necrosis
The following ocular signs may be noted on slit-lamp examination:
Eyelids: Trichiasis, distichiasis, meibomian gland dysfunction, blepharitis
Conjunctiva: Papillae, follicles, keratinization, subepithelial fibrosis, conjunctival shrinkage,
foreshortening of fornices, symblepharon, ankyloblepharon
Cornea: Superficial punctate keratitis, epithelial defect, stromal ulcer, neovascularization,
keratinization, limbitis, conjunctivalization, stromal opacity, perforation
Ref: Medscape
-post MI developed palpitation ECG pic (VT) Ttt:
A) Metoprolol Adenosine
B)
C)
D)
Answer: A ( bb )
- Metformin action at the cellular level:
A) inhibit tyrosine kinase
B)activate adenylate kinase
C) decrease ATP
D)
Answer: It inhibits purified AMP deaminase . Furthermore , a known inhibitor of AMPD stimulated
Glucose uptake and fatty acid oxidation .
What antihypertensive decreases preload as well as causing vasodilatation:
A)
B)
C)
D)
Answer: ACEI , CARVIDILOL

446

genetics:
female with ductal carcinoma Doctor want treat her, what is the gene responsible for that cancer? (No
BRCA1 in options)
1- tp53 I don't remember the remaining
Answer:
Changes in other genes (OTHER THAN BRACA GENES):

TP53: The TP53 gene makes a protein called p53 that helps stop the growth of abnormal
cells. Inherited mutations of this gene cause Li-Fraumeni syndrome. People with this
syndrome have an increased risk of breast cancer, as well as other cancers such as
leukemia, brain tumors, and sarcomas (cancers of bones or connective tissue). This is a
rare cause of breast cancer.
CHEK2: The Li-Fraumeni syndrome can also be caused by inherited mutations in the
CHEK2 gene. Even when it doesnt cause this syndrome, it can increase breast cancer
risk when its mutated.
PTEN: The PTEN gene normally helps regulate cell growth. Inherited mutations in this
gene cause Cowden syndrome, a rare disorder in which people are at increased risk for
both benign and malignant breast tumors, as well as growths in the digestive tract,
thyroid, uterus, and ovaries. Defects in this gene can also cause a different syndrome
called Bannayan-Riley-Ruvalcaba syndrome thats not thought to be linked to breast
cancer risk. The syndromes caused by mutations in PTEN can be grouped together as
PTEN Tumor Hamartoma Syndrome.
CDH1: Inherited mutations in this gene cause hereditary diffuse gastric cancer, a
syndrome in which people develop a rare type of stomach cancer at an early age. Women
with mutations in this gene also have an increased risk of invasive lobular breast cancer.
STK11: Defects in this gene can lead to Peutz-Jeghers syndrome. People affected with
this disorder develop pigmented spots on their lips and in their mouths, polyps in the
urinary and gastrointestinal tracts, and have an increased risk of many types of cancer,
including breast cancer.
PALB2: The PALB2 gene makes a protein that interacts with the protein made by the
BRCA2 gene. Defects in this gene can lead to an increased risk of breast cancer. It isnt
yet clear if PALB2 gene mutations also increase the risk for ovarian cancer and male
breast cancer.

447

Ref: http://www.cancer.org/cancer/breastcancer/moreinformation/breastcancerearlydetection/breastcancer-early-detection-risk-factors-you-cannot-change

mode of inheritance for Wilson's disease The answer :

A-autosomal recessive

answer A
4- What's the bound between T-RNA and m-RNA
A) Hydrophobic
B) Hydrogen
C)Convalent
answer : B ?
Autosomal recessive disease : Both parents are carrier and phenotype normal what the chance they have
a kid with a disease ?
A- 25
answer :A?
2- What come with Turner syndrome ?
A.Hypothyroid
B.DM
C.Addison's
answer : A
3-Which of the following is considered in estimating the response of nucleoside reverse transcriptase
inhibitors
A.HIV RNA level in the serum
B.genotype of HIV
answer : A?
1st degree cousins came for pre-marriage counseling, they are worried about hereditary disease so they
must be screened for:
Answer: a-thalassemia

448

449

450

Un-classified Questions

451

Newly added Qs 7th update

9 december Qs
1- patients with epilepsy, which of the following receptors most likely is stimulated?
- Protein G
- glutamate
- serotonin
- kinase
Answer: - glutamate
2- patient 3 weeks after URTI develop rash, knee pain, hematuria. What' the
Dx?
- hypersensitivity vasculitis
Answer
Hypersensitivity vasculitis is thought to be mediated by immune complex deposition. In this form of
vasculitis, circulating antigens in the body (produced by factors such as medications, infections, and
neoplasms) induce antibody formation.
autoantibodies, such as antineutrophil cytoplasmic antibody (ANCA), may be associated with
disease manifestations
symptoms such as fever, arthralgia, arthritis, myalgia, abdominal pain, diarrhea, hematochezia,
cough, hemoptysis, sinusitis, paresthesia, weakness, and hematuria
causes: idiopathic,Antibiotics,Upper respiratory tract infections (particularly beta-hemolytic
streptococcal infection) and viral hepatitis (particularly hepatitis C),Collagen-vascular
diseases,IBD,Malignancy
[1]

3- patient taking anticonvulsant, develop hairstism, what' the agent?


- phenytoin
- barbiturates
- valporate
- phenobarbitol
Answer: phenytoin
4- case of digoxin toxicity
Answer
Cardiovascular: Accelerated junctional rhythm, asystole, atrial tachycardia with or without block, AV
dissociation, first-, second- (Wenckebach), or third-degree heart block, facial edema, PR
prolongation, PVCs (especially bigeminy or trigeminy), ST segment depression, ventricular
tachycardia or ventricular fibrillation
Central nervous system: Dizziness (6%), mental disturbances (5%), headache (4%), apathy,
anxiety, confusion, delirium, depression, fever, hallucinations
Dermatologic: Rash (erythematous, maculopapular [most common], papular, scarlatiniform,
vesicular or bullous), pruritus, urticaria, angioneurotic edema
Gastrointestinal: Nausea (4%), vomiting (2%), diarrhea (4%), abdominal pain, anorexia
Neuromuscular & skeletal: Weakness
Ocular: Visual disturbances (blurred or yellow vision)
Respiratory: Laryngeal edema
5. sign of osteoporosis on X-ray
Answer: DEXA scan ??
Osteopeni
Radiographic features
Decreased bone density can be appreciated by decreased cortical thickness, loss of bony trabecula
in early stages in radiography. Bones like vertebra, long bones (proximal femur), calcaneum and
tubular bones are usually looked for evidence of osteoporosis.

452

Plain film
not a sensitive modality, as more than 30-50% bone loss is required to appreciate decreased bone
density on radiograph
vertebral osteoporosis manifests as:
pencilling of vertebrae
loss of cortical bone (picture frame vertebra) and trabecular bone (ghost vertebra)
compression fractures and vertebra plana
loss of trabecula in proximal femur area which is explained by Singh's index (which can also be
seen in the calcaneum
in tubular bones (especially metacarpals), there will be thinning of cortex
cortical thickness <25% of whole thickness of metacarpal signifies osteoporosis (normally 25-33%)
6- child with eye itching for one month (no other symptoms in scenario), and have Hx of asthma,
what's the Dx?
- bacterial conjunctivitis
- viral conjunctivitis
- venral conjunctivitis
Answer: venral conjunctivitis
Vernal conjunctivitis is long-term (chronic) swelling (inflammation) of the outer lining of the eyes. It is
due to an allergic reaction.
often occurs in people with a strong family history of allergies. These may include allergic rhinitis,
asthma, and eczema
7- tttx of de queverian syndrome((is a tenosynovitis of the sheath or tunnel that surrounds two
tendons that control movement of the thumb))
Answer
forearm-based thumb spica splint with the interphalangeal joint free as well as a concurrent trial of
nonsteroidal antiinflammatory drugs (NSAIDs) for pain relief
if persistante local glucocorticoid injection
For patients with persistent symptoms despite splinting and one or two glucocorticoid injections,
surgical therapy may help relieve symptoms
8.The most accurate diagnostic investigation For ectopic pregnancy ?
A-Culdocentesis
B- Pelvic U/S
C- Endometrial biopsy
D- Serial B-HCG
E- Laparoscopy
Answer: E
Laparoscopy remains the criterion standard for diagnosis; however, its routine use on all patients
suspected of ectopic pregnancy, However initial diagnosis of ectopic pregnancy is a clinical
diagnosis made based upon serial serum human chorionic gonadotropin (hCG) testing and
transvaginal ultrasound (TVUS)
9. A 14 years female, with 6 month history of lower mid abdominal pain , the pain is colicky radiate
to the back and upper thigh, begin with onset of manse and last for 2-4 days, , physical examination
of abdomen and pelvis normal, normal secondary sex development, what is the most likely
diagnosis?
a)
Primary dysmenorrhea
b)
Secondary dysmenorrhea.
answer: A
Primary dysmenorrhea refers to the presence of recurrent, crampy, lower abdominal pain that
occurs during menses in the absence of demonstrable disease that could account for these
symptoms.

453

Secondary dysmenorrhea has the same clinical features, but occurs in women with a disorder that
could account for their symptoms, such as endometriosis, adenomyosis, or uterine fibroids.
10.
Female patient known to have Bicornuate uterus present in labor , give History of kicking in
lower abdomen and on Examination there is round object in fundus on auscultation the heart
positive in the umbilicus of his mother , what is the most likely presentation ?
a)
Face
b)
Vertex
c)
Breach
answer: C
11.
Case of cystitis. http://www.mayoclinic.org/diseases-conditions/cystitis/basics/definition/con20024076
12.
Case of prostate cancer http://www.mayoclinic.org/diseases-conditions/prostatecancer/basics/definition/con-20029597
13.
Case about testicular torsion http://www.mayoclinic.org/diseases-conditions/testiculartorsion/basics/definition/con-20033130
14.
Lady with menses every 15 days
a)
Menometrorrhea
b)
Polymenorhea
c)
Hypermenorhea
answer: B
Polymenorrhea is the medical term for cycles with intervals of 21 days or fewer
Menometrorrhagia is a condition in which prolonged or excessive uterine bleeding occurs irregularly
and more frequently than normal. It is thus a combination of metrorrhagia and menorrhagia.
Hypermenorrhea, also known as menorrhagia, is a disruption in the normal menstrual flow of girls
and women.
15. Premenstrual syndrome :
a)
More in the first half of menses
b)
More in the 2nd half of menses
answer: B
16. Children diagnosed to be hypertensive :
a)
BP Above 90th percentile
b)
BP Above 95th percentile
answer : A
17. What is the injection that is routinely given to new-born to inhibit haemorrhage:
a)
Vitamin K
b)
Vitamin C
c)
Vitamin D
d)
Vitamin E
answer: A
18. Child with URTI is complaining of bleeding from nose, gum and bruising the treatment is:
a)
Prednisolone
b)
IVIG
answer : B
ITP
19. Cellulitis occurring about the face in young children (6-24 months) and associated with fever
and purple skin discoloration is MOST often caused by
a)
group A beta hemolytic streptococci
b)
Haemophilus influenzae type B
c)
streptococcus pneumoniae
d)
staphylococcus aureus

454

e)
pseudomonas
answer : A
20. Patient in dental clinic received local anesthesia and give numbness below eye , maxilla and
part of the nose most likely nerve blocked is :
a)
Sphenopalatine
b)
Infraorbital
answer: B
21. Patient with lesion above the left eye brows , first lymph node to be
examined is :
a)
Parotid
b)
Mental
c)
Submandibular
answer :A
22. Attention Deficit Hyperactivity Disorder ( they give me the symptom not the diagnosis ) child
what is the management?
a)
Ecitalpram
b)
Atomoxetine
c)
Olanzapine
d)
Clonazepam
answer: B
first time: methylphenidate, dextroamphetamine
Atomoxetine (Strattera) has become a second-line
23. 4 years old brought by his parents, height < 5th percentile, they ask if he will remain short.
what you will do initially :
a)
Parental height
answer : i think check Somatmedin C
24. 8 years old girl presented with fever, numerous bruises over the entire body and pain in both
legs. Physical examination reveals pallor and ecchymosis and petechiae on the face, trunk and
extremities. Findings on complete blood count includes a haemoglobin of 6.3 g/dl, white cell count of
2800/mm3 and platelet count of 29,000/mm3. Which of the following would be the MOST
appropriate treatment?
answer :
25. Patient with hematuria and cough and sputum with saddle nose deformity your diagnosis is ?
a)
Wegener's granulomatosis
answer: A
26. Child with mild trauma develop hemarthrosis, in past history of similar episode DX ?
a)
Platelets dysfunction
b)
Clotting factor deficiency
Answer : B hemophilia
27. 3 years old his parents has TB as a pediatrician you did PPD test after
72 hr you find a 10mm induration in the child this suggest
a)
Inconclusive result
b)
Weak positive result
c)
Strong positive result
answer: C
28. Child with aspirin intake overdose ...what kind of acid base balance:
a)
Metabolic alkalosis
b)
Metabolic acidosis
c)
Respiratory alkalosis
d)
Respiratory acidosis
answer: B

455

Acid-Base Disorders in Salicylate Toxicity Adults: Metabolic acidosis AND Respiratory alkalosis
Children: Metabolic acidosis
If fasting=>starvation ketosis may develop
Salicylates directly stimulate the respiratory center to cause hyperventilation (respiratory alkalosis)
which is dose-dependent. This stimulation is much more pronounced in adults than in children.
http://www.anaesthesiamcq.com/AcidBaseBook/ab8_6c.php
Initial respiratory alkalosis followed by metabolic acidosis
the most common abnormality, especially in adults, is a mixed acid-base disturbance (a primary
respiratory alkalosis plus a primary metabolic acidosis)
29. Child with high-grade fever for 5 days and sore throat, on examination there was tonsillitis and
white patches on the gingiva. No LN enlargement, ASO is negative. The most likely causative
organism is:
a)
Coxsackie virus.
b)
Herpes simplex virus.
c)
EBV.
answer : B
30. CPR in child according to American heart association in presence of 02 rescuer:
a)
15 compression and 2 ventilation
b)
30 compression and 2 ventilation
answer: A
31. A man runs for a long distance then develops pain in the thigh with loss of hair diagnosis :
a)
DVT
b)
Acute leg Ischemia
answer : chronic leg ischemia
32. Postmenopausal women with bone metastasis, BMD T score -3. Came with vertebral fractures.
What is your appropriate management?
A.
Estrogen
B.
Bisphosphonate
Answer: B ?

33. G3P2+0. Her first visit was on the 20th week of gestation. She has history of two premature
deliveries. Her cervical length was 30 mm. what is your appropriate management?
A.
Strict bed rest
B.
Terminate her pregnancy
C.
Immediate cerclage
D.
Inject her with progesterone
answer: D or C
Several studies have indicated that the likelihood of preterm delivery increases with decreasing
cervical length. A cervical length of 2530 mm before 32 weeks gestation seems to increase the risk
of preterm delivery. If examination and ultrasound show that you have an abnormally short cervix,
and youre less than 24 weeks pregnant, your practitioner may recommend cerclage, a procedure
in which she stitches a band of strong thread around your cervix to reinforce it and help hold it
closed. However, theres a lot of controversy about whether cerclage should be used in this
situation."
34. A patient was found to have a bilateral hypo-echoic cysts; by ultrasound. What is the next step?
A.
MRI
B.
CT
C.
Biopsy

456

answer: CT
35. A scenario clearly describing a case of pelvic abscess. How will you drain it?
A.
Laparoscopy
B.
Laparotomy
C.
Colpotomy
D.
Medications?
answer : A or B
Procedures used for drainage of the abscess include:
Ultrasound-guided aspiration and drainage: usually the abscess would be rectally drained in men,
and in females it would be drained vaginally.[5][6]
CT-guided aspiration and drainage. Percutaneous drainage often uses a trans-gluteal approach.[7]
Endoscopic ultrasound-guided drainage (EUS-guided drainage). Evidence supporting this as an
effective, minimally invasive option is growing.[8][9]
Laparotomy or laparoscopy with drainage of abscess may be required in some cases.
36. 40 year-old female, completed her family. She has endometrioma. Presented complaining of
mild dysmenorrhea and severe pain during intercourse. What is the most appropriate management?
A.
Removal of the cyst and ablation of the endometriosis lesions
B.
TAH + BSO
answer : B Oophorectomy
37. A lady had progressive sadness over the past 2 years. She has hopelessness, insomnia,
decreased appetite, low self-esteem and suicidal ideation. Diagnosis?
A.
Minor depression
B.
Depressive disorder
C.
Dysthymia
answer : C
38. A LONG scenario about a patient with high temperature, high WBC, low RBC
and low PH. What are you going to do?
A.
Bone marrow aspiration
B.
Cultures
Answer : B
39. When does acute episode of rheumatic fever occur?
A.
In case of pharyngitis
B.
When bacteria invade joints
C.
When bacteria coat the myocardium
Answer : A
40. In acute PE with no signs of DVT; what will you give?
A.
IM warfarin
B.
IM heparin
C.
Warfarin + heparin
D.
Streptomycin
Answer : C
41. A thyroid-disease patient developed pain while moving her shoulder and decreased range of
motion. What is the diagnosis?
A.
Impingement syndrome
B.
Rotator cuff tear
C.
Adhesive capsulitis
Answer : C
42. (Golf elbow) + Difficulty in flexing the wrist. What is your next step?
A.
Refer to orthopedician
B.
Refrain golf
C.
Do an x-ray
D.
Suggest surgery
Answer: B
43. A patient presented to ER with ingestion of multiple iron tablets. What is your next step?
A.
Induce emesis

457

B.
Ipecac syrup
C.
Gastric lavage
D.
Wait and monitor
Answer:D if asymptomatic wait 6 h then discharge
44. A patient came with eosinophilia and generalized body aches. What is the causative organism?
A.
Entamoeba
B.
Giardia
C.
Oxyuris
D.
?
Answer: helminth
45. A 35 year-old smoking man has white plaques in his mouth. What will you do?
A.
Excisional biopsy
B.
Close follow up
C.
?lymph node
Answer: A , to roll out cancer
46. Enlarged medial superficial inguinal lymph nodes. What would you like to examine?
A.
Anal area
B.
Gluteal area
C.
Medial thigh
D.
Leg
Answer: A
47. Angioedema is a side effect of which drug?
A.
BB
B.
CCB
C.
ACEI
Answer: C
48. A scenario telling that a patient has viral infection and you will use an inhaled antiviral. What
anti-viral are you going to use?
A- Zanamivir
Answer:
http://emedicine.medscape.com/article/1966844-overview#a1
49. Pt with odorless vaginal disch, grey-white. Spores on wet mount.
-Candida
-Othet opts
Answer: candida
50. Pt with foul vaginal discharge, greenish color. Microscopy flagellate organisms. What's the
treatment: (trach vaginitis)
-Oral metronidazole
51. Pregnant lady 39 weeks presented with high blood pressure for the first time.
No proteinuria or seizures, wts her dx:
-Gestational hypertension
52. When to swap for GBS in pregnant ladies:
-25 wks
-30 wks
-35 wks
-40 wks
Answer: 35 wks
CDC has recommended routine screening for vaginal strep B for all pregnant women. This
screening is performed between the 35th and 37th week of pregnancy
53. What drug is safe during pregnancy:
-arythromycin
-ciprofluxacine
-cemetidine
-One more opt
Answer: erythromycin

458

Ampicillin,Clindamycin,Erythromycin,Penicillin,Gentamicin,Ampicillin-Sulbactam
,Cefoxitin,Cefotetan,Cefazolin
54. Pt with trauma, femoral/hip fracture. Lt leg short and laterally rotated. What muscle is
responsible for lateral rotation:
-gluteus maximus
-rectus femorus
-gracilus
-adductor magnus
Answer: gluteus Maximus
The lateral rotators are: the superior gemellus, inferior gemellus, obturator externus, obturator
internus, quadratus femoris, gluteus maximus and the piriformis.
55. Child with FOOSH. He has pain on affected arm and shoulder. X-ray shows midclavicular
fracture with segments overriding. What's the mang:
-sling
-surgery
-closed reduction
Answer http://www.uptodate.com/contents/claviclefractures?source=see_link&sectionName=FRACTURES+OF+THE+MIDDLE+THIRD+%28MIDSHAFT%29+OF+TH
E+CLAVICLE&anchor=H6#H6
56. Pt with leg pain for a while. X-ray shows periosteal elevation and onion skin. Wts the mng?
A- IV antibiotic
B- Warm comp, rest and elevation
C- Steroids
Multilayered periosteal reaction, also known as a lamellated or onion skin periosteal reaction,
demonstrates multiple concentric parallel layers of new bone adjacent to the cortex, reminiscent of
the layers on an onion. The layers are thought to be the result of periods of variable growth.
It has been associated with:
osteosarcoma
acute osteomyelitis
Ewing sarcoma
Langerhans cell histiocytosis (LCH)
57. Pt with PIP, DIP swollen and painful. Tests show high uric acid. Wts the treatment:
allopuranol
Answer:

58. Pseudogout What type of calcium crystals?


A- carbonate
B- pyrophosphate
answer : B
59. Pt with multiple trauma post MVA, vitally unstable. First step:
-IVF
60. Pt was anemic, being given blood transfusion. Developed fever, chills, burning at site of IV line,
what to do:
-stop transfusion and give crystalloids
Answer :

459

Stop transfusion as soon as reaction is suspected


Infuse normal saline via IV site using new tubing; aggressive fluid resuscitation is ordered to
maximize renal perfusion.
Monitor vital signs and urine output
Examine blood bag to see if patient was the intended recipient.
Return donor blood back to the blood bank to determine whether the corrrect unit of blood
was administered.
IV furosemide and low-dose dopamine may be ordered to increase renal perfusion (maintain
urine output at 30-100 ml/hr).

If there is any suggestion (eg, clerical mistake, hypotension, pink plasma or urine) that an AHTR is
possible, and even before laboratory tests are available, generous fluid replacement with saline
(100 to 200 mL/hour) to support a urine output above 100 to 200 mL/hour should be initiated
immediately, in an attempt to prevent the development of acute oliguric renal failure. The beneficial
effect of urinary alkalinization in patients with marked hemoglobinuria is uncertain.
Vigorous supportive care is also important for the treatment of AHTRs. If there has been massive
hemolysis and clinical or laboratory signs of disseminated intravascular coagulation, cautious and
early heparinization (10 units/kg per hour) for the next 12 to 24 hours may be of value, although
there are no recent studies on the effectiveness of this intervention.
A vasopressor, such as low-dose dopamine, may be required. If massive intravascular hemolysis
has already occurred, hyperkalemia is likely, and cardiac monitoring and acute hemodialysis may be
required. Accordingly, the patient's renal function and coagulation profile (eg, prothrombin time,
partial thromboplastin time, fibrinogen, platelet count) should be monitored frequently.

61. 50 yo w Painless loss of vision in one eye, with headache and pain when touching the hair on
the same side. Wts next:
-topical steroids
-oral steroids
-brain CT
Answer: The universally accepted treatment of giant cell arteritis (GCA) is high-dose corticosteroid
therapy,The major justification for the use of corticosteroids is the impending danger of blindness in
untreated patients. Patients who present with visual symptoms have a 22-fold increased chance of
visual improvement if therapy is started within the first day. Damage may be irreversible if treatment
is delayed beyond 48 hours. http://emedicine.medscape.com/article/332483-treatment
once it started to affect the vision >> IV steriod
62. child with oral and tonsillar ulcers and vesicles, fever. Dx:
-Herpangia
Answer : Though herpangina can be asymptomatic, symptoms usually associated are high fever
and sore throat. A small number of lesions (usually 2 - 6) form in the back area of the mouth,
particularly the soft palate or tonsillar pillars. The lesions progress initially from red macules to vesicles
and lastly to ulcerations which can be 2 4 mm in size. The lesions heal in 7 10 days.
63. Dew drops on rose petals vaginal lesions, dx:
A- herpes simplex
B- syphillis
C_ chanchroid lesion
D- herpangia
Answer: A
Herpes Simplex viruses generally cause mucocutaneous infection, that is, cells of the skin and the
mucous membranes are infected. This can manifest as cold sores on the lips, or as genital sores.
The typical rash has been described as "dew drops on a rose petal", it consists of vesicles (blisters)
that are initially clear and then crust over, typically with yellowish exudate. These vesicles are
generally painful, and further, the area of skin and/or mucosa and the subcutaneous tissues in the

460

region where the rash will appear commonly becomes sensitive and even swollen before eruption of
the vesicles. Tissue swelling may increase as the rash blossoms, and then, generally over a course
of a week to 2 weeks, resolves completely - leaving no scarring.
http://en.citizendium.org/wiki/Herpes
64. Barking cough treatment after rec epi:
- Steroids
- antibiotics
Answer: steroids
65. Pt with large tongue, anemia, numbness in feet. What is the best treatment:
- vit B12
66. Pt with anemia, high billirubin, positive direct and indirect coombs
Photo of samear shoing spherocytosis, wts the dx:
A-AIHA (rt answer since +ve coombs)
B-spherocytosis
Answer : A
67. Pt with starry sky pattern on biopsy > burkhits
Wts the mutation:
- C-myc gene ( rt answer)
- abl2
- Bcr- abl

68. Pt obese with DM wt treatment:


- metformin (rt answer)
Other ops
69- tttx of ITP
Answer http://www.uptodate.com/contents/immune-thrombocytopenia-itp-in-adults-initial-treatmentand-prognosis?source=search_result&search=ITP&selectedTitle=2~150
70- 4 cases about IDA
Answer http://www.uptodate.com/contents/causes-and-diagnosis-of-iron-deficiency-anemia-in-theadult?source=search_result&search=IDA&selectedTitle=1~150
71- Rx of patient with lung ca stage IIIb came with sudden lower back pain?
- MRI only
- MRI with steroid
- radiotherapy
Answer RADIOTHERAPY
http://www.uptodate.com/contents/non-small-cell-lung-cancer-treatment-stage-iv-cancer-beyond-thebasics?source=outline_link&view=text&anchor=H10#H10
72- loss of sensation over the angle of mandible, which nerve affected?
- lesser occipital
- greater occipital
- 3rd occipital
- greater auricular
Answer : greater auricular
73- case of fanconi syndrome
Answer:http://emedicine.medscape.com/article/981774-overview

461

74- muscle of knee extension?


- Quadriceps
- biceps femoris
Answer : quadriceps
75- case of pneumonia, what your finding on the auscultation?
A - dispred crackles
B - bronchial breath sound
C - absence of vesicular breath sound
Answer b
On physical examination, approximately 80 percent are febrile, although this finding is frequently
absent in older patients and temperature may be deceptively low in the morning. A respiratory rate
above 24 breaths/minute is noted in 45 to 70 percent of patients and may be the most sensitive sign
in older adult patients; tachycardia is also common. Chest examination reveals audible crackles in
most patients, while approximately one-third have evidence of consolidation. However, no clear
constellation of symptoms and signs has been found to accurately predict whether or not the patient
has pneumonia.

76- case of burn, how much fluids you will give in the 1st 8 hours?
Answer:

77- read about how to differentiate bw carpal tunnel syndrome and thoracic outlet obstruction
Answer http://www.uptodate.com/contents/carpal-tunnel-syndrome-clinical-manifestations-anddiagnosis?source=search_result&search=carpal+tunnel+syndrome&selectedTitle=2~130
http://www.uptodate.com/contents/overview-of-thoracic-outletsyndromes?source=search_result&search=thoracic+outlet+obstruction&selectedTitle=1~150
78- snowstorm appearance in pregnant what's the Dx?
Answer: seen in complete hydatidiform mole

462

79- case of tibial collateral ligament injury


Answer
Medial collateral ligament (MCL) injuries of the knee are very common sports-related injuries and it
is the most commonly injured knee ligament. Injuries to the MCL occur in almost all sports and in all
age groups. Contact sports such as hockey, wrestling, rugby, football, and judo are responsible for
the most MCL injuries.
in PE:
- Inspection and palpation of the knee(presence and location of point tenderness, localized soft
tissue swelling, deformity, or ecchymosis). A large joint effusion indicates an associated intraarticular injury.
-The integrity of the MCL is tested with a valgus stress, Testing should be performed in full
extension and at 30 of flexion.If the valgus stress test reveals laxity at 30 degrees of flexion, the
superficial portion of the MCL may be injured. Laxity at 0 degrees of flexion suggests injury to the
deeper structures of the MCL and a possible disruption of the anterior cruciate ligament (ACL),
which acts as a secondary restraint to valgus stress.

Classification systems include the following:


American Medical Association Committee on the Medical Aspects of Sports (1966)
Grade 1 - 0-5 mm of opening
Grade 2 - 5-10 mm of opening
Grade 3 - Greater than 10 mm of opening
O'Donoghue classification
Grade 1 - Few torn fibers, structurally intact
Grade 2 - Incomplete tear, no pathologic laxity
Grade 3 - Complete tear, pathologic laxity
reference:
http://www.uptodate.com/contents/medial-collateral-ligament-injury-of-the-knee#H11
http://emedicine.medscape.com/article/89890-clinical#b4

463

80- lady with spot of hair loss over the scalp with normal underlying skin, what's the Dx?
- alopecia areata
Answer
Alopecia areata is a recurrent nonscarring type of hair loss that can affect any hair-bearing area and
can manifest in many different patterns.
The presence of smooth, slightly erythematous (peach color) or normal-colored alopecic patches is
characteristic.
referance:
http://emedicine.medscape.com/article/1069931-clinical#b4

81- most reliable screening for prostate cancer is:


- PR examination
Answer
The American Cancer Society (ACS) recommends that men have a chance to make an
informed decision with their health care provider about whether to be screened for prostate
cancer. The decision should be made after getting information about the uncertainties, risks,
and potential benefits of prostate cancer screening. Men should not be screened unless they
have received this information. The discussion about screening should take place at:
Age 50 for men who are at average risk of prostate cancer and are expected to live at least
10 more years.
Age 45 for men at high risk of developing prostate cancer. This includes African Americans
and men who have a first-degree relative (father, brother, or son) diagnosed with prostate
cancer at an early age (younger than age 65).
Age 40 for men at even higher risk (those with more than one first-degree relative who had
prostate cancer at an early age).
After this discussion, those men who want to be screened should be tested with the
prostate-specific antigen (PSA) blood test. The digital rectal exam (DRE) may also be done
as a part of screening.
If, after this discussion, a man is unable to decide if testing is right for him, the screening
decision can be made by the health care provider, who should take into account the patients
general health preferences and values.
Assuming no prostate cancer is found as a result of screening, the time between future
screenings depends on the results of the PSA blood test:
Men who choose to be tested who have a PSA of less than 2.5 ng/mL may only need to be
retested every 2 years.
Screening should be done yearly for men whose PSA level is 2.5 ng/mL or higher.
82- Scenario, flashes & floaters in the eye, how to treat?
Answer: posterior vitreous detachment
http://www.geteyesmart.org/eyesmart/diseases/floaters-flashes/causes.cfm
83- postmenopausal lady taking tamoxofin, which of the following u will carefully assess?
- vaginal bleeding
Answer A
because it may increase the risk of uterine malignancy
Tamoxifen may increase the risk of the following, particularly in women over age 50 years:
Cancer of the uterus (endometrial cancer and sarcoma).
Blood clots within deep veins (deep vein thrombosis), usually in the legs, which can travel to the
lungs (pulmonary embolism).

464

Reference:
http://www.uptodate.com/contents/tamoxifen-druginformation?source=outline_link&view=text&anchor=F224618#F224618
84- theoretically which of the following cancer will prevented by vaccination?
A- ALL
B- CML
C- adult T cell leukemia
D- myocois something
Answer : C

85- patient with pheocromocytoma And high catecholamine in urine Initial medical management :
A- ACEI
B-aldosterone blocker
Answer : from Medscape: the definitive treatment for pheocromocytoma is surgical resection of the
tumor, and usually cures the hypertension. the initial medical treatment is only as preoperative
treatment with Alpha and beta blockers.
Start alpha blockade with phenoxybenzamine 7-10 days preoperatively
Provide volume expansion with isotonic sodium chloride solution
Encourage liberal salt intake
Initiate a beta blocker only after adequate alpha blockade, to avoid precipitating a
hypertensive crisis from unopposed alpha stimulation
Administer the last doses of oral alpha and beta blockers on the morning of surgery
86-smokers obese patient can't exercise with family history of MI came with
vague chest pain . But ECG is normal Next step ?
1- 24 hours ECG
2- exercise with ECG
3- perfusion cardiac scan
Answer : from Master the boards: perfusion cardiac scan, when the patient cant exercise .
Abnormality will be detected by seeing decreased thallium uptake.
Nb: if the question is describing a scenario in the ER, Acute chest pain do ECG and Cardiac
enzymes .
if the question is describing a scenario in the clinic or chest pain etiology is not clear or the ECG is
not diagnostic then do exercise tolerance test.
87-gunshot with wound bowel perforation , What antibiotics you should give?
Answer: from medscape:its dirty traumatic wound , Antibiotics should cover aerobic and anaerobic
organisms: Metronidazole is typically used in combination with an aminoglycoside(Gentamicin) to
provide broad gram-negative and anaerobic coverage. OR Metronidazole +Cefoxitin (a secondgeneration cephalosporin) OR Metronidazole + Cefoperazone ( a third-generation cephalosporin).
88-best imaging for cyst in the breast
1-US
2-mammogram
Answer: US
89-adult male on multiple drug came with violaceous maculopapular lesion in the trunk Diagnosis?
A- erythema multiforme
B-toxic epidermal necrolysis
Answer : it could be toxic epidermal necrolysis .

465

90-16 years old female . Fever and Chronic diarrhea for 10 months Post meal para umbilical pain
Sometimes blood mixed with stool
A-crohn
B- chronic pancreatitis
Answer A .
91-sickle cell anemia came with hepato splenomegaly And low platelets- HGB -WBC?
A- splenectomy
B-blood transfusion
Answer: A
92- man with hyperthyroidism, presented with thyroid nodule showing high radioactive uptake, how
would you manage him?
A- antithyroid medication.
B- radioactive iodine.
C- thyroidectomy.
Answer : C , from medscape: Hot nodule (high radioactive uptake) - A hot toxic nodule may require
medical therapy before surgical removal; the patient should receive suppressive doses of antithyroid
medications often require antithyroid medications before surgery.
93- C cell found in which type of thyroid cancer?
A- Papillary.
B- Follicular.
C- Medullary.
Answer: C
94- organism found in soil water?
A- schistosoma.
Answer: Ascaris lumbricoides
95- cataract surgery complication?
A- endophalmitis.
Answer : Complications that may occur with cataract surgery include:
Infection in the eye (endophthalmitis).
Swelling and fluid in the center of the nerve layer (cystoid macular edema).
Swelling of the clear covering of the eye (corneal edema).
Bleeding in the front of the eye (hyphema).

466

96- female with pustules on her face, which type of acne is this?
A- inflammatory.
B- infectious.
Answer from toronto note derma: inflammatory type 3 .
type 1: comedonal, sparse, no scarring .
type 2: comedonal, papular, moderate -+ little scarring.
type 3: comedonal,papular, and pustular with scarring.
type 4: nodulocystic acne, risk of severe scarring.
97- which one of those analgesics can makes the symptoms of acute cholecystitis worse?
A- acetaminophen.
B- morphine.
Answer: B
98- patient presented to the ER, unable to talk, his face is blue, what is the next step in the
management?
A- open mouth check for any foreign body.
99- baby with decreased air in the lung, abdomen looks scaphoid, what is the diagnosis?
A- diaphragmatic hernia.
B- dextocardia.
Answer A
100- pt with psoriasis, took a medication then developed generalized psoriasis covering all his body
surface, what is the percentage of the involved body surface?
A- 30%.
B-50%
C- 70%
D- 90%
Answer:D
Pustular psoriasis
http://www.kevinmd.com/blog/2014/05/mksap-64yearold-man-rapidly-spreading-rash.html
101- pregnant lady in the week 41 of gestation, effacement is 50%, 2 cm dilated for the past 2
weeks, now the effacement is 60%, dilated 3 cm, the fetal condition is good based of CTG findings,
what is your next step in her management?
A- give oxitocin and amniotomy.
B- give . And amniotomy.
C- give epidural then CS.
Answer from Toronto note Obstetrics , give oxitocin and amniotomy .
102- child who's lethargic and losing his concentration, Hgb is 10.5, what to give?
A- IM iron.
B- oral ferrous sulphate.
C- fortified cereal.
Answer: B
the patient is symptomatic, and the initial treatment for IDA is oral Ferrous Sulfate
http://www.fpnotebook.com/hemeonc/peds/pdtrcanm.htm
103- couples will get marry, they were relatives, which screening test should be done before
marriage?
Answer usually choose the answer which contains a test for an autosomal recessive disease.
104- child with delay in walking, on examination there is bowing in his legs, labs showing normal ca
normal phosphor and elevated alkaline phosphates, what is the diagnosis?
A- Rickets.
Answer : A

467

calcipenic rickets is often but not always associated with low serum calcium levels, while
phosphopenic rickets is characterized by low serum levels of phosphorus.
Early on in the course of rickets, the calcium (ionized fraction) is low. However, this level is often
within the reference range at the time of diagnosis,The phosphorus level is invariably low for age,
unless recent partial treatment or recent exposure to sunlight has occurred. Alkaline phosphatase
levels are uniformly elevated.
Reference:
http://www.uptodate.com/contents/overview-of-rickets-inchildren?source=outline_link&view=text&anchor=H1#H1
http://emedicine.medscape.com/article/985510-workup#c8

105- which of SSRIs drugs suitable for young age and children?
A- fluoxetine.
Answer is A
http://www.webmd.com/depression/selective-serotonin-reuptake-inhibitors-ssris-for-childhood-andadolescent-depression
106- pt with fear of automobiles?
A- specific phobia.
B- GAD.
Answer A- specific phobia.
107- female who experienced palpitation, sweating while she is in a meeting, she became anxious
and refuses to attend any meeting after that, what is the diagnosis?
A- specific phobia.
B- GAD.
Answer : B- GAD.
108- Child with painful ear and runny nose and mild cough what will u give him?
A.
Antibiotic
B.
Antihistamine and decongestant
C.
Paracetamol
Answer: A
source: http://emedicine.medscape.com/article/994656-overview
109- young pt presented with fever, vertigo, nystagmus no hearing loss, for the past 2 days,
cerebellum signs are present, what is the diagnosis?
A- central vertigo.
B- neuritis.
C- menieres.
D- BPPV.
Answer from medscape : A- central vertigo .
110- Child with red bulging tympanic membrane, ear pushed down and forward, what is the dx?
Answer : Mastoiditis - From Lecture note
111- child with generalized swelling, long scenario of nephritic syndrome you are suspecting
minimal change nephropathy, what you will find in the biopsy?
(options are long "2 lines long for each")
answer: -ve EM , focal fusion , loss of foot process
112- child with cola colored urine, which test should you perform first?
A- urinary microscopy.
B- renal function test.
C- renal biopsy.

468

Answer: A- urinary microscopy.


113- female with cervical proliferation, +ve herpes simplex virus, +ve
Chlamydia, which factor contribute to the risk?
A- human papilloma virus.
B- herpes simplex type2.
C- Chlamydia infection.
Answer : A- human papilloma virus.

114- old man with multiple fractures during the last 2 years, he is k/c of osteoporosis. What should
you give him at this stage?
A- Alendronate.
B- vitamin D.
C- estrogen.
Answer: A- Alendronate.
115- child with sickle cell, what is the lifelong treatment to prevent infections?
A- penicillin and immunization.
116- child with pain and swelling in his hands and foot (sickle cell disease),. forgot the
question..
117- child with vesicles in his oral mucosa, what is the diagnosis?
A- herpes simplex type 1.
118- condition which increasing the incidence of TB recurrence?
A- HIV.
119- pt with gram-negative diplococcic infection, which abx should be given?
gram-negative diplococci are Neisseria / antibiotic third-generation cephalosporin (eg, ceftriaxone,
cefotaxime) or pencilin .
120- pt with ECG showing bradycardia, what should you give?
A- atropine.
B- cardioversion.
check the ACLS Algorithm .

469

121- pt with obstructive lung disease (FEV1), what is your best advice for him?
A- stop smoking.
Answer:
All patients with COPD should be advised to quit smoking, educated about COPD, and given a
yearly influenza vaccination, In addition to pneumococcal polysaccharide vaccine.
pharmacological management:
- For all patients with COPD, short-acting bronchodilator (eg, beta-agonist, anticholinergic agent) be
prescribed for use as-needed for relief of intermittent increases in dyspnea
- for patient insufficient to control symptoms or who have other high-risk predictors, including severe
or very severe airflow obstruction or two or more exacerbations in the previous year,
we prefer the long-acting inhaled anticholinergic agent to the twice daily long-acting beta agonists.
- For patients who continue to have symptoms or have repeated exacerbations despite an optimal
long-acting inhaled bronchodilator regimen,
we suggest adding an inhaled glucocorticoid .
- For symptomatic patients with GOLD Stage II, III, or IV COPD, we recommend pulmonary
rehabilitation.
- long-term oxygen therapy in all patients with COPD who have chronic hypoxemia (Grade 1A).
reference:
http://www.uptodate.com/contents/management-of-stable-chronic-obstructive-pulmonary-disease#H40
122- pt with STEMI, what you will find in the ECG?

470

Answer:
https://www.heart.org/idc/groups/heart-public/@wcm/@mwa/documents/downloadable/ucm_467056.pdf
123- 45 years old lady, found a lump in the upper outer area of her right breast, it was large, firm
with irregular border, not tender, no palpable lymph node, what should you do?
A- FNA.
B- lumpectomy and radiation.
C- mastectomy.
Answer: B- lumpectomy and radiation.
124- male pt presented with testicular swelling, his abdomen was distended, hyperactive bowel,
gave a hx of constipation and vomiting, what you will do?
A- consult general surgeon.
B- consult urologist.
Answer: A- consult general surgeon.( intestinal obstruction )
125- female pt, k/c of sickle cell, experiencing acute cholecystitis attacks from time to time, on US
there is 7 gall bladder stones, one of them is 2 cm large, now she is asymptomatic, what is your
plan?
A- cholecystectomy.
126- female pt, k/c of crohns disease, presented to the ER with "symptoms of bowel obstruction due
to adhesions including constipation, vomiting " what is the diagnosis?
A- bowel obstruction.

127- 45 years old male, came for regular check up, apart from bronchial asthma, his bp is 125/80,
ha1c is 5.9, when is the next time he should check his blood glucose level?
A- 3 months
B- 6 months
C- 12 months
D- 36 months
blood glucose level daily home screening
HA1C every 3 months .
128. (3 scenarios about testicular torsion)

129- Sore throat and pancytopenia in women taking antithyroid , which one ?
A-Methimazole
B-propranolol
Answer: A-Methimazole .
130- Endometriosis definition ?

471

The presence of tissue that normally grows inside the uterus (womb) in an abnormal anatomical
location. Endometriosis is very common and may not produce symptoms, or it may lead to painful
menstruation. It has also been associated with infertility. Endometriosis occurs most commonly
within the Fallopian tubes and on the outside of the tubes and ovaries, the outer surface of the
uterus and intestines, and anywhere on the surface of the pelvic cavity. It can also be found, less
often, on the surface of the liver, in old surgery scars or, very rarely, in the lung or brain.
Endometriosis occurs in the reproductive years. The average age at diagnosis is 2530.Endometriosis may be suspected by during a physical examination; it is confirmed by surgery,
usually laparoscopy; available treatments include medication for pain, hormone therapy, and surgery
http://www.medicinenet.com/script/main/art.asp?articlekey=3240
131- Neonate was on breast feeding after 2 weeks has irritability and decrease feeding diagnosed
with meningitis .. What is the organism ?
A- Niesseria
B- listeria
C- strepococcal pneumonia ( i chose listeria )
Answer: B
GEL
G: GBS
E: E.coli
L: Listeria
http://emedicine.medscape.com/article/1176960-overview#a5

132- Best x ray to show fracture rib


A-AP
B-PA
C- Oblique
Answer: c
45 oblique view on expiration is recommended for radiographic imaging of patients with clinical
signs of fracture, e.g. evaluation of lower rib fractures, while 45 oblique view during fast breathing is
recommended for suspected upper rib fractures.
http://www.ncbi.nlm.nih.gov/pmc/articles/PMC3529706/
133- Pt with pic of anemia and thrombocytopenia - blood film showed schistocytes what dx ?
Answer : DIC

472

134- Pt came with 3 hours hx of anxiety , diaphoretic , tachypneic , what is the cause ?
A- Sympathomemitcs
B- organophostate
C- anticholinergics
Answer A

135- Kid dm type 1 brought to school clinic unconscious , last insulin dose unknown What to do ?
A- IV dextrose
B- RL
C- insulin
D- urgent referral to hospital
In the school clinic I.V set could be available :-/ so I.V dextrose 25%
136- Q about seminoma of testis
(Germ cell tumor )Malignant; painless, homogenous testicular enlargement; most common testicular
tumor, most common in 3rd decade, never in infancy. Large cells in lobules with watery cytoplasm
and fried egg appearance. placental ALP. Radiosensitive. Late metastasis, excellent prognosis.
first aid for usmle step 1
137- Pregnant 10wks has bleeding and fetus delivered , os is opened and still some remnants ?
What to do ?
With missed, incomplete, or inevitable abortion present before 13 weeks' gestation, the standard
therapy has been suction D&C (medscape).
Women with an incomplete, inevitable, or missed abortion can be managed surgically, with
medication, or expectantly. All three management approaches are effective, but treatment is
completed more quickly with surgical management and involves fewer medical visits. The choice of
method is typically based upon patient preference
138. Calculation of BMI and what degree pf obesity
Underweight BMI <18.5 kg/m2.
Normal weight BMI 18.5 to 24.9 kg/m2.
Overweight BMI 25.0 to 29.9 kg/m2.
Obesity BMI 30 kg/m2. THEN
Obesity class I BMI of 30.0 to 34.9 kg/m2.
Obesity class II BMI of 35.0 to 39.9 kg/m2. (from here and above morbid obesity)
Obesity class III BMI 40 kg/m2. This type of obesity is also referred to as severe, extreme, or
massive obesity.

473

139. Pt diagnosed as HTN and started meds came back with high glucose. Wt was he given:
- Thiazides
uptodate:
thiazide and Beta blockers are also associated with impaired glucose tolerance and an increased
risk of new onset diabetes, with the exception of vasodilating beta blockers such as carvedilol and
nebivolol( perhaps thiazide is more important here)
140. Pt diabetic for years and was just dx as htn, wt to give:
A- ACEI- pril drugs
Answer : ACE inhibitors are first-line therapy in all patients who have HF or asymptomatic LV
dysfunction, in all patients who have had an ST elevation MI, in patients with a non-ST elevation MI
who have had an anterior infarct, diabetes, or systolic dysfunction, and in patients with proteinuric
chronic kidney disease
141. Pt with high cholesterol on treatment, has muscles aches wt was she given:
A- statins
Fibrate + statins = more risk
statin alone also known to elevate liver enzymes
(Master the board 3rd edition page 69)
142. Polymyalgia Rheumatica question, wt supports the dx
Pain and stiffness in shoulders and hips, often with fever, malaise, weight loss. Does not cause
muscular weakness. More common in women > 50 years old; associated with temporal (giant
cell) arteritis.
FINDINGS :high ESR, high CRP, normal CK.

143. Typical symptoms of RA, wts the dx.


- RA
Rheumatoid arthritis is a chronic inflammatory disorder autoimmune disorder that typically affects
the small joints in your hands and feet on both sides of your body.
symptoms: tender swelling joint, morning stiffness last for hours, rhomatoid nodules, fever .
complication: osteoporosis ,carpal tunel syndrom, heart proplem,lung diseas.
blood test: high CRP, anti-cyclic citrullinate d peptide (anti-CCP) antibodies.
medication:NSAIDs. Side effects ringing in your ears, stomach irritation, heart problems, and liver
and kidney damage.
Steroids. Side effects may include thinning of bones, weight gain and diabetes.
Disease-modifying antirheumatic drugs (DMARDs).. include methotrexate (Trexall), leflunomide
(Arava), hydroxychloroquine (Plaquenil) and sulfasalazine (Azulfidine).
Side effects vary but may include liver damage, bone marrow suppression and severe lung
infections.
144. Pt with PCO (OCPs?) was on progesterone and now is off it, at risk of wt:
- endometrial ca
- cervical ca
Answer : Cervical cancer There appears to be an increased risk for developing cervical cancer
among women who have taken OCs
http://www.uptodate.com/contents/risks-and-side-effects-associated-with-estrogen-progestincontraceptives?source=preview&language=en-US&anchor=H20&selectedTitle=2~150#H20
145. N. Meningitis meningicocemia, prophylaxis for family:
-rifampin
146. Pt with tibiofibular fracture, plaster of paris applied, came back with extreme pain, wts the sign
to ur dx:
A- loss of pulse
B- loss of sensation
C- loss of movement
D- pain out of proportion

474

Answer : D
147. 18 y/o married missed her period for two months, came with rt sided abd pain wts the dx:
- ruptured ectopic
148. Similar question to previous Q but what's the test to order:
- urine hcg
149-Zone of liver affected by hypoxia
answer : zone 3
150-chromosome in DM type 2 ?
chromosome 20
151-vginal bleeding week 10 fundus 15cm closed os what is the diagnosis
_threatened abortion.

152-internal iliac injured which is affected


A- anterior thigh ms
B- ovary
C- bladder
D- anterior abdomen
Answer : C
153- RSV ttt
http://www.mayoclinic.org/diseases-conditions/respiratory-syncytial-virus/basics/treatment/con-20022497
154-female abdominal pain examination tender nodular retroverted uterus what investigation?
A- laparoscopy
B- hysteroscopy

475

C- hysterosalpengiogram
answer : A
155-viral knockout which of the following
IL2
IL3
IL4
IL5
answer : C

156-depression patient came to psychiatrist the doctor asked him about his mentality what did the
doctor examined?
A- insight
B- compliance
C- depression
Answer A
157-milestone > baby healthy run to the doctor play a role model as his father cant complete a
sen"tence cant eat with spoon
Answer: 10 months
158- Hypopigmentation and loss of sensation in forearm with ulnar nerve thickness?
Laprosy
159-Hepatitis prevented by vaccine:
-Hepatitis B.
160- women has lichen sclerosis which cancer she will get:
-squamous cell carcinoma
-Adenocarcenioma carcinoma
-Adeno-squamous carcinoma
answer :
Squamous cell carcinoma
n females, vulval lichen sclerosus is associated with an increased risk of vulval cancer or anal
cancer (squamous cell carcinoma, SCC
161- male after prostectomy through venous plexus the metastasis will go to:
A-Skull
B-Vertebral body
C-Lung
answer : B
prostatic venous plexus drains into the internal iliac vein which connects with the vertebral venous
plexus, this is thought to be the route of bone metastasis of prostate cancer.wiki
162- pic / pink patch under collar lumb:
-erythrasma
-fungal infection
- vetiligo
answer:
The patches of erythrasma are initially pink, but progress quickly to become brown and scaly (as
skin starts to shed), which are classically sharply demarcated. Erythrasmic patches are typically
found in intertriginous areas (skin fold areas - e.g. armpit, groin, under breast).
163- pt with photophobia and tearing got injured by his brother finger nail what is the treatment:
-steriod ointment
- steriod drop
- antiviral
- fitting contact lens
answer :
may put antibiotic eye drops or ointment in your eyes or use steroid eyedrops to reduce
inflammation and reduce the chance of scarring.

476

164- chid drink cow milk his hemoglobin low and MCV low which type of anemia he will has:
-Iron deficiency anemia
Answer:
Iron deficiency anemia

165- best sentence describe case-control study:


One of the most significant triumphs of the case-control study was the demonstration of the link
between tobacco smoking and lung cancer
166- child sit and support his head , laughing and cooing :
A- 4
B- 6
C- 8
D- 16
Answer:C
167- aspirin in high dose cause hyperthermia through which mechanism: (Read about it)
Salicylates are neurotoxic,
CNS toxicity is related to the amount of drug bound to CNS tissue. It is more common with chronic
than acute toxicity.
Acidosis worsens CNS toxicity by increasing the amount of salicylate that crosses the blood-brain
barrier and increases CNS tissue levels.
There is an association between taking aspirin for viral illnesses and the development of Reye
syndrome.in child
168- child has itching and all student in his class got the same infection: Sarcopits scapi
Answer :
Scabies
169- mitral stenosis cause enlargement in which chamber :
A- left atrium
B- right atrium
C- left ventricle
D- right ventricle
Answer A
Chest radiographic findings suggestive of mitral stenosis include left atrial enlargement (eg, double
shadow in the cardiac silhouette, straightening of left cardiac border due to the large left atrial
appendage
170- women in phenobarbital what you will do while breastfeeding:
A- stop drug
B- continue drug
C- ween child 3 week before starting breastfeeding.
D- stop it one month before starting breastfeeding.
Answer : B
171-which drug is safe for pregnant women:
A-cemitidin
B- cefoxizime
C- and two another choices.
Answer:a

477

478

479

172- pregnant women with UTI what is the best drug:


nitrofurantoin
173- pt with DKA he start to breath rapidly to buffer his acidosis through :
A- oxygen
B- carbon monoxide
C- carbon dioxide
D- nitrogen
Answer : C
174- in emergency department pt come with closed head trauma and loss of consciousness what is
the first thing to do:
A-intubation and hyperventilation
B-assess airway
Answer:B
175- viral gastroenteritis prevented by which vaccine :
A-Rota vaccine
Answer : a
Rotavirus vaccine
A vaccine that helps protect children against gastroenteritis caused by the rotavirus is now part of
the routine childhood vaccination schedule.
This vaccine is given as a liquid that is dropped into a baby's mouth. It is given in two doses, with
the first given at two months and another at three months.
Two possible side effects of the vaccine are diarrhoea and irritability, but these are usually mild and
short-lived.
http://www.nhs.uk/Conditions/Rotavirus-gastroenteritis/Pages/Prevention.aspx
176- child his height and weight below normal besides growth hormone what you will order:
- somatocin c
- aldosterone
- insulin
- testosterone
answer : A
178-80 years women had yellow watery foul smelling vaginal discharge:
A- bacterial vaginosis
B- trichomonas vaginalis
C- atrophic vaginitis
answer: B
179. Distichiasis?
A- Malrotation of eye lashes
B- DePigmentation of eyelashes
C- Extra row of eyelashes
answer C
Distichiasis is a rare disorder defined as the abnormal growth of lashes from the orifices of the
meibomian glands on the posterior lamella of the tarsal plate

180. Case Necrotizing fasciitis treatment?


A- Imipenem & metronidazole
B- Ampicillin & gentamicin
C- Pipracillin & tazobactam

480

D- Penicillin and smth


Answer : c

181. Pt saying that she had a protruded jaw and she wants a cosmetic surgery, when you examine
her her jaw is normal with no protrusion and you say that she doesn't need the surgery She went
already to 2 doctors who also refused to do the surgery. She is done blepharoplasty & other
cosmetic surgery before.What does she have?
A- Body dysmorphic disorder
answer: Body dysmorphic disorder
182. Papillary thyroid ca mostly associated with which of the following
Hurthle cell
http://www.nature.com/modpathol/journal/v24/n2s/full/modpathol2010129a.html
183. Pt with swelling in the thyroid , She is euthyroid what will you do ?
A- Thyroid lobectomy
B- FNA Biopsy
C- Excisional biopsy
answer : B
184-lung curve for cardiac at what level of costal cartilage:
A- 3
B- 4
C- 5
D- 6
Answer B
185-Temporal arteritis, confirmatory dx?
A- Biopsy temporal muscle
B- Biopsy temporal artery
Answer B
Superficial temporal artery biopsy (TAB) is the criterion standard for making a diagnosis of temporal
arteritis. TAB should be obtained almost without exception in patients in whom GCA is suspected
clinically
186- Fractured pelvis, injury to gonadal artery, what organ affected? (not sure of the Q)
A- ovary?
Answer: the pelvis forms one major ring and two smaller rings of bone that support and protect the
bladder, intestines and rectum.
Organ pelvic protect (Bladder, lower colon, lymph nodes, uterus, and vagina)

187- Herpes keratitis of eye, scenario with picture, how to treat?


Answer:
The mainstay of therapy is antiviral treatment either in the form of topical therapy with trifluridine 1%
eight to nine times a day or oral administration of acyclovir or valacyclovir for 10 to 14 days. If
trifluridine drops are used, care is to be taken to ensure antiviral drops are discontinued within 10-14
days due to corneal toxicity. Epithelial debridement of the dendrites may also be utilized in

481

conjunction with antiviral therapy to help reduce viral load. Topical corticosteroids are
contraindicated in the treatment of active HSV epithelial keratitis.
http://eyewiki.aao.org/Herpes_Simplex_Virus_Keratitis#Management
188-Child opaque lens with signs of inflammation?
A- Cataract
B- Neuroblastoma
C- Retinoblastoma
Answer:C
189-Pregnant, developed edema from inguinal to ankle what to give her?
A- Heparin
B- Warfarin
Answer: A
(Uptodate)Pregnancy and the puerperium are well-established risk factors for deep vein thrombosis
(DVT) and pulmonary embolism (PE), which are collectively referred to as venous thromboembolic
disease (VTE).
Initial management of suspected VTE during pregnancy depends on the degree of clinical
suspicion, whether anticoagulation is contraindicated, and whether PE, DVT, or both are suspected.
For pregnant women, we recommend adjusted dose subcutaneous low molecular weight heparin
(SC LMWH), rather than adjusted dose intravenous unfractionated heparin (IV UFH) (Grade 1B) or
vitamin K antagonists (Grade 1A). We recommend against the use of oral direct thrombin inhibitors
(eg, dabigatran) or anti-Xa inhibitors (eg, rivaroxaban, apixaban) in pregnant women (Grade 1C).
We suggest that anticoagulant therapy continue at least six weeks postpartum (Grade 2C). We
suggest a total duration of anticoagulant therapy of at least three to six months for women whose
only risk factors for VTE were transient (eg, pregnancy) (Grade 2C). Patients with persistent risk
factors for VTE may require longer therapy.
Thrombolytic therapy should be reserved for pregnant or postpartum patients with life-threatening
acute PE (ie, persistent and severe hypotension due to the PE))
190-Patient with chest pain, ischemic, presented after 6 h what to give him?
A- Aspirin
B- thrombolytics
Answer: A

192-Child with hairless spot, mother noted she was pulling her hair when stressed, what to give
her?

482

Lithium
Lorazepam
Other antiepileptic mentioned
Answer: SSRI
Citalopram
Fluvoxamine
Escitalopram
Paroxetine
Sertraline
Fluoxetine
(Uptodate) Trichotillomania is an intriguing psychosomatic entity in which there is an irresistible
desire to pull out the hair from the scalp, eyelashes, eyebrows and other parts of the body. The
process results in an instant release of tension, a sense of relief and security. However, non-scaring
alopecia is its clinical presentation. The development of trichobezoar following ingestion of the
pulled hair is its salient complication in a few cases. Subsequently, it may cause symptoms
pertaining to the gastrointestinal tract culminating in intestinal obstruction, perforation, pancreatitis
and obstructive jaundice. The Rapunzel syndrome (trichobezoar) may occur when gastrointestinal
obstruction is produced by a rare manifestation of a trichobezoar with a long tail that extends to or
beyond the ileocecal valve. In most cases in children, trichotillomania +/- trichobezoar is a habit
disorder and thus has a better prognosis. However, in adults the psychopathology is usually deeper
and thus entails a poor prognosis. The diagnosis is made after taking a thorough history, noting the
clinical features and evaluating a hair-root examination, where telogen hair is (almost) completely
lacking, which distinguish trichotillomania from other hair disorders. Treatment modalities vary in
childhood and adult varieties. Apart from psychotherapy, the drug treatment involves several agents
including selective serotonin reuptake inhibitors (SSRIs) and domipramine. Trichobezoar/Rapunzel
syndrome requires surgical intervention.
193-Case of herpes type 1, what to give?
A- Oral antiviral
B- Topical steroids
answer: A
reference:
http://emedicine.medscape.com/article/218580-medication#2
http://www.uptodate.com/contents/treatment-of-herpes-simplex-virus-type-1-infection-inimmunocompetent-patients#H26
194-Patient came to u her mother and her sister diagnosed with breast cancer, came for counseling,
what to do now?
BRCA?
MRI
PET scan
answer: US and mamogram if present + genatic counciing
195-Old with osteoporosis risk scenario, has spine compressive fracture, what to give her to
protect?
A- Bisphosphonate, calcium, vit D Other
http://www.m.webmd.com/a-to-z-guides/spinal-compression-fractures-preventing

483

196-SVT scenario what most important to test?


A- TSH
Answer
Electrolyte levels - Should be checked because electrolyte abnormalities can contribute to
paroxysmal supraventricular tachycardia (paroxysmal SVT)
CBC help to assess anemia
TSH to asses hyperthyroidism
197-MVA with shock and fracture femur, what will be increased?
A- ALP
B- CK
C- ALT
Answer A
198-Patient with fear of motor vehicle, increasing in intensity, what is the dx?
A- Specific phobia
199-Haloperidol toxicity, uprolling of eyes and dyskinesia, what is this?
A- Tardive dyskinesia
B- Neuroleptic malignant syndrome
Answer: B
(Emedicine) Administration of neuroleptic medications can result in any of the consequences listed
in Physical; however, certain combinations of medications (eg, lithium + haloperidol, anticholinergics
+ haloperidol), depot preparations (eg, fluphenazine and haloperidol), and stronger neuroleptics (eg,
haloperidol) are more likely to produce adverse effects, including neuroleptic malignant syndrome
(NMS)
200-Patient afraid of diseases and germs, what dx?
A- Obsessive compulsive
B-Anxiety disorder
C-Specific phobia
Answer: c
http://www.apa.org/monitor/julaug05/fears.aspx
201-Psychiatric patient swallowed 2 safety pins, found on duodenum what to do?
A- Immediate laparotomy
B- Admit and observe
C- Charcoal
Answer:B
202-Preventing child from drinking before bed, encourage to go to toilet before bed, all these
measures to help child with?
Enuresis

203-DM with high BP and deteriorating renal what to give?


A- ACEI (lisinopril)
Scientists have made great progress in developing methods that slow the onset and progression of
kidney disease in people with diabetes. Drugs used to lower blood pressure can slow the
progression of kidney disease significantly. Two types of drugs, angiotensin-converting enzyme
(ACE) inhibitors and angiotensin receptor blockers (ARBs), have proven effective in slowing the

484

progression of kidney disease. Many people require two or more drugs to control their blood
pressure. In addition to an ACE inhibitor or an ARB, a diuretic can also be useful. Beta blockers,
calcium channel blockers, and other blood pressure drugs may also be needed.
An example of an effective ACE inhibitor is lisinopril (Prinivil, Zestril), which doctors commonly
prescribe for treating kidney disease of diabetes. The benefits of lisinopril extend beyond its ability
to lower blood pressure: it may directly protect the kidneys' glomeruli. ACE inhibitors have lowered
proteinuria and slowed deterioration even in people with diabetes who did not have high blood
pressure.
An example of an effective ARB is losartan (Cozaar), which has also been shown to protect kidney
function and lower the risk of cardiovascular events.
http://www.medicinenet.com/script/main/mobileart.asp?articlekey=101334&page=6
204-Dyslipidemia on tx, developed muscle weakness, whats the drug hes taking?
Statins
205-Breast ca what LN you'll examine to exclude lymphatic spread?
A- Ant
B- Post
C- Lateral
Answer A
206-Polycystic kidney inheritance?
A- Dominant
207-Child above 90th percentile in height, long scenario and cardiac abnormality with fatigueetc ?
A- Marfan syndrome
208-Multiparous with cervical dysplasia, has chlamydia and HSV 2, what is the cause of her
dysplasia ?
A- Chlamydia
B- HSV
C-HPV
Answer:C
(Uptodate) Human papillomavirus (HPV) is the major etiologic agent of cervical precancer and
cancer. The association between HPV and cervical neoplasia is so strong that most other
behavioral, sexual, and socioeconomic covariables have been found to be dependent upon HPV
infection and do not hold up as independent risk factors.
HPV infection is necessary but not sufficient to develop cervical neoplasia. The two major factors
associated with development of high-grade CIN and cervical cancer are the subtype of HPV and
persistent infection. Environmental factors (eg, cigarette smoking) and immunologic influences also
appear to play a role.
Low-oncogenic-risk HPV subtypes, such as HPV 6 and 11, do not integrate into the host genome
and only cause low-grade lesions (eg, low-grade SIL and CIN 1) and benign genital warts
High-oncogenic-risk HPV subtypes, such as 16 and 18, are strongly associated with high-grade
lesions, persistence, and progression to invasive cancer, but also cause low-grade lesions.
The primary approach to prevention of CIN and cervical cancer is HPV vaccination. Although HPV
is a sexually transmitted infection, condoms are only partially protective. For women with CIN,
appropriate monitoring and treatment are used as secondary prevention of cervical cancer.

209-Humerus fracture, how will he present?


A- Waiters tip hand
B- Wrist drop
C- Claw hand
Answer:
Mid shaft humerus > radial nerve injury > wrist drop
Surgical neck > axillary nerve
Medial epicondyle > ulnar nerve > claw hand

485

Supra condylar > median n > ape hand


210. Most Important risk factor for cardiac?
A- LDL
B- total cholesterol
C- BP
D- smoking
Answer: C
211. burn pt, 70 kg, circumferential upper and lower left limbs + anterior trunk, how much fluid youll
give in the first 8 hours?
A- 6 liter
B- 8 liter
C- 10 liter
Answer A
212. AIDS patient 34 w pregnant her CD count dropped to 200 what will u do:
A- Book for CS
B- CS when spontaneous labor
C- Vaginal delivery
Answer: I think A
http://www.uptodate.com/contents/hiv-and-pregnancy-beyond-the-basics
https://aidsinfo.nih.gov/contentfiles/hivandpregnancy_fs_en.pdf
213. Pregnant lady, fall from stairs, and started to have vaginal bleeding, Diagnosis?
A- Placenta abruption
214. Young boy with pain in his knee, aspiration of fluid reveal yellowish and turbid appearance,
Diagnosis?
A- septic arthritis
215.- case of closed Fx of ulnar and radius, what's the tttx??
The following are specific indications for operative treatment:
Fracture of both bones (ie, radius and ulna)
Fracture dislocations, Monteggia fracture dislocations, and Galeazzi fracture dislocations
Isolated radius fractures
Displaced ulnar shaft fractures
Delayed union or nonunion
Open fractures
Fractures associated with a compartment syndrome, irrespective of the extent of
displacement
Multiple fractures in the same extremity, segmental fractures, and floating elbow
Pathologic fractures
http://emedicine.medscape.com/article/1239187-treatment

216. Lactating women the doctor prescribed phenytoin for seizures regarding breast feeding she
should
1 stop breast feeding
2 feed after 8 hours
Answer 1
Phenytoin may cause harm to an unborn baby, but having a seizure during pregnancy could
harm both mother and baby. Tell your doctor right away if you become pregnant while taking
this medicine.
If you become pregnant while taking phenytoin, your name may be listed on a pregnancy
registry. This is to track the outcome of the pregnancy and to evaluate any effects of
phenytoin on the baby.

486

Phenytoin can make birth control pills less effective. Ask your doctor about using non
hormonal birth control (condom, diaphragm with spermicide) to prevent pregnancy while
taking this medicine.
Phenytoin can pass into breast milk and may harm a nursing baby. You should not breastfeed while you are using this medicine.
http://www.drugs.com/phenytoin.html
217. A pt diagnosed with n. Gonorrhea what other infection you should look for ?
answer : chlamydia
(master the board)
218. A neonate has eye infection gm -ve diplococci ?
A- Iv cipro
B- IM something
C- local antibx
Ceftriaxone 2550 mg/kg IV/IM, single dose (max 125 mg)
Alt: cefotaxime, single dose
219. 64 ys male with medial enlarged prostate, all labs normal, what to do?
http://emedicine.medscape.com/article/437359-treatment
220. Best medication to give with analgesics?
Metoclopramide
Answer : i think its right because the analgesic drugs mimic peptic ulcer.

221. At inguinal canal base, what artery you can find?


a- iliac
b- femoral
answer : if they ask about artery at the base i think its femoral , but if the ask about which artery
pass through the canal the answer will be 3 arteries: artery to vas deferens (or ductus deferens),
testicular artery, cremasteric artery. But if they just asked about the base of the canal the answer will
be >> The Canal Boundaries :The anterior wall is formed by the aponeurosis of the external
oblique, and reinforced by the internal oblique muscle laterally.The posterior wall is formed by the
transversalis fascia.The roof is formed by the transversalis fascia, internal oblique and transversus
abdominis.The floor is formed by the inguinal ligament (a rolled up portion of the external oblique
aponeurosis) and thickened medially by the lacunar ligament
http://teachmeanatomy.info/abdomen/areas/the-inguinal-canal/

487

222.Child with epilepsy on anticonvulsant What you will change in his vaccines
A.Change opv to ipv
B. Dtp
C. Remove all vaccines
D. Remove all live vaccines
answer : B
source: http://www.vaccines.gov/basics/safety/should/
223.Female had vaginitis coming with fever , rash
A. Toxic shock syndrome
answer: A
source: http://emedicine.medscape.com/article/169177-clinical
224.Pt with +ve PPD for the first time and -ve CHEST X-RAY. No signs or symptoms of TB, wt to
do:
A. reassure
B. INZ for 6 mns
C. Others
Answer: B
source: http://www.uptodate.com/contents/treatment-of-latent-tuberculosis-infection-in-hiv-uninfectedadults?source=see_link
225. Child with itchy scalp and scales, other classmates affected, dx:
A.
-tinea capitis
B.
-scabies
answer : A
226. staph saprophyticus vaginal infection, whats a risk factor for it:
A.
-septicides in condoms
B.
-douching habits
Answer: A
sorce: http://cid.oxfordjournals.org/content/40/6/896.full
227. Bacteroids in gunshot wound abdomen, what antibiotics:
- clindamycin
- Others ops
Answer : Antibiotics (eg, cefotetan, metronidazole hydrochloride, gentamicin sulfate, vancomycin
hydrochloride, ampicillin sodium-sulbactam sodium)
http://emedicine.medscape.com/article/2036859-medication#showall
228. Pt diagnosed with ovarian germ cell theca , what other finding?
A- chronic salpingitis
B- endometrial hyperplasia
answer : B
hyperestrogenic findings including: hyperplastic endometrium and abnormal uterine bleeding, breast
tenderness, postmenopausal bleeding, menstrual abnormalities, and in children sexual precocity
http://www.uptodate.com/contents/sex-cord-stromal-tumors-of-the-ovary-granulosa-stromal-celltumors?source=machineLearning&search=granulosa+stromal+cell+tumor&selectedTitle=1~10&sectionRank
=1&anchor=H10#H20
229. months baby came for vaccine was delivered at 34 and his weight was low, he is gaining
weight 90 g each month i guess ! Regarding vaccines ?
A- delay 2 months from usual
B- as usual
C- half dose
Answer : B

488

keep in mind that if preterm babies get the infections that vaccines can prevent, they have a
greater chance of having disease-related problems. All of the available vaccines are safe when
given to preterm and low birth weight babies. Any side effects associated with the vaccines are
similar in both full-term and preterm babies.
https://www.healthychildren.org/English/safety-prevention/immunizations/Pages/Immunizations-ForPreterm-Babies.aspx
230. Kid with RF has PSGN with casts, you will find ?
Poststreptococcal glomerulonephritis (PSGN) is usually diagnosed based upon:
The clinical findings of acute nephritis include hematuria with or without red blood cell casts,
variable degrees of proteinuria, edema, and hypertension.
Documentation of a recent GAS infection includes either a positive throat or skin culture or
serologic tests (eg, anti-streptolysin [ASO] or streptozyme test).
Although a low C3 and/or CH 50 (total complement) level are consistent with a diagnosis of PSGN,
these complement components may also be decreased in other forms of glomerulonephritis,
including membranoproliferative glomerulonephritis.
http://www.uptodate.com/contents/poststreptococcal-glomerulonephritis
231. Sickler with VOC dehydrated and his hemoglobin is 3.5 what the next step in management ?
A- PRBcs transfusion
B- analgesia and IV fluids
Answer : IV fluid to rehydration then blood transfusion
https://www.nhlbi.nih.gov/files/docs/guidelines/sc_mngt.pdf
232. Child is doing fine, his brother died while heading to work !what we should investigate for ?
A- Hypertrophic cardiomyopathy
Familial hypertrophic cardiomyopathy is a heart condition characterized by thickening
(hypertrophy) of the heart (cardiac) muscle. Thickening usually occurs in the interventricular
septum, which is the muscular wall that separates the lower left chamber of the heart (the left
ventricle) from the lower right chamber (the right ventricle). In some people, thickening of the
interventricular septum impedes the flow of oxygen-rich blood from the heart, which may lead to an
abnormal heart sound during a heartbeat (heart murmur) and other signs and symptoms of the
condition. Other affected individuals do not have physical obstruction of blood flow, but the
pumping of blood is less efficient, which can also lead to symptoms of the condition. Cardiac
hypertrophy often begins in adolescence or young adulthood, although it can develop at any time
throughout life.
The symptoms of familial hypertrophic cardiomyopathy are variable, even within the same family.
Many affected individuals have no symptoms. Other people with familial hypertrophic
cardiomyopathy may experience chest pain; shortness of breath, especially with physical exertion;
a sensation of fluttering or pounding in the chest (palpitations); lightheadedness; dizziness; and
fainting.
While most people with familial hypertrophic cardiomyopathy are symptom-free or have only mild
symptoms, this condition can have serious consequences. It can cause abnormal heart rhythms
(arrhythmias) that may be life threatening. People with familial hypertrophic cardiomyopathy have an
increased risk of sudden death, even if they have no other symptoms of the condition. A small
number of affected individuals develop potentially fatal heart failure, which may require heart
transplantation
233. Pt with RF has Mitral regur , valve was 0.7 cm and other findings in aortic valve ? What is the
proper management ?
A-Total valve replacement
B- valvoplasty
C- balloon
Answer: http://www.uptodate.com/contents/mitral-regurgitation-beyond-the-basics

489

234.Pt reported that he was diagnosed with pancreatic cancer and received chemo and did not
improve and wants to do invx , his medical report was free (he was lying) , and when he confronted
by reports , he run ? Dx :
most probably malingering
235. Case of lady delivered macrosomic baby , What is the reliable method of diagnosing
postpartum hemorrhage ?
A- Visual assessment of blood loss
B- maternal pulse
C- hemoglobin
D- creatinine
answer : B
http://www.uptodate.com/contents/overview-of-postpartum-hemorrhage#H100822618
236. TTTof OM in a child ?
Answer: amoxicillin
Antibiotics should be prescribed for bilateral or unilateral AOM in children aged at least 6
months with severe signs or symptoms (moderate or severe otalgia or otalgia for 48 hours or
longer or temperature 39C or higher) and for nonsevere, bilateral AOM in children aged 623 months
On the basis of joint decision-making with the parents, unilateral, nonsevere AOM in children
aged 6 -23 months or nonsevere AOM in older children may be managed either with
antibiotics or with close follow-up and withholding antibiotics unless the child worsens or
does not improve within 48-72 hours of symptom onset
Amoxicillin is the antibiotic of choice unless the child received it within 30 days, has
concurrent purulent conjunctivitis, or is allergic to penicillin; in these cases, clinicians should
prescribe an antibiotic with additional -lactamase coverage
Clinicians should reevaluate a child whose symptoms have worsened or not responded to
the initial antibiotic treatment within 48-72 hours and change treatment if indicated
In children with recurrent AOM, tympanostomy tubes, but not prophylactic antibiotics, may
be indicated to reduce the frequency of AOM episodes
Clinicians should recommend pneumococcal conjugate vaccine and annual influenza
vaccine to all children according to updated schedules
Clinicians should encourage exclusive breastfeeding for 6 months or longer
http://emedicine.medscape.com/article/859316-treatment

237. child with hydrocephalus progressively increasing in between the 3rd and 4th ventricle - which
area is blocked ?
I chose cerebral aqueduct
Answer : Non-communicating hydrocephalus also called "obstructive" hydrocephalus occurs
when the flow of CSF is blocked along one or more of the narrow passages connecting the
ventricles. One of the most common causes of hydrocephalus is "aqueductal stenosis." In this case,
hydrocephalus results from a narrowing of the aqueduct of Sylvius, a small passage between the
third and fourth ventricles in the middle of the brain.
http://www.ninds.nih.gov/disorders/hydrocephalus/detail_hydrocephalus.htm
238.the foramen than present between the 3rd and 4th ventricle is?
Answer : aqueduct of sylvius
The third ventricle also communicates with the fourth ventricle through the narrow cerebral (sylvian
or mesencephalic) aqueduct.
http://www.cerebromente.org.br/n02/fundamentos/ventriiii_i.htm
239. Case of rheumatic fever -ttt?
A- Penicillin and high dose of aspirin
http://www.uptodate.com/contents/acute-rheumatic-fever-treatment-and-prevention

490

240. pic of glaucomatous cupping ,,, Answer:

http://www.optic-disc.org/tutorials/glaucoma_evaluation_basics/page13.html
241. case of multiple myeloma
Answer:
http://emedicine.medscape.com/article/204369-overview#a1
242. case of lactose intolerance
Answer :
http://www.medicinenet.com/script/main/mobileart.asp?articlekey=7809
243. Origin of right atrium embryo ?
A- Cordis in heart
B- change of transposition of GA
C-ectopic cordis
Answer: non of the above the origin of the right atrium is Sinus venosus
Primitive heart tube -has 5 dilatations.
1.truncus arteriosus-gives aorta and pulmonary trunk
2.bulbus cordis-smooth parts of rt & left ventricles
3.primitive ventricle-trabeculated parts of rt &left ventrcles.
4.primitive atrium- gives trbeculated parts of rt & left atria.
5.sinus venosus- gives the smooth part of rt atrium, coronary sinus, oblique vein of
left atrium
244. Treatment of high myopia:: name of complicated optha surgeries ?
Answer: treatment of high myopia:
Eyeglasses or
Contact Lenses or
Refractive Surgery.
245. Spontaneous lips swelling?
Answer:
Swollen lips may have a range of different causes, from mild conditions to more serious disorders,
including infections, allergy, inflammation, injuries or an underlying medical condition.
346.191-Patient with IBD, mild to moderate, what is the mainstay of treatment?
A- Surgery
B- Antibiotics
Answer :

491

For more details >


http://www.uptodate.com/contents/management-of-mild-to-moderate-ulcerativecolitis?source=outline_link&view=text&anchor=H432583317#H432583317
247. questions about meniere's disease
Answer:
http://www.mayoclinic.org/diseases-conditions/menieres-disease/basics/definition/con-20028251
248.Q45. Patient given yellow fever vaccine, started vomiting and SOB.. Cant remember the rest of
Q
Answer :
http://www.nhs.uk/conditions/yellow-fever/pages/prevention.aspx
249. A scenario about a patient who deals with flowers and got pricked by rose thorn, presented
with redness at the prick site. which of the following is the cause:
A. Sporothrix schenckii .
Answer : A

250. Hemorrhagic vesicles on tympanic membrane


Answer: Bullous Myringitis
source: http://www.msdmanuals.com/professional/ear,-nose,-and-throat-disorders/middle-ear-andtympanic-membrane-disorders/myringitis
251. F/u of blood lipids and F/u of blood sugar
Answer :
http://www.e-mercy.com/images/cholesterol-metabolism/research/2/Guidelines-to-Interpret-Results-fromHealth-Screening.pdf

252. High triglyceride what to add to statins?


Answer:
HMG CoA reductase inhibitor medications (statins) are most effective in lowering LDL cholesterol,
mildly effective in increasing HDL cholesterol, and mildly effective in lowering triglycerides.
Examples include pravastatin (Pravachol), lovastatin (Mevacor), atorvastatin (Lipitor), rosuvastatin
(Crestor), and simvastatin (Zocor).
Fibric acid derivative medications such as gemfibrozil (Lopid) and fenofibrate (Tricor) are most

492

effective in lowering triglycerides, effective in increasing HDL, and minimally effective in lowering
LDL levels.
Nicotinic acid (Niacin), known by the names Niacin, Niaspan, or Slo-Niacin, is most effective in
increasing HDL, effective in lowering triglycerides, and mildly to moderately effective in lowering
LDL levels
http://www.emedicinehealth.com/script/main/mobileart-emh.asp?articlekey=114550&page=7
253. Case of PROM at 32 weeks what to do ?
A- Sterile speculum exam
B- vaginal exam
C- chemical investigation of liquor
Answer: C if the presentation of this pt gush of fluid or leakage
(American Associated Family Medicine)

254. pt with multiple complains, CNS, GIT,, what is the diagnosis?


A- somatization.
Answer:

493

16 December Q.
1- Loss of sense of smell which lobe affected ?
A- Frontal
B- Occipital

494

C- Parietal
D- Temporal
Answer: temporal
Temporal lobesenses of smell and sound, as well as processing of complex stimuli like faces and
scenes
Frontal lobeconscious thought; damage can result in mood changes, social differences, etc. The
frontal lobes are the most uniquely human of all the brain structures.
Parietal lobeplays important roles in integrating sensory information from various senses, and in
the manipulation of objects; portions of the parietal lobe are involved with visuospatial processing
Occipital lobesense of sight; lesions can produce hallucinations
Limbic systememotion, memory, emotional expression, resolve conflict from frustration. "Odors
often trigger emotional reactions and memories" (Elaine & Katja, 2015).
Insula"island inside" pain, taste, hunger, visceral functions, social emotions, time perception and
awareness "connects to the cortex and the limbic systems" (Blanc et. al., 2014).
2- HELLP syndromes:
A- Hypertension,........, low enzyme
B- Hypertension, ,....., high enzyme
C- Hemolysis , Elevate liver enzyme, low platlet
Answer: C
3- Minimal investigation for Monitor pre-eclampsia ?
to diagnose preeclampsia, you have to have high blood pressure and one or more of the following
complications after the 20th week of pregnancy:
Protein in your urine (proteinuria)
A low platelet count
Impaired liver function
Signs of kidney trouble other than protein in the urine
Fluid in the lungs (pulmonary edema)
New-onset headaches
Visual disturbances
Previously, preeclampsia was only diagnosed if a pregnant woman had high blood pressure and
protein in her urine. However, experts now know that it's possible to have preeclampsia, yet never
have protein in the urine.
A blood pressure reading in excess of 140/90 mm Hg is abnormal in pregnancy. However, a single
high blood pressure reading doesn't mean you have preeclampsia. If you have one reading in the
abnormal range or a reading that's substantially higher than your usual blood pressure your
doctor will closely observe your numbers. Having a second abnormal blood pressure reading four
hours after the first may confirm your doctor's suspicion of preeclampsia. Your doctor may have you
come in for additional blood pressure readings and blood and urine tests.
Tests that may be needed
If your doctor suspects preeclampsia, you may need certain tests, including:
Blood tests. These can determine how well your liver and kidneys are functioning and
whether your blood has a normal number of platelets the cells that help blood clot.
Urine analysis. A single urine sample that measures the ratio of protein to creatinine a
chemical that's always present in the urine may be used to make the diagnosis. Urine
samples taken over 24 hours can quantify how much protein is being lost in the urine, an
indication of the severity of preeclampsia.
Fetal ultrasound. Your doctor may also recommend close monitoring of your baby's growth,
typically through ultrasound. The images of your baby created during the ultrasound exam
allow your doctor to estimate fetal weight and the amount of fluid in the uterus (amniotic
fluid).
Nonstress test or biophysical profile. A nonstress test is a simple procedure that checks how
your baby's heart rate reacts when your baby moves. A biophysical profile combines an

495

ultrasound with a nonstress test to provide more information about your baby's breathing,
tone, movement and the volume of amniotic fluid in your uterus
mayo clinic
4- Parafollicular cell secrete which type of tumor?
A.Follicular
B.Medullary
Answer: B
Parafollicular cells secrete calcitonin, a hormone that lowers the level of calcium in the blood
5- scenario about short stature, webbed neck what's chromosome?
Answer: 45X ( Turner Syndrome )
6- scenario about Down syndrome: single palmar crease...etc what's the diagnosis ?
Down syndrome
7- athletic come for check up all thing normal except Xanthelasma on achllis tendon and
cholesterol?
-LDL receptor
-Apo ll
-Apo c
Answer: if they asked about mutation the answer will be LDL receptor
Associated with severe hypercholesterolaemia and elevated LDL levels
Patients with xanthelasma had higher levels of cholesterol, LDL-C, and apo B, and lower levels of
HDL2-C than control subjects. The prevalence of the apo E4/E3 phenotype was higher in cases
than in controls (P < 0.05). Patients with xanthelasma had a higher prevalence of personal and
familiar history of cardiovascular disease and were more overweight than control subjects. A
stepwise discriminant analysis disclosed an independent association of xanthelasma with lower
HDL-C, HDL2-C, and HDL3-C levels in men, and with higher total cholesterol and lower HDL2-C
levels in women
8- which type of anemia associated with chemotherapy and Radiotherapy ?
Answer: Aplastic anemia
9- infant periumbilical hernia , what you will ?
A- put plastic in mid abdomen
B- reassurance
C- do hernia repair before start school
Answer: reassurance
10- what is papanicolaou smear ? ( Choices: how many sample and how many area ?)
answer: http://www.uptodate.com/contents/screening-for-cervicalcancer?source=search_result&search=pap+smear&selectedTitle=1~139#H3213283

11- definition of insomnia?


answer: According to guidelines from a physician group, insomnia is difficulty falling asleep or
staying asleep, even when a person has the chance to do so. People with insomnia can feel
dissatisfied with their sleep and usually experience one or more of the following: fatigue, low energy,
difficulty concentrating, mood disturbances, and decreased performance in work or at school
12- calculates the deficit for child case?

496

13- treatment of juvenile rheumatoid arthritis?


answer: Pharmacologic management consisting of:
nonsteroidal anti-inflammatory drugs (NSAIDs),
disease-modifying antirheumatic drugs (DMARDs),
biologic agents,
intra-articular and oral steroids
source: http://emedicine.medscape.com/article/1007276-treatment
15- treatment of orbital pseudotumor?
A- Radiation
B- Antibiotics
C- surgical excision
D- Systemic steroid
answer : D
source:
http://eyewiki.aao.org/Nonspecific_Orbital_Inflammation_(Idiopathic_orbital_inflammation,_Orbital_inflam
matory_syndrome,_Orbital_pseudotumor)
16- type of elbow joint
A- hinge
B- saddle
C- Pivot
Answer: A
source: http://teachmeanatomy.info/upper-limb/joints/elbow-joint/
17- ECG finding of ostium secundum atrial septal defect
A- LT axis deviation
B- RBBB
C- LVH
D- Delta wave
answer: B
source: http://www.metealpaslan.com/ecg/asden.htm
18- 2 Q about meningitis how to interpretation of types of meningitis and how to treat children with
meningitis?
source: http://emedicine.medscape.com/article/961497-treatment#d10
19- about cyanotic heart disease.
A- TOF
B- ASD
C- VSD
D- PDA
Answer: A

497

source: Master the boards: USMLE step 2, Second Edition, pediatric page 412
20- baby sit briefly, crawl , move object from hand to hand , but can't do pincer grasp
A- 4 month
B- 6 month
C- 7 month
B - 9 month
answer: B (Toronto notes 2014 P5)
21- 42 years old female complaining of amenorrhea, night sweat and flushing for the last 6 months.
What is the most likely diagnosis?
A.
Hypothyroid
B.
Hypoprolactinemia >> new choices
C.
Congenital adrenal Hyperplasia
D.
Pheochromocytoma
Answer : A
Primary hypothyroidism will lead to decreased level of T3, T4 and increase level of TSH. TSH and
FSH both of them have the same alpha unit, so when level of TSH is increased it will go and attach
to FSH receptor and will work like FSH, this will lead to appear of menopause symptoms.
22- MVA when to do diagnostic peritoneal lavage:
A- Hypotensive patients.
B- All MVA patients.
C- Unconscious patients with severe head injury.
D- Conscious patients with abdomen pain.
answer : A
DPL can be used to evaluate both blunt and penetrating abdominal trauma in patients who are
hemodynamically unstable or who require urgent surgical intervention for associated extraabdominal injuries. DPL can rapidly confirm or exclude the presence of intraperitoneal hemorrhage.
Thus, the patient with a closed head injury, the unstable patient who has been in a motor vehicle
accident, or the patient with a pelvic fracture and potential retroperitoneal hemorrhage can be
appropriately triaged to emergency laparotomy.
http://emedicine.medscape.com/article/82888-overview
23- Location of saphenous vein:
A- Anterior to medial malleolus.
B- Posterior to medial malleolus.
C- Posterior to lateral malleolus.
D- Anterior to lateral malleolus.
answer A
http://emedicine.medscape.com/article/80393-overview

498

24- baby thirsty with tachycardia, sunken eye.. volume loss:


A.1%
B.<3%
C.5-9%
D.9%
Answer : D
Symptom
Degree of Dehydration
Mild (< 3% body
weight lost)

Moderate (3-9% body


weight lost)

Severe (>9% body weight


lost)

Mental status

Normal, alert

Restless or fatigued,
irritable

Apathetic, lethargic,
unconscious

Heart rate

Normal

Normal to increased

Tachycardia or
bradycardia

Quality of pulse

Normal

Normal to decreased

Weak, thready, impalpable

Breathing

Normal

Normal to increased

Tachypnea and hyperpnea

Eyes

Normal

Slightly sunken

Deeply sunken

Fontanelles

Normal

Slightly sunken

Deeply sunken

Tears

Normal

Normal to decreased

Absent

Mucous
membranes

Moist

Dry

Parched

Skin turgor

Instant recoil

Recoil < 2 seconds

Recoil >2 seconds

Capillary refill

< 2 seconds

Prolonged

Minimal

Extremities

Warm

Cool

Mottled, cyanotic

http://emedicine.medscape.com/article/801012-clinical#showall
25- baby ride tricycle draw circle but cant draw square :
A.3 years
B.4 years
answer A
illustrated textbook
27- baby said hi when he entered the clinic, imitates his mother, feeds his doll, refers to himself
ME and say eye:
A.12 months
B.15 months
C.18 months
D. 24 months
answer : D
illustrated textbook
28- which dye used to examine the cornea ?
fluorescein dye
Reference: https://www.nlm.nih.gov/medlineplus/ency/article/001017.htm

499

29- Baby with cellulitis and purple rash:


Staph aureus
Strep pneumococcus
Group A beta hemolytic strep
answer C ?
In individuals with normal host defenses, the most common causative organisms are group A
streptococci (GAS) and S aureus. Group B Streptococcus cellulitis occurs in infants younger than 6
months, because their immune responses are not fully developed, and it may also be seen in adults
with comorbidities such as diabetes or liver disease. For infantile cellulitis, presentations may
include sepsis.
http://emedicine.medscape.com/article/214222-overview#a4
30- patient with pertussis best swap
A- Nasal swab
B- Nasopharyngeal
C- Tracheal
Answer B
The culture specimen should be obtained by using deep nasopharyngeal aspiration or by holding a
flexible swab (Dacron or calcium alginate) in the patient's posterior nasopharynx for 15-30 seconds
or until a cough is produced.
http://emedicine.medscape.com/article/967268-workup#c9
31- woman underwent lung lavage hemosiderin laden macrophages seen indicates.. ?
A Heart failure
answer A
heart failure lead to diffuse alveolar hemorrhage syndromes
http://www.uptodate.com/contents/the-diffuse-alveolar-hemorrhagesyndromes?source=see_link&sectionName=Bronchoalveolar+lavage&anchor=H12#H12
32- MVA patient with spleen injury stage 1 vitally stable:
Observation in ICU
Observation in surgical ward
answer : B
Hemodynamically stable Hemodynamically stable patients with low-grade (I to III) blunt or
penetrating. splenic injuries without any evidence for other intra-abdominal injuries, active contrast
extravasation, or a blush on CT, may be initially observed safely. In general, patients who meet the
criteria for observation but who require intervention to manage extra-abdominal injuries (eg, leg
fracture stabilization) can also be safely observed.
http://www.uptodate.com/contents/management-of-splenic-injury-in-the-adult-trauma-patient
33- epileptic patient with gingival bleeding and white gain which medication:
a-phenytoin
answer A
http://reference.medscape.com/drug/dilantin-phenytek-phenytoin-343019#5
http://emedicine.medscape.com/article/1076264-overview#showall
34- patient taking carbamazepine developed generalized rash and peeling of epidermis:
answer Steven Johnson syndrome
http://emedicine.medscape.com/article/1197450-overview#a5

500

35- patient with HTN 3 readings N; 146, K:3:


Primary hyperaldosteronism.
Primary HTN
. answer A
http://emedicine.medscape.com/article/127080-workup#showall
36- Patient with fracture, constipation and abdominal pain, high serum CA:
A Hyperparathyroidism
Answer: A
source: Master the boards USMLE step 2 CK
37- patient with bilateral abdominal mass:
a.
Polycystic kidney disease.
Answer: A
source: http://emedicine.medscape.com/article/244907-overview
38- patient with BPH best investigation:
A.
Cystoscopy
B.
Annual renal function
C.
Annual prostate antigen
D.
Beta-blocker
answer : C
source: http://emedicine.medscape.com/article/437359-workup
and Toronto notes 2014 FM17
39- patient with metabolic acidosis with high anion gap (aspirin toxicity)
answer:
Activated charcoal and Alkaline diuresis with extra KCl
source: http://www.merckmanuals.com/professional/injuries;-poisoning/poisoning/aspirin-and-othersalicylate-poisoning
40- female with severe pain during her period and heavy bleeding on examination nodules in
uterosacral ligament:
a- Endometriosis.
Answer: A
Signs and symptoms of Endometriosis: Cyclic pelvic pain, abnormal heavy bleeding and nodular
uterus or adnexal masses.
Diagnosis: laparoscopy (dark brown clusters of lesions called Endometrioma Chocolate Cyst)
Treatment: NSAIDs, OCPs, Danazol androgen derivative, leuprolide acetate leupron both are
used to decrease FSH & LH.
source: Master the boards: USMLE STEP 2 CK
41- 25 years old female with 2 cm mass in upper right breast movable and firm negative family
history, for 2 months:
A.
fibroadenoma.
Answer: A
source: Toronto Notes 2014
42- 40 years old female with 3 months mass 2.5 cm firm, mammogram and
US normal what to do ?
A.
follow up after two cycles
B.
obtain biopsy
Answer: B
Source: answered by GS resident.

501

43- female delivered her baby 4 months ago breastfeeding needs contraception and concerned
about not having her period?
A.
Reassure and counsel about contraception
B.
Order prolactin level
Answer: B
Women who breastfeed have a delay in resumption of ovulation postpartum. This is believed to be
due to prolactin-induced inhibition of pulsatile gonadotropin-releasing hormone release from the
hypothalamus.
source: http://www.uptodate.com/contents/overview-of-postpartum-care
44- female with right upper abdomen pain and fever no jaundice.. what is the management :
A.
Emergent surgery.
B.
Iv fluid and antibiotics.
C.
Discharge
Answer: A
Definitive treatment: Surgery within 3 days.
Initial management: IV fluids and Abx.
source: Toronto Notes 2014
45- patient with history of 2 years depression, decreased appetite, low self- esteem
A.
Major depression
B.
Depression something
C.
Dysthymia
Answer: C
source: Toronto notes 2014 PS11
46- 60 years old female with distal phalangeal joint swelling and shoulder pain and knee pain
x ray showed narrow joint space and osteophytes:
A- Rheumatoid arthritis
B- Osteoarthritis
Answer : B Toronto notes
47- patient had throat infection 2 weeks ago was developed hematuria how to treat:
Corticosteroids
Thiazide
Answer: C
the diagnosis is post strep glomerulonephritis
During the acute phase of the disease, restrict salt and water. If significant edema or hypertension
develops, administer diuretics. Loop diuretics increase urinary output and consequently improve
cardiovascular congestion and hypertension.
For hypertension not controlled by diuretics, usually calcium channel blockers or angiotensinconverting enzyme inhibitors are useful. For malignant hypertension, intravenous nitroprusside or
other parenteral agents are used.
Other features of therapy are as follows:
Indications for dialysis include life-threatening hyperkalemia and clinical manifestations of uremia
Restricting physical activity is appropriate in the first few days of the illness but is unnecessary once
the patient feels well
Steroids, immunosuppressive agents, and plasmapheresis are not generally indicated
Reference:
http://emedicine.medscape.com/article/240337-treatment#d8

502

48- treatment of chronic pain ?


treatment include:
Physiotherapy, NSAIDs and Acetaminophen, Antidepressants, Anticonvulsants, Muscle relaxant
and Opioids.
answer: https://www.asra.com/page/46/treatment-options-for-chronic-pain
49- Child with mild jaundice ,splenomegaly and echogenicity in the gallbladder .what is the type of
anemia?!
A.Sickle cell anemia
B.Thalassemia
Answer: A Toronto notes
50-potential side effect of amitriptyline:
A: Dystonia
B: Weight gain
Answer: B Dizzy, Drowsiness, Dry Mouth, Headache, Increased Hunger, Taste Problem, Weight
Gain http://www.webmd.com/drugs/2/drug-8611/amitriptyline-oral/details/list-sideeffects
51- A patient had weight gain, because she cant taste the food or smell. Examination is normal,
she was seen by neuro and psychiatry with no diagnosis. Whats her diagnosis?
A.Meningioma
B.Aneurysm
C.Malingering
Answer: C The most common goals of people who malinger in the emergency department are
obtaining drugs and shelter. In the clinic or office, the most common goal is financial compensation.
and choices A&B can be diagnosed easly.
http://emedicine.medscape.com/article/293206-clinical
52- A patient with heartburn taking antacids. She had rheumatic fever 1 week ago and was started
on aspirin. What side effect she can develop?
A- Constipation
B- Diarrhea
C- Dry mouth
D- Galactorrhea
Meaning of Q is SE of which DRUG!! antacid or aspirin??
Answer: Aspirin SE: Conditions of Excess Stomach Acid Secretion, Nausea, Vomiting, Heartburn,
Irritation of the Stomach (Cramps)
Antacid SE: cause nausea, constipation, diarrhea, or headache. Diarrhea is more common with this
product than constipation http://www.webmd.com/drugs/2/drug-76860-769/antacid-oral/aluminummagnesiumantacid-simethicone-oral/details#side-effects
53- Regarding the previous case, what should be given ?
A.
Misprostol
Answer: A
Prevents stomach ulcers caused by nonsteroidal anti-inflammatory drugs (NSAIDs).
source: https://en.wikipedia.org/wiki/Misoprostol
54-Treatment of trigeminal neuralgia?
A- Prednisiolone
B- Nalaxone
Answer :Carbamazepine and oxcarbazepine are considered first-line therapy in trigeminal neuralgia
(TN) http://emedicine.medscape.com/article/1145144-treatment#d9

503

55- Mountain climber who has hypoxia, which of the following liver zones is most affected by
hypoxia ?
A- Central of acini zone II
B- Peripheral of acini zone II
C- Sinusoidal
answer : Zone 1 encircles the portal tracts where the oxygenated blood from hepatic arteries enters.
Zone 3 is located around central veins, where oxygenation is poor. Zone 2 is located in between.
56- Patient with vesicles in forehead and supraorbital region for one day, what will u do?
A.Antiviral
B.Antiviral and refer to ophthalmic
C.Reassure
Answer: A Varicella is self-limited in healthy children. A vaccine is available for infants, children, and
adults and is routinely used for disease prevention. Adults should be treated with systemic acyclovir.
Although acyclovir may speed the cutaneous course of zoster, pain control is most important for
patients with this disease. First aid step2 ck
57- Patient with hyperpigmented non pruritic papules in the dorsum of the hands not resolved with
antifungal
A.
tinea corporis
B.
lichen planus
Answer: B?
lichen planus is hyperpigmented lesions thats doesnt resolved by antifungals but it cause pruritis!!
Treatment: Topical corticosteroids with occlusion or intradermal steroid injections.
source: Toronto notes 2014 D17
58- Which of the following is a tumor marker for prostate cancer?
A.
CEA
B.
alpha fetoprotein
C.
acid phosphatase
Answer: Prostate specific antigen PSA http://www.cancer.gov/about-cancer/diagnosisstaging/diagnosis/tumor-markers-fact-sheet
59- Vwb deficiency which will be affected ?
In the great majority of cases, vWD is an inherited condition. The vWF gene is located near the tip
of the short arm of chromosome 12. The gene is composed of 52 exons and spans a total of 180kb
of the human genome; therefore, it is similar in size to the FVIII gene. Expression of the vWF gene
is restricted to megakaryocytes, endothelial cells, and, possibly, placental syncytiotrophoblasts. A
partial, nonfunctional duplication (pseudogene) is present on chromosome 22.
vWF exists as a series of multimers varying in molecular weight between 0.5-kd (dimer) and 20
million kd (multimer). The building blocks of multimers are dimers, which are held together by
disulfide bonds located near the C-terminal end of each subunit
60- clear case of absence seizure then asked what is going to happen to this patient if given
fentanyl :
A- glutamate receptors activation
B- seizure activity due to toxic neurotransmitters release
C- demyelination.
Answer: B
Epidural Fentanyl is local anesthetic : MOA Due to its high lipid solubility it rapidly binds dorsal horn
receptors in the spinal cord. G- protein coupled receptors & inhibit adenylate cyclase
NB: its not related to absence seizure.

504

61- A case of pediatric nephrotic syndrome with edema, what will you give him ?
A.steroid trial
B.diuretic
Answer: A most common in children is minimal change dis treat it by Steroids. First aid step2 ck
62- A patient was stabbed in his abdomen, o/e hes vitally stable, and some of the mesentery is out.
what will u do?
A.exploratory laparotomy
B.wound exploration
C.observe
Answer: A
63- Seven weeks pregnant lady c/o vaginal bleeding with tissue. Her cervix was open and you can
see some product of conception. Her fundal height is equal to 7 to 8 weeks.
A.
Threatened abortion
B.
Incomplete abortion
C.
Missed abortion
answer : B
complete abortion: no product of conception found.
incomplete abortion: some product of conception found
inevitable abortion: product of conception intact, dilated cervix, vaginal bleeding.
threatened abortion: product of conception intact, no cervix dilatation, intrauterine bleeding.
missed abortion: death of fetus, but all products of conception present in uterus.
septic abortion: infection of the uterus and surrounding area.
Source: Master the boards USMLE step 2 CK
64- child take overdose of isoniazid and toxicity symptoms?
Answer: seizure include status epilepticus
http://www.uptodate.com/contents/isoniazid-inhpoisoning?source=outline_link&view=text&anchor=H25#H25
65- When does a pregnant patient do GDM ?
A.
12 weeks
B.
16 weeks
C.
20 weeks
D.
28 weeks
Answer: D (24 -28 weeks)
Source: toronto notes 2014 OB6
66- Patient with exophthalmos and swollen lids and you can feel its pulse, TFT
normal. Whats your diagnosis ?
A.
Hyperthyroid
B.
Cellulitis
C.
Cavernous sinus thrombosis
Answer: C
source: The wills eye manual 6th edition

505

67- A patient with incontinence, distended bladder and difficulty to initiate urination:
A.
Reflex
B.
Urge
C.
Overflow
D.
Stress
Answer: C

68- A mother took her child to the well baby clinic, he was laughing with his mother. When the
doctor came he cried. How old is he ?
A.
2 mo
B.
4 mo
C.
6 mo
Answer: C stranger anxiety
source: Toronto notes 2014 P5
69-Definition of postpartum hemorrhage:
A.
More than 500 ml post SVD
B.
Less than 500 ml post CS
C.
More than 500 ml post CS
D.
Less than 500 ml post SVD
Answer: A
postpartum hemorrhage: loss of >500 ml post SVD or >1000 ml post CS
Source: Toronto notes 2014 OB 48
70-Positive predictive value means?
Answer: is the probability that subjects with a positive screening test truly have the disease
71- Which of the following organisms is seen in patients with chronic granulomatous disease?
A.Cl. Difficle
B.Staph aurues
Answer: B
72- Mother with GBS and had a baby who has irritability and agitation and fever. What will you do?
A.Give antibiotics
B.Do cultures
Answer:B http://www.cdc.gov/groupbstrep/about/symptoms-diagnosis-treatment.html

506

73- A case of pyelonephritis, what is the next step?


A.Admit and give antibiotics
B.Do investigations
C.Give him antibiotics at home
Answer: Treatment with fluids and oral antibiotics may be given on an outpatient basis if children are
not vomiting and not markedly ill so it depend on the case. It is prudent to order urinalysis (and urine
culture in those with abnormal findings) in all febrile boys younger than 6 months and febrile girls
younger than 24 months with fever lasting more than 48 hours. medscape
74- 25 years old lady presented with severe abdominal pain and regular menstruation, in US mass
in the ovary with hair, what is the next step?(dermoid cyst)
A.
Laparoscopy
Answer: A
75- You have a patient and you took her permission to examine her. What are you doing?
A.Taking informed consent
B.Being efficient in you job
Answer: The very act of a patient entering a doctor's chamber and expressing his problem is taken
as an implied (or implicit) consent for general physical examination and routine investigations. But,
intimate examination, especially in a female, invasive tests and risky procedures require specific
expressed consent. Expressed (explicit) consent can be oral or written.
76- A patient was injured and now has scrotal pain, whats the most likely diagnosis?
A.Testicular torsion
Answer:A http://www.uptodate.com/contents/evaluation-of-the-acute-scrotum-inadults?source=machineLearning&search=scrotal+pain&selectedTitle=1~25&sectionRank=1&anchor=H67141
0222#H2012423711
77- A patient with unilateral parotid swelling, hes post-cholecystectomy. Saliva was cloudy ( I think)
Culture from parotid saliva was negative?
A.Sarcoid granuloma bi
B.Bacterial culture negative
C.Cancer no constitutional
D.Sjogren syndrome bi
Answer: Acute sialadenitis?
The most common causes are postoperative dehydration, radiation therapy, and
immunosuppression (eg, diabetes mellitus, organ transplant, chemotherapy, human
immunodeficiency virus)
http://www.uptodate.com/contents/salivary-gland-stones
78- there is TB outbreak in a region, however you tested one patient and you found him to be
negative for TB, what are you going to give him:
A- Bacillus Calmette-Gurin vaccine
B-Rifampin chemoprophylaxis
Answer:B

507

79- feature of severe depression?


Answer:
psychotic: with hallucinations or delusions
chronic: lasting 2 yr or more
catatonic: at least two of: motor immobility, excessive motor activity, extreme negativism or mutism,
peculiarities of voluntary movement, echolalia or echopraxia
melancholic: quality of mood is distinctly depressed, mood is worse in the morning, early morning
awakening, marked weight loss, excessive guilt, psychomotor retardation
atypical: increased sleep, weight gain, leaden paralysis, rejection hypersensitivity
Source: Toronto notes 2014 PS10
80- baby was delivered 30 weeks and has resp symptoms what will be the main cause ?
A- asthma
B- decreased pulmonary surfactant
Answer: B
81- knee reflex?
Answer: L 3-4 but mainly L4
82- Looks of dorsiflexion of foot which nerve injury?
pt can't do dorsiflexion & eversion:
A)common peroneal N
B)Deep perineal N
Answer: B http://www.getbodysmart.com/ap/muscularsystem/footmuscles/fibularistertius/tutorial.html
83- multiple sclerosis Present to ER what to give?
answer : IV methylprednisolone
84- cystic fibrosis mode of inheritance?
Answer: autosomal recessive
85- smoker came with whitish patches - what to do (I think: Stomatitis nicotina)
A- excision biopsy
B- antibiotic
C- nothing to do
D- close follow up
Answer: A you have to do a biopsy to exclude Ca http://www.dermnetnz.org/reactions/nicotinestomatitis.html
86- A case of heat stroke?
A-warm fluid
B-core cooling whole body
C-electrolytes replacement
Answer : B uptodate http://www.uptodate.com/contents/exertional-heat-illness-in-adolescents-andadults-management-and-prevention?source=see_link#H12884520
87- a clear case of septic arthritis - high WBC high ESR?
A.aspiration
B.ASO
Answer: A definitive diagnosis UpToDate
88- Triple therapy for gastric ulcer in 10 yo boy?
Answer: Triple therapy regimens for H pylori consist of a PPI, amoxicillin, and clarithromycin for 7-14
days.

508

89- carpal tunnel syndrome .. pt can't work and write which muscles affected:
A- Thenar eminence
B- interossei palmar muscle
C- interossei dorsal muscle
Answer: A Compression of the median nerve as it runs deep to the transverse carpal ligament (TCL)
causes atrophy of the thenar eminence, weakness of the flexor pollicis brevis, opponens pollicis,
abductor pollicis brevis
90- pt has a cat and then she C/O eye pain and erythema in the eye ?
A- allergic conjunctivitis
B- contact dermatitis
C- bacterial conjunctivitis
Answer: A
91- what is the cause of death due to flames fire :
A.
smoke inhalation injuries
Answer: A
source: http://www.nfpa.org/press-room/reporters-guide-to-fire-and-nfpa/consequences-of-fire
92- a case of myocardial infarction .. what is the complications if he didn't managed after six hour ?
A.myocardial rupture
B.arrhythmias
Answer:
Pericarditis:
Post infarction pericarditis usually begins several days after the infarct, due to an inflammatory
exudate in the pericardium.
Ventricular septal rupture:
Acute ventricular septal rupture can occur usually several days following the acute infarction, due to
softening of the necrotic portion of the septum.
Complications may occur due to ischemic or injured tissue and therefore may begin within 20
minutes of the onset of M.I., when myocardial tissue injury begins. These complications include
arrhythmias and heart block (due to injured or ischemic conduction system tissue), and hypotension
and congestive heart failure (due to ischemic or injured muscle tissue, resulting in abnormal filling
{"diastolic dysfunction"} or abnormal emptying {"systolic dysfunction"}).
Referance:
http://www.brown.edu/Courses/Bio_281-cardio/cardio/handout4.htm
93-A patient presented with history of repetitive coles of his garage around 10 times and moved
around his house a lot. What is the most likely diagnosis?
a. obsessive compulsive disorder
94- A patient presented with history of panic attack of job meeting and she become anxious. What is
the most likely diagnosis?
A- specific anxiety disorder
B- panic attack
Answer: A
95- a case of painful nodules in groin area and pain relieved after punching and discharge came out
of it:
A.Hidradenitis suppurativa.
B.furunculus
Answer: A http://dermnetnz.org/acne/hidradenitis-suppurativa.html
96- primary hyperparathyroidism what will be the lab results:
Answer: calcium and PTH both high
Reference : uptodate

509

97- baby come with ear pain and Discharge, in examination erythema and edema in the ear canal
what is the diagnosis ?
A- otitis externa
Answer: A
Source: http://emedicine.medscape.com/article/994550-overview
98- Which bone make the ankle joint ? ( every option was made of 3 bones)
The true ankle joint is composed of three bones, anterior view: the tibia which forms the inside, or
medial portion of the ankle; the fibula which forms the lateral or outside portion of the ankle and the
talus underneath.
The true ankle joint is responsible for the up-and-down motion of the foot.
99- mechanism of action of inhaled antivirus?
block the ion channel function of the M2 protein of influenza A virus , thus interfering with
corresponding specific steps in the viral life cycle. The neuraminidase inhibitors are novel drugs,
designed on the basis of the three-dimensional structure of the influenza A and B neuraminidase
The mechanism by which antiviral drugs interrupt the replicative cycle of influenza is illustrated. M2
inhibitors prevent the M2-mediated acidification of the interior of the virus while it resides in
endosomes and the subsequent uncoating of the viral genome, thus inhibiting viral replication.
Neuraminidase inhibitors (NAIs) prevent cleavage of sialic acid residues and thus newly formed
virus cannot be released from the cell surface to infect adjacent cells; also, virus particles remain
associated to one another.
100- lateral movement of one eye of a child when you close the other eye?
A-squint (strabismus)
B- nystagmus
Answer: A. By the direction of the squinting (turning) eye:
An eye that turns inwards is called an esotropia.
An eye that turns outwards is called an exotropia.
An eye that turns upwards is called a hypertropia.
An eye that turns downwards is called a hypotropia.
101- You performed a pudendal nerve block on a woman in labor, which of the following structures
will be fully sensitive and not blocked by the anesthesias?
A. Perineal body
B. Urogenital diaphragm
C. Rectum
Answer:C perineum, vulva, scrotum/vagina those are blocked.
uptodate
102- Man got new glass , after one week he cannot see well by the new glass , after examination
change the glass and referred to ophthalmologist.. What is the cause of change the old glass?
A. Glucose in lens
B. Cataract
C. Glaucoma
d- keratitis
Answer : A
From ophthalmic consultant
103- Old lady postmenopausal with osteoarthritis and risk for osteoporosis, what you will do:
A. calcium ,TSH ,dihydroxy vit D
B. bisphosphonate, vit D, calcium
C. DEXA scan
Answer: C We recommend pharmacologic therapy for postmenopausal women with a history of
fragility fracture or with osteoporosis based upon bone mineral density (BMD) measurement (Tscore -2.5)

510

Uptodate
104- Duchenne muscular dystrophy:
Answer: Proximal muscle wasting
105- Cover test +, what is the complication of the condition?
Answer: Amblyopia
the patient has Squint and the treatment is batching but if the treatment was ignored or delayed the
patient will develop amblyopia.
106- pt with polyhydramnios what atresia?
A.kidney
B.esophagus
C. Duodenal
Answer: C

107- surgeon take graft from rectus muscle which artery should be dissected?
A.superior epigastric
B.inferior epigastric
C.superficial epigastric
Answer B The deep superior epigastric vessels are not used as the pedicle for the free flap
because it is of smaller caliber than the inferior vessels and a greater amount of skin can be
harvested with the inferior system. medscape
108-Vaccine of Hep. A missed second dose what to do ?
Answer: have it as soon as possible but you don't need to start with the first dose again
source: family medicine Consultant.
109- Painless genital ulcer + lymph nodes enlargement ?
A.syphilis (chancre)
Answer: A
110- ECG showing ventricular tachycardia the patient is unstable what is the management ?
Answer:
Hemodynamic compromise: electrical cardioversion.
No hemodynamic compromise: electrical cardioversion, lidocaine, amiodarone, type Ia agents
(procainamide, quinidine).
Source: Toronto notes 2014 C20

511

111- pt with rt kidney 14 Cm and left kidney 7 cm, Arteriography: Renal artery stenosis
what to do?
A.CT angio (same as Arteriography)
B.Ct abdomen
C.Biopsy
Answer: Treat http://www.uptodate.com/contents/treatment-of-unilateral-atherosclerotic-renal-arterystenosis
112- A scenario about a patient who deals with flowers and got pricked by rose thorn, presented
with redness at the prick site. which of the following is the cause?
A. Sporothrix schenckii .
Answer is A
Sporotrichosis (also known as "Rose gardener's disease"[1]) is a disease caused by the infection of
the fungus Sporothrix schenckii
Reference: https://en.wikipedia.org/wiki/SporotrichosiS
113- A patient who presented with ciliary flush, bilateral eye redness and pain. Examination
revealed keratic precipitate and presence of cells in the anterior chambers (classical scenario of
uveitis) what is the treatment:
A.
systemic corticosteroid
B.
cyclopentolate with topical steroids
Answer: B
source: Toronto notes 2014 OP20
114- a three weeks old boy presented with scrotal asymmetry. In examination both testicles were
palpable in the scrotum. (thats it, with no more information). What are you going to do next in
examination:
A- trans-illumination test
B- let the patient to cry/cough to see if there's any bulging
Answer: the answer is Reassurance!! because anatomically one of the testicles hanged higher than
the other! but if you got no choices except these two choose B. or you can do UltraSound.
115- patient presented with foreign body sensation in his eyes, itchiness and grittiness. (not sure if
mentioned watery discharge or no). he is taking (a drug) and antihistamine. Also he mentioned that
he sit in front the screen for six hours daily. What is the cause of his disease? ( the scenario was not
very clear whether it is allergic or just simple dryness)
Amast cell degranulation and histamine release.
BCorneal dryness
Answer: B
Source: http://www.webmd.com/eye-health/eye-health-dry-eyes
116- a table that has 100 exposed and 80 affected and 500 not exposed and 6 affected. What's the
ratio of exposed to non exposed
44:1
55:1
66:1
answer: A, I got 100% in research.
117- Case of gonorrhea, what are you going to give his close contacts:
A- Rifampin chemoprophylaxis
B- Isolate all contacts for 4 weeks
C- Meningococcal vaccine
Answer: Treat the patient and the partner. oral cefixime and azithromycin is used for treatment of
the partner.
Source: http://www.uptodate.com/contents/treatment-of-uncomplicated-gonococcalinfections?source=search_result&search=gonorrhea+treatment&selectedTitle=1%7E150#H4144433

512

118- a pregnant lady in labor. Multipara and gravida, after presentation to you she had spontaneous
rupture of amniotic membrane. in examination she is 5 cm dilated with 100% effacement of the
cervix with station zero . After three hours still the same and no change. What is your management:
A- expectant management
B- oxytocin
C- prostaglandin E2 (or I2 not sure)
D- CS
answer: B
source: http://www.webmd.com/baby/guide/inducing-labor?page=2
119- after CS, on the 5th day there was discharge from the wound. In examination abdominal
structure can been seen through the wound. What is the diagnosis
a- bowel fistula
b- wound dehiscence
Answer: A
source: http://emedicine.medscape.com/article/179444-clinical
120- after vaginal delivery the patient is complaining of urine coming out of the vagina during the
micturition what is your diagnosis:
a- viscovaginal fistula
b- urterovaginal fistula
c-uretherovainal fistula
d-rectovaginal fistula
Answer: A
source: http://emedicine.medscape.com/article/267943-overview#a7
121- Doctor asked his patient to stand on his toes, which nerve is he testing ?
Answer:
Toe Dorsifelxion by Deep perineal N
Toe Planter flexion by Tibial N
So By Tibial in this case.
http://www.orthobullets.com/spine/2002/lower-extremity-spine-and-neuro-exam
122- after appendectomy a patient got abdominal infection by enterococcus faecium. He is allergic
to penicillin what are you going to give him:
Aceftriaxone
Bvancomycin
CCefotaxime
DTMP-SMX or metronidazole (not sure which one of them was mentioned)
Answer: B
Source: http://www.nhstaysideadtc.scot.nhs.uk/Antibiotic%20site/penhypers.htm
123- patient complaining of hematuria with WBC in urine and the culture is negative. Cystoscopy,
revealed submucosal hemorrhage what is the cause:
Acystolithiasis
Binterstitial cystitis
Answer: B
source: http://emedicine.medscape.com/article/2055505-overview
124- pregnant lady 32 weeks presented with regular abdominal pain and dilation of cervix without
premature rupture of the membrane what is the management ?
A- restrict movement with steroid
B- hydration with steroid
C- oxytocin with IV antibiotic
D- restrict movement with antibiotic IV
Answer: A

513

125- which layer in the scalp the nerves are found in :


A-epicranial aponeurosis
B- connective tissues
C- skin
D- pericranium
Answer: B
source: http://emedicine.medscape.com/article/834808-overview
126- long scenario about patient presented dry cough after being diagnosed with HTN what is the
cause:
a- furosemide
b- ACEI (they mentioned the drug name )
Answer: B
source: http://www.rxlist.com/ace_inhibitors-page2/drugs-condition.htm
127- Indirect hernia sac relation to spermatic cord:
A- Anterior medial
B- Anterior lateral
C- Posterior medial
D- Posterior lateral
answer: A
source: http://fitsweb.uchc.edu/student/selectives/Luzietti/hernia_inguinal_indirect.htm
128- Patient diagnosed with duodenal ulcer he was prescribed medication 1 month ago now he
have gynecomastia which medication:
A.
Cimetidine
B.
lansopraloe
Answer: A
source: http://www.drugs.com/sfx/cimetidine-side-effects.html
129- Patient with acute MI typical scenario, the analgesia he was given will work on: ( most likely
asking about morphine)
A.
inhibition of COX
B.
work on P-peptide or receptor can't remember exactly
Answer: ?
The precise mechanism of the analgesic action of morphine is unknown. However, specific CNS
opiate receptors have been identified and likely play a role in the expression of analgesic effects.
Morphine first acts on the mu-opioid receptors.
Source: http://www.drugbank.ca/drugs/DB00295

130- Old patient that presented with abdominal pain from time to time starts in the left mid abdomen
radiates to the back, whenever he have the attacks, he lies down on that side and bend his body
position like a baby, no vomiting, diarrhea or wt loss what is the diagnosis:
A- duodenal ulcer
B- gastric ulcer
C- chronic pancreatitis
D- mesenteric thrombosis
Answer: C

514

131- Patient with damaged valve after a tooth extraction he got infective endocarditis, what is the
organism:
A.strep viridans
B.staph aureus
Answer: A These account for 50%80% of IE cases. Streptococcus viridans (eg, S. anguis, S.
milleri, S. mutans, S. mitior) make up the normal bacterial flora of the pharynx and upper respiratory
tract. Tonsillectomy, dental extraction, and dental cleaning can result in bacteremia and lead to
infection.
132- Patient with grey vaginal discharge, erythema of skin:
A.candida
B.Bacterial vagainosis
Answer: B
133- Patient with severe painful vesicles on genital area:
A.syphilis
B.HSV
Answer: B
134- Baby, 3rd day after delivery got a purulent eye discharge what is the organism:
A.chlamydia
B.gonorrhea
Answer: B
Gonorrhoeal infection - typically, 1-5 days after birth but it may occur later: hyperacute conjunctival
injection and chemosis, lid oedema and severe purulent discharge. There may be associated
corneal ulceration and perforation.
Chlamydial infection Most common cause, 5-14 days after birth (some report up to 28 days after
birth): unilateral/bilateral watery discharge which becomes copious and purulent later on. There may
be associated preseptal cellulitis and, less commonly, rhinitis, otitis and pneumonitis. The eyes are
usually less inflamed than in the case of gonococcal infection.
135- Old man with DM, has redness in calf area, raised and painful, tender:
- Cellulitis
- Diabetic neuropathy
answer : Erysipelas if demarcated

136- Patient with DM, how to prevent progressive renal disease:


Answer: ACEI
Toronto NP29
137- (24) year old mother presented with her child who is diagnosed with Down syndrome
CLINICALLY, she's asking about the risk of down in her next child, what is the best investigation:
- karyotype this child
- karyotype the mother and child
- do US next pregnancy
- do amniocentesis next pregnancy
Answer: A
If a patient has had a trisomy 21 pregnancy in the past, the risk of recurrence in a subsequent
pregnancy increases to approximately 1 percent above the baseline risk determined by maternal
age. Diagnosis of a chromosome-21 translocation in the fetus or newborn is an indication for
karyotype analysis of both parents. If both parents have normal karyotypes, the recurrence risk is 2
to 3 percent. If one parent carries a balanced translocation, the recurrence risk depends on the sex
of the carrier parent and the specific chromosomes that are fused.

515

138- Mother brought her child, 9 months, fisting hand and crossed legs, she mentioned that after his
birth he didn't cry:
A.cerebral palsy
B.Down syndrome
Answer: A
http://kidshealth.org/parent/medical/brain/cerebral_palsy.html#
139- Muscle responsible of internal rotation of left lower leg:
A.Gluteus maximums
B.gluteus minimus
C.rectus femurs
Answer: B http://www.ptcentral.com/muscles/musclelegs.html
140- 5 years old boy presented with sore throat, he was discharged home, culture was done
showing Group A meningococcus, the physician called the family to inform them he finds that the
child is asymptomatic, the best treatment is:
A.penicillin
B.single dose ceftriaxone
Answer: B
141- Patient with irritably, has delusion and auditory hallucination, now he have flights of idea:
- neurosis
- psychosis
- dissociative disorder
Answer: B
http://www.uptodate.com/contents/clinical-manifestations-differential-diagnosis-and-initial-managementof-psychosis-in-adults?source=search_result&search=psychosis&selectedTitle=1~150#H261969368
142- Man feeling depressed for 3 months, he is fighting with 2 of employee in job, 4 months ago he
became the manager in his job:
A.
Depression
B.
Adjustment
Answer: B
source: http://www.mayoclinic.org/diseases-conditions/adjustment-disorders/basics/symptoms/con20031704

143- Patient with nodulocystic acne with scar what is the treatment:
- oral isotreotoin
- oral antibiotics
- topics antibiotics
Answer: A
Treatment of nodulocystic acne can be challenging and may require a combination of medicines.
Topical treatment is usually ineffective.
The recommended treatment for nodulocystic acne is isotretinoin, which should be commenced
early to prevent scarring. The treatment is required for at least five months, and further courses are
sometimes necessary
http://www.dermnetnz.org/acne/nodulocystic-acne.html

516

144- Long scenario about patient with eye problems, he have myopia, bilateral decrease vision,
glasses was advice, after month he became not not satisfied, there is refractive error and
astigmatism:
- keratoconus
- keratoglobus
- keratoectais
- kerato*****
Answer: A
As the disease progresses, patients experience difficulties with their spectacle correction and
contact lens fitting. Patients may require frequent changes in spectacles due to progressive myopia
and astigmatism
http://www.uptodate.com/contents/keratoconus?source=search_result&search=keratoconus&selectedTitle
=1~14
145- Scrape of skin can be done in which of the following:
Answer- scabies
http://emedicine.medscape.com/article/1109204-workup#c8
146- character of MS murmur:
-diastolic low pitched rumbling
Reference: talley
147- 34 years old Female after examination with Pap smear you found ASCUS what is your next
step:
- Cone biopsy
- colposcopy
- repeat later
- do HPV test
Answer: D
http://www.uptodate.com/contents/follow-up-of-low-grade-abnormal-pap-tests-beyond-the-basics#H3
148- Had history of HPV when she was young, you did Pap test and found nothing what to do now:
A- do nothing
B- repeat every 5 years
C- repeat annually
Answer: C

149- Female while giving birth full dilatation and effacement the child heartbeat decrease from
baseline what is the best analgesic: I don't remember the scenario exactly
- pudendal
- paracervical
- general
- narcotic anesthesia
Answer:
150- UTI patient, results showed, lactose + non fermenting gram negative bacilli:
A.
- klebsiella
B.
- proteus
Answer: A
source: http://www.asp.mednet.ucla.edu/files/view/guidebook/MicrobiologyOverview.pdf

517

151- Poor prognosis schizophrenia:


Answer:
factors affecting prognosis of schizophrenia:
gradual onset, early age, chronic course >2 years, history of schizophrenia, positive family history,
depression, negative symptoms.
152- Patient with heart failure and AF, you added digoxin, what is the benefit:
A.
- decrease heart failure
B.
- slow ventricular rate
C.
- decrease ventricular efficacy
Answer: B
source: Toronto notes 2014 C17
153- Baby with enuresis what to do:
A.
Urine culture
B.
urine analysis
Answer: B
Source: http://emedicine.medscape.com/article/1014762-workup
154- in epidural anesthesia the anesthesiologist hit just lateral to spinal processes which structure
he will injured?!
A- ligamentum flavum
B- posterior ligament
C- interspinous ligament.
D- interior ligament
Answer: A
155- knee aspiration in young boy showing labs result: wbc > 75, >25 neutrophils .. Typical scenario
how you gonna treat:
- oral antibiotic
- IV antibiotic
- surgical drainage with IV antibiotic
Answer : Medical management of infective arthritis focuses on adequate and timely drainage of the
infected synovial fluid, administration of appropriate antimicrobial therapy, and immobilization of the
joint to control pain.
Reference :
http://emedicine.medscape.com/article/236299-treatment

156- Patient with TB, how to prevent her dorm friends from having it:
- no need they want get it
- isolate all contact for 4 weeks
- immunization
- penicillin and other antibiotics like it.
Answer ?
Household contacts of patients with MDR-TB have a particularly high risk for tuberculosis, 7.8%
within 4 years in a study from Lima, Peru. Limited data are available on regimens for the treatment
of patients exposed to MDR-TB. However, if treatment is initiated, at least 2 drugs should be given,
and the index isolate should be susceptible to all drugs used
http://emedicine.medscape.com/article/230802-treatment
[81]

157- Baby while delivery you touched: nose mouth chin, what is the presentation:
A- face
B- brow
Answer : A
http://emedicine.medscape.com/article/262341-overview#showall

518

158- Long case of boy bleeding epistaxis and ecchymosis with long lab results showing anemia
thrombocytopenia and leukopenia, what is the diagnosis:
A- IDA
B- aplastic
C- hypoplastic
D- hemolytic
Answer : B
http://emedicine.medscape.com/article/198759-clinical#showall
159- Patient lost elevation of shoulder and external rotation what is the diagnosis:
A- rotator cuff
B- impingement
Answer : A ?
Passive and active joint restriction in all directions of ROM is caused by a frozen shoulder or
glenohumeral synovitis.
Restriction in internal rotation suggests an impingement syndrome due to rotator cuff
tendinitis.
Inability to perform active abduction suggests a rotator cuff tear or a frozen shoulder.
http://emedicine.medscape.com/article/328253-clinical
160- puptic ulcer medication cause erectile dysfunction and decrease lipido?
A.
cimetidine
answer: A
source: http://www.webmd.com/erectile-dysfunction/guide/drugs-linked-erectile-dysfunction
161- Patient with breast lump painless increased in size >3 cm I don't remember the details:
A- fibroadenoma
B- cancer
C-fibrocystic changes
Answer: A
http://www.medicinenet.com/breast_lumps_in_women/page2.htm

162- patient complaining of hip pain after long periods of using the hip it keeps him awake at night
and have prolonged hours of stiffness in the morning:
A- osteoporosis
B- osteoarthritis
Answer : B
http://emedicine.medscape.com/article/330487-clinical#b1
163- Patient with amenorrhea and discharge from her breast with high prolactin level what to do
next:
A.
check estrogen level
B.
exclude pituitary lesion
C.
TSH level
Answer: C
source: Master the boards USMLE step 2 CK
164- child with diarrhea!!
http://emedicine.medscape.com/article/928598-differential

519

165- Patient ingested naloxone 0.1 ml/kg came to hospital comatose which overdose he got:
- morphine
- bendyrl
- methanol
Answer: A
Naloxone is used to treat a narcotic overdose in an emergency situation. This medicine should not
be used in place of emergency medical care for an overdose
http://www.drugs.com/naloxone.html
166- Patient ingested toxic substance of relative presented after 8 hours with LOC what is the next
step:
A- ipecac
B- activated charcoal
C- gastric lavage
Answer: B When you dont know what to do in toxicology, give charcoal.
Master the boards step2 CK
167- Girl after fight with her friend she doesn't talk to her or call her she avoid her and when her
friend comes to the same place she goes out what is that:
A.
Avoidance
B.
Denial
C.
Depression
Answer: A
Avoidance: mentally or physically avoiding something that causes distress.
Denial: refusing to acknowledge that an event has occurred
168- Long scenario of child with cola color urine what you gonna do next:
- urine sedimentation rate
- RFT
- renal biopsy
Answer : B
http://emedicine.medscape.com/article/240337-workup

169- First sign and symptoms of anesthesia toxicity :


Headache
Tongue fasciculation
Answer : circumoral numbness
http://emedicine.medscape.com/article/1844551-overview
170- 13 y o with hypertension ?
A.
Decrease salt and thiazide
Answer: A
http://www.uptodate.com/contents/treatment-of-hypertension-in-children-and-adolescents

520

171- HIV patient with meningitis ,what is the organism?


Answer:
Different forms of meningitis are associated with HIV infection. They may be classified as follows,
according to the etiologic agent:
Cryptococcal
Tuberculous
Syphilitic
Listeria species
Lymphomatous
Aseptic
Reference: medscape
172- Mother 34 weeks pregnant got pneumonia infection during pregnancy resolved what types of
immunity will give the baby?
Answer: Natural Passive
Reference : https://en.m.wikipedia.org/wiki/Immunity_(medical)
173- Pt with constipation increase weight thinning of hair ?
A.
Hypothyroidism
Answer: A
174- antibiotic contraindicated in pregnancy?
A.
Tetracycline
Answer: A
Source: http://www.rxlist.com/sumycin-drug/warnings-precautions.htm
175- Resolved hip A virus what the microscope will show after six month ?
A- Central necrosis
B- Peripheral
C- Normal
Answer: C
176- Patient got rapid swelling response after a bee bite what type of hypersensitivity ?
A- 1
B- 2
C- 3
D- 4
Answer: A
sorce: http://mcb.berkeley.edu/courses/mcb150/Lecture20/Lecture20(6).pdf

177- 4 years old child says hahalllo (stuttering)what are you going to do?!
A- observation (right answer )
B- Auditory evaluation
C- Speech therapy
Answer: A
its common for children to stutter from 2-5 years old and it's simply part of learning to use language
and putting words together to form sentences.
source: http://www.webmd.com/parenting/stuttering
178- Child with recurrent infection eczema and low platelet?!
a- Wiskott Aldrich Syndrome
answer: A
source: http://emedicine.medscape.com/article/888939-overview

521

179- meningitis with gram +bacilli?


Answer: Listeria monocytogenes
Reference: step up to medicine
180- Neuraminidase inhibitor?
neuraminidase found on the surface of influenza viruses that enables the virus to be released from
the host cell. Neuraminidases are enzymes that cleave sialic acid groups from glycoproteins and are
required for influenza virus replication.!
181- Man present With hepatosplenomegaly ,blue nodules and neck swelling, what is the
investigation you are going to do?(rubella)!
serology!
If it was EBV the answer is The heterophile antibody test (eg, the Monospot test)
Reference: http://emedicine.medscape.com/article/222040-workup#c3
182- patient injured her eye with fingernail (picture of cornea with fluorescein dye)
A.Corneal abrasion
B.Corneal laceration
C.Corneal ulceration
Answer:A
183- Patient with pain when moving one eye and decreased vision:
A.
Optic neuritis
Answer: A
source: http://emedicine.medscape.com/article/1217083-overview
184- baby with symptoms of peptic ulcer his father has peptic ulcer he is on PPI
, metronidazole .. what to add:
A.
clarithromycin
B.
tetracycline.
answer: A
source: http://www.ncbi.nlm.nih.gov/pubmed/9651466
185- abnormal connection between aorta and pulmonary trunk:
A.
PDA
Answer: A
186- preterm delivery before ?
A- 34
B- 32
C- 37
D- 23
Answer : C
Preterm: b/w 25 to 27
Term: b/w 38 to 42
Postterm : after 42
Reference : master the board

187- Which of the following medication decrease mucus production in a patient with emphysema?
A- Cromolyn sodium

522

B- steroids
Answer: http://www.uptodate.com/contents/role-of-mucoactive-agents-in-the-treatment-ofcopd?source=machineLearning&search=Mucolytic+agents&selectedTitle=1~50&sectionRank=2&anchor=H2
6#H518133719
188- digoxin toxicity treatment:
Source: http://bestpractice.bmj.com/best-practice/monograph/338/treatment/step-by-step.html
189- cold induced urticaria drug:
answer : Treatment for cold urticaria includes
taking antihistamines
avoiding cold air and water.
source: http://www.mayoclinic.org/diseases-conditions/cold-urticaria/basics/treatment/con-20034524
200- patient not hyperventilating PH: LOW, Bicarb: low co2: normal:
A.
metabolic acidosis
B.
metabolic alkalosis
C.
respiratory acidosis
D.
respiratory alkalosis
answer: metabolic acidosis
source: https://herzing.blackboard.com/bbcswebdav/pid-5299257-dt-content-rid-11795236_1/courses/062132-A-PN108-1/ABGebook.pdf
201- Picture not clear for shoulder asking for the diagnosis:
A.Rotator cuff tear
B. acromioclavicular
Answer : A

202- Blood film attached, asking for diagnosis:


- leishmaniasis
- malaria
- lymphoma
- leukemia
Answer:
Malaria > http://www.cdc.gov/dpdx/malaria/gallery.html
Lymphoma and leukemia > http://library.med.utah.edu/WebPath/HEMEHTML/HEMEIDX.html

203- cup picture attached asking for the diagnosis:

523

- glaucomatous
- papillitis
- diabetic retinopathy
Answer: look for pictures of each.
glaucoma, which is in most cases associated with an increase in intraocular pressure, often
produces additional pathological cupping of the optic disc. The pink rim of disc contains nerve fibers.
The white cup is a pit with no nerve fibers.
Glaucomatous disk appears as increased in cup/ disk ratio which shows the blood vessles coming
out from the periphery of the cup
Papillitis is a term used when the optic disc (portion of the optic nerve that enters the retina)
becomes inflamed in one of the eyes. Papillitis is often unilateral (affecting only one eye) and is
associated with Optic Neuritis.
diabetic retinopathy is the result of damage to the tiny blood vessels that nourish the retina. They
leak blood and other fluids that cause swelling of retinal tissue and clouding of vision. The condition
usually affects both eyes. (flame shaped hemorrhages in the retina).
204. alk which enzyme confirm that elevation from liver?
Answer: gamma-glutamyl transpeptidase (GGT) or 5'-nucleotidase (5'-NT)
205. Adolescent received one dose of varicella came after 12 months:
A.
Give the other dose
B.
Repeat the 2 doses
C.
Repeat all series
Answer A
What is the recommended schedule for vaccinating a child? What about adults?
For children, the first dose should be given at age 12 months with a second dose given at age 4
through 6 years. The second dose could be given earlier, if necessary, as long as there is a 3month interval between doses. All children age 13 years and older as well as adults without
evidence of immunity should also have documentation of 2 doses of varicella vaccine, separated
by a minimum interval of 4 weeks.
http://www.immunize.org/askexperts/experts_var.asp
206- Which part of the female urethra is more susceptible to be damaged ( in an intervention I
cant remember what ?
Urethral injury Intraoperative urethral perforation or laceration occurs in approximately 0.9
percent of midurethral retropubic sling placements. This type of complication may be diagnosed
during cystourethroscopy. When recognized immediately, in our practice, we do not complete the
procedure, since we consider this a high risk situation for urethral mesh erosion. We then leave a
urethral catheter in place for seven days. The midurethral sling may be performed at a later time
after urethral healing.
Rarely, urethral injury is not recognized immediately or injury occurs postoperatively due to mesh
erosion. These injuries often present several weeks or months postoperatively with periurethral
discomfort, hematuria, voiding difficulty, or irritative voiding symptoms. Urodynamic evaluation may
reveal evidence of outflow obstruction. Management includes excision of the eroded portion of the
mesh and urethroplasty with the use of local flaps where necessary.
http://www.uptodate.com/contents/surgical-management-of-stress-urinary-incontinence-in-womenretropubic-midurethralslings?source=see_link&sectionName=Urethral+injury&anchor=H4701082#H4701082

207- patient with loss of shoulder passive and active movement ?


Answer:

524

Adhesive capsulitis (also known as Frozen shoulder) Movement of the shoulder is severely
restricted, with progressive loss of both active and passive range of motion.[1] The condition is
sometimes caused by injury, leading to lack of use due to pain, but also often arises spontaneously
with no obvious preceding trigger factor (idiopathic frozen shoulder),Risk factors for frozen shoulder
include tonic seizures, diabetes mellitus, stroke, accidents, lung disease, connective tissue
diseases, thyroid disease, and heart disease. Treatment may be painful and taxing and consists of
physical therapy, occupational therapy, medication, massage therapy, hydrodilatation or surgery. A
physician may also perform manipulation under anesthesia, which breaks up the adhesions and
scar tissue in the joint to help restore some range of motion. Pain and inflammation can be
controlled with analgesics and NSAIDs.
208- Glaucoma drugs side effect ?
Answer: http://www.glaucoma.org/treatment/glaucoma-medications-and-their-side-effects.php
209. 12 week pregnant w high blood pressure ?
Answer:

525

526

THANK YOU,
SMLE Q BANK Group

527

Vous aimerez peut-être aussi